questions - uhs.edu.kh you suspect lyme disease, most likely contracted by which of the following?...

313
DIRECTIONS (Questions 1 through 24): Each of the numbered items or incomplete statements in this section is followed by answers or completions of the statement. Select the one lettered answer or completion that is best in each case. Clue: Questions 1 and 2 are simple and straightfor- ward, each with only one possible correct answer. For this type of question, you can actually answer the question before looking at the choices. Then read the choices to verify that your answer is there. Finally, review all the other choices to be sure that none is better than the one you selected. 1. You have just confirmed the diagnosis of cystic fibrosis in a 3-year-old child. The parents are concerned about future pregnancies. You explain to them that the pattern of genetic transmission of cystic fibrosis is (A) autosomal dominant (B) autosomal recessive (C) X-linked recessive (D) X-linked dominant (E) autosomal recessive in some families and X-linked in others 2. A 12-year-old boy just returned from Boy Scout Camp in Wisconsin. He now has fever, myal- gia, and a 10-cm skin lesion which looks like a target. You suspect Lyme disease, most likely contracted by which of the following? (A) ingestion of unripe fruit (B) ingestion of spoiled fruit (C) drinking of contaminated water (D) the bite of a tick (E) the bite of a mosquito Clue: Unlike the preceding two questions, the answer to Question 3 cannot be anticipated before viewing the list of suggested answers, because there are many possible completions to the statement. Nevertheless, the question is simple and straightforward. 3. A 2-year-old boy presents with extremity swelling and proteinuria and is found on urine analysis. Minimal-change disease is suspected and you explain to the child’s parents that this diagnosis (A) is the most common cause of nephrotic syndrome in childhood (B) has a peak incidence in children between 10 and 15 years of age (C) usually results in end-stage renal dis- ease in 5–10 years (D) is characterized by normal serum lipids and cholesterol (E) typically has a poor response to corti- costeroid treatment Clue: Pay attention to key words when you read a question. Question 4 contains the key words “most likely.” 4. You have just prescribed phenytoin for a 12- year-old boy with new onset of epilepsy. Of the following side effects, which is most likely to occur in this patient? (A) lymphoma syndrome (B) Raynaud phenomenon (C) acute hepatic failure (D) gingival hyperplasia (E) optic atrophy Questions 2

Upload: others

Post on 14-Nov-2020

1 views

Category:

Documents


0 download

TRANSCRIPT

Page 1: Questions - uhs.edu.kh You suspect Lyme disease, most likely contracted by which of the following? (A) ingestion of unripe fruit (B) ingestion of spoiled fruit (C) drinking of contaminated

DIRECTIONS (Questions 1 through 24): Each ofthe numbered items or incomplete statements inthis section is followed by answers or completionsof the statement. Select the one lettered answer orcompletion that is best in each case.

Clue: Questions 1 and 2 are simple and straightfor-ward, each with only one possible correct answer.For this type of question, you can actually answerthe question before looking at the choices. Thenread the choices to verify that your answer is there.Finally, review all the other choices to be sure thatnone is better than the one you selected.

1. You have just confirmed the diagnosis of cysticfibrosis in a 3-year-old child. The parents areconcerned about future pregnancies. Youexplain to them that the pattern of genetictransmission of cystic fibrosis is

(A) autosomal dominant(B) autosomal recessive(C) X-linked recessive(D) X-linked dominant(E) autosomal recessive in some families

and X-linked in others

2. A 12-year-old boy just returned from Boy ScoutCamp in Wisconsin. He now has fever, myal-gia, and a 10-cm skin lesion which looks like atarget. You suspect Lyme disease, most likelycontracted by which of the following?

(A) ingestion of unripe fruit(B) ingestion of spoiled fruit(C) drinking of contaminated water(D) the bite of a tick(E) the bite of a mosquito

Clue: Unlike the preceding two questions, the answerto Question 3 cannot be anticipated before viewingthe list of suggested answers, because there are manypossible completions to the statement. Nevertheless,the question is simple and straightforward.

3. A 2-year-old boy presents with extremityswelling and proteinuria and is found on urineanalysis. Minimal-change disease is suspectedand you explain to the child’s parents that thisdiagnosis

(A) is the most common cause of nephroticsyndrome in childhood

(B) has a peak incidence in childrenbetween 10 and 15 years of age

(C) usually results in end-stage renal dis-ease in 5–10 years

(D) is characterized by normal serum lipidsand cholesterol

(E) typically has a poor response to corti-costeroid treatment

Clue: Pay attention to key words when you read aquestion. Question 4 contains the key words “mostlikely.”

4. You have just prescribed phenytoin for a 12-year-old boy with new onset of epilepsy. Ofthe following side effects, which is most likelyto occur in this patient?

(A) lymphoma syndrome(B) Raynaud phenomenon(C) acute hepatic failure(D) gingival hyperplasia(E) optic atrophy

Questions

2

Page 2: Questions - uhs.edu.kh You suspect Lyme disease, most likely contracted by which of the following? (A) ingestion of unripe fruit (B) ingestion of spoiled fruit (C) drinking of contaminated

Questions: 1–12 3

5. A term infant is born to a mother who has beenusing crack cocaine. This infant is at increasedrisk for which of the following?

(A) anemia(B) intrauterine growth retardation(C) hypercalcemia(D) macrosomia(E) postmaturity

6. You examine an 18-year-old male college stu-dent with a 5-day history of fever, sore throat,and fatigue. Physical examination reveals anexudative tonsillitis and bilateral enlarged andslightly tender posterior cervical lymph nodes.The spleen is palpable 3 cm below the rib cage.Which agent is most likely responsible for thispatient’s illness?

(A) Group A β-hemolytic streptococcus(B) Adenovirus(C) Toxoplasma gondii(D) Epstein-Barr virus(E) Corynebacterium diphtheriae

7. A term infant requires intubation in the deliv-ery room after aspiration of thick meconiumand is brought to the neonatal intensive careunit. Which of the following is the most likelyrisk factor for meconium aspiration syndromein this infant?

(A) chromosomal anamoly(B) congenital heart disease(C) cystic fibrosis(D) fetal distress(E) tracheoesophageal fistula

8. A newborn infant with stigmata of Down syn-drome has a heart murmur. Which of the fol-lowing cardiac lesions is most likely in this baby?

(A) hypoplastic left heart syndrome(B) total anamolous venous return(C) coarctation of the aorta(D) anamolous coronary artery(E) atrioventricular defect

9. Amother of a 2-month-old wants more informa-tion about immunizations. Which of the follow-ing statements regarding immunization againstHaemophilus influenzae type b (Hib) is correct?

(A) It is indicated for high-risk children only.(B) Hib vaccine can be administered effec-

tively as early as 2 months of age.(C) Hib vaccine should not be given to children

who have had allergic reactions to eggs.(D) Hib vaccine should not be administered

to children with a history of reaction toDTaP immunization.

(E) Hib vaccine should be deferred forinfants with history of febrile seizures.

10. Acellular pertussis vaccine is recommendedfor infants, children, adolescents, and adults.Compared to the previously available wholecell vaccine, which of the following bestdescribes these products?

(A) they are more immunogenic(B) they are less expensive(C) they are associated with fewer side effects(D) they require fewer doses(E) they can be combined with the varicella

vaccine for the infant under age 1 year

11. It is recommended that young infants shouldsleep in the supine rather than in the prone posi-tion. This is based on data suggesting that theprone position is associated with an increasedincidence of which of the following?

(A) delayed eruption of the first deciduous teeth(B) gastroesophageal reflux and aspiration(C) macrognathia(D) strabismus(E) sudden infant death

12. The feeding of honey to infants less than 6 monthsof age has been associated with which of thefollowing?

(A) anaphylaxis(B) hypernatremia(C) botulism(D) jaundice(E) listeriosis

Page 3: Questions - uhs.edu.kh You suspect Lyme disease, most likely contracted by which of the following? (A) ingestion of unripe fruit (B) ingestion of spoiled fruit (C) drinking of contaminated

4 1: Warm-Up Questions and Exam-Taking Skills

13. A1-month-old infant presents with fever of 39°Cand vomiting. He was born at term vaginally toan 18-year-old mother who did not have prena-tal care. On examination, he is alert but fussyand cries with palpation of his abdomen. He isuncircumcised and both testes are descended.An evaluation for sepsis and meningitis isperformed. Urine analysis shows 50–100 WBC/HPF with positive leukocyte esterase andnitrites. CSF examination is normal and culturesfrom blood, urine, and CSF are pending. Youtell his mother that he has urinary tract infectionand she asks why this happened. Which of thefollowing is correct in explaining this infant’smost likely risk for urinary tract infection?

(A) The mother was colonized with GroupB streptococcus and did not receiveintrapartum prophylaxis.

(B) The infant is uncircumcised.(C) The infant has prune belly syndrome.(D) The infant has galactosemia.(E) There is a family history of vesicoureter-

al reflux.

14. A3-month-old infant presents with poor growthand inadequate weight gain. There is no historyof vomiting or diarrhea. Except for the appear-ance of malnutrition and lack of subcutaneous fat,the physical examination is normal. What is themost likely cause of this child’s failure to thrive?

(A) renal disease(B) a metabolic disorder(C) tuberculosis(D) an endocrine disorder(E) a nonorganic cause

15. A 2-year-old child is being evaluated becausethe mother notes that her right eye has beenturning in. Physical examination documentsstrabismus with a right esotropia. Attempts tovisualize the fundi are unsuccessful, but it isnoted that the red reflex is replaced by a yellow-white pupillary reflex in the right eye. This childmost likely has which of the following?

(A) retinitis pigmentosa(B) retinoblastoma(C) rhabdomyosarcoma

(D) severe hyperopia(E) severe myopia

16. A 2-year-old child is admitted because of weak-ness proceeding to coma. According to the par-ents, he had been well until several hours prior toadmission, when they noted diarrhea, cough,wheezing, and sweating. Physical examinationreveals a comatose child with diffuse weaknessand areflexia. Pupils are pinpoint and unrespon-sive. Examination of the chest reveals generalizedwheezing. Oral secretions are copious. Which ofthe following should you administer at this time?

(A) adrenaline(B) atropine(C) cefotaxime(D) methylprednisolone(E) edrophonium

17. A3-week-old infant is admitted with vomiting of5 days’ duration. Physical examination reveals arapid heart rate, evidence of dehydration, andambiguous genitalia. Serum electrolytes are Na+

120 meq/L, K+ 7.5 meq/L, HCO3− 12 meq/L,

BUN 20 mg/dL. In addition to intravenous fluidreplacement with normal saline, administration ofwhich of the following would be most important?

(A) diuretics(B) potassium exchange resin(C) glucose and insulin(D) antibiotics(E) hydrocortisone

18. A previously well 12-year-old girl presents toclinic because of painful swellings on the front ofthe legs of about 3 days’ duration. Examinationreveals tender erythematous nodules, 1–2 cm indiameter, on the extensor surfaces of the lowerlegs. The remainder of the physical examinationis unremarkable. Which of the following is mostlikely to confirm the cause of this condition?

(A) stool smear and culture(B) urine analysis and BUN(C) throat culture(D) slit-lamp examination of the eye(E) echocardiogram

Page 4: Questions - uhs.edu.kh You suspect Lyme disease, most likely contracted by which of the following? (A) ingestion of unripe fruit (B) ingestion of spoiled fruit (C) drinking of contaminated

Questions: 13–23 5

19. An 18-year-old boy presents with cough, chestpain, and low-grade nightly fevers of severalweeks duration. He has a 4-year history ofsmoking two packs of cigarettes per day. Chestx-ray reveals a large mass in the mediastinumwith extension into the right upper chest. Whichof the following is the most likely diagnosis?

(A) adenocarcinoma(B) squamous cell carcinoma(C) small cell carcinoma(D) lymphoma(E) metastatic Wilms tumor

20. A 12-year-old child is seen because of a rash andsevere headache which began 2 weeks afterreturning from vacation in Massachusetts. Theskin lesion began as a red macule on the thigh,which gradually expanded over 1 week to reachapproximately 15 cm in diameter with red bordersand central clearing. The lesion is slightly painful.A few days after the onset of the skin manifesta-tion, the child developed severe headache, myal-gias, arthralgias, and malaise. Low-grade feverwas present. The mother recalls that the child wasbitten by a tick about 1 week prior to the onset ofsymptoms. This patient’s disorder is probablybest treated with which of the following?

(A) corticosteroids(B) diphenhydramine(C) methotrexate(D) nonsteroidal anti-inflammatory drugs(E) doxycycline

21. An 8-year-old child is hospitalized because ofparoxysms of severe colicky abdominal painwhich does not radiate to the back or the groin.Physical examination is unremarkable exceptfor generalized abdominal tenderness. Anexploratory laparotomy reveals an edematousintestine without specific lesions. The appendixappears normal but is removed. Postoperativelythe abdominal pain persists, and hematuriadevelops. Values for BUN and creatinine arenormal. On the second postoperative day,tender swelling of both ankles and knees isnoted. Which of the following additional find-ings would most likely be present in this child?

(A) shock(B) meningitis(C) hepatitis(D) a purpuric rash(E) hemorrhagic pancreatitis

22. A 10-year-old boy has been having episodes ofrepetitive and semipurposeful movements of theface and shoulders. The parents believe thesemovements are worse when the child is underemotional stress. They also volunteer that they havenever noted the movements while the patient isasleep. The movements have been present for morethan 6 months. The parents are now especiallyconcerned because the child has developedrepetitive episodes of throat clearing and snorting.Physical and neurologic examinations are entirelynormal. During the examination you note that thechild has some blinking of the right eye, twitchingof the right face, and grimacing. You ask him tostop these movements, and he is temporarily suc-cessful in doing so, but the movements recur. Thehome situation, social history, and child’s devel-opment and social adjustment appear normal. Ahead CT scan is normal. Of the following, whichwould be the most appropriate next step?

(A) order an electroencephalogram(B) prescribe carbamazepine(C) prescribe corticosteroids(D) prescribe haloperidol(E) refer the child to a psychiatrist

23. A 3-month-old infant is hospitalized because ofrecurrent right focal seizures that are now gen-eralized in nature. Birth and perinatal historyare unremarkable. You note that the child hasa flat, purplish-red skin lesion on the left sideof the face extending onto the forehead. Theremainder of the examination including a com-plete neurologic examination is within normallimits. The results of a lumbar puncture arenormal. You order a CT scan of the head andanticipate seeing which of the following?

(A) agenesis of the corpus callosum(B) a porencephalic cyst(C) gyriform calcifications(D) hydrocephalus(E) normal findings

Page 5: Questions - uhs.edu.kh You suspect Lyme disease, most likely contracted by which of the following? (A) ingestion of unripe fruit (B) ingestion of spoiled fruit (C) drinking of contaminated

6 1: Warm-Up Questions and Exam-Taking Skills

24. On routine examination of the children of amigrant farm worker, you notice that a 12-year-oldchild who has received little previous medicalcare is short and mentally retarded. Physical exam-ination reveals that the liver is enlarged to 5 cmbelow the right rib cage, and the spleen is enlarged6 cm below the left rib cage. Lumbodorsal kypho-sis is prominent. The child has a peculiar facieswith thick lips and a large tongue. Attempts tovisualize the retina are unsuccessful becauseof clouding of the corneas. You expect thatexamination of this child’s urine will revealwhich of the following?

(A) dermatan and heparan sulfate(B) galactose(C) mannose(D) the odor of maple syrup(E) the odor of sweaty feet

DIRECTIONS (Questions 25 through 30): Each setof matching questions in this section consists of alist of several numbered items introduced by 5–26lettered options. For each numbered item selectthe one lettered option with which it is most closelyassociated. Each lettered option may be selectedonce, more than once, or not at all.

Selection of answers for Questions 25 through 27

(A) miliaria rubra(B) verrucae vulgaris(C) condyloma acuminatum(D) molluscum contagiosum(E) pityriasis rosea

25. Small (pinhead to 1 cm), pearly papules withtranslucent tops and waxy, whitish materialinside, distributed on the face and anteriortrunk; some lesions are umbilicated

26. Soft, flesh-colored papular or pedunculatedlesions around the genitalia and rectum

27. Oval, maculopapular lesions oriented with thelong axis along skin tension lines

Selection of answers for Questions 28 through 30

(A) ABO incompatibility(B) α1-antitrypsin deficiency(C) biliary atresia(D) breastfeeding jaundice(E) breast milk jaundice(F) choledochal cyst(G) cholelithiasis(H) Crigler-Najjar syndrome(I) cystic fibrosis(J) Dubin-Johnson syndrome

(K) erythroblastosis (Rh incompatibility)(L) galactosemia

(M) glucose-6-phosphate dehydrogenasedeficiency

(N) hepatitis(O) hereditary spherocytosis(P) hypothyroidism(Q) physiologic hyperbilirubinemia(R) sepsis

28. A 3-day-old term, healthy infant is noted to bejaundiced. Physical examination is otherwisenormal. Laboratory values: Hb 16.8 g/dL; retic-ulocytes 1.0%; bilirubin unconjugated 8.5 mg/dL,conjugated 0.8 mg/dL.

29. A 5-week-old infant has been jaundiced forabout 2 weeks. He has been asymptomatic andphysical examination is otherwise normal.Laboratory values: Hb 14.2 g/dL; reticulocytes1.2%; bilirubin unconjugated 4.5 mg/dL, con-jugated 5.5 mg/dL; ALT 25 IU/L, AST 75 IU/L.Abdominal ultrasound examination reveals anormal-size liver; gallbladder is not visualized.

30. An otherwise well 4-week-old infant hasremained jaundiced since day 3 of life despitetwo exchange transfusions and continuous pho-totherapy. Laboratory values: Hb 14 g/dL; retic-ulocytes 1.0%; bilirubin unconjugated 16 mg/dL,conjugated 0.2 mg/dL; ALT 15 IU/L, AST 40IU/L. A Coombs test prior to the first exchangetransfusion was negative. Ultrasound exami-nation reveals a normal liver and gallbladder.

Page 6: Questions - uhs.edu.kh You suspect Lyme disease, most likely contracted by which of the following? (A) ingestion of unripe fruit (B) ingestion of spoiled fruit (C) drinking of contaminated

7

Answers and Explanations

1. The correct answer is (B). Cystic fibrosis (CF) isan autosomal recessive disorder with a diseaseincidence in the Caucasian population of about1:1500 and a corresponding carrier state of about1:20. Currently, CF represents the most commonlethal genetic disease in the Caucasian popula-tion. The disease is much less common amongAfrican Americans and Asians. (McMillan, 1490;Rudolph, 1967)

2. Like Question 1, this question has only onepossible correct answer, and you should havebeen able to come up with that answer beforelooking at the list of choices. The answer is (D),bite of a tick. Borrelia burgdorferi, the spirochetethat causes Lyme disease, is transmitted tohumans by the bite of a tick, most commonlyIxodes species, although in some geographicareas, other ticks such as Ambylomma ameri-canum (the lone star tick) have been incrimi-nated. (AAP:Red Book, 428–433)

3. The correct answer is (A). This question is simplein that it deals with well-known and impor-tant clinical features of a common disease—minimal-change nephrotic syndrome (MCNS).It is straightforward in that not only is one ofthe listed choices (A) clearly the best, but theother four choices all are incorrect. Minimal-change disease is the most common cause ofnephrotic syndrome in childhood, and accountsfor more than all other causes combined. Thepeak incidence is between 2 and 5 years of age.The prognosis is very favorable, and theprocess rarely progresses to end-stage renal dis-ease. Serum lipids and cholesterol are elevated,as they are with other causes of nephrotic syn-drome. Finally, the disease characteristically

responds well to treatment with corticosteroids,with only a small minority of patients failing toremit. (Rudolph, 1691–1693; McMillan, 1796–1797)

4. The correct answer is (D). Optic atrophy is nota recognized complication of phenytoin ther-apy. Acute hepatic failure, a lymphoma-likesyndrome, and Raynaud phenomenon all havebeen noted rarely with this drug. Gingivalhyperplasia is a common and troublesome sideeffect, which often can be minimized by scrupu-lous dental hygiene. If the examinee knew thata lymphoma-like syndrome has been reportedwith phenytoin and focused in on that withoutcarefully considering all subsequent choices, heor she might have selected (A). You can avoidsuch errors by carefully reading the questionand asking yourself, “What are the commonside effects of this drug?” even before looking atthe choices. Knowing that gingival hypertro-phy is a very common side effect of phenytoinwould be sufficient knowledge to answer thequestion correctly. Another way to approachthis question would be to ask yourself, “Of thefollowing side effects, which is most frequent?”(Rudolph, 1292; McMillan, 2054)

5. This is another straightforward, completely fac-tual question. Only one choice is correct. As amatter of fact, two answers are not only incor-rect, they are exact opposites of what actuallyhappens with cocaine, so it should be easy to beconfident that they are incorrect. Infants born towomen using cocaine, especially crack cocaine,have an increased incidence of prematurity (notpostmaturity) and low birth weight (not macro-somia). Intrauterine growth retardation withdisproportionate decrease in head size is noted

Page 7: Questions - uhs.edu.kh You suspect Lyme disease, most likely contracted by which of the following? (A) ingestion of unripe fruit (B) ingestion of spoiled fruit (C) drinking of contaminated

8 1: Warm-Up Questions and Exam-Taking Skills

in infants exposed prenatally to cocaine. Thecorrect answer is (B). (Rudolph, 2023)

6. Although this question also is quite straight-forward, it potentially is more difficult than thepreceding questions because several of theagents listed can explain many of the features ofthis adolescent’s illness. If you read the instruc-tions carefully, you noted that you were askedto select the one best answer, which does notimply that all other choices are totally withoutmerit. Consider which of the above choices bestfits the clinical scenario, and therefore, which ismost likely responsible for this patient’s illness.

The correct answer is (D). The agent mostlikely responsible for this child’s illness is theEpstein-Barr virus (EBV). The clinical pictureis strongly suggestive of mononucleosis.Although this patient could be infected withgroup A streptococcus or adenovirus, severalfeatures are much more characteristic of EBVinfection than either of these. They include thefact that the child is a college student, thepresence of splenomegaly, and the fact thatthe adenopathy is posterior rather than anteri-or and is only slightly tender. While thesefindings also could be explained by toxoplas-mosis, this diagnosis is rarely confirmed as acause of acute exudative tonsillitis and cervi-cal adenitis in the United States. Although it isappropriate to think of diphtheria in patientswith acute exudative tonsillitis, there is noth-ing specific in this case to suggest that diag-nosis. Infectious mononucleosis is clearly themost likely diagnosis. (Rudolph, 1035–1038)

7. (D) Fetal distress is the major risk factor formeconium aspiration. The mechanism involvesthe loss of anal sphincter tone, passage of meco-nium into the amniotic fluid, and aspirationby the distressed, gasping infant during theprocess of birth. The thick meconium obstructsthe airways, causing tachypnea, retractions,and grunting.

This is the type of question in which a littleknowledge can go a long way. If you knew thatmeconium aspiration was a relatively commonproblem in the delivery room, you could elim-inate (C) cystic fibrosis (a relatively uncommondisease) as its cause. (Meconium ileus, which is

associated with cystic fibrosis, has nothing to dowith meconium aspiration.) If you realized thataspiration of meconium can only occur before orduring delivery, you also could eliminate (B)congenital heart disease and (A) neonatalmeningitis, as neither of these generally causedistress during delivery. Finally, you should beable to figure out that a tracheoesophageal fis-tula, with or without associated esophagealatresia, would lead to aspiration of saliva, milk,or gastric contents after birth but would not pre-dispose to aspiration of meconium. (Rudolph, 203)

A severe pneumonia following meconiumaspiration occurs as an in utero response to sig-nificant hypoxic or ischemic stress. Infants whohave fetal distress, thick meconium, andAPGAR scores of less than 7 at 1 and 5 minutesare at increased risk for meconium aspirationsyndrome. When meconium staining of amni-otic fluid is noted, the appropriate approach tocare of the infant according to The NeonatalResuscitation Program (NRP) of the AmericanAcademy of Pediatrics and American HeartAssociation include intubation when the infantis not vigorous (defined as having poor respi-ratory efforts, poor muscle tone, and a heartrate less than 100 beats/minute). (Rudolph, 194)

8. (E) The overall incidence of congenital heart dis-ease in the general population is less than 1% butabout 40% of children with Down syndromehave heart defects. The most common lesions inchildren with Down syndrome include atri-oventricular septal defects, ventricular septaldefects and atrial septal defect, or patent ductusarteriosus. Atrioventricular septal defects (AVcanal) is most often seen in these children,making up approximately 60% of the congenitalheart disease found in trisomy 21 but accountingfor less than 3% of congenital heart defects in thegeneral population. (Rudolph, 732)

9. Prior to the initiation of conjugate H influenzaetype b (Hib) vaccine in 1990, infection caused bythis pathogen was a major cause of morbidityand mortality in the young infant. A Hib poly-saccharide vaccine, first licensed in 1985 wasnot effective in infants less than 2 years of age;unfortunately, the greatest incidence of Hibmeningitis was in this age group. The conjugate

Page 8: Questions - uhs.edu.kh You suspect Lyme disease, most likely contracted by which of the following? (A) ingestion of unripe fruit (B) ingestion of spoiled fruit (C) drinking of contaminated

Answers: 6–16 9

vaccines used a process of bonding the polysac-charide to a protein carrier which served as amore effective antigen, greatly improving theimmunogenecity in the young child. It is esti-mated that 95% of Hib-immunized children willdevelop protective antibody after a primaryseries starting when the child is 2 months of age.

The correct answer to this question, there-fore, is (B). It is now recommended that all infantsbe immunized with the conjugated Hib vaccine.Depending on the brand of vaccine used, infantsshould receive a series of two or three immuniza-tions between 2 and 6 months of age, followed byone booster dose at 12–15 months of age.(AAP:Red Book, 314–315)

10. The correct answer is (C). Current pertussis vac-cines used in the United States are acellular prod-ucts which contain two or more purified Bordetellapertussis immunogens. These vaccines have alower risk of adverse events when compared topreviously used whole cell pertussis vaccines. Interms of immunogenecity, efficacy, and number ofdoses, they do not differ but they are more expen-sive than whole cell products. (Rudolph, 43, 45)

11. (E) While the etiology of sudden infant deathsyndrome (SIDS) is not completely elucidated,since 1992, The American Academy ofPediatrics has recommended that infants beplaced to sleep on their backs. Since that time,the frequency of prone sleeping has decreasedfrom greater than 70% to approximately 20% ofUS infants, and the SIDS rate has decreased bymore than 40%. (Rudolph, 936)

12. Cases of botulism predominantly occuring ininfants less than 6 months of age, have followedintroduction of nonhuman milk sources inbreastfed infants. Honey is considered a poten-tial risk for infection. Because honey can containspores of Clostridium botulinum, this productshould not be given to infants under 1 year ofage. (AAP:Red Book, 258, 863)

13. The risk factor for urinary tract infection forthe infant in this scenario is the fact that he is anuncirmcised male. Uncircumcised boys in thefirst year of life have a greater than eightfoldhigher incidence than girls or circumcised boys.

It is suggested that the presence of the foreskinallows for easier bacterial colonization of theperiurethral region. While urinary tract infec-tions can occur in infants with galactosemia orprune belly syndrome, there is no informationgiven to suggest that either diagnosis is presentin the infant. Vesicoureteral reflux is present in25% of infants with urinary tract infection andfamilial risk is described but is not the likelyrisk for this infant. Urinary tract infections arecaused by Escherichia coli in 90% of cases andmaternal colonization is a risk for Group Bstreptococcal infection. (Rudolph, 306, 1668–1669)

14. (E) Failure to thrive (FTT) is a common pediatricproblem characterized by poor growth, espe-cially in regard to weight gain. Today, in theUnited States, nonorganic causes of FTTaccount for 30% to more than 50% of the casesin most series and are responsible for morecases than any other etiology. Nonorganiccauses encompass a diverse spectrum extend-ing from poverty and lack of food, through poorparenting skills and misguided feeding to frankneglect or abuse. Knowledge of the commonreasons for failure to thrive allows one to elim-inate the other choices because none of the otherchoices listed accounts for more than 5% or 10%of cases. If gastrointestinal problems had beena choice, the question would have been moredifficult, because gastrointestinal disordersaccount for up to 25% of cases of FTT in mostseries. (McMillan, 1048–1049; Rudolph, 7–8)

15. (B) Retinoblastoma is the most likely cause ofthis child’s strabismus and white pupillaryreflex (leukokoria, cat’s eye reflex). Althoughrhabdomyosarcoma may involve the orbit, it isextrinsic to the globe and does not cause awhite pupillary reflex. Other causes of leukoko-ria include visceral lava migrans (Toxocara canisinfection) and retrolental fibroplasia. Althoughretinoblastoma is rare, the association withleukokoria is a classic and important pediatricentity with which all students, pediatric houseofficers and practitioners should be familiar.(Rudolph, 2395–2396)

16. This format is common on national medicalexaminations. It makes the question difficult

Page 9: Questions - uhs.edu.kh You suspect Lyme disease, most likely contracted by which of the following? (A) ingestion of unripe fruit (B) ingestion of spoiled fruit (C) drinking of contaminated

10 1: Warm-Up Questions and Exam-Taking Skills

because it requires recall rather than recognition.The question asks you to identify a disease orcondition but does not provide a list of diag-noses from which to choose; instead, it pro-vides a list of associated findings, in this case,treatments. To answer the question correctlyyou must analyze the clinical findings andrecall the disease rather than selecting (recog-nizing) it from a list.

The sudden onset of neurologic signs orsymptoms in a previously well toddler alwaysought to raise suspicion of a toxin or poisoning.The correct answer is (B). This child is a victim oforganophosphate poisoning. Organophosphateand carbamate insecticides are widely usedthroughout the United States and are importantcauses of poisoning in children. These drugsproduce both muscarinic effects (rhinorrhea,wheezing, pulmonary edema, salivation, vom-iting, cramps, bradycardia, and pinpointpupils) as well as nicotinic effects (twitching,weakness and paralysis, convulsions, coma,and respiratory failure). These children ofteningest the toxic substance unobserved, and thediagnosis must be suspected on the basis of theclinical picture even when there is no history ofingestion or exposure. Atropine will reverse themuscarinic effects of these agents and is a use-ful part of treatment. Edrophonium is a shortand rapidly acting cholinergic drug used diag-nostically to reverse the muscle weakness ofmyasthenia gravis. Myasthenia, however,would not explain the pinpoint pupils, saliva-tion, wheezing, bradycardia, or convulsions.(Rudolph, 373–374)

17. (E) The child described probably has congeni-tal adrenal hyperplasia (CAH), an inborn meta-bolic error of the adrenal cortex. The acidosis(HCO3

− 12 meq/L) helps to rule out pyloricstenosis as the cause of the emesis, as mostinfants with pyloric stenosis have a metabolicalkalosis. The enzyme deficiency in CAH resultsin decreased production of cortisol and otheradrenal cortical hormones and secondary hyper-trophy of the adrenal gland. Accumulation ofandrogen-like precursors of cortisol during fetaldevelopment leads to masculinization of thefemale fetus and ambiguous genitalia, which isan important clue in this case. The low serum

sodium and high serum potassium levels are clas-sic findings in this condition, reflecting the lack ofmineralocorticoids. In addition to the use ofsaline, administration of a mineralocorticoid suchas cortisone or hydrocortisone is critical. The ele-vated serum potassium level usually respondsrapidly to administration of saline and steroids,and specific therapy with exchange resins or glu-cose and insulin usually is unnecessary.

As did the preceding question, this ques-tion tests the examinee’s ability of recall ratherthan recognition, a more difficult but clinicallymore relevant skill. Instead of providing a list ofdiseases or syndromes as possible answers, itprovides a list of additional features or findings,one of which is associated with the disorder inquestion. In this case, as in the preceding ques-tion, the feature to be selected is the appropriatetherapy. The question tests more than the exam-inee’s ability to recite the treatment of hyper-kalemia. It tests his or her ability to analyze theclinical situation, make a correct diagnosis, setpriorities, and tailor therapy to the specificpathophysiology involved. (Rudolph, 2032–2041)

18. Again, this question requires recall rather thanrecognition. The stem of the question gives noinformation except the age and sex of the patientand a description of the skin lesions—tendererythematous nodules on the extensor surfacesof the legs. On the basis of these data you mustdecide what disease the patient most likely has.Which of the following best fits the skin lesionsdescribed: erythema nodosum, rheumatic nod-ules, subcutaneous fat necrosis, hematomas,septic emboli, or Henoch-Schöenlein purpura?

The correct answer is (C). To answer thisquestion you must not only identify the rashas erythema nodosum (an uncommon but notrare disease) but you must also know thatgroup A β-hemolytic streptococcal infection isa common cause. Erythema nodosum is areactive phenomenon characterized by tender,erythematous nodules 1–2 cm in diameter. Thelesions usually are on the extensor surfaces ofthe extremities and are more common on thelegs. This rash is seen in a variety of infectionsincluding histoplasmosis, tuberculosis, coccid-ioidomycosis, and group A streptococcalinfection. Today, the most common cause in

Page 10: Questions - uhs.edu.kh You suspect Lyme disease, most likely contracted by which of the following? (A) ingestion of unripe fruit (B) ingestion of spoiled fruit (C) drinking of contaminated

Answers: 17–22 11

an otherwise well child in the United States isgroup A streptococcal infection. (McMillan, 911;Rudolph, 1237)

19. The differences between adults and childrenare frequently emphasized in medical training.The differences between adults and adolescentsshould also be recognized. The differentialdiagnosis for many conditions, such as anintrathoracic mass seen in this case, is agedependent. While carcinoma of the lung is aleading cause of intrathoracic mass in adults, itis very rare in adolescents, even those whohave a significant smoking history. That elimi-nates choices (A), (B), and (C). While it is truethat Wilms tumor frequently metastasizes tothe lung, this malignancy almost always pres-ents in the first few years of age and would beunheard of in an 18-year-old. That leaves lym-phoma as the only remaining choice and themost likely diagnosis. (D) is the correct answer.Other causes, such as tuberculosis, histoplas-mosis, and sarcoid need to be considered butwere not listed as choices. (Rudolph, 1608)

20. Here again, you are required to make a diagno-sis but are not given a list of diseases from whichto choose. You should analyze the data, identifythe important features, and generate a list ofmost likely diagnoses. The major problemsappear to be fever, a localized rash, andmeningeal inflammation (headache and stiffneck). The malaise, fatigue, lethargy, generalizedlymphadenopathy, and arthralgia are less spe-cific. Of note is the fact that the child was bittenby a tick a week prior to the onset of the illness.If the tick bite is related to the illness, it wouldsuggest an infectious etiology. The systemic find-ings, the central clearing of the rash, and the timecourse permit us to rule out a simple cellulitis.Knowledge of the common infections carried byticks as well as the epidemiology of such infec-tions is essential to correctly answering thequestion. Rocky Mountain spotted fever, Lymedisease, tularemia, babesiosis, and Coloradotick fever are all spread by ticks, but only Lymedisease fits with the localized rash described—erythema migrans. This disease is caused by thespirochete B burgdorferi and over 90% of casesoriginate from 10 states in the US (Connecticut,

Delaware, Maryland, Massachusetts, Minnesota,New Jersey, New York, Pennsylvania, RhodeIsland, and Wisconsin). The organism is suscepti-ble to a number of antibiotics, including amoxi-cillin and doxycycline. The correct answer is (E).(McMillan, 1171–1173; Rudolph: Color Plate, 22:1212–1213)

21. This question also challenges you to identify adisease without providing a list of diagnosesfrom which to choose. What disease do youbelieve this child most likely has: juvenile idio-pathic arthritis, inflammatory bowel disease,cystic fibrosis, Henoch-Schönlein purpura(HSP), Salmonella infection?

The correct answer is (D). This child hasanaphylactoid purpura, also known as HSP.This is an important and not rare pediatric enti-ty, well known to pediatricians and pediatricresidents but not so well known by students.The question is difficult because the scenariogiven is infrequent although well recognized inthis disorder. The major features of this diseaseare colicky abdominal pain, nephritis, arthritis,and a characteristic purpuric rash limited to thearea below the waist. The purpuric rash listedas a possible answer does not specify locationor distribution, but is still the best answer. Ifyou missed this question, was it because youwere not familiar with HSP or because you didnot recognize it from this presentation? Theonly atypical feature in this case is that the childwas taken to the operating room. When abdom-inal pain is the first complaint, diagnosis is vir-tually impossible until other features appear.(Rudolph:Color Plate, 12:1212–1213)

22. What disorder do you believe this child probablyhas: psychomotor seizures, Tourette syndrome,drug abuse, brain tumor, or psychologic disorder?

The child most likely has Tourette syn-drome, a disorder characterized by blinking,twitching, grimacing, and jerking movementsthat often have a repetitive and semipurpose-ful character. Like simple habit tics, the move-ments usually can be voluntarily suppressedmomentarily, disappear during sleep, and aremade worse by emotional tension. These fea-tures could mislead the examinee to assume apsychologic etiology. Ultimately, the musclesof respiration and swallowing become

Page 11: Questions - uhs.edu.kh You suspect Lyme disease, most likely contracted by which of the following? (A) ingestion of unripe fruit (B) ingestion of spoiled fruit (C) drinking of contaminated

12 1: Warm-Up Questions and Exam-Taking Skills

involved so that throat clearing, coughing,snorting, hiccups, and other noises are com-mon. Coprolalia, echolalia, and spitting areclassic features but are not always present. Thecorrect answer is (D). Haloperidol is the drugof choice for Tourette syndrome, although notall patients require this therapy. Haloperidolrelieves symptoms in 80% of patients, cloni-dine has also been found to be helpful in somepatients. Corticosteroids are not appropriate inthis patient. Referral to a child psychiatristmay be a useful adjunct but has not beenshown to have consistent positive effects. AnEEG is helpful to diagnose epilepsy, but in thiscase, the child can stop the movements whenasked, so the movements are clearly notindicative of seizure activity. (Rudolph, 462)

23. This question deals with a rare but dramaticpediatric syndrome. What is the significance ofthe hemangioma on one side of the face? If youcan identify the disease, can you then antici-pate the findings on CT scan? Do you think thischild has congenital toxoplasmosis, holopros-encephaly, Sturge-Weber disease, subdural effu-sions, porencephalic cyst? Providing a list ofdiagnoses would have changed the questionfrom one of recall to one of recognition.

The association of a unilateral facial heman-gioma, particularly in the distribution of thetrigeminal nerve, and focal seizures suggestsSturge-Weber disease, also referred to as Sturge-Weber-Dmitri syndrome and encephalotrigemi-nal angiomatosis. Incidentally, national exami-nations often use eponyms for diseases andsyndromes even when other specific namesexist. Examples include Down syndrome ratherthan trisomy 21 and Werdnig-Hoffmann dis-ease for spinomuscular atrophy. Sturge-Weberdisease is characterized by a port-wine capillarynevus on the face (classically in the distributionof the first division of the trigeminal nerve),focal seizures on the contralateral side, andintracranial calcifications on the ipsilateral side.Therefore, the correct answer to Question 23 is(C), gyriform calcifications. The intracranialpathology is caused by hemangiomatouschanges of the meninges. This is a congenitaldisorder, probably of nongenetic basis. Theseizures often are very difficult to control. Other

common features include mental deficiencyand a contralateral hemiparesis. (Rudolph, 2347)

24. This question is exceedingly difficult, so yourfirst task is to establish a probable diagnosis. Ifyou were not able to deduce the diagnosis fromthe question, how about from the list of answers?

The correct answer is (A). The childdescribed has Hurler syndrome, a form ofmucopolysaccharidosis. This rare, autosomalrecessive disorder is characterized by growthretardation that generally starts after the firstyear of life. Classically, facial features becomecoarse and eventually appear gargoyle-like.Hepatosplenomegaly results from the accu-mulation of mucopolysaccharide and often isstriking. Bone and joint involvement withkyphosis and joint contractures are frequent.Corneal clouding results from the depositionof mucopolysaccharide in that organ. Theaccumulation of mucopolysaccharide withinthe brain leads to mental retardation.

If you were not familiar with Hurler syn-drome and did not know that it is characterizedby dermatan and heparan sulfate in the urine,you would not be able to answer the question.On the other hand, you might know that infor-mation and still not be able to answer the ques-tion if you could not successfully recall the dis-ease and match the features to the patient in thequestion. Exploring each potential answer andtrying to recall the conditions with which it isassociated could help. (Rudolph, 2329)

25. (D) In a matching question with six or fewerchoices, it is practical to read and briefly thinkabout each lettered choice before attemptingto answer the numbered questions.

The lesions of molluscum contagiosumare typically quite small, from pinhead size to

TABLE 1-1.

Urinary Finding Disease

Dermatan and heparan sulfate Hurler syndromeGalactose GalactosemiaMannose MannosidosisOdor of maple syrup Branched-chainaminoacidemiaOdor of sweaty feet Isovaleryl CoAdehydrogenase deficiency

Page 12: Questions - uhs.edu.kh You suspect Lyme disease, most likely contracted by which of the following? (A) ingestion of unripe fruit (B) ingestion of spoiled fruit (C) drinking of contaminated

Answers: 23–30 13

5 or 10 mm in diameter. Larger lesions do occurbut are infrequent. The lesions usually have aneasily recognized appearance: round, dome-shaped papules with a translucent top and awaxy, whitish material inside. Umbilication iscommon, especially of larger lesions. The con-dition is caused by a DNA pox virus and isspread by direct contact with an infected indi-vidual. Lesions may occur anywhere but aremost common on the arms and trunk.(McMillan, 831; Rudolph, 1056, 1220)

26. (C) Condyloma acuminatum are soft, fleshy,papular, or pedunculated lesions occurringaround the genitalia and/or rectum. Althoughthese lesions are caused by the human papillo-mavirus and are sexually transmitted in ado-lescents and older children, it is now believedthat most cases in infants and very young chil-dren are not sexually acquired but are ratheracquired during passage through the birthcanal. (Rudolph, 267)

27. (E) The typical lesion of pityriasis rosea is anovoid, pink papule or plaque with fine scales.Lesions typically follow tension lines on theskin, giving the appearance of the branches of apine tree or a Christmas tree on the patient’sback. A single lesion appearing a week or twobefore other lesions is a common occurrence andis referred to as a herald patch. (Rudolph, 1181)

28. In this type of matching question, up to 26options are presented for each item. The choicesrepresent a long differential diagnosis for theinfant with hyperbilirubinemia. When the list ofoptions is long (more than six), it becomes inef-ficient and time consuming to evaluate eachpossible lettered choice for each numbered item.However, it is helpful to scan or preview the listof options. Then, for each numbered item,decide what the best answer would be and lookfor it in the list of possible choices.

It is clear that the 3-day-old term infant inthis question has unconjugated hyperbiliru-binemia but is otherwise well and has no evi-dence of hemolysis. The most likely cause ofthese findings would be physiologic jaundice,a generally benign condition of neonates asso-ciated with hepatic immaturity and a peak

bilirubin level of less than 13 mg/dL on day oflife 3 or 4 for a term infant and 15 mg/dL orless on day 5–7 for a preterm infant. Sincephysiologic jaundice is one of the options list-ed (Q), the examinee need look no further.However, if time permits, scanning the list forother potential answers would be a wise safe-ty measure. Although it is true that some ofthe other conditions listed, such as breast milkjaundice or Crigler-Najjar syndrome, couldcause similar findings, we are not told that theinfant is being breast-fed, and Crigler-Najjarsyndrome is exceedingly rare. Physiologicjaundice is clearly the most likely cause andtherefore the best choice. (McMillan, 199–200)

29. (C) This 5-week-old infant has persistent mixedhyperbilirubinemia, suggesting a hepatic disor-der. The normal liver enzymes indicate anobstructive rather than an inflammatory condi-tion. Finally, the inability to visualize a gallblad-der on ultrasound examination makes biliaryatresia the only plausible diagnosis. Prolongedjaundice in the otherwise healthy neonate cansignal a potentially lethal hepatic disorder and inthe case of biliary atresia, it is essential to under-stand that timely diagnosis is key in improvingthe prognosis for affected infants. Interventionwith hepatic portoenterostomy which can pre-vent the progression to liver failure is associatedwith poorer outcomes in infants whose diagno-sis is made beyond 2 months of age. (Rudolph,1506–1507; McMillan, 199–200)

30. (H) This infant has had severe, persistentunconjugated hyperbilirubinemia for 4 weeksbut is otherwise well. The normal serum levelsof conjugated bilirubin and hepatic enzymesrule out most forms of liver disease (obstructiveor inflammatory), and there is no evidence ofhemolysis. The Coombs test was negative, andthe hyperbilirubinemia is too severe and pro-longed for either a blood group incompatibilityor breast milk jaundice. Such a course forneonatal jaundice is very rare, and thereforeone must consider rare causes. Crigler-Najjarsyndrome, a congenital deficiency of hepaticenzymes involved in conjugation of bilirubin,is the only disorder that could explain thispatient’s findings. (Rudolph, 166, 1489)

Page 13: Questions - uhs.edu.kh You suspect Lyme disease, most likely contracted by which of the following? (A) ingestion of unripe fruit (B) ingestion of spoiled fruit (C) drinking of contaminated

14 1: Warm-Up Questions and Exam-Taking Skills

AAP. Circumcision policy statement. Pediatrics 1999;103:686–693.

Craig JC, Knight JF, et al. Effect of circumcision of urinarytract infection in preschool boys. J Pediatr. 1996;128:23–27.

SELECTED READINGS

Page 14: Questions - uhs.edu.kh You suspect Lyme disease, most likely contracted by which of the following? (A) ingestion of unripe fruit (B) ingestion of spoiled fruit (C) drinking of contaminated

CHAPTER 2

General PediatricsCatalina Kersten, MD

15

This chapter is designed to include topics not covered in other chapters, and to reinforce importantconcepts that may be discussed elsewhere in the book.

Page 15: Questions - uhs.edu.kh You suspect Lyme disease, most likely contracted by which of the following? (A) ingestion of unripe fruit (B) ingestion of spoiled fruit (C) drinking of contaminated

16

DIRECTIONS (Questions 1 through 88): Each ofthe numbered items or incomplete statements inthis section is followed by answers or by comple-tions of the statement. Select the one letteredanswer or completion that is best in each case.

1. A 6-year-old girl presents with unilateral non-painful, nonsuppurative conjunctivitis andpreauricular lymphadenitis. What is the mostlikely causative organism?

(A) Mycobacterium avium(B) Bartonella henselae(C) Adenovirus(D) Staphylococcus aureus(E) Chlamydia trachomatis

2. You suspect the diagnosis of Werdnig-Hoffmandisease in an infant with severe hypotonia.Which other finding will support this diagnosis?

(A) normal deep tendon reflexes(B) seizures(C) fasciculations of the tongue(D) recurrent fevers(E) atrophy of the optic nerve

3. A 16-month-old girl presents with acute onset oftruncal ataxia with vomiting, nystagmus, anddysarthria. She is afebrile and has no nuchalrigidity. Which of the following historical itemswould help you to identify a cause for thesesymptoms?

(A) elevated lead level at the age of one year(B) febrile seizure episode at the age of

13 months

(C) sore throat with blisters on palate 3 weeksago

(D) febrile illness with rash 2 months ago(E) first MMR vaccination 1 month ago

4. You follow a patient with craniosynostosis andcongenital malformations of the head and face inyour clinic. What other congenital malforma-tions will you most likely find in this patient?

(A) trachea and esophagus malformations(B) heart malformations(C) genitourinary tract malformations(D) extremity malformations(E) spine malformations

5. A 12-year-old boy has migratory arthritis withred, warm, and swollen joints. He has serologicevidence of recent group A streptococcal infec-tion. Arthritis in this condition is characterizedby which of the following?

(A) usually nonpainful(B) heals without deformity(C) appears after the fever subsides(D) seen only in patients with concurrent

carditis(E) involves large and small joints equally

6. A 2-month-old infant has severe dyspnea andcyanosis. Chest roentgenogram reveals minimalcardiomegaly and a diffuse reticular pattern ofthe lung fields. Which of the following bestexplains these findings?

(A) acute viral myocarditis(B) hypoplastic left heart syndrome

Questions

Page 16: Questions - uhs.edu.kh You suspect Lyme disease, most likely contracted by which of the following? (A) ingestion of unripe fruit (B) ingestion of spoiled fruit (C) drinking of contaminated

Questions: 1–13 17

(C) pulmonary artery atresia(D) total anomalous pulmonary drainage

with venous obstruction(E) transposition of the great arteries

7. You suspect the diagnosis of a brain tumor in a2-year-old girl with a recent history of ataxia,slurred speech, and early morning vomiting.Which statement about childhood brain tumorsis true?

(A) Most are located in the midline and/orbelow the tentorium cerebri.

(B) Brain tumors are a rare type of cancer inchildhood.

(C) Signs of increased intracranial pressureare rare on presentation.

(D) Seizures are the presenting complaint inmost cases.

(E) Most cases occur in the first year of life.

8. An 8-month-old child has vomiting and scream-ing episodes for 12 hours. Physical examinationreveals a sausage-shaped mass in the right upperquadrant. Which of the following would be mostuseful?

(A) passage of nasogastric tube(B) examination of a stool specimen for ova

and parasites(C) blood culture(D) abdominal ultrasound(E) barium enema study

9. A 4-year-old boy has failed to grow and hasevidence of exocrine pancreatic insufficiency.What is the most likely cause for this?

(A) acute pancreatitis(B) biliary atresia(C) Swachman-Diamond syndrome(D) congenital absence of the pancreas(E) cystic fibrosis

10. A 5-year-old girl presents with fever andheadache. Imaging of the brain reveals a ring-enhancing lesion. Which of the following isthe most likely underlying condition in thischild?

(A) chronic renal failure(B) idiopathic or familial epilepsy(C) congenital cyanotic heart disease(D) chronic or recurrent tonsillitis(E) Langerhans cell histiocytosis

11. A 5-year-old girl diagnosed with pauciarticularjuvenile idiopathic arthritis has a positive anti-nuclear antibody test. Which of the followingwould most likely be found in this patient?

(A) pericarditis(B) nephritis(C) uveitis(D) splenomegaly(E) lymphadenopathy

12. A 2-year-old African-American child presentswith painful swelling of the hands and feet.Laboratory evaluation reveals hemoglobin of9 g/dL with white blood cell count of 11,500and platelet count of 250,000. Which additionallaboratory test will support your diagnosis?

(A) skeletal survey(B) VDRL testing(C) bone marrow aspiration(D) hemoglobin electrophoresis(E) serum calcium measurement

13. A 2-week-old infant presents with apnea. Theinfant was born at term after an uncomplicatedpregnancy. The mother of this baby had rhin-orrhea and cough that started 3 weeks ago andnow she has a severe persistent cough withpost-tussive emesis. Which treatment shouldbe initiated?

(A) ceftriaxone(B) amoxicillin(C) azithromycin(D) vancomycin(E) amantidine

Page 17: Questions - uhs.edu.kh You suspect Lyme disease, most likely contracted by which of the following? (A) ingestion of unripe fruit (B) ingestion of spoiled fruit (C) drinking of contaminated

18 2: General Pediatrics

14. An 18-month-old toddler has microcytic anemia.Which dietary history finding best explainsthis?

(A) pica(B) lack of fresh vegetables in the diet(C) intake of inadequate amounts of fruit

juice(D) intake of excessive amounts of vitamin C(E) intake of large amounts of unmodified

cow’s milk

15. A 2-year-old toddler has a large abdominalmass and pancytopenia. Which of the follow-ing diagnoses would most likely be establishedby bone marrow aspiration?

(A) hepatoblastoma(B) neuroblastoma(C) renal cell carcinoma(D) rhabdomyosarcoma(E) Wilms tumor

16. A 14-month-old boy has a 4-month history ofintermittent diarrhea. He frequently has explo-sive bowel movements containing food parti-cles. He is growing well, is otherwise healthy,and has a normal physical examination. Whatshould be the next step?

(A) reassurance of parents(B) stool culture(C) total serum qualitative immunoglobulin

measurement(D) qualitative fecal fat(E) prescribe oral antidiarrheal agent

17. A child with polyosteotic fibrous dysplasia ofthe bones and abnormal skin pigmentation isdiagnosed with McCune-Albright syndrome.What other problem is this patient most likelyto develop?

(A) anemia(B) deafness(C) precocious puberty(D) multiple neurofibromas(E) chronic glomerulonephritis

18. A normal 6-month-old infant has a continuouscardiac murmur and bounding peripheralpulses. What step should be taken next?

(A) karyotype evaluation(B) surgical or catheter correction of the

defect(C) life-long endocarditis prophylaxis for at-

risk procedures(D) repeating examination at the age of

12 months(E) reassuring of the parents

19. A newborn has delayed passage of meconiumstools and barium enema radiograph showsdilated proximal colon and small obstructeddistal colon. What should be the next diagnostictest?

(A) abdominal CT-scan(B) stool studies(C) rectal suction biopsies(D) sweat chloride testing(E) chromosome analysis

20. An 8-year-old boy is referred for new-onsetseizures. Which of the following would mostlysupport a diagnosis of complex partial (psy-chomotor) seizures?

(A) normal mental state, consciousness, andresponsiveness during seizure

(B) a brief tonic-clonic phase(C) automatisms(D) three-per-second spike-and-wave pat-

tern on EEG(E) normal mental state, consciousness, and

responsiveness after seizure

21. A 14-year-old female has progressive headaches.Examination shows bilateral papillary edema.CT-scan of the brain is normal. What should bethe next diagnostic test?

(A) lumbar puncture with opening pressure(B) MRI of the brain(C) orbital CT-scan(D) urine toxicology screen(E) serum beta HCG measurement

Page 18: Questions - uhs.edu.kh You suspect Lyme disease, most likely contracted by which of the following? (A) ingestion of unripe fruit (B) ingestion of spoiled fruit (C) drinking of contaminated

Questions: 14–29 19

22. An infant is born to a mother who is HBsAgpositive. What should be the next step?

(A) Check hepatitis B serology on infant andgive hepatits B immune globulin ifindicated.

(B) Give infant hepatitis B immuneglobulin.

(C) Vaccinate infant with hepatitis B vaccine.(D) Give infant hepatitis B immune globulin

and hepatitis B vaccine.(E) Start infant on formula and discourage

breastfeeding.

23. A 38-week infant is born to a mother with ges-tational diabetes. Birth weight is 4255 g. Whatwould you expect to see most commonly inthis infant?

(A) neural tube defect(B) small left colon syndrome(C) cardiomegaly(D) hydronephrosis(E) renal dysplasia

24. A newborn has been diagnosed with aniridia.Which of the following tests should be per-formed on this patient?

(A) chest radiograph(B) alpha-fetoprotein measurement(C) renal function testing(D) testicular examination(E) renal ultrasound

25. A 12-year-old girl develops jaundice, progres-sive tremors, and emotional lability. You aremost likely to find which of the followingduring physical examination?

(A) head circumference greater than 95thpercentile

(B) brown discoloration of the limbic regionof the cornea

(C) bilateral conductive hearing loss(D) generalized lymphadenopathy(E) sacral hair tuft and dimple

26. A 12-month-old infant is unable to sit by herselfand parents have noticed an exaggerated star-tle response. What are you most likely to findon physical examination?

(A) holosystolic murmur(B) absent knee-jerk reflex(C) syndactyly(D) cherry red macular spot(E) bilateral inguinal hernias

27. An infant has been diagnosed with congenitalhypoparathyroidism. What are you most likelyto find on evaluation?

(A) microcephaly(B) hyponatremia(C) hyperkalemia(D) goiter(E) candidiasis

28. A 14-month-old boy has severe eczema, recur-rent sinus and ear infections, and thrombocy-topenia. What is the inheritance pattern of thisdisorder?

(A) X-linked(B) autosomal dominant(C) autosomal recessive(D) random mutation(E) multifactorial

29. A 12-year-old girl has had progressive muscleweakness over the past weeks. She has also devel-oped an erythematous, scaly rash on the face,arms and thighs, and a lacy rash on her uppereyelids. What is the next best laboratory study?

(A) rheumatoid factor(B) erythrocyte sedimentation rate (ESR)(C) urine analysis(D) serum creatinine kinase(E) antinuclear antibody (ANA) panel

Page 19: Questions - uhs.edu.kh You suspect Lyme disease, most likely contracted by which of the following? (A) ingestion of unripe fruit (B) ingestion of spoiled fruit (C) drinking of contaminated

20 2: General Pediatrics

30. A 12-month-old girl has been diagnosed withtransient erythroblastopenia of childhood(TEC). Which statement about this disorder iscorrect?

(A) Corticosteroid treatment is usuallybeneficial.

(B) Red blood cell transfusions may benecessary.

(C) Hepatosplenomegaly is usually present.(D) Spontaneous recovery is uncommon.(E) Parvovirus infection has been associated

with this disease.

31. A 2-year-old girl is listless and pale. You obtaina complete blood count and find that thepatient has severe megaloblastic anemia. Whatadditional history explains this?

(A) eats only organically grown products(B) drinks exclusively goat milk(C) has required phototherapy in neonatal

period(D) has required multiple antibiotics for

middle ear infections(E) is an infant of a diabetic mother

32. A 12-month-old child has had poor weight gain.The child started to have loose stools at the ageof 8 months and has a very poor appetite. Onexamination, you see a clingy, irritable childwith very little subcutaneous fat and a protu-berant abdomen. What is the next best test?

(A) IgA-endomysial antibody(B) urine analysis(C) sweat chloride(D) quantitative immunoglobulins(E) fecal blood

33. A 7-year-old girl develops secondary nocturnalenuresis. What is the next best study?

(A) renal ultrasound(B) voiding cystourethrogram(C) abdominal radiograph(D) urine analysis(E) creatinine clearance

34. A 5-year-old girl suffers from a second episodewith meningococcal meningitis. What is thebest next laboratory study?

(A) quantitative immunoglobulin levels(B) T-cell subset analysis(C) CH50

(D) quantitative nitroblue tetrazolium test(E) delayed hypersensitivity skin testing

35. A 2-year-old child is referred to you for evalu-ation of child abuse. On physical examination,you find a pale child with diffuse petechiaeand bilateral proptosis with periorbital ecchy-moses. Which of the following statements istrue about this condition?

(A) Age at presentation correlates directlywith survival.

(B) A full skeletal survey should beobtained next.

(C) Hematuria is a common finding.(D) It usually presents between 4 and 8 years

of age.(E) Spontaneous regression has occurred in

some children.

36. Which study is the most important to obtain ina 2-year-old child with Beckwith-Wiedemannsyndrome and an abdominal mass?

(A) hepatobiliary scintigraphy(B) upper gastrointestinal endoscopy(C) urine catecholamine levels(D) serum alpha-fetoprotein level(E) voiding cystourethrogram

37. A 9-year-old African-American child presentswith anemia and stroke. What is the most likelyfinding with hemoglobin electrophoresis?

(A) HbS 45%(B) HbA 65%(C) HbA2 15%(D) HbF 15%(E) HbC 45%

Page 20: Questions - uhs.edu.kh You suspect Lyme disease, most likely contracted by which of the following? (A) ingestion of unripe fruit (B) ingestion of spoiled fruit (C) drinking of contaminated

Questions: 30–45 21

38. A 2-year-old boy from Sudan has failure tothrive, chronic diarrhea, and severe candidiasis.You suspect HIV infection. Which of the fol-lowing organisms would most likely be foundon stool examination?

(A) rotavirus(B) Salmonella(C) Cryptosporidium species(D) Giardia(E) Yersinia enterocolitica

39. A 5-year-old boy has severe pharyngitis andculture is positive for group A streptococci. Ofthe following suppurative and nonsuppura-tive complications of group A streptococcalpharyngitis and skin infections, which is asso-ciated only with pharyngeal infections?

(A) scarlatina(B) toxic shock syndrome(C) rheumatic fever(D) necrotizing fasciitis(E) glomerulonephritis

40. A 7-year-old boy has abdominal pain and a rashthat started several days ago. On examination,you notice a palpable purpuric rash over hiscalves and buttocks with swelling of bothankles. Abdominal examination is unremark-able. What is the most likely laboratory finding?

(A) decreased platelet count(B) hypochromic microcytic anemia(C) elevated blood urea nitrogen and

creatinine(D) low C3 complement levels(E) normal clotting parameters

41. A mother with mild mental retardation has a10-year-old son with severe mental retardation.The boy is tall, has a long face with prominentjaw and large ears. Which statement about hiscondition is true?

(A) Premutation carriers generally havephenotypic manifestations.

(B) Inheritance is autosomal dominant.(C) It is the most common form of inherited

mental retardation.

(D) Affected males typically havemicroorchidism.

(E) A single gene point mutation is thecause of this syndrome.

42. A 4-month-old female infant has generalizedhypotonia, small ears, inner epicanthal folds,clinodactyly, and wide-spaced first and secondtoes. For which of the following problems isshe most at risk?

(A) leukemia(B) patent ductus arteriosus(C) seizure disorder(D) hearing loss(E) gastrointestinal tract anomalies

43. What is the most important test to obtain adiagnosis in a 14-year-old girl with primaryamenorrhea, and short stature, who has a his-tory of repaired coarctation of the aorta ininfancy?

(A) sweat chloride testing(B) karyotyping(C) fluorescent in situ hybridization (FISH)

of chromosome 22q11(D) pelvic ultrasonography(E) lymphocyte subset analysis

44. What are the blood requirements for transfusionof a patient with hypocalcemia, heart defect,and recurrent infections?

(A) leukocyte depleted(B) HLA matched(C) CMV negative(D) O negative(E) irradiated

45. What hematologic abnormality should yoususpect in a newborn with bilateral absence ofradii?

(A) thrombocytopenia(B) anemia(C) neutropenia(D) pancytopenia(E) lymphopenia

Page 21: Questions - uhs.edu.kh You suspect Lyme disease, most likely contracted by which of the following? (A) ingestion of unripe fruit (B) ingestion of spoiled fruit (C) drinking of contaminated

22 2: General Pediatrics

46. What laboratory abnormality do you expectto find in a 3-year-old child with severemental retardation, coarse facies, hazy corneas,hepatosplenomegaly, and multiple skeletalx-ray abnormalities?

(A) increased serum homocystine(B) deficiency of leucocyte hexosaminidase A(C) urinary excretion of dermatan sulfate

and heparan sulfate(D) deficiency of liver glucose-6-phosphatase

activity(E) increased serum uric acid

47. A 3-month-old infant is brought to the hospitalbecause of altered mental status changes.Examination shows a sleepy baby who is difficultto arouse. Fundoscopic examination shows reti-nal hemorrhages. Examination otherwise is unre-markable. What is the best next diagnostic test?

(A) spinal tap(B) hematology profile with smear review(C) CT-scan of the head(D) skull radiographs(E) EEG

48. A previously healthy 5-year-old girl has acuteonset of edema and oliguria. Laboratory studiesreveal hypoalbuminemia and hypercholes-terolemia. Which of the following is the majorcause for mortality with this condition?

(A) bacterial peritonitis(B) acute renal failure(C) hyperlipidemia(D) congestive heart failure(E) hypertension

49. A previously healthy 9-year-old boy has haddiarrhea for 6 weeks that started after hereturned from camp. He has had anorexia,abdominal cramps with abdominal disten-sion, and a 4 pound weight loss. His stoolsare large, foul-smelling but do not containblood. What is the best treatment?

(A) avoidance of lactose in diet(B) gluten-free diet(C) oral prednisone

(D) trimethroprim-sulfamethoxazole(E) metronidazole

50. A 16-year-old adolescent has morbid obesity.Which of the following conditions is the mostcommon cause for pulmonary insufficiency inobese adolescents?

(A) pneumothorax(B) gastric esophageal reflux disease(C) congestive heart failure(D) asthma(E) sleep apnea

51. The parents of an 8-year-old boy are concernedabout their son’s short stature. What should bethe most important next step?

(A) measurement of body mass index(B) determination of genital maturation stage(C) bone age measurement(D) determination of height velocity(E) determination of weight/height ratio

52. A7-year-old boy has chronic fecal soiling but onlyrarely has a voluntary bowel movement. What isthe most common explanation for his problem?

(A) Hirschsprung disease(B) functional fecal retention(C) hypothyroidism(D) lead poisoning(E) iron therapy

53. A 6-month-old boy is found to have very lowlevels of IgG, IgM, and IgA. Which of the fol-lowing organisms is most likely to cause prob-lems in this patient?

(A) enterovirus(B) herpesvirus(C) Shigella(D) Escherichia coli(E) Mycobacterium tuberculosis

54. An infant is born to a mother who acquiredprimary CMV infection during pregnancy.What will be the most likely finding in thisinfant?

Page 22: Questions - uhs.edu.kh You suspect Lyme disease, most likely contracted by which of the following? (A) ingestion of unripe fruit (B) ingestion of spoiled fruit (C) drinking of contaminated

Questions: 46–61 23

(A) hepatosplenomegaly and jaundice(B) subclinical infection(C) microcephaly and intrauterine growth

retardation(D) sensorineural hearing loss(E) thrombocytopenia and purpura

55. A healthy adolescent is found to have elevatedblood pressure on several occasions. Whichstatement is correct?

(A) Obesity is rarely associated with hyper-tension among adolescents.

(B) Essential hypertension is the most com-mon cause of hypertension amongadolescents.

(C) The incidence in Caucasian adolescentsis twice that of African-Americanadolescents.

(D) Most pediatric patients with hyperten-sion are symptomatic.

(E) An adolescent with hypertension shouldnot participate in sports.

56. A 14-year-old girl has irregular menstrualbleeding since menarche 1 year ago. What isthe most common cause for this?

(A) immature hypothalamic-pituitary-ovarian axis

(B) polycystic ovarian syndrome(C) blood dyscrasia(D) systemic illness(E) sexually transmitted disease

57. A 3-year-old girl has a mild febrile illness withmild URI symptoms. She has an erythematousrash on both cheeks. Her pregnant mother hadarthralgias of the hands wrists, knees, andankles a week ago. What should be the nextaction?

(A) Closely monitor the child’s sibling whohas spherocytosis.

(B) Exclude the child from daycare untilrash has resolved.

(C) Exclude the pregnant daycare providersuntil no further cases are diagnosed for2 weeks.

(D) Give the mother IVIG.(E) Give the sibling with spherocytosis

IVIG.

58. A 14-year-old boy has an acutely painful andswollen scrotum. What should be the next step?

(A) fine needle aspiration(B) bone marrow aspiration(C) surgical exploration(D) oral antibiotics(E) bed rest and analgesia

59. A 16-year-old boy presents with fever, fatique,and sore throat. Examination reveals exuda-tive pharyngitis, generalized lymphadenopa-thy, and mild splenomegaly. Laboratory studiesshow elevated WBC count with presence ofatypical lymphocytes. What is the best action?

(A) no participation in contact sports fornext 2–4 weeks

(B) 2-week treatment with oral prednisone(C) 2-week treatment with oral acyclovir

and prednisone(D) 10 days of oral penicillin(E) strict bed rest

60. A 4-year-old girl had bloody diarrhea for sev-eral days. One week later she develops perior-bital edema and fatique. What is the most likelylaboratory finding?

(A) elevated PT and PTT(B) positive ANA(C) decreased C3 and C4(D) hypoalbuminemia(E) anemia and thrombocytopenia

61. An infant with failure to thrive has rectal pro-lapse. What test will most likely provide thediagnosis?

(A) abdominal CT-scan(B) rectal biopsies(C) liver function testing(D) barium enema study(E) sweat chloride test

Page 23: Questions - uhs.edu.kh You suspect Lyme disease, most likely contracted by which of the following? (A) ingestion of unripe fruit (B) ingestion of spoiled fruit (C) drinking of contaminated

24 2: General Pediatrics

62. A 16-month-old toddler has painless rectalbleeding. His stools have currant jelly consis-tency. Physical examination of the patient,including rectal examination, is completelynormal. A routine barium study is normal.Which of the following is true for this condition?

(A) Plain abdominal radiographs are usu-ally diagnostic.

(B) It is the most common congenital gas-trointestinal anomaly.

(C) The abnormality typically is locatedwithin 1 cm of the ileocecal valve.

(D) Most common presentation is partial orcomplete bowel obstruction.

(E) It usually becomes clinically apparentafter 2 years of age.

63. A 5-day-old infant boy is jaundiced. The totalbilirubin level is 14 mg/dL and the directbilirubin is 4 mg/dL. Which of the followingtests is the most appropriate?

(A) blood type and direct antibody test onthe infant’s blood

(B) blood type and direct antibody test onthe mother’s blood

(C) urine analysis and culture(D) hepatitis serology(E) examination of infant’s blood smear

64. An 18-year-old boy presents with acute severechest pain. EKG and enzyme studies confirman acute myocardial infarction. Cardiac cather-ization reveals a coronary artery aneurysmwith thrombosis. Which constellation of symp-toms in his past could explain this finding?

(A) conjunctivitis, fever, cervicallymphadenopathy

(B) meningitis, conjunctivitis, pallor(C) cervical lymphadenopathy, hepatitis,

rash(D) fever, irritability, pancreatitis(E) hepatosplenomegaly, rash, conjunctivitis

65. A 2-year-old girl has persistent seborrheic der-matitis in the diaper area. In addition she haschronically draining infected ears. In which

location do bony lesions most often occur inpatients with this disorder?

(A) ribs(B) femur(C) sternum(D) skull(E) humerus

66. A 3-year-boy with severe hypotonia and mildmental retardation is severely obese. He isobsessed with eating and does not have a senseof satiation. What abnormality will you mostlikely find?

(A) macrocephaly(B) height greater than 95%(C) large hands and feet(D) thyromegaly(E) micropenis and cryptorchidism

67. A 3-year-old girl develops petechiae andbruises on her extremities while she is recov-ering from a cold. She is brought to medicalattention after she has a transient nosebleed.Physical examination shows a toddler withwidespread petechiae and bruising who oth-erwise looks healthy. What is the best treat-ment for this patient?

(A) plasmapheresis(B) intravenous gammaglobulin(C) vincristine and methotrexate(D) intravenous antibiotics(E) platelet transfusion

68. A 3-month-old infant has persistent stridor.What is the most likely cause?

(A) vascular ring(B) laryngomalacia(C) tracheomalacia(D) laryngeal cleft(E) subglottic stenosis

69. A newborn girl with ambiguous genitalia hassevere vomiting with weight loss. What willbe the most likely finding?

Page 24: Questions - uhs.edu.kh You suspect Lyme disease, most likely contracted by which of the following? (A) ingestion of unripe fruit (B) ingestion of spoiled fruit (C) drinking of contaminated

Questions: 62–77 25

(A) decreased urinary excretion of17-ketosteroids

(B) decreased plasma dehydroepiandros-terone sulfate level

(C) increased plasma cortisol(D) hyponatremia and hyperkalemia(E) decreased serum ACTH

70. A 9-year-old has hematuria and an increasedserologic titer to antistreptolysin O (ASO).What will be the most likely finding?

(A) decreased serum C3(B) IgA deposits in kidney biopsy(C) decreased serum albumin(D) decreased urinary protein/creatine ratio(E) hypercalciuria

71. A 6-year-old girl with short stature has webbingof the neck, a low posterior hairline, a broad chest,and cubitus valgus. Which organ is affected mostfrequently in patients with this syndrome?

(A) heart(B) kidneys(C) ovaries(D) thyroid(E) intestines

72. A 15-year-old female has a 1 year history ofsecondary amenorrhea. She is an avid gymnastand has an intese fear of becoming fat. Herweight is at 80% of ideal body weight. For whatlong-term irreversible complication is thispatient at greatest risk?

(A) cardiac arrythmias(B) hypothyroidism(C) visual impairment(D) infertility(E) osteoporosis

73. A newborn boy was diagnosed prenatally withbilateral hydronephrosis, distended bladder,and oligohydramnios. What will be the mostlikely diagnosis?

(A) urethral strictures(B) anterior urethral valves

(C) prune-belly syndrome(D) posterior urethral valves(E) meatal stenosis

74. A healthy 2-day-old infant has multiple, firm,yellow-white papules on an erythematous basethat are widely dispersed over much of theskin. What will be the most likely microscopicfinding?

(A) multinucleated giant cells(B) eosinophilic infiltrate(C) cytoplasmic inclusion bodies(D) gram-positive cocci in clusters(E) IgA deposits

75. A 13-month-old toddler has a tibia fractureafter an insignificant fall. Other family mem-bers have blue sclerae and recurrent fracturesin childhood. For what other problem is thistoddler at increased risk?

(A) presenile hearing loss(B) mental retardation(C) seizures(D) recurrent pneumonia(E) hydrocephalus

76. A 4-week-old male infant has vomiting and ahypochloremic metabolic alkalosis. What is thenext best study?

(A) urine organic acids(B) urine 17-hydroxy progesterone(C) stool culture(D) abdominal ultrasound(E) head ultrasound

77. A newborn boy has deficiency of the abdomi-nal muscles and urinary tract abnormalities.What other anomaly will you most likely find?

(A) imperforate anus(B) undescended testes(C) mental retardation(D) congenital heart disease(E) congenital aganglionic megacolon

Page 25: Questions - uhs.edu.kh You suspect Lyme disease, most likely contracted by which of the following? (A) ingestion of unripe fruit (B) ingestion of spoiled fruit (C) drinking of contaminated

26 2: General Pediatrics

78. A 15-month-old boy has strabismus and awhite pupillary reflex. Ophthalmologic exam-ination reveals a white retinal mass. Thispatient is at increased risk for development ofwhich other tumor?

(A) leukemia(B) lymphoma(C) osteosarcoma(D) Ewing sarcoma(E) rhabdomyosarcoma

79. A previously healthy 3-month-old infant devel-ops generalized weakness with difficulty insucking, swallowing, and crying, and laboredbreathing. No fever is present. Which studywill most likely provide the diagnosis?

(A) stool culture(B) blood culture(C) head CT-scan(D) nerve conduction velocity testing(E) cerebrospinal fluid analysis

80. A 3-year-old boy presents with acute right legpain and a limp. There is no history of trauma.He holds his right hip in external rotation andflexion and he has mild restriction of range ofmotion. He appears otherwise well and isafebrile. His WBC is normal and ESR is 25 mm/h.What is the best treatment option at this time?

(A) intravenous antibiotics(B) surgical drainage of the right hip joint(C) anti-inflammatory drugs and bed rest(D) oral antibiotics(E) intra-articular corticosteroids

81. You have followed a 7-month-old infant whohas failed to gain weight. Birth weight was3250 g; the child currently weighs 5.5 kg. Inyour office, the baby takes an 8-oz bottle withease and does not vomit. What is the next beststep?

(A) placement of nasogastric feeding tube(B) hospitalization of the child with unlim-

ited feedings(C) contact child protective services for

placement in foster care

(D) a barium swallowing study(E) scheduled return visit in 1 month

82. A patient with streptococcal pharyngitis devel-ops tender red bumps along her entire tibia.What is the most likely diagnosis?

(A) sarcoidosis(B) cellulitis(C) thrombophlebitis(D) insect bites(E) erythema nodosum

83. A2-year-old child develops apnea, cyanosis, andloss of consciousness with repeated generalizedclonic jerks after being scolded by his mother. Onexamination, the child appears completely normal.What is the best treatment option?

(A) tegretol(B) valproic acid(C) antiarrhythmics(D) cardiac pacemaker(E) counseling of parents

84. A 16-year-old high school soccer player com-plains of chronic knee pain that has not beenassociated with an injury. The pain is worseupon going upstairs and after sitting for pro-longed periods. The only abnormal finding onexamination is peripatellar tenderness. What isthe best next action?

(A) arthroscopy(B) thigh strengthening exercise(C) knee brace(D) immobilization with cast(E) anti-inflammatory drugs

85. You evaluate an 8-year-old girl with hyperactiv-ity and inattentiveness. Which of the followingmanifestations is required to make a diagnosis ofattention-deficit hyperactivity disorder?

(A) occurrence before the age of 10 years(B) concurrent learning disability(C) impulsivity(D) history of birth trauma(E) a sibling with the diagnosis of ADHD

Page 26: Questions - uhs.edu.kh You suspect Lyme disease, most likely contracted by which of the following? (A) ingestion of unripe fruit (B) ingestion of spoiled fruit (C) drinking of contaminated

Questions: 78–99 27

86. You counsel the new parents of a baby boy withhypospadias about circumcision. Which informa-tion will you most likely share with the parents?

(A) There is clearly an increased risk forpenile cancer in uncircumcised males.

(B) Urinary tract infections are 10–15 timesmore common in uncircumcised infants.

(C) Circumcision reduces the risk of sexuallytransmitted diseases.

(D) Complications following circumcisionare very rare.

(E) Circumcision can be safely done in infantswith hypospadias.

87. A14-year-old boy has had several measurementsof blood pressure. His systolic blood pressurehas been above 99th percentile for age and dias-tolic blood pressure has ranged between 90th to94th percentile. What should you advise thisyoung man?

(A) Complete restriction of exercise is necessary.(B) Patient can participate in competitive

sports if there are no signs of targetorgan damage.

(C) Participation in competitive sports needto be restricted until hypertension isunder adequate control.

(D) Complete restriction of exercise withexception of isometric activities isnecessary.

(E) Full participation in all sports withoutrestrictions.

DIRECTIONS (Questions 88 through 112): Eachset of matching questions in this section consistsof a list of five to eight lettered options followedby several numbered items. For each numbereditem select the one lettered option with which it ismost closely associated. Each lettered option maybe selected once, more than once, or not at all.

Questions 88 through 93

(A) Ventricular septal defect(B) Atrial septal defect(C) Bicuspid aortic valves(D) Coronary aneurysm

(E) Third-degree heart block(F) Aortic aneurysm(G) Supravalvular aortic stenosis(H) Pulmonary stenosis

88. Williams syndrome

89. Neonatal lupus

90. Noonan syndrome

91. Kawasaki disease

92. Turner syndrome

93. Marfan syndrome

Questions 94 through 98

(A) 6 weeks of age(B) 8 weeks of age(C) 12 weeks of age(D) 16 weeks of age(E) 6 months of age(F) 8 months of age(G) 12 months of age

94. Sits without support

95. Hands together in midline

96. Bangs two cubes

97. Thumb–finger grasp

98. Disappearance of Moro reflex

Questions 99 through 104

(A) Hb 12 g/dL; WBC 11,500/mm3; platelets160,000/mm3; reticulocytes 1%

(B) Hb 12 g/dL; WBC 11,500/mm3; platelets25,000/mm3; reticulocytes 1%

(C) Hb 5.5 g/dL; WBC 3000/mm3; platelets35,000/mm3; reticulocytes 0.5%

(D) Hb 5.5 g/dL; WBD 8000/mm3; platelets400,000/mm3; reticulocytes 0.5%

(E) Hb 8 g/dL; WBC 19,500/mm3; platelets170,000/mm3; reticulocytes 14%

Page 27: Questions - uhs.edu.kh You suspect Lyme disease, most likely contracted by which of the following? (A) ingestion of unripe fruit (B) ingestion of spoiled fruit (C) drinking of contaminated

28 2: General Pediatrics

99. Idiopathic thrombocytopenic purpura

100. Normal 2-year-old child

101. Sickle cell disease, not in crisis

102. Iron deficiency anemia

103. Acute lymphoblastic leukemia

104. Acquired aplastic pancytopenia

Questions 105 through 107

(A) Congenital aganglionic megacolon(B) Duodenal atresia(C) Jejunoileal atresia(D) Intestinal malrotation(E) Meconium ileus

105. Cystic fibrosis

106. Midgut volvulus

107. Enterocolitis

Questions 108 through 112

(A) Sensorineural deafness(B) Limb abnormalities(C) Short stature(D) Glaucoma(E) Hydrocephalus(F) Tram-track calcifications(G) Saddle nose(H) Skin, eye, mouth infection

108. Neonatal herpes virus infection

109. Congenital cytomegalovirus infection

110. Congenital toxoplasmosis

111. Congenital syphilis

112. Congenital varicella

Page 28: Questions - uhs.edu.kh You suspect Lyme disease, most likely contracted by which of the following? (A) ingestion of unripe fruit (B) ingestion of spoiled fruit (C) drinking of contaminated

29

1. (B) This patient presents with Parinaud ocu-loglandular syndrome. This syndrome is themost common atypical presentation of catscratch disease (CSD) but can also occur withtularemia, tuberculosis, and syphilis. CSD isone of the most common causes of subacutelymphadenitis in children. Bartonella henselae isthe organism that has been associated with theclinical syndrome of CSD. Bartonella henselae istransmitted among cats by fleas and bacteremiccats transmit the disease to humans throughsaliva. Human-to-human transmission does notoccur. The conjunctiva is the site of inoculationin oculoglandular syndrome and the conjunc-tivitis is typically painless and nonsuppurative.A conjunctival granuloma may be present at theinoculation site. Lymphadenopathy most oftenoccurs in the preauricular nodes and less com-monly submandibular. (English, 27:123–127, 2006)

2. (C) Werdnig-Hoffmann disease is an autoso-mal recessive disorder affecting the anteriorhorn cells and the motor nuclei of the brain-stem. Loss of motor function begins in infancyand progresses fairly rapidly, leading in mostcases to ventilatory failure within the first2 years of life. Clinical features include hypo-tonia, weakness or paralysis, hyporeflexia, andmuscle fasciculations, which are most readilynoted in the tongue. Seizures, optic atrophy,and fever are not features of this disorder. Therealso is a rare, late-onset, more slowly progres-sive degenerative disorder of the anterior horncells referred to as Kugelberg-Welander disease.(Behrman, 2075–2076)

3. (C) This patient most likely has acute cerebel-lar ataxia. This occurs primarily in childrenbetween the age of 1–3 years and is a diagno-sis of exclusion. It is often preceded by a viralillness 2–3 weeks prior to onset of symptomsand is thought to represent an autoimmuneresponse to the viral agent affecting the cere-bellum. Recognized viral etiologies includevaricella, coxsackievirus, or echovirus infec-tion. Other infectious causes of acute ataxiathat must be excluded are cerebellar abscessand acute labyrinthitis which is associated withmiddle ear infection. Toxic causes of acuteataxia such as alcohol, thallium, and phenytoinmust be ruled out as well. Brain tumors of thefrontal lobe and cerebellum as well as neurob-lastoma may all present with ataxia. (Behrman,2020)

4. (D) Craniosynostosis is the condition of pre-mature fusion of one or more of the cranialsutures and occurs both as an isolated abnor-mality and in association with a wide variety ofcongenital malformations, syndromes, andchromosomal abnormalities. Excluding abnor-malities of the head and face, the extremitiesare the organs most frequently involved withassociated defects. The most common and bestknown such syndrome is the autosomaldominant Apert syndrome. Craniosynostosisplus extensive syndactyly of the fingers andtoes occur with this syndrome. (Behrman,1992–1993)

Answers and Explanations

Page 29: Questions - uhs.edu.kh You suspect Lyme disease, most likely contracted by which of the following? (A) ingestion of unripe fruit (B) ingestion of spoiled fruit (C) drinking of contaminated

30 2: General Pediatrics

5. (B) The arthritis of acute rheumatic fever is apainful acute migratory polyarthritis. Althoughany joint can be involved, it is primarily thelarge joints of the extremities that are affected inmost cases. Pain and swelling in one joint sub-sides as another joint becomes symptomatic.Eventually, all joints heal without deformity orother permanent sequelae. Fever and arthritisusually occur concomitantly but may occur inthe presence or absence of carditis. (Behrman, 876)

6. (D) The clinical findings described are classicfor the entity of total anomalous pulmonaryvenous return with obstruction of the veins. Inthis condition, the pulmonary veins drain tothe right rather than the left atrium. Aftermixing with systemic venous return in the rightatrium, some of the oxygenated pulmonaryvenous blood shunts across the foramen ovale(which is kept open by the increased right atrialpressure), providing a right-to-left shunt ofpartially oxygenated blood into the systemiccirculation. In many cases, the pulmonaryvenous return is not directly into the rightatrium but rather takes a devious route, oftencoursing below the diaphragm before reach-ing the right atrium. In such instances, venousobstruction is the rule, and cyanosis resultsboth from the right-to-left shunt and from wet,congested lungs. A diffuse reticular pattern tothe lung fields is characteristically seen on radi-ograph. Although the heart may be consider-ably enlarged in patients without venousobstruction, it is characteristically normal oronly minimally enlarged in those withobstruction. (Behrman, 1538–1539)

7. (A) Brain tumors, although rare in the first yearof life, are the second most common type ofcancer in childhood, exceeded only by leukemia.More than half of the tumors in children arelocated below the tentorium, and about three-quarters are in the midline. For this reason,increased intracranial pressure from obstructionof the third or fourth ventricle is a common find-ing. Seizures can be the presenting complaintbut are not in the majority of cases. (Behrman,1702–1703)

8. (E) The infant described most likely has anintussusception of the intestine. This conditionis most frequent in the second half of the firstyear of life and involves the telescoping of onesegment of bowel into another, most frequentlythe ileum into the colon (ileocolic intussuscep-tion). Intermittent abdominal pain and vomit-ing are common features. Mild fever andleukocytosis are frequent. Very often the intus-susceptum can be palpated as a sausage-shaped mass. Circulation to the intussusceptedbowel can be impaired, resulting in dischargeof a bloody, mucous stool—the so-called“currant jelly” stool. Barium enema is the pro-cedure of choice because it can be therapeuticas well as diagnostic. Under fluoroscopic visu-alization, the radiologist usually is able toemploy the hydrostatic pressure of the enemato reduce the intussuscepted bowel. (Behrman,1242–1243)

9. (E) Cystic fibrosis is the most common cause ofpancreatic insufficiency in childhood, account-ing for almost all cases. Biliary atresia is notassociated with pancreatic insufficiency.Congenital hypoplasia or absence of the pan-creas is recognized but rare. Schwachman syn-drome is a rare condition of unknown causecharacterized by pancreatic insufficiency andneutropenia. Acute pancreatitis usually doesnot lead to pancreatic insufficiency. (Behrman,1299–1300)

10. (C) Brain abscess is an infrequent but not raredisorder of childhood. Most childhood brainabscesses occur between age 4 and 8 years.Recognized predisposing conditions includepenetrating head injury, brain surgery, immun-odeficiency, cystic fibrosis, right-to-left intrac-ardiac shunts, and infection of the middle ear,mastoid, or facial sinuses. Right-to-left intrac-ardiac shunts in children with cyanotic con-genital heart disease bypass the macrophage-filtering mechanism of the lungs and increasethe access of bacteria to the brain. In thesechildren, cerebral hypoxia and focal encephalo-malacia also may predispose to infection.(Behrman, 2047–2048)

Page 30: Questions - uhs.edu.kh You suspect Lyme disease, most likely contracted by which of the following? (A) ingestion of unripe fruit (B) ingestion of spoiled fruit (C) drinking of contaminated

Answers: 5–16 31

11. (C) There are three classic clinical presentationsof juvenile idiopathic arthritis (JIA)—acute sys-temic, pauciarticular, and polyarticular. Thefigure demonstrates the typical knee arthritisseen in a girl with pauciarticular disease (seeFigure 2-1). The frequency of ANA seropositiv-ity is highest in girls with younger age at onsetof disease. Among children who have oligoarthri-tis and uveitis there is highest prevalence of pos-itive ANAs. Determination of ANAseropositivityis therefore helpful in identifying children mostat risk for chronic uveitis. (Cassidy, 224)

12. (D) The child described has the classic hand-foot syndrome seen in infants and toddlerswith sickle cell disease. Dactylitis, presumablysecondary to infarction of the small bones,causes painful swelling of the hands and feet.Hemoglobin electrophoresis would show pres-ence of high levels of HbS. (Behrman, 1624–1625)

13. (C) There are many causes of apnea in infants.The extent of the diagnostic workup depends onmany factors such as history and physical exam-ination findings. In this case, the symptoms ofthe mother should raise significant concern forpresence of pertussis, a well-recognized causefor apnea in young infants. Pertussis, caused byB pertussis, is endemic and the annual incidencehas risen in the past year. Coughing adolescentsand adults currently are the major reservoir forB pertussis. Erythromycin for 14 days in the pasthas been the standard treatment but currentrecommendations suggest azithromycin is

effective and a shorter duration of therapy isneeded. It should be instituted when pertussisis suspected or confirmed to limit the spread ofinfection; the clinical benefit to the patient whohas been coughing for longer than 2 weeks isneglible. New vaccines recently have beenlicensed for booster immunizations of adolescentsand adults (Tdap) and will hopefully serve toreduce the risk of exposure of young infants to thepool of susceptible older patients. (Behrman, 908–912)

14. (E) Inadequate dietary iron is the leading causeof iron deficiency anemia in children. Milk haslow iron content, and the iron in cow’s milk isnot well absorbed. If a large percentage ofdietary calories come from milk, the diet is aptto be low in iron. Additionally, microscopicgastrointestinal blood loss associated with theintake of unmodified cow’s milk can contributeto iron deficiency. (Behrman, 1614–1615)

15. (B) Although rhabdomyosarcoma and infre-quently hepatoblastoma, can metastasize to bone,few solid cancers of childhood spread to bone asregularly as neuroblastoma. Additionally, whereasbone metastases of other tumors tend to be focal,involvement by neuroblastoma frequently is dif-fuse. In some series of neuroblastoma, bonemarrow involvement has exceeded 50% of cases.Thus, bone marrow aspiration, even in theabsence of radiographic evidence of osseousinvolvement, is more likely to be diagnostic inneuroblastoma than in any other solid cancer ofchildhood. (Behrman, 1709–1710)

16. (A) The patient described most likely haschronic nonspecific diarrhea (CNSD) also calledtoddler’s diarrhea. The diagnosis of thiscommon condition is based on history and phys-ical examination. The presence of normalgrowth and normal physical examination in theabsence of other signs or symptoms despite 4months of diarrhea makes this the most likelydiagnosis. Therefore, reassurance of the parentsshould occur and no further workup is neces-sary. It is important that in the process of thisoffice visit, the physician reviews the dietnormal for this age including amounts of fruitjuices. Antidiarrheal agents are rarely appropri-ate for infants and children. (Keating, 25:5–13, 2005)

Figure 2-1(Courtesy of Mary Anne Jackson, MD)

Page 31: Questions - uhs.edu.kh You suspect Lyme disease, most likely contracted by which of the following? (A) ingestion of unripe fruit (B) ingestion of spoiled fruit (C) drinking of contaminated

32 2: General Pediatrics

17. (C) The McCune-Albright syndrome consists offibrous dysplasia of bone, multiple, large pig-mented nevi (generally on only one side ofthe trunk), and precocious puberty, which ismore common in females than in males. Otherendocrine disorders occur less frequently andinclude hyperthyroidism and hyperadrenalism(Cushing syndrome). (Behrman, 1867)

18. (B) The combination of the continous murmurcombined with the wide pulse pressure is typ-ical for a patent ductus arteriosus (PDA). Theincreased left ventricular output, coupled withrunoff from the aorta through the ductus, pro-duces a widened pulse pressure and a bound-ing or collapsing pulse. The increased flow tothe lungs and back to the left ventricle causeshypertrophy of that chamber rather than of theright ventricle. Other more rare conditions suchas coronary arteriovenous fistulas, aberrant leftcoronary artery with massive right coronarycollaterals, and truncus arteriosus with torren-tial pulmonary flow may display similar dynam-ics as PDA. Spontaneous clusure of the ductusafter infancy is extremely rare and patientsrequire surgical or catheter closure. Thesepatients do not need life-long SBE prophylaxis.(Behrman, 1510–1512, 1569)

19. (C) Congenital megacolon (Hirschsprung dis-ease) is the result of congenital absence of gan-glion cells in a segment of large bowel. Absentor deficient peristalsis in the affected segmentresults in functional obstruction. This causesconstipation and distention of bowel proximalto the aganglionic area. It is this chronicallydistended bowel that has led to the namemegacolon. Severe cases present as neonatalintestinal obstruction. Practically all normal,full-term born neonates pass meconium in thefirst 48 hours of life. Hirschprung diseaseshould be considered in any full-term infantwith delayed passage of stool. Rectal manom-etry and rectal suction biosy are the easiest andmost reliable indicators of Hirschprung disease.(Behrman, 1239–1240)

20. (C) Complex partial seizures is the currentname for what previously was termed tempo-ral lobe or psychomotor seizures. Complex par-

tial seizures are characterized by alterations ofmental status, consciousness, or responsiveness.During the seizure, there may be confusion,emotional reactions, feelings of detachment,and hallucinations. Automatisms (semipur-poseful but inappropriate motor acts) are fre-quent. There is no tonic or clonic component.Postictal confusion is common. The usual EEGfinding is spike-wave activity over one or bothtemporal lobe regions. A three-per-secondspike-and-wave pattern on EEG is characteris-tic of absence seizures. (Behrman,1995–1996)

21. (A) This patient has pseudotumor cerebriwhich is a clinical syndrome that mimicsbrain tumors. It is characterized by increasedintracranial pressure with normal size, posi-tion, and anatomy of the ventricles. It is causedby impaired reabsorption of cerebrospinal fluidwith or without increased intracerebral bloodvolume. The causes of pseudotumor cerebriare numerous and involve metabolic, toxic,infectious, hematologic, and anatomic etiolo-gies. A cause is not always identified. The mostcommon symptom is headache. Optic atrophyand blindness are the most serious complica-tions. Treatment options include serial spinaltaps to relieve the pressure, acetazolomide, andsteroids. (Behrman, 2048–2049)

22. (D) Hepatitis B vaccines are highly efficacious inpreventing perinatal transmission of HBV. Infantsborn to HBsAg-positive mothers should receivethe first dose within 12–24 hours after birth.These infants also should receive hepatitis Bimmune globulin (HBIG) in the first day of life.Although HBV is found in breast milk, breast-feeding has not been shown to raise the risk ofHBV transmission. (Stellwagen, 27:89–97, 2006)

23. (C) Infants of diabetic mothers tend to be largeas a result of increased body fat and enlargedviscera. They may have any of the diverse man-ifestations of hypoglycemia. A greater inci-dence of respiratory distress syndrome appearsin infants of diabetic mothers. Approximately30% of these infants have cardiomegaly andheart failure occurs in 5% to 10% of diabeticmothers. Infants of diabetic mothers have athreefold increased incidence of congenital

Page 32: Questions - uhs.edu.kh You suspect Lyme disease, most likely contracted by which of the following? (A) ingestion of unripe fruit (B) ingestion of spoiled fruit (C) drinking of contaminated

Answers: 17–29 33

anomalies. Cardiac malformations and lum-bosacral agenesis are among the most common.Neural tube defects, renal anomalies, duodenalor anorectal atresia, holoprosencephaly, andsmall left colon syndrome also may occur. Theincidence of Down syndrome may also beincreased. (Behrman, 613–614)

24. (E) Aniridia involves the whole eye and shouldnot be thought of as an isolated iris defect. Iristissue usually is present but is hypoplastic. Itusually occurs bilaterally and vision is severelyimpaired. Aniridia can either be a sporadicoccurrence or can have an autosomal dominantinheritance pattern. One-fifth of the patientswith sporadic anirida will develop Wilmstumor. These patients will need surveillancewith renal ultrasound every 4–6 months. Thereis also an association between Wilms tumor,aniridia, genitourinary anomalies, and mentalretardation (WAGR acronym). (Behrman, 2089)

25. (B) The patient most likely has Wilson disease.Wilson disease or hepatolenticular degenera-tion is an autosomal recessive disorder ofcopper metabolism. Copper accumulates in thebrain, liver, and cornea where deposits are visi-ble as brown rings (Kayser-Fleischer). Untreated,this disorder ultimately leads to death second-ary to hepatic, neurologic, renal, or hematologiccomplications. Chelation therapy with oralpenicillamine should start as early as possible.(Behrman, 1321–1322)

26. (D) The patient most likely has the infantileform of Tay-Sachs disease. Tay-Sachs is amember of the family of lipid storage diseases.These diseases have a deficiency of a lysosomalhydrolase enzyme, leading to the lysosomalaccumulation of specific sphingolipids. Commonfeatures of these disorders are neurodegenera-tion and organomegaly. Tay-Sachs disease iscaused by deficiency of hexosaminidase Aand isinherited as an autosomal trait with a carrier fre-quency of 1/25 in the Ashkenazi Jewish popu-lation. The infantile form of this disease will oftenbecome manifest in early childhood with loss ofmilestones, increased startle reaction (hyperacu-sis), and cherry-red spot on retinoscopy. (Behrman,462–463)

27. (E) The 22q11 deletion syndrome is character-ized by aplasia or hypoplasia of the thymusand parathyroid glands: this results fromdysmorphogenesis of the third and fourth pha-ryngeal pouches during early embryogensis.Other organs that are formed during the sametime period are frequently affected as well.Facial abnormalities, esophageal atresia, bifiduvula, and congenital heart disease are some ofthe anomalies associated with this syndrome.Patients with complete 22q11 deletion syn-drome have problems early in life withincreased susceptibility to infectious agentssuch as viruses and fungi. Microcephaly,hyponatremia, hyperkalemia, and goiter arenot associated with this syndrome. (Behrman,694)

28. (B) The patient described most likely hasWiskott-Aldrich syndrome. This syndrome ischaracterized by thrombocytopenia, atopic der-matitis, and severe immunodeficiency. Theinheritance pattern is X-linked and affects onlyboys. Severe atopic dermatitis and recurrentinfections usually become manifest during thefirst year of life. These children are susceptibleto infections with encapsulated bacteria such aspneumococci, resulting in frequent otitismedia, sinusitis, pneumonia, and sepsis.Infections with Pneumocystis carinii, herpesvirus, and cytomegalovirus can also become aproblem. The only curable treatment is bonemarrow transplantation. (Behrman, 699)

29. (D) The patient has the classical symptoms ofjuvenile dermatomyositis (JDM). Patients withthis disorder usually present with a combina-tion of malaise, fever, fatigue, muscle weak-ness, and rash. Determination of serum levelsof muscle enzymes is important for diagnosisand monitoring the disease course once ther-apy has been instituted. Rheumatoid factortests in children with JDM are almost alwaysnegative. Antinuclear antibodies (ANAs) canbe positive in JDM, however, this would not bethe first test of choice. Erythrocyte sedimenta-tion rate is a nonspecific indicator of inflam-mation and of little help in making a diagnosisof JDM. Renal abnormalities are rare in JDM.(Cassidy, 407, 424)

Page 33: Questions - uhs.edu.kh You suspect Lyme disease, most likely contracted by which of the following? (A) ingestion of unripe fruit (B) ingestion of spoiled fruit (C) drinking of contaminated

34 2: General Pediatrics

30. (B) Corticosteroid therapy is frequently bene-ficial in the treatment of Diamond-Blackfansyndrome (congenital pure red cell aplasy),but does not appear to have any value in thetreatment of TEC. TEC is a transient disorderand usually resolves spontaneously by 1–2months. Diamond-Blackfan syndrome canbecome a chronic disease if there is noresponse to prednisone. Hepatosplenomegalyis not a feature of TEC. Red blood cell transfu-sions may be necessary for TEC. However, par-vovirus infection has been associated withconditions resulting in low hemoglobin con-centrations. These include, among others, tran-sient aplastic crisis in children with homolyticanemias and chronic erythroid hypoplasia inimmunodeficient patients. Parvovirus infec-tion has not been associated with TEC.(Behrman, 1606–1608)

31. (B) Megaloblastic anemia in children almostalways results from a deficiency of folic acid,vitamin B12, or both. In the peripheral blood,red blood cells are large and frequently hyper-segmented neutrophils appear. Folic acid isabundant in many foods, including green veg-etables, fruits, and animal organs. Human andcow milk provide adequate amounts of folicacid, but goat milk is clearly deficient in folicacid. Vitamin B12 is present in many foods anddietary deficiency is therefore rare. It can beseen in vegans who do not consume any animalproducts. (Behrman, 1611)

32. (A) The child described most likely has celiacdisease or gluten-sensitive enteropathy. Thisdisorder is the result of small bowel mucosaldamage caused by sensitivity to dietary glutenand has an incidence in the United States ofapproximately 1:10,000. The incidence appearshigher in Europe. Symptoms typically startafter gluten is introduced in the diet. The clin-ical spectrum of this disease is wide but failureto thrive, diarrhea, irritability, vomiting, andanorexia are common. The sensitivity andspecificity of serum IgA-endomysial antibodytesting is very high, but histologic findings onsmall bowel biopsy remain the standard formaking the diagnosis. (Behrman, 1264–1266)

33. (D) Secondary nocturnal enuresis is defined bythe occurrence of nighttime bedwetting afterbeing dry for a minimum of 6 months. Anorganic cause can only be found in 2% to 3% ofpatients with nocturnal enuresis. Any organicfactors or disease should be ruled out duringthe office visit. Every child with enuresisshould have a urine analysis. Diabetes mellitus,diabetes insipidus, or urinary tract infectionscan be quickly diagnosed or ruled out. (Schmitt18:183–190, 1997)

34. (C) A defect in complement function should besuspected in any patient with recurrent pyogenicinfections or recurrent Neisseria meningitidisor N gonorrhea infections. Testing for totalhemolytic complement activity (CH50) is a goodscreening tool for complement abnormalities.(Behrman, 728–729)

35. (E) The child presented most likely has metasta-tic neuroblastoma. The median age at diagnosisis 2 years. Age, stage of disease at diagnosis, andbiology of the tumor are prognostic factors inneuroblastoma. Children under the age of 1 yearwith limited disease and without mycn geneamplification have a 95% survival. Imaging withCT scan or MRI, bone scans, and bone marrowbiopsy are part of the staging for this disease.Bone radiographs may be abnormal but are notused for staging purposes. Hematuria is not acommon feature of this tumor. There are anumber of well-documented cases in infants whohad complete regression of their tumor withoutmedical intervention. (Behrman, 1709–1710)

36. (D) Beckwith-Wiedemann syndrome is char-acterized by neonatal overgrowth. Infants arelarge for gestational age, have macroglossia,hepatosplenomegaly, and often difficult to con-trol hypoglycemia. Children with this disorderare at increased risk for development of Wilmstumor, hepatoblastoma, and adrenocortical car-cinoma. Serum alpha-fetoprotein is elevated inabout 90% of patients with hepatoblastoma.(Behrman, 1712, 1725)

37. (D) The presented patient most likely has sicklecell disease. As many as 10% of patients withsickle cell anemia will exhibit sequelae of

Page 34: Questions - uhs.edu.kh You suspect Lyme disease, most likely contracted by which of the following? (A) ingestion of unripe fruit (B) ingestion of spoiled fruit (C) drinking of contaminated

Answers: 30–42 35

strokes. The diagnosis of sickle cell diseasecan be made by hemoglobin electrophoresis.A patient who is homozygous for the sicklecell gene has no normal hemoglobin (HbA1),will have a high amount of sickle hemoglobin(HbS), and an increased amount of fetal hemo-globin (HbF). Acarrier of the sickle cell gene willhave about 50% HbA1 and less than 50% HbS.(Behrman, 1624–1627)

38. (C) The detection of Cryptosporidium in the stoolof a patient with chronic diarrhea and malnu-trition in the setting of human immunodefi-ciency virus (HIV) infection would be mostlikely. This water borne protozoa can been seenin otherwise healthy children but diarrhea isusually self-limited in such cases. Rotavirus,Giardia, Salmonella, and Yersinia have all beenassociated with gastroenteritis in healthy chil-dren and recovery of these organisms wouldnot support a diagnosis of HIV infection. (RedBook, 271)

39. (C) The incidence of group A streptococcal dis-ease depends on the age group, the season, theclimate, and degree of contact with infectedindividuals. Suppurative and nonsuppurativecomplications from group A streptococcal dis-ease have increased. Streptococcal skin infec-tions can cause impetigo, cellulitis, erysipelas,and necrotizing fasciitis. Nonsuppurative com-plications from streptococcal skin infectionsinclude scarlatina, poststreptococcal glomeru-lonephritis, and toxic shock syndrome. Acuterheumatic fever is a nonsuppurative compli-cation from streptococcal pharyngitis but is notassociated with streptococcal skin infections.(Behrman, 873–874)

40. (E) The child described most likely hasHenoch-Schönlein purpura (HSP). HSP typi-cally develops in a previously healthy childwith a distinctive rash that often involves theextensor surfaces of the lower extremities.Arthritis or arthralgia develops in 75% of casesand most patients have some gastrointestinalsymptoms. Intussusception can be a seriouscomplication occurring in about 5% of patients.Renal involvement is detected in about half ofthe patients and can range from mild to very

severe. One percent of patients will developend-stage renal disease. The diagnosis can bedifficult to make when gastrointestinal symp-toms or arthritis precede the rash. Laboratorystudies are typically not helpful in making thediagnosis. Clotting functions are generallynormal and platelet counts in these patientsare normal or elevated. Remember, HSP is asso-ciated with nonthrombocytopenic purpura(see Figure 2-2). (Behrman, 826–828; Zitelli, 248–250)

41. (C) The patient described has signs and symp-toms that are very suggestive of fragile X syn-drome. Fragile X syndrome is the most frequentcause of hereditary mental retardation. Thissyndrome is caused by amplification of CGCtriplets on the X chromosome. Both male andfemale premutation carriers of this syndrometypically have no phenotypic manifestations.Premutations can expand to full mutationsonly in female meiotic transmission. Affectedmales typically develop testicular enlargementafter puberty. (Behrman, 388; Jones, 160–162)

42. (D) The described patient has classical signsand symptoms of trisomy 21. The occurrence oftrisomy 21 or Down syndrome increases withadvancing maternal age. There is no singlephysical finding that makes a diagnosis ofDown syndrome; rather, the combination ofminor and major anomalies often leads to adiagnosis. All the listed anomalies are more

Figure 2-2(Courtesy of Mary Anne Jackson, MD)

Page 35: Questions - uhs.edu.kh You suspect Lyme disease, most likely contracted by which of the following? (A) ingestion of unripe fruit (B) ingestion of spoiled fruit (C) drinking of contaminated

36 2: General Pediatrics

common in Down syndrome, however, hearingloss affecting more than 60% of patients is themost common. About 40% of patients have car-diac abnormalities with endocardial cushiondefect being the most common. (Jones, 7–11)

43. (B) The patient has classical findings of Turnersyndrome. Patients with this syndrome have anXO karyotype. Short stature is the cardinal find-ing in all girls with Turner syndrome. Ovariandysgenesis is present and sexual maturationusually fails to occur, although a small percent-age of girls may have spontaneous breast devel-opment and menstrual periods. Other clinicalfeatures that may be present are webbed neck,low hairline, and wide-spaced nipples. Cardiacdefects occur in about half of the patients andbicuspid aortic valves are the most commonlyfound abnormality. Renal abnormalities, suchas horseshoe kidney, occur in a majority ofpatients. (Behrman, 1931–1934; Jones, 76–81)

44. (E) The patient described most likely has 22q11deletion syndrome. Dysmorphogenesis of thethird and fourth pharyngeal pouches duringearly embryogenesis leads to thymic andhypoparathyroid aplasia in varying degrees ofseverity. When absence of the thymus is com-plete, patients suffer from a severe combinedimmunodeficiency (SCID) with abnormalitiesof both B-lymphocyte and T-lymphocyte func-tions. These infants are at risk for graft versushost disease (GVHD) from nonirradiated bloodproducts and may actually develop GVHDfrom maternal derived cells in their circulation.22q11 deletion syndrome can be part of theCATCH 22 syndrome (cardiac, abnormal facies,thymic hypoplasia, cleft palate, hypocalcemia),which includes the broad clinical spectrum ofconditions with chromosome 22q11.2 deletions.(Behrman, 694)

45. (A) Thrombocytopenia absent radius (TAR)syndrome is one of the congenital thrombocy-topenia syndromes. The thrombocytopeniaseverity usually lessens with advancing age.Patients may occasionally have other congeni-tal abnormalities. Wiskott-Aldrich syndrome(WAS) is another congenital thrombocytope-nia syndrome in which boys are affected with

thrombocytopenia, eczema, and severe immun-odeficiencies. Congenital amegakaryocyticthrombocytopenia usually presents in the firstweeks of life with petechiae and purpura. (Jones,364–365; Behrman, 1672)

46. (C) The patient described has all the features ofmucopolysaccharidosis I (MPS I) or Hurler syn-drome. The MPS are inherited disorders char-acterized by deficiency of lysosomal enzymesneeded to break down glycosaminoglycans, theintralysosomal accumulation of glycosamino-glycans, and excessive urinary excretion of gly-cosaminoglycans such as dermatan sulfate,heparan sulfate, or keratan sulfate. The MPSdisorders share many clinical features althoughin varying degrees of severity. The course isoften chronic and progressive, many organs areinvolved, and typically organomegaly andabnormal facial features and dysostosis multi-plex on skeletal radiographs are present. Hurlersyndrome is the most severe in this group ofdisorders and is due to a deficiency of alpha-l-iduronidase. (Behrman, 483–486; Jones, 524–527)

47. (C) Approximately 85% of shaken babies haveretinal hemorrhages. Other findings that arehighly suggestive of shaking injury are poste-rior rib fractures, methaphysial fractures (bucket-handle fractures), and subdural hematomas.These patients may present with coma, convul-sions, apnea, and increased intracranial pressure.Often there are no external marks. All childrenunder the age of 2 years in whom physical abuseis suspected should have an initial roentgeno-logic bone survey done and a repeat study in7–10 days to reveal healing of fractures that mayhave been missed on initial film. (Behrman,123–124, American Academy of Pediatrics Section onRadiology, 105:1345–1348, 2000)

48. (A) About 75% of nephrotic syndrome in chil-dren 1–12 years of age is due to minimal-changedisease. The mortality in minimal-changenephrotic syndrome is about 1%–2%. Most ofthe mortality is related to an increased suscep-tibility for infections, with bacterial peritonitisbeing the most serious one. The majority of theremainder of deaths is due to thromboembolicevents. Hyperlipidemia is associated with

Page 36: Questions - uhs.edu.kh You suspect Lyme disease, most likely contracted by which of the following? (A) ingestion of unripe fruit (B) ingestion of spoiled fruit (C) drinking of contaminated

Answers: 43–55 37

nephrotic syndrome, but is generally not asso-ciated with clinical complications. Acute renalfailure can occur with intravascular volumedepletion and decreased renal perfusion but iseasily treatable. (Roth, 23:237–247, 2002)

49. (E) The patient most likely has Giardia intesti-nalis infection. Giardiasis occurs worldwideand is endemic in developing countries wheresanitation is poor. It is also the most commonparasite identified in stool specimens in theUnited States. Transmission generally occursthrough ingestion of cysts from unwashedhands that were in contact with infected feces.Outbreaks can occur from contaminated drink-ing water and recreational pools. It causes sig-nificant morbidity in child-care centers andchronic residential institutions. The clinical man-ifestations of giardiasis can range from asymp-tomatic colonization to acute or chronic diarrheaand malabsorption. The current treatment is oralmetronidazole for 10 days. (Behrman, 1125–1127)

50. (E) Obesity in children is on the rise and is reach-ing epidemic proportions in the United States.Current estimates indicate that 20%–25% of chil-dren between the age of 6–19 years are obese.Obesity is a significant risk factor for many seri-ous medical problems. These patients are at riskfor psychologic problems, pulmonary insuffi-ciency, skeletal abnormalities, metabolic diseases,cardiovascular diseases, and malignancies. Sleepapnea resulting from upper airway obstructioncan be a dangerous complication and is the mostcommon cause for pulmonary insufficiency inobese patients. (Klish, 19:312–315, 1998)

51. (D) Following and graphing a child’s growth isan important tool to assess the general health ofthe child. Standard growth curves from growthdata of different ethnic groups are available.However, the distinction between normal andabnormal growth is not always easy to make.The single most critical factor to determinewhether the growth of a child is normal is todetermine the height velocity. An easy way todecide if the height velocity is normal is byobserving whether the height is crossing per-centiles on the linear growth curve. (Rose, 26:404–413, 2005)

52. (B) Constipation is a source of frustration forpatients, families, and health-care workers. Hardstools, pain with defecation, or failure to passthree stools per week are usually labeled as con-stipation. There are functional and nonfunctionaletiologies of constipation. Functional fecal reten-tion is the most common explanation for child-hood constipation. This can first start aroundpotty training time or as a normal avoidancetechnique in the context of painful anal inflam-mation. It may be accompanied by early satiety,increasing irritability, and abdominal pain.Unabated functional fecal retention eventuallyleads to encopresis. (Abi-Hanna 19:23–31, 1998)

53. (A) The patient most likely has X-linked agam-maglobulinemia. Patients with this immunode-ficiency typically have no symptoms during thefirst 6–9 months of life because of passive trans-fer of maternal antibodies. Thereafter, they aresusceptible for severe infections with extracel-lular pyogenic organisms such as Streptococcuspneumoniae and Haemophilus influenzae. Fungalinfections typically do not occur and viral infec-tions are generally handled well with the excep-tion of hepatitis viruses and enteroviruses.(Behrman, 689–690)

54. (B) Apprixmately 1%–4% of pregnant womenin the United States acquire primary CMV infec-tion during pregnancy. A fetus may becomeinfected as a result of primary and recurrentmaternal infection, but risk for fetal infection isgreatest with maternal primary CMV infec-tion. Only 5% of all infected infants show thefull spectrum of congenital CMV disease withintrauterine growth retardation, prematurity,hepatosplenomegaly, jaundice, thrombocytope-nia, purpura, microcephaly, and intracranial cal-cifications. Another 5% of congenitally infectedinfants have mild infection. About 90% of con-genitally infected infants have subclinical butchronic CMV infection. Sensorineural hearingloss occurs in approximately 7% of infectedinfants and can even occur in infants who wereasymptomatic at birth. (Behrman, 1066–1069)

55. (B) Hypertension is classified as primary (essen-tial) or secondary. Essential hypertension is adiagnosis of exclusion. Essential hypertension is

Page 37: Questions - uhs.edu.kh You suspect Lyme disease, most likely contracted by which of the following? (A) ingestion of unripe fruit (B) ingestion of spoiled fruit (C) drinking of contaminated

38 2: General Pediatrics

the most common cause of hypertension inadults but is a significant pediatric diagnosisonly in the adolescent age group. The youngerthe patient and the more severe the hyperten-sion, the more likely a secondary cause will befound. Coarctation of the aorta is responsible forabout a third of the cases of neonatal hyperten-sion. As in adults, hypertension in African-American adolescents is about twice ascommon as in Caucasian adolescents. Mostpediatric patients with hypertension are asymp-tomatic. Sports participation should not be lim-ited on the basis of hypertension alone. Regularexercise should be part of lifestyle recommen-dations, especially if the hypertension is associ-ated with obesity. (Norwood, 23:197–208, 2002)

56. (A) Dysfunctional uterine bleeding (DUB) ismost common in the first 2 years after menarche.Almost all cases of DUB during adolescence aredue to anovulatory cycles. The most commonreason for anovulatory cycles at this age groupis immaturity of the hypothalamic-pituitary-ovarian axis. There are other causes for anovu-lation that must be ruled out. Polycystic ovariansyndrome (PCOS) affects about 5%–10% ofwomen and is associated with irregular mensesand physical signs of hyperandrogenism.Systemic illness, especially if associated with sig-nificant weight loss, can lead to anovulation. Themost common endocrine disorder associatedwith anovulation is hypothyroidism but hyper-prolactinemia and Cushing syndrome are alsoassociated. Abnormal uterine bleeding cansometimes be the first presenting symptom of ablood dyscrasia. These disorders most oftenpresent as regular but heavy menses. Sexuallytransmitted diseases as well as complications ofpregnancy can present with irregular and/orpainful menses. (Rimsza 23:227–232, 2002)

57. (A) The child and mother exhibit classicalsymptoms of human parvovirus B19 infectionalso called fifth disease or erythema infectio-sum. Prodromal symptoms are usually mildand diagnosis is often made when the charac-teristic rash appears. Initally the rash starts onthe face and children have what is oftendescribed as a “slapped-cheek” appearance.The rash spreads readily to the rest of the body

with a diffuse macular erythema. Central clear-ing of the macules then occurs rapidly andgives the rash a lacy, reticulated appearance.Joint symptoms are very common in adults andmay be the only clinical manifestation of par-vovirus B19 infection. The primary target ofparvovirus B19 is the erythroid cell line.Children with erythema infectiosum are mostinfectious prior to the onset of the rash, there-fore, exclusion from school or day care afterthe rash develops is not indicated. Pregnantcaregives in day care or schools need not beexcluded. Parvovirus B19 has also been associ-ated with hydrops fetalis in pregnant women.It also can cause a more serious transient aplas-tic crisis in patients with chronic hemolytic dis-eases. Postexposure prophylaxis with IVIG forpregnant women or patients with chronichematologic disorders is not indicated. (Behrman,1048–1050)

58. (C) Testicular torsion is the most common causeof testicular pain and swelling in adolescentboys. It rarely occurs under the age of 10 years.It is caused by excessive mobility of the testisbecause of inadequate fixation within the scro-tum. After several hours, ischemia to the testiswill occur and spermatogenesis is lost. Rapidsurgical exploration is indicated when there issuspicion of testicular torsion to preservefunction. (Behrman, 1818–1819)

59. (A) Signs and symptoms in this patient are con-sistent with infectious mononucleosis most likelycaused by Epstein-Barr virus (EBV). A presump-tive diagnosis can be made with typical clincalpresentation and presence of atypical lymphocy-tosis. Serologic testing can confirm the diagnosis.Very few healthy adolescents with infectiousmononucleosis experience severe complications.The most feared complication is splenic rupturewhich has a reported occurrence rate of 0.2% inadults and is usually associated with blunt traumato the abdomen. Participation in contact sportsshould therefore be avoided for several weeks oruntil splenomegaly has resolved. Corticosteroidsare indicated only if there is severe airway obstruc-tion because of tonsillar enlargement. Acyclovirdoes not alter the course of the disease. (Behrman,1064–1066)

Page 38: Questions - uhs.edu.kh You suspect Lyme disease, most likely contracted by which of the following? (A) ingestion of unripe fruit (B) ingestion of spoiled fruit (C) drinking of contaminated

Answers: 56–65 39

60. (E) The patient exhibits signs and symptoms ofhemolytic uremic syndrome (HUS) after anepisode of gasteroenteritis most likely causedby enterohemorrhagic strain of E coli. This syn-drome is characterized by microangiopathichemolysis, thrombocytopenia, and acute renalfailure. The typical history, clinical picture, andlaboratory findings confirm the diagnosis inmost patients. Treatment is mainly supportive,frequent peritoneal dialysis needs to be initi-ated. Mortality is less than 10% with aggressivemanagement of the acute renal failure. Most ofthe patients will recover normal renal function.(Behrman, 1746–1747)

61. (E) Rectal prolapse can occur with many dis-orders such as acute diarrhea, Ehlers-Danlossyndrome, chronic constipation, ulcerative coli-tis, and cystic fibrosis. For an infant with failureto thrive, the most common diagnosis is cysticfibrosis. However, most cases of rectal prolapseare idiopathic. Usually the protruding rectalmucosa can be reduced by gentle pressure.Controlling the pancreatic steatorrhea withpancreatic enzyme replacement provides ade-quate treatment in cystic fibrosis. Surgical inter-vention is occasionally required. Sweat chloridetesting remains the fastest and easiest way todiagnose cystic fibrosis. The test may be diffi-cult in the first few weeks of life because oflow sweat volumes. The procedure requirescare and accuracy and is therefore best done ina laboratory with significant experience. (Behrman,1289; 1448–1449)

62. (B) The presented patient has signs and symp-toms of Meckel diverticulum, a remnant of theomphalomesenteric duct or the vitelline duct. Inthe fetus this duct connects the gut to the yolksac. Meckel diverticulum is the most commoncongenital gastrointestinal anomaly and occursin 2%–3% of all infants. They are located within100 cm of the ileocecal valve. Most of the symp-tomatic Meckel diverticula are lined with acid-secreting mucosa. The most common symptomis painless rectal bleeding caused by ulcerationof the adjacent normal ileal mucosa. Symptomsusually arise in the first 2 years of life, but canoccur throughout the first decade. Confirmationof a Meckel diverticulum can be difficult. Plain

abdominal radiographs and routine bariumstudies are usually not helpful. The most sensi-tive study is a scan with technetium-99mpertechnetate, which is taken up by the mucus-secreting cells of the ectopic gastric mucosa.Surgical excision is the treatment for sympto-matic Meckel diverticula. (Behrman, 1236–1237)

63. (C) The differential diagnosis of jaundice isextensive, but may be divided by elevation ofunconjugated (indirect) or conjugated (direct)bilirubin. Elevation of conjugated bilirubin isalways pathologic. Causes of direct hyperbiliru-binemia generally are infectious or metabolic innature or otherwise related to intra- or extra-hepatic pathology. Isoimmune hemolysis is acause of unconjugated hyperbilirubinemia.(Behrman, 1309–1311; American Academy of PediatricsSubcommittee on Hyperbilirubinemia, 114, 297–316,2004)

64. (A) Kawasaki disease, also known as mucocu-taneous lymph node syndrome, is the mostcommon cause of acquired heart disease in pedi-atrics. Criteria for diagnosis are fever for greaterthan 5 days plus four of the following five fea-tures: conjunctivitis, cervical adenitis, mucousmembrane changes, a polymorphous rash of thetrunk and extremities, and distal extremitychanges. The illness can be divided into threephases. The acute phase is characterized byfever, conjunctivitis, cervical adenopathy, rash,mucous membrane changes, extremity swelling,and erythema which are seen in the first 10 days.A subacute phase occurs over the next 7-14 daysand is manifested by a decrease in fever, devel-opment of thrombocytosis, and distal extremityskin desquamation. Coronary artery aneurysms,which occur in approximately 25% of untreatedpatients develop 10-40 days into the course. Earlytreatment with intravenous immunoglobulinand salicylates reduces the risk of developingcoronary artery aneurysms. (Behrman, 823–826)

65. (D) This patient most likely has Langerhans cellhistiocytosis (LCH). Three classes of childhoodhistiocytosis are recognized. They all have incommon a prominent accumulation of cells of themonocyte-macrophage lineage. Accumulationcan occur in different organs. Eosinophilic

Page 39: Questions - uhs.edu.kh You suspect Lyme disease, most likely contracted by which of the following? (A) ingestion of unripe fruit (B) ingestion of spoiled fruit (C) drinking of contaminated

40 2: General Pediatrics

granuloma, Hand-Schüller-Christian disease,and Letterer-Siwe disease are all included inthe Class I histiocytoses and are commonlydescribed as LCH. LCH can present as local-ized or generalized disease. The skeleton isinvolved in 80% of the patients and bonylesions are most commonly seen in the skull.Chronically draining infected ears can occurwith destruction in the mastoid area. About50% of patients have skin involvement duringtheir course, which is often a seborrheic der-matitis of the scalp or diaper region. (Behrman1727–1730)

66. (E) Prader-Willi syndrome is characterized byhypotonia, hypogonadism, obesity, shortstature, mental retardation, and undescendedtestis in males. Although there may be feedingdifficulties in infancy, these children eventu-ally develop excessive appetites and obesity.The obesity accentuates the appearance of amicropenis. Hypotonia becomes less markedwith time. Hands and feet are generally verysmall. Microcephaly can be an occasionalabnormality. Approximately 70% of affectedindividuals have a microdeletion of the longarm of the paternal derived chromosome 15.The majority of the remainder are due to twomaternal copies of the long arm of chromo-some 15 and no paternal copy. (Jones, 223–227)

67. (B) This patient most likely has idiopathicthrombocytopenic purpura (ITP). ITP is the mostcommon cause of isolated thrombocytopenia inotherwise healthy children. At least half of thecases are preceded by a viral infection. Thereare no other abnormal physical findings besidesthe bleeding manifestations. For acute ITP thethree most common used therapeutics are pred-nisone, IVIG, and anti-d immunoglobulin.Platelet transfusions are only indicated in the rareinstance of life-threatening bleeding. (Buchanan,26:395–402, 2005)

68. (B) Stridor is a musical inspiratory sound of asingle pitch produced by narrowing of theextrathoracic airway. Laryngomalacia is themost common cause of persistent stridor ininfants, and is usually present by 6 weeks of

age. Laryngomalacia is caused by underdevel-opment of the cartilaginous support of thesupraglottic airway structures. It generally is abenign disorder, which resolves as the carti-laginous support develops, usually by the ageof 2 years. Diagnosis is confirmed by directvisualization of the larynx during an endo-scopic procedure. (Behrman, 1409–1410)

69. (D) Congenital adrenal hyperplasia (CAH) withsalt-losing disease should be considered in anyinfant with failure to thrive and ambiguousexternal genitals. CAH is a family of autosomalrecessive disorders of adrenal steroidogenesisthat lead to cortisol deficiency. Because of thecortisol deficiency, ACTH levels are increasedwith resulting adrenocortical hyperplasia andoverproduction of intermediary steroid metabo-lites. Deficiency of 21-hydroxylase accounts for90% of CAH cases. CAH can lead to a salt-losing disease such as should be suspected inthis patient. Laboratory studies would showhyponatremia and hyperkalemia. The diagno-sis can be confirmed by increased urinary secre-tion of 17-ketosteroids and increased plasmalevels of dehydroepiandrosterone sulfate(DHEAS). Glucocorticoids should be adminis-tered to inhibit the excessive production ofandrogens and patients with salt-losing diseasemay also require a mineralocorticoid. (Behrman,1909–1914)

70. (A) The differential diagnosis of hematuria inchildren includes glomerular diseases, extra-glomerular renal diseases, and various nonrenaldiseases. The glomerular diseases include acuteor chronic glomerulonephritis, IgAnephropathy,benign familial hematuria, Alport syndrome,systemic vasculitis, and hemolytic uremic syn-drome. Hypercalciuria can also cause hema-turia. Serum complement, both total and C3, isdecreased in conditions such as poststreptococcalglomerulonephritis, systemic lupus erythemato-sus, and bacterial endocarditis. Hematuria with apositive ASO and decreased C3 levels confirms adiagnosis of poststreptococcal glomerulonephritis.(Behrman, 1740–1741)

Page 40: Questions - uhs.edu.kh You suspect Lyme disease, most likely contracted by which of the following? (A) ingestion of unripe fruit (B) ingestion of spoiled fruit (C) drinking of contaminated

Answers: 66–75 41

71. (C) The patient described has typical findingsof Turner syndrome. Half of the patients withTurner syndrome only have one X chromosome(45,X). The other half of patients have a varyingamount of mosaicism. At birth these infantsare often small with low birth weight and havecharacteristic edema of the dorsa of the handsand feet and loose skin folds at the nape of theneck. If the diagnosis is not made during child-hood, these girls will come to medical attentionat puberty because sexual maturation fails tooccur. In the absence of one X chromosome,the death of oocytes occurs much more rap-idly and nearly all oocytes will be gone by theage of 2 years. The ovaries eventually aredescribed as “streaks” and consist mostly of con-nective tissue. Associated defects with Turnersyndrome are common. About 30% of patientshave cardiac anomolies, most often bicuspedaortic valves. Renal malformations and autoim-mune thyroid disease are also commonly found.Increased incidence of inflammatory bowel dis-ease and gastrointestinal bleeding have beendescribed as well. (Behrman, 1931–1934)

72. (E) The patient has key diagnostic criteria ofanorexia nervosa as pulished in the DSM-IV.These criteria include fear of becoming obese,disturbance in body image perception, bodyweight greater than or equal to 15% belowexpected, and in females the absence of at leastthree consecutive menstrual cycles. An estimated0.5% of adolescents and young adults haveanorexia nervosa based on DSM-IV criteria.Disturbances in almost any organ can be seen.Congestive heart failure, electrolyte distur-bances, and cardiac arrhythmias contribute tothe acute mortality, which is about 10%. Allthese complications are reversible with appro-priate management. Amenorrhea can play amajor role in the development of osteopeniaand osteoporosis and this is the one complicationthat can have long-term irreversible implications.(Fisher, 27:5–14, 2006)

73. (D) Posterior urethral valves are the mostcommon cause of severe obstructive uropathyin newborns. They only occur in boys and inabout 30% of patients will lead to end-stage

renal disease or chronic renal insufficiency. Ifthe obstruction is very severe, the diagnosis issometimes made in utero and these infantscarry the worst prognosis because they oftenhave associated oligohydramnios and pul-monary hypoplasia. Urethral strictures inmales are usually the result of trauma and theyare exceptional in females. Anterior urethralvalves are very rare. Meatal stenosis is anacquired condition that can occur after cir-cumcision and almost never leads to obstruc-tive uropathy. (Behrman, 1802–1803)

74. (B) The neonate most likely has erythema tox-icum which is a very common, benign, self-limited eruption. It occurs in approximately50% of full-term infants and preterm infantsare affected less frequently. Lesions may besparse or numerous and can involve the entireskin but palms and soles are usually spared.The peak incidence occurs on the second day oflife, but new lesions may erupt during the firstseveral days of life. The cause of erythema tox-icum is unknown. The course is brief and notreatment is necessary. The pustules representcollections of eosinophils in the stratumcorneum or deeper in the epidermis. Culturesof the pustules are sterile. (Behrman, 2162)

75. (A) This patient most likely has osteogenesisimperfecta (OI) type I. OI is the most commongenetic cause of osteoporosis and is causedby defects in type I collagen. There are fourrecognized forms of OI. Type I is the mildest andcan be found in large pedigrees such as in thisclinical scenario. Blue sclerae, recurrent fracturesin childhood, and presenile hearing loss(30%–60%) may occur in multiple family mem-bers. Seizures, mental retardation, and hydro-cephalus are not part of this disorder. Thesepatients have mild short stature compared tononaffected family members. Type II is muchmore severe and most infants are stillborn or diein the first year of life. Type III is the most severe,nonlethal form and results in significant physicaldisability. These patients have extreme shortstature. Type IV is moderately severe and maypresent at birth with in utero fractures. Type IVpatients have moderate short stature. Patients

Page 41: Questions - uhs.edu.kh You suspect Lyme disease, most likely contracted by which of the following? (A) ingestion of unripe fruit (B) ingestion of spoiled fruit (C) drinking of contaminated

42 2: General Pediatrics

with Type II OI often die in early life and have areduced life span because of cardiopulmonarycomplications. Patients with Type I and Type IVOI have a normal life span. (Behrman, 2336–2338)

76. (D) The infant presented most likely has pyloricstenosis. Pyloric stenosis is four times ascommon in boys as in girls. There is a geneticpredisposition; infants of mothers with pyloricstenosis have a much higher incidence ofpyloric stenosis than the general population. Itoften presents as nonbilious vomiting in thefirst weeks of life. The vomiting may becomeprojectile and classically a hypochloremic meta-bolic alkalosis will develop. The infants gener-ally appear hungry and experienced examinerscan feel the thickened pyloric muscle (olive) inmost of the cases. After the alkalosis has beencorrected, pyloromyotomy can be performed.Elevated urine organic acids can be found incertain metabolic disorders such as organicacidemias, but they would not lead to alkalosis.Measurement of serum or urine 17-hydroxyprogesterone would be helpful to diagnoseadrenal insufficiency. Adrenal insufficiency maycause persistent vomiting but would not causealkalosis. Children with gastroenteritis typicallyhave diarrhea and are more prone to develop ametabolic acidosis. (Behrman, 1229–1231)

77. (B) The infant most likely has prune-belly syn-drome, also known as Eagle-Barrett triad. Thissyndrome occurs mostly in males; only 5%of the patients are females. It is characterizedby a deficiency in abdominal muscles, unde-scended testes, and urinary tract abnormali-ties. Imperforate anus, mental retardation,congenital heart disease, and congenital agan-glionic megacolon (Hirschprung disease) arenot associated with this syndrome. (Behrman,1801–1802)

78. (C) This patient most likely has retinoblastoma.Retinoblastoma is the most common primarymalignant intraocular tumor of childhood andis rarely diagnosed beyond the age of 2 years.Bilateral involvement is evident in 20%–30%of patients. Approximately 30% of retinoblas-tomas are hereditary. Patients with hereditaryretinoblastoma have a significantly increased

risk of developing osteosarcoma. This is thoughtto be related to loss of heterozygosity of the Rbgene which is a tumor suppressor gene.(Behrman, 1722–1723)

79. (A) The infant most likely suffers from botu-lism. Botulism is an acute flaccid paralysis causedby the neurotoxin produced by Clostridium botu-linum. Infant botulism is a life-threatening con-dition that often is misdiagnosed at onset. In themajority of cases, sepsis is the suspected diag-nosis at admission. Botulinum toxin is neuro-toxic; it binds irreversibly at cholinergic synapses,blocks the release of acetylcholine, and causesimpaired neuromuscular and autonomic trans-mission. Sensory nerves are not affected andbulbar involvement is severe. In infants, inhala-tion and subsequent swallowing of airborneclostridial spores most likely cause the diseaseand prevention is therefore difficult. The onlyavoidable source of botulinum spores is honey.Honey should not be given to children under theage of 1 year. Routine laboratory studies includ-ing cerebrospinal fluid analysis are normal.Nerve conduction velocity and sensory nervefunction are normal in botulism. The diagnosisis established by demonstrating the presence ofC botulinum organisms or toxin in stool. (Behrman947–950)

80. (C) Transient synovitis is the most commoncause for limping in a healthy child at this age.Transient synovitis classically occurs 1–2 weeksafter a nonspecific upper respiratory tract infec-tion. Transient synovitis is a diagnosis of exclu-sion. Osteomyelitis and septic arthritis mustbe excluded. Patients with transient synovitisare usually afebrile or may have a low-gradefever. Laboratory studies are usually normal,although a mild elevation in the erythrocytesedimentation rate may be seen. Ultrasound ofthe hip can demonstrate a hip joint effusion.Treatment of this disorder is symptomatic withnonsteroidal anti-inflammatory medications.(Behrman, 808–809)

81. (B) This child most likely has nonorganic (noknown medical condition) failure to thrive(FTT) which is far more common than organic(marked by an underlying medical condition)

Page 42: Questions - uhs.edu.kh You suspect Lyme disease, most likely contracted by which of the following? (A) ingestion of unripe fruit (B) ingestion of spoiled fruit (C) drinking of contaminated

Answers: 76–87 43

FTT. Nonorganic FTT is due to lack of adequatecaloric intake. Typically, there is an abnormal-ity in the infant–mother relationship. The best-offered option for this child with a weight of5.5 kg (far below the fifth percentile) is to hos-pitalize the infant. Unlimited amount of caloriesshould be given, weight should be followedclosely and the mother–child interaction shouldbe carefully observed. Typically, children withnonorganic FTT who are in a controlled healthyenvironment will gain more than 2 oz/dayfor the first week. It is reasonable and practi-cal to avoid laboratory studies as long as thechild is showing good weight gain with thisapproach. None of the other offered choicesare valid options at this point in time. (Behrman,133–134)

82. (E) The patient most likely has erythemanodosum, which is characterized by pretibialtender erythematous nodules. Common infec-tious diseases that have been associated witherythema nodosum are streptococcal pharyngi-tis, tuberculosis, and other infectious diseasesdue to Yersinia, histoplasmosis, and coccid-ioidomycosis. Other associated noninfectiousdiseases are inflammatory bowel disease, sar-coidosis, and spondyloarthropathy. (Behrman, 794)

83. (E) The patient most likely had a breath-holding spell. These are always provoked byupsetting or scolding a child. The peak age foroccurrence is 2 years. The child typically has abrief shrill followed by apnea, cyanosis, andloss of consciousness. Posturing and general-ized clonic jerks may occur during an episode.The clinical setting and the occurrence of apneaand cyanosis usually differentiate breath-holding spells from seizure disorders. The bestapproach is to do a thorough examination fol-lowed by an explanation to the parents of themechanism of breath-holding spells. Breath-holding spells can recur and it is important forparents not to reinforce the child’s behavior.(Behrman, 2010)

84. (B) Patellofemoral pain syndrome is the mostcommon cause of chronic anterior knee pain.As in this patient, it typically worsens upongoing up the stairs, after sitting for prolonged

periods, or after squatting or running. The find-ing of peripatellar tenderness on examinationconfirms the diagnosis. Intensity of treatmentis based on the severity of the symptoms andis focused on improving strength and flexi-bility of the vastus medialis muscle. (Behrman,2310–2311)

85. (C) Attention-deficit hyperactivity disorder(ADHD) is characterized by problems withtask attendance, motoric overactivity, andimpulsivity. According to DSM-IV criteria, thesymptoms must be evident before the age of7 years and must be present in at least two dif-ferent settings, such as school and home. Theymust also impair the child’s functioning. Theprevalence of comorbidities is high. Oppositionaland aggressive behaviors, anxiety, mood dis-orders, and concurrent learning difficulties aresome of the more common ones. Comorbiditiesdo not have to be present to meet the diagnos-tic criteria for ADHD. The presence of anotherchild in the family with ADHD or a history ofbirth trauma are not criteria of the diagnostics.(Behrman:107–110)

86. (B) There are many controversies around thesubject of circumcision in newborn boys. TheAmerican Academy of Pediatrics issued a cir-cumcision policy statement in 1999 and reaf-firmed the statement in 2005. The only solidevidence regarding the benefit of circumcisionis that urinary tract infections (UTI) are muchless common in circumcised infants than inuncircumcised infants. The increased risk forUTI in uncircumcised infants is primarily inthe first 6 months, although the risk remainsincreased at least through the age of 5 years.There is no solid evidence that circumcisionsignificantly lowers the risk for sexually trans-mitted diseases or penile cancer. Boys withhypospadias should not be circumcised. (Behrman,1814; American Academy of Pediatrics Task Force onCircumcision, 103:686–693, 1999)

87. (C) This patient has severe hypertension withsystolic blood pressure above the 99th per-centile for age. Youth with severe hypertensionneed to be restricted from competitive sportsand highly static activities until their hypertension

Page 43: Questions - uhs.edu.kh You suspect Lyme disease, most likely contracted by which of the following? (A) ingestion of unripe fruit (B) ingestion of spoiled fruit (C) drinking of contaminated

44 2: General Pediatrics

is under control and they have no evidence oftarget organ damage. Cardiovascular condi-tioning is less strenuous than competitive ath-letics and youth with severe hypertension mayparticipate in cardiovascular conditioning.Youth who have significant hypertension (95thto 98th percentile) but have no target organdamage may participate in competitive athletics.(American Academy of Pediatrics Committee on SportsMedicine and Fitness, 99:637–638, 1997)

88. (G) Supravalvular aortic stenosis is the leastcommon type of aortic stenosis. It may be spo-radic, familial, or associated with Williams syn-drome. Other features of Williams syndromeinclude mental retardation, elfin facies, and idio-pathic hypercalcemia of infancy. (Behrman, 1516)

89. (E) An infant born to a mother with lupus maydevelop the neonatal lupus syndrome. Associatedproblems are cutaneous lesions, liver disease,thrombocytopenia, neutropenia, pulmonarydisease, neurologic disease, and congenitalheart block. Except for the congenital heartblock which often requires cardiac pacing,these associated problems usually resolve with-out intervention. (Behrman, 813)

90. (H) Noonan syndrome is characterized by shortstature, webbing of the neck, pectus excava-tum, characteristic facies, and cardiac defects,most commonly pulmonary valvular stenosis.(Behrman, 1513, 1925, 1935)

91. (D) Kawasaki disease leads to aneurysms ofthe coronary or systemic arteries in up to 25%of untreated patients. Three to four percent ofpatients with Kawasaki disease, treated withintravenous gammaglobulin, will developaneurysms. (Behrman, 1591)

92. (C) Clinical features of Turner syndromeinclude edema of the dorsa of the hands andfeet, webbing of the neck, low posterior hair-line, broad chest, and short stature. Sexual mat-uration fails to occur at the expected age. Aboutone-third of the girls with Turner syndromehave bicuspid aortic valves. (Behrman, 1931–1934)

93. (F) Classical features of Marfan syndromeinclude tall stature, ocular abnormalities, arachn-odactyly, and scoliosis. Morbidity of Marfan syn-drome is substantial; about 80%–100% of patientswill develop aortic root dilatation. (Behrman,2338–2340)

94. (E), 95. (C), 96. (F), 97. (F), 98. (C) Sitting with-out support is on average mastered at 6 monthsof life. Generally a 3-month-old infant can bringthe hands together in the midline. An average8-month-old child can bang two cubes and willhave a thumb–finger grasp. The Moro reflex istypically absent by 12 weeks of life. (Behrman,33–34)

99. (B) In idiopathic thrombocytopenic purpura(ITP) the platelet count is decreased, usuallybelow 60,000/mm3, while the remainder of theblood count is normal unless there has beensignificant bleeding, in which case the Hb maybe decreased and the reticulocyte countincreased. Most pediatric cases of ITP occur inthe first 5–6 years of life. (Behrman, 1670–1671)

100. (A) An Hb concentration of 12 g/dL and aWBC count of 11,500/mm3 are within statisti-cal “normal” limits for a 2-year-old child (mean12.5; 10th percentile 11.5). (Behrman, 1605)

101. (E) In children with sickle cell disease the Hb isusually between 7 and 10 g/dL, and the WBCcount is between 15,000 and 25,000/mm3. Thereticulocyte count is increased except in thepresence of an aplastic crisis, when the Hb will belower with low reticulocyte count. (Behrman, 1605)

102. (D) In the presence of iron deficiency anemia, thereticulocyte count typically is low. Hemoglobinvalues of 5 g/dL are not uncommon, and valuesas low as 2 g/dL are seen occasionally in verysevere cases. Striking increases in plateletcounts have been noted in children with irondeficiency anemia. (Behrman, 1614–1616)

103. (C) Acute lymphoblastic leukemia (ALL) oftenpresents with anemia and thrombocytopenia.The total WBC count may be increased, normal,

Page 44: Questions - uhs.edu.kh You suspect Lyme disease, most likely contracted by which of the following? (A) ingestion of unripe fruit (B) ingestion of spoiled fruit (C) drinking of contaminated

Answers: 88–110 45

or decreased. In about 10% of the cases, theperipheral WBC count is below 3000/mm3 atthe time of presentation. A more definitivediagnosis of ALL in the absence of increasedWBC count and absence of lymphoblasts canonly be made by examination of the bonemarrow. (Behrman, 1694–1696)

104. (C) As the name implies, in pancytopenia, allblood elements are quantitatively diminished.The low reticulocyte count reflects failure ofthe bone marrow, which is the usual cause ofpancytopenia. (Behrman, 1642–1646)

105. (E) Meconium ileus is intestinal obstruction inthe newborn caused by impacted meconiumin the small bowel, usually the ileum. The con-dition is essentially always associated withcystic fibrosis. In this disease, meconium isabnormally thick and sticky, partly because ofabnormal glycoproteins and partly because ofpancreatic insufficiency, and accumulates inthe intestinal lumen producing bowel obstruc-tion even before birth. About 10%–15% ofinfants with cystic fibrosis have meconiumileus at birth. (Behrman, 1440)

106. (D) Malrotation of the intestines is an abnor-mality that results from a failure of counter-clockwise rotation of the fetal intestine as itreturns to the abdominal cavity at about week10 of gestation. The condition often is associ-ated with duodenal obstruction secondary toconstricting peritoneal bands. However, in asmall percentage of cases, obstruction resultsfrom a midgut volvulus. This is a potentiallydevastating complication in which the mobile,malrotated bowel twists about the superiormesenteric artery. Infarction and necrosis ofmajor segments of bowel may occur. Midgutvolvulus is a surgical emergency. (Behrman, 1235)

107. (A) Enterocolitis is a well-recognized compli-cation of congenital aganglionic megacolon(Hirschsprung disease) that occurs primarilyin undiagnosed or inadequately managedpatients. Severe recurrent diarrhea secondaryto enterocolitis may be the presenting complaint

in a young infant with congenital megacolon.Recognition of the underlying abnormality isimportant, as the mortality of secondary ente-rocolitis (toxic megacolon) can be quite high.(Behrman, 1239–1240)

108. (H) Neonatal herpes virus infections are usu-ally acquired at the time of delivery and manyof the mothers do not have evidence of genitalherpes lesions. Fifty percent of infants born tomothers who have their primary infection andare shedding virus at the time of delivery, willbecome infected compared to less than 5% ofbabies born to mothers with recurrent genitalherpes infection. The infection usually mani-fests within the first month of life. Three majorcategories exist. The localized skin, eye, andmouth infection is present in about 30%–40% ofinfants at onset. Isolated CNS infection anddisseminated infection are the other two cate-gories. The morbidity and mortality are high,especially with the systemic form of infections.A high index of suspicion for herpes shouldarise when dealing with infants and sepsisunresponsive to antibiotics. (Behrman, 1053–1054)

109. (A) Cytomegalovirus (CMV) is the most commonidentified etiologic agent in congenital infections.Only 5% of infants with congenital CMV have thefull spectrum of hepatosplenomegaly, jaundice,petechia, purpura, and microcephaly. The major-ity of patients are born with subclinical but chronicCMV infections. The most common sequel of con-genital CMV infection is sensorineural hearingloss. (Behrman, 1067–1068)

110. (E) Congenital infection with Toxoplasma gondiican range from asymptomatic infection to severeneonatal disease. As with the other congenitalinfections, patients can have hepatosplenomegaly,thrombocytopenia, and CNS involvement.Intracranial calcifications with congenital tox-oplasmosis are usually diffuse. A unique prob-lem with congenital toxoplasmosis is theoccurrence of hydrocephalus in some infants.(Behrman, 1147–1150)

Page 45: Questions - uhs.edu.kh You suspect Lyme disease, most likely contracted by which of the following? (A) ingestion of unripe fruit (B) ingestion of spoiled fruit (C) drinking of contaminated

46 2: General Pediatrics

111. (G) The early manifestations of congenitalsyphilis can involve many different organ sys-tems. Late manifestations are the consequenceof chronic inflammation of bone, teeth, andcentral nervous system. Hutchinson teeth andsaddle nose are some of the characteristic find-ings that occur at later age. (Behrman, 978–981)

112. (B) Most of the stigmata of congenital varicellasyndrome can be attributed to viral-inducednervous system injury. Stigmata involve mainlythe skin, extremities, eyes, and brain. Extremitiesmay be shortened and malformed, often coveredwith zigzag scarring (cicatrix). (Behrman, 1059)

Page 46: Questions - uhs.edu.kh You suspect Lyme disease, most likely contracted by which of the following? (A) ingestion of unripe fruit (B) ingestion of spoiled fruit (C) drinking of contaminated

CHAPTER 3

The NeonateEugenia K. Pallotto, MD

The neonatal period is defined as the period of life from birth until 4 weeks (28 days) of age. This isa vulnerable period of time in the life of a human since the transition from fetal to extrauterine life involvesmany complex biochemical and physiologic processes. The newborn infant must adapt from dependence onthe maternal placental circulation to self-sufficient functioning of all physiologic systems.

There are many unique disease processes that present during this time period along with a unique set ofnormal laboratory and physiologic parameters. Many problems of the newborn infant are a result of abnormalitiesin the transition to extrauterine life either due to prematurity, the effects of an immature immunological system,congenital anomalies, or adverse effects of the events surrounding delivery. Understanding of the uniquephysiology and pathophysiology associated with this period of life is imperative for the physician to be ableto appropriately diagnose and treat the newborn infant.

47

Page 47: Questions - uhs.edu.kh You suspect Lyme disease, most likely contracted by which of the following? (A) ingestion of unripe fruit (B) ingestion of spoiled fruit (C) drinking of contaminated

48

DIRECTIONS (Questions 1 through 88): For eachof the multiple choice questions in this sectionselect the one lettered answer that is the bestresponse in each case.

1. An infant is diagnosed with a given disorderbelow. Which of these poses the greatest recur-rence risk for this patient’s future siblings?

(A) Hirschsprung disease(B) cystic fibrosis(C) ventricular septal defect(D) trisomy 21(E) trisomy 13

2. A term infant with microcephaly, jaundice, andthrombocytopenia is thought to have congeni-tal CMV infection. Your attending physiciannotes though that 1.5% of all newborns mayhave asymptomatic congenital cytomegalovirusinfection. Which of the following is the mostcommonly reported sequelae of such infections?

(A) chorioretinitis(B) sensorineural hearing loss(C) thrombocytopenia(D) poor growth(E) liver failure

3. A 2-year-old infant has acquired sensorineuralhearing loss. His mother is asking what themost likely cause could be. What is the leastlikely etiology of acquired sensorineural hear-ing loss in a toddler?

(A) hyperbilirubinemia(B) congenital infection(C) aminoglycoside therapy(D) neuroblastoma(E) late-onset group B streptococcal meningitis

4. The pediatric surgeon is requesting an echocar-diogram on a hospitalized newborn with a con-genital defect of the gastrointestinal tract.Which of the following defects has the highestincidence of associated cardiac defects?

(A) omphalocele(B) congenital volvulus(C) Hirschsprung disease(D) gastroschisis(E) pyloric stenosis

5. You are a practicing pediatrician in a statewhere cystic fibrosis is not a routine part ofnewborn screening. Which of the followingsymptoms in a newborn infant would promptyou to test for cystic fibrosis?

(A) pneumonia(B) intrauterine growth retardation(C) meconium ileus(D) wheezing(E) hypochloremic alkalosis

6. An infant is born precipitously to a motherwithout prenatal care and you are requested todetermine the infant’s gestational age. Of thefollowing, which physical finding is mostindicative of a full-term infant?

Questions

Page 48: Questions - uhs.edu.kh You suspect Lyme disease, most likely contracted by which of the following? (A) ingestion of unripe fruit (B) ingestion of spoiled fruit (C) drinking of contaminated

Questions: 1–12 49

(A) veins and tributaries are seen over theabdomen

(B) long lanugo is present on the back(C) palpable breast tissue of less than 1 cm(D) pitting edema over the tibia(E) soft ear pinnae, easily folded

7. A 1-week-old term newborn is in your office fora well-child assessment, physical examinationfindings are consistent with oral candidiasis(thrush). Which of the following is a correctstatement regarding thrush in a term newborn?

(A) responds well to topical therapy withnystatin

(B) requires systemic therapy withamphotericin B

(C) requires both topical (nystatin) and sys-temic (amphotericin) therapy

(D) requires investigation to rule out 22q11deletion syndrome

(E) requires no treatment

8. During the delivery room resuscitation of avigorous term newborn, which of the followingshould be performed first?

(A) Verify the airway is clear, dry, and stim-ulate the infant.

(B) The heart rate should be auscultated.(C) Breath sounds should be auscultated.(D) The mouth and trachea should be

suctioned.(E) Assess color and administer oxygen if

necessary.

9. A low-risk newborn infant has pathologicunconjugated hyperbilirubinemia, which isappopriately diagnosed within the first fewhours of life due to the astute observation ofrapid, progressive jaundice by the nurse. She isasking why the infant was not jaundiced imme-diately after birth. Which of the following bestdescribes the major route for excretion of biliru-bin in the fetus in utero?

(A) via the kidney(B) transplacental passage(C) degradation to biliverdin

(D) reincorporation into hemoglobin(E) hepatic secretion and storage in the

intestinal lumen

10. You are the attending physician for a newborninfant with hemolytic jaundice. The mother didnot receive prenatal care with this pregnancy orher prior pregnancy. The direct Coombs test ispositive. The mother’s blood type is A- and thebaby’s blood type is O+. Her first baby didnot have hemolytic jaundice. What is the mostlikely cause of the hemolytic jaundice?

(A) ABO incompatibility(B) Toxoplasmosis(C) Rh incompatibility(D) rubella(E) hereditary spherocytosis

11. The blood bank has received an order for anintrauterine transfusion. A fetus with which ofthe following would most likely require trans-fusion prior to birth?

(A) erythroblastosis fetalis(B) sickle cell anemia(C) spherocytosis(D) fetal distress and bradycardia(E) congenital heart disease

12. The obstetrician performing the intrauterinetransfusion is counseling the mother regard-ing complications of this procedure. Which ofthe following is the most common complicationof intrauterine transfusion?

(A) a transfusion reaction (mismatch)(B) graft-versus-host reaction(C) premature onset of labor(D) acquired immunodeficiency syndrome

(AIDS)(E) renal failure

Page 49: Questions - uhs.edu.kh You suspect Lyme disease, most likely contracted by which of the following? (A) ingestion of unripe fruit (B) ingestion of spoiled fruit (C) drinking of contaminated

50 3: The Neonate

13. During a routine prenatal visit a mother statesshe has been reading about kernicterus. She isvery concerned that her infant will developkernicterus if she provides breast milk for theinfant since she has also read that some breast-fed infants have high bilirubin levels. Which ofthe following is most predictive for the devel-opment of kernicterus?

(A) hyperbilirubinemia within the first24 hours of life

(B) peak conjugated bilirubin level(C) peak unconjugated bilirubin level(D) duration of hyperbilirubinemia(E) hemoglobin level immediately after birth

14. A medical student in your office is assessing a1-week-old term infant. The mother has beenexclusively breast-feeding and the student isnot sure how to interpret the change in theinfant’s weight compared to the birth weight.Which of the following is the expected weightflux for an infant in the first week of life?

(A) gain approximately 30 g/day(B) gain approximately 60 g/day(C) neither gain nor lose weight(D) lose approximately 5%–10% of its birth

weight(E) lose approximately 15% of its birth weight

15. A 3-day-old infant is requiring phototherapyfor hyperbilirubinemia. Which of the followingrisk factors best predicts the occurrence of ABOisoimmune hemolytic disease in a newborn?

(A) first pregnancy(B) more than four pregnancies(C) prior Rh disease(D) maternal blood type is O and infant is

type A(E) preexisting maternal anemia

16. The patient in Question 15 was hospitalizedfor 2 weeks, treated with phototherapy for12 days, and required a red cell transfusionduring the hospitalization. Which of the followingis the most common serious late clinical mani-festation of ABO disease?

(A) kernicterus(B) congestive heart failure(C) gallstones(D) bilirubinuria(E) iron deficiency

17. Prevention of bilirubin encephalopathy or ker-nicterus is one of the goals for the appropriatediagnosis and treatment of hyperbilirubine-mia. Which of the following mechanisms has arole in preventing these adverse outcomes?

(A) Unconjugated bilirubin is not lipid soluble.(B) Unconjugated bilirubin is tightly bound

to albumin.(C) Unconjugated bilirubin is tightly bound

to hemoglobin.(D) The blood–brain barrier is impermeable

to unconjugated bilirubin.(E) Unconjugated bilirubin is rapidly

metabolized by cerebrospinal fluid.

18. You are formulating a differential diagnosis fora newborn infant with respiratory distress.Which of the following is most closely associ-ated with the development of neonatal respi-ratory distress syndrome (hyaline membranedisease)?

(A) gestational age(B) birth weight(C) cesarean section delivery(D) maternal diabetes(E) meconium in the amniotic fluid

19. Which of the following statements is trueregarding infants of comparable weight andgestational age (> 1500 g) in the United Statesregarding mortality rate?

(A) There is no difference in mortality ratesbetween males and females and AfricanAmericans and Caucasians.

(B) Males have a higher mortality rate thanfemales, and African Americans have ahigher mortality rate than Caucasians.

(C) Males have a lower mortality rate thanfemales, and African Americans have alower mortality rate than Caucasians.

Page 50: Questions - uhs.edu.kh You suspect Lyme disease, most likely contracted by which of the following? (A) ingestion of unripe fruit (B) ingestion of spoiled fruit (C) drinking of contaminated

Questions: 13–25 51

(D) Males have a higher mortality rate thanfemales, and African Americans have alower mortality rate than Caucasians.

(E) Males have a lower mortality rate thanfemales, and African Americans have ahigher mortality rate than Caucasians.

20. A 29-week-gestation infant is being resusci-tated in the delivery room. Surfactant is giventhrough the endotracheal tube. Which of thefollowing is the most physiologically activecomponent of surfactant?

(A) surfactant protein A(B) surfactant protein B(C) neutral lipid(D) water(E) phospholipid

21. A25-week-gestation infant is born to a 25-year-oldprimigravida, who has had preeclampsia.Which of the following is a true statementdescribing the neonatal mortality rate for thisinfant?

(A) It decreases with increasing gestationalage from 30 weeks through 43 completedweeks of gestation.

(B) It is not related to birth weight.(C) It is not related to race.(D) It is higher than the mortality rates of

adolescents.(E) It has not changed significantly since

1980.

22. During the resuscitation of a preterm infant, amedical student in the delivery room is askingwhy surfactant is so important to neonatal lungphysiology. Which of the following is true ofsurfactant production as it relates to respiratorydistress syndrome in a premature infant?

(A) Surfactant is synthesized and stored inthe type I alveolar cells.

(B) Surfactant is synthesized and stored inthe type II alveolar cells.

(C) Surfactant is produced by the pulmonaryalveolar macrophages.

(D) Surfactant is stored in the interstitialspaces in the lungs.

(E) Surfactant is not produced until afterlabor ensues.

23. A mother presents in active labor, she did notreceive prenatal care and is unsure when herlast menstrual period was. Using ultrasound,the estimated gestational age is 30 weeks.Which of the following best describes the aver-age birth weight of a 30-week gestation infant?

(A) 500 g(B) 1000 g(C) 1500 g(D) 2000 g(E) 2500 g

24. A mother presents for prenatal care with a com-plicated medical history as listed in the followingchoices. Which of the conditions most predis-poses her fetus to congenital heart disease?

(A) hypertension(B) diabetes mellitus(C) atherosclerotic coronary vascular disease(D) anemia(E) rheumatoid arthritis

25. On a discharge examination you hear a heartmurmer and consult the cardiologist. Anechocardiogram was notable for a small ven-tricular septal defect and a patent foramenovale. You are notifying the parents of theresults and explain to them the role of the fora-men ovale in fetal life. Which of the followingstatements are true?

(A) Blood flows through the foramen ovalefrom the right ventricle to the left ventricle.

(B) Blood flows through the foramen ovalefrom the left ventricle to the right ventricle.

(C) Blood flows through the foramen ovalefrom the left atrium to the right atrium.

(D) Blood flows through the foramen ovalefrom the right atrium to the left atrium.

(E) Blood must pass through the foramenovale for blood to enter the right atriumfrom the umbilical vein.

Page 51: Questions - uhs.edu.kh You suspect Lyme disease, most likely contracted by which of the following? (A) ingestion of unripe fruit (B) ingestion of spoiled fruit (C) drinking of contaminated

52 3: The Neonate

26. A full-term newborn has a diffuse rash on theday of anticipated discharge. You diagnose ery-thema toxicum and must discuss this findingwith the parents. Which of the following is true?

(A) more common among term than prema-ture infants

(B) usually associated with fever and a gen-eral toxic state

(C) uncommon before the fifth day of life(D) usually associated with an elevated

peripheral white blood cell count(E) manifested in less than 10% of newborns

27. Amother is hospitalized with high blood pressueat 35 weeks gestation. The perinatologist recom-mends obtaining a lecithin–sphingomyelin ratioof the amniotic fluid to aid in the decision fordelivery. For which of the following systemsdoes the lecithin–sphingomeylin ratio indicatematurity?

(A) central nervous system(B) lungs(C) liver(D) kidneys(E) immunologic system

28. A hospitalized neonate is given the diagnosis ofbronchopulmonary dysplasia (BPD). Which ofthe following best describes the pathophysiol-ogy of BPD?

(A) An inflammatory insult to the lungs latein fetal development.

(B) Failure of development of pulmonaryarterioles during early fetal life.

(C) Failure of development of the bronchialbuds during early fetal life.

(D) Intrauterine viral infection.(E) The use of oxygen and positive-pressure

breathing in the treatment of respiratorydistress syndrome.

29. A 32-week-gestation infant is now 24 hoursold and has had progressive respiratory dis-tress. Given the infant’s clinical course and theradiographic appearance of the lungs the deci-sion has been made, in consultation with the

neonatologist, to give surfactant replacementtherapy for respiratory distress syndrome.Which of the following is true?

(A) is considered experimental(B) is only useful in infants with birth

weight less than 1500 g(C) has no known complications(D) has not been shown to reduce mortality

in very low birth weight infants(E) requires tracheal intubation to administer

30. You are examining the chest x-ray of a 4-hour-old infant born at 30 weeks gestation. Theinfant is breathing 100 times per minute whilebreathing 100% oxygen and intubation is immi-nent. You suspect respiratory distress syndrome.Which of the following is the characteristicroentgenographic findings of the infant withrespiratory distress syndrome?

(A) lobar atelectasis and interstitial edema(B) bilateral patchy densities and pneu-

mothorax(C) diffuse reticulogranular changes and air

bronchograms(D) diffuse hyperaeration and cardiomegaly(E) cardiomegaly and interstitial edema

31. After surfactant therapy, the infant with respi-ratory distress syndrome is treated with con-tinuous positive airway pressure. Which of thefollowing is the major goal of continuous pos-itive airway pressure?

(A) prevent infection(B) prevent pneumothorax(C) improve cardiac output(D) raise arterial Po2

(E) raise arterial Pco2

32. A 30-week-gestation infant with respiratorydistress syndrome is weaning from the venti-lator. He currently has an oxygen saturation of100% while breathing 50% oxygen on minimalventilator settings. In a premature infant withrespiratory distress syndrome, which of the fol-lowing may be an adverse effect of supple-mental oxygen therapy?

Page 52: Questions - uhs.edu.kh You suspect Lyme disease, most likely contracted by which of the following? (A) ingestion of unripe fruit (B) ingestion of spoiled fruit (C) drinking of contaminated

Questions: 26–38 53

(A) alveolar proteinosis(B) atelectasis(C) fire or explosion(D) kernicterus(E) retinopathy of prematurity

33. The father of a 26-week-gestation infant, whorequired intubation at birth, is asking why theinfant’s lungs were not mature. Which of thefollowing is the pathophysiology mechanismof respiratory distress syndrome in the prema-ture infant?

(A) increased production of pulmonarysurfactant

(B) decreased production of pulmonarysurfactant

(C) increased metabolism of pulmonarysurfactant

(D) decreased metabolism of pulmonarysurfactant

(E) rerouting of pulmonary surfactant to thesystemic circulation

34. A healthy term infant is circumcised and expe-riences excessive blood loss eventually requir-ing transfusion. The most likely diagnosis iswhich of the following?

(A) factor IX deficiency(B) factor VIII deficiency(C) von Willebrand disease(D) disseminated intravascular coagulopathy(E) protein C deficiency

35. An irregular red reflex is noted on the initialexamination of an infant. The infant is referred tothe opthalmologist for evaluation of a cataract.Which of the following is most likely to be asso-ciated with a cataract in the newborn?

(A) maple syrup urine disease(B) glucose-6-phosphate dehydrogenase

deficiency(C) phenylketonuria(D) galactosemia(E) propionic acidemia

36. A mother has brought her infant into youroffice for the first newborn visit. She is veryconcerned about a pigmented skin lesion thatwas not discussed with her in the hospital.Which of the following is true of pigmentedlesions known as slate gray spots?

(A) They never occur in white infants.(B) They are identified in over 40% of

African American infants.(C) They consist of small, well-demarcated

lesions approximately 2 mm in diameter.(D) Malignant degeneration is common.(E) The most common site of occurrence is

the nape of the neck.

37. A newborn infant presents with cyanosis andmild tachypnea at about 6 hours of life. Theinfant is placed in 95% oxyhood and satura-tions normalize. Which of the following is themost likely diagnosis in this infant?

(A) cyanotic congenital heart disease(B) lung disease(C) central nervous system disease(D) liver disease(E) methemoglobinemia

38. A35-week-gestation infant is delivered weighing3.9 kg, with an omphalocele and a large tongue.No other abnormalities are detected. Which ofthe following is the most likely diagnosis?

(A) congenital hypothyroidism(B) trisomy 18(C) trisomy 13(D) fetal alcohol syndrome(E) Beckwith-Wiedemann syndrome

Page 53: Questions - uhs.edu.kh You suspect Lyme disease, most likely contracted by which of the following? (A) ingestion of unripe fruit (B) ingestion of spoiled fruit (C) drinking of contaminated

54 3: The Neonate

39. A 3-week-old infant is noted to have micro-cephaly, cerebral calcifications on skull x-ray,and blindness. Which of the following is themost likely cause of these findings?

(A) bilateral subdural hemorrhages(B) cerebral agenesis(C) cytomegalovirus infection(D) erythroblastosis(E) primary microcephaly

40. A 7-day-old infant develops white, cheesypatches on the tongue and buccal mucosa withmild inflammation of the mucosa. Which ofthe following organisms is most likely thecause of these oral lesions?

(A) Candida albicans(B) Listeria monocytogenes(C) Escherichia coli(D) group A streptococcus(E) group B streptococcus

41. You are interviewing a family with a newborninfant. They are new to your practice and youelicit a family history of seizures in both themother and maternal grandfather. Duringexamination of the infant you note a skin find-ing that may be associated with the family his-tory of seizures. Which of the following is themost likely skin finding in this infant?

(A) hypopigmented patch(B) harlequin color change(C) salmon patch on the nasal glabella(D) hemangioma of the thigh(E) pustular melanosis

42. A term, otherwise healthy, neonate has isolatedpremature synostosis of the sagittal suture.Which of the following is the most likely asso-ciated condition?

(A) scaphocephaly(B) increased intracranial pressure(C) microcephaly(D) hydrocephalus(E) subdural effusions

43. A healthy newborn is noted to have numerous3 mm vesicles on the chest and neck, alongwith several similar-sized hyperpigmentedmacules in the same distribution. Which of thefollowing is the most likely diagnosis?

(A) mucocutaneous herpes simplex infection(B) acne neonatorum(C) erythema toxicum(D) incontinentia pigmenti(E) transient neonatal pustular melanosis

44. A newborn infant gags and chokes with feed-ings. He has just had an apneic spell during afeed resulting in significant cyanosis. Which ofthe following is the most likely diagnosis?

(A) proximal esophageal atresia(B) hypoplastic left heart syndrome(C) group B streptococcal sepsis(D) neonatal herpes simplex infection(E) pyloric stenosis

45. As a medical student in the newborn nursery,you correctly identify a scalp swelling as acephalohemtoma. Your attending physician thenasks you to describe how you would differenti-ate this from caput succedaneum. Which of thefollowing best describes the way to differentiatea cephalohematoma from caput succedaneum?

Figure 3-1(Courtesy of Angela Myers, MD)

Page 54: Questions - uhs.edu.kh You suspect Lyme disease, most likely contracted by which of the following? (A) ingestion of unripe fruit (B) ingestion of spoiled fruit (C) drinking of contaminated

Questions: 39–51 55

(A) absence of a history of prolonged or dif-ficult labor

(B) limitation of swelling to the area overone bone

(C) a normal neurologic examination(D) a prolonged prothrombin time(E) a normal lumbar puncture

46. A 40-year-old couple is in your office forgenetic counseling prior to having their firstchild. They are very concerned about advanc-ing maternal age and the possible genetic prob-lems it could cause for their child. Which ofthe following is the most likely disorder asso-ciated with advancing maternal age?

(A) autosomal recessive disorders(B) nondisjunction chromosome disorders(C) autosomal dominant disorders(D) X-linked disorders(E) inborn errors of metabolism

47. The same couple in Question 46 also is inquiringabout the genetic affects of the father’s age. Whichof the following is the most important disorderassociated with advancing paternal age?

(A) autosomal recessive disorders(B) nondisjunction chromosome disorders(C) autosomal dominant disorders(D) X-linked disorders(E) inborn errors of metabolism

48. A normal newborn is screened for hypo-glycemia after birth as per the normal nurseryprotocol at your hospital. In the asymptomaticterm neonate, evaluation and treatment ofhypoglycemia should be initiated when theglucose level is at or below which of the fol-lowing levels?

(A) 10 mg/dL(B) 20 mg/dL(C) 40 mg/dL(D) 60 mg/dL(E) 80 mg/dL

49. A nurse working in the newborn nursery callsyour office because she is concerned about theappearance of the feet of a newborn infant justdelivered. Based on her description, the infanthas congenital clubfoot. Which of the followingsystems is most commonly also involved withthis disorder?

(A) the central nervous system(B) the hematopoietic system(C) the gastrointestinal system(D) the cardiovascular system(E) the respiratory system

50. A newborn infant has micrognathia, glossop-tosis, and cleft soft palate. These findings areconsistent with Pierre Robin sequence. Whichof the following life-threatening events is asso-ciated with these findings?

(A) heart failure(B) seizures(C) intestinal obstruction(D) metabolic acidosis(E) upper airway obstruction

51. A consultant in the neonatal intensive care unitis recommending a trial of pyridoxine for apatient. Which of the following problems in anewborn infant might respond to a pharmaco-logic dose of pyridoxine?

(A) blindness(B) seizures(C) jaundice(D) rash(E) urinary retention

Page 55: Questions - uhs.edu.kh You suspect Lyme disease, most likely contracted by which of the following? (A) ingestion of unripe fruit (B) ingestion of spoiled fruit (C) drinking of contaminated

56 3: The Neonate

52. The geneticist is evaluating a patient in thenewborn intensive care unit. The possibility ofa disorder with a mitochondrial inheritancepattern has been discussed with the family.Which of the following is true of mitochondrialinheritance?

(A) These disorders commonly follow apaternal line of inheritance.

(B) These disorders have not been identifiedin humans thus far.

(C) Only females are affected.(D) Organ systems with low energy

demands are the most affected.(E) These disorders commonly follow a

maternal line of inheritance.

53. A family with an infant in the neonatal inten-sive care unit is very concerned that their childwill have long-term neurologic abnormalities.Of the following, which correlates best withsubsequent neurologic abnormalities?

(A) fetal bradycardia(B) failure to breathe at birth(C) a low 1-minute Apgar score(D) a low 5-minute Apgar score(E) seizures in the first 36 hours of life

54. An 8-day-old infant develops inflammatorypapules and pustules on the forehead, nose,and malar areas of the face. The child is other-wise well, and the remainder of the physicalexamination is normal. Which of the follow-ing is the most likely diagnosis?

(A) congenital syphilis(B) impetigo(C) neonatal acne(D) staphylococcal pustulosis(E) tuberous sclerosis

55. You are counseling a mother with an abnor-mal serum alpha-fetoprotein test during afollow-up prenatal visit. The maternal serumalpha-fetoprotein test is most useful in diag-nosing which of the following?

(A) duodenal atresia(B) clubfoot

(C) cleft lip and palate(D) myelomeningocele(E) alveolar proteinosis

56. A newborn with trisomy 21 develops biliousemesis on the first day of life. An abdominalx-ray reveals a “double bubble sign.” Which ofthe following is the most likely diagnosis?

(A) annular pancreas(B) duodenal atresia(C) gastric volvulus(D) pyloric stenosis(E) Hirschsprung disease

57. A 1700-g infant was asphyxiated at birth, aftera successful resuscitation, the infant hadnumerous apneic episodes. On the third day oflife, the infant began to vomit. Abdominal dis-tention and bloody stools were noted. Which ofthe following is the most likely diagnosis?

(A) congenital aganglionic megacolon(B) intussusception(C) necrotizing enterocolitis(D) Shigella enteritis(E) volvulus

58. A 2-month-old infant is in your office for awell-child examination. The mother is askingyou about a red macule on the infant’s cheek.She describes it as getting larger over the pastfew weeks. You suspect this is a capillary hema-gioma. The natural history of an elevated capil-lary or cavernous hemangioma is best describedby which of the following statements?

(A) no significant change in size after birth.(B) an increase in size during the first few

years after birth and then regression.(C) an increase in size during the first decade

of life and then no further change.(D) a slow but progressive increase in size

throughout life.(E) a slow but progressive decrease in size

starting shortly after birth.

Page 56: Questions - uhs.edu.kh You suspect Lyme disease, most likely contracted by which of the following? (A) ingestion of unripe fruit (B) ingestion of spoiled fruit (C) drinking of contaminated

Questions: 52–65 57

59. A full-term, ill appearing, infant presents withvesicles and large bullae shortly at birth. Whichof the following is most likely?

(A) herpes simplex virus(B) gram-negative infection(C) epidermolysis bullosa(D) Staphylococcus aureus(E) maternal diabetes

60. A newborn infant presents with skin lesionstypical of erythema toxicum and you wish toconfirm the diagnosis. Wright stain of the con-tents from a lesion of erythema toxicum usuallywill reveal which of the following cells?

(A) basophils(B) eosinophils(C) lymphocytes(D) immature lymphocytes(E) polymorphonuclear leukocytes

61. A mother with hyperparathyroidism just deliv-ered a term infant who appears healthy. Whichof the following problems is likely to developin her infant?

(A) hypercalcemia(B) hypocalcemia(C) parathyroid carcinoma(D) hyperthyroidism(E) hyperparathyroidism

62. A screening bilirubin test is sent on a term new-born who appears jaundiced at 72 hours of life.You unexpectedly identify conjugated hyper-bilirubinemia. Which of the following is asso-ciated with conjugated hyperbilirubinemia?

(A) sequestrated blood(B) tyrosinemia(C) hereditary spherocytosis(D) Rh incompatibility(E) physiologic jaundice

63. A newborn infant is being evaluated forambiguous genitalia. While awaiting results ofchromosome analysis, the infant developshyponatremia, hypoglycemia, and low bloodpressure. What is the most likely diagnosis?

(A) 21-hydroxylase deficiency(B) severe hypospadius(C) 3-beta-hydroxysteroid dehydrogenase

deficiency(D) 11-beta-hydroxylase deficiency(E) 17-alpha-hydroxylase deficiency

64. You are evaluating a small-for-gestational-ageinfant for tachycardia. There is no prenatal carehistory available, but based on the infant’ssigns and symptoms and the obstetrician’sdescription of the mother you suspect neonatalthyrotoxicosis. Thyrotoxicosis in the first day oflife most likely occurs in an infant born to amother with which of the following diagnoses?

(A) with untreated hypothyroidism(B) with untreated Graves disease(C) with Graves disease being treated with

antithyroid medications(D) with euthyroid goiter(E) receiving iodides as therapy for chronic

bronchitis

65. During a prenatal care visit, a mother notifiesyou that her partner has been diagnosed withherpes simplex virus (HSV). She is askingabout risks for the infant if she has HSV. Whichof the following is the most important risk fac-tors for neonatal HSV infection?

(A) prematurity(B) cold sore on mother’s lip at time of

delivery(C) primary maternal genital infection dur-

ing pregnancy(D) fetal scalp monitoring(E) rupture of membranes greater than

4–6 hours

Page 57: Questions - uhs.edu.kh You suspect Lyme disease, most likely contracted by which of the following? (A) ingestion of unripe fruit (B) ingestion of spoiled fruit (C) drinking of contaminated

58 3: The Neonate

66. As a medical student in the neonatal intensivecare unit, you report a calcium level in a 1-day-old term newborn that was flagged as abnor-mal by the laboratory. Which of the followingis true of serum calcium levels in the newborn?

(A) High parathyroid hormone (PTH) levelssuppress the serum calcium.

(B) The serum calcium promptly decreasesafter delivery.

(C) The serum calcium remains essentiallyunchanged over the first 3 days.

(D) The serum calcium is usually higher onday 2 than day 1.

(E) Infants of diabetic mothers have anaccentuated PTH response resulting inhypercalcemia.

67. A mother of a 3-day-old newborn, born athome with a midwife, is calling your officebecause she is concerned about constipation.You are deciding whether to evaluate thisinfant for Hirschsprung disease and ask herabout meconium passage. Meconium passagedelayed beyond which of the following timepoints to the diagnosis of Hirshsprung disease?

(A) during the process of birth(B) within a few minutes of birth(C) 2–4 hours of life(D) 12–18 hours of life(E) 24–36 hours of life

68. On the fifth day of life, an infant is noted tohave a violaceous, circumscribed, subcutaneousnodule immediately beneath fading forcepsmarks on one cheek. The infant is otherwisewell appearing. Which of the following is themost likely diagnosis?

(A) an abscess(B) a hemangioma(C) a pericytoma(D) periorbital cellulitis caused by

Haemophilus influenzae(E) subcutaneous fat necrosis

69. Your office is notified by the state screeningprogram that a newborn in your practice is pre-sumped positive for hypothyroidism. Whichof the following is true of congenital hypothy-roidism?

(A) Affected infants are usually clinicallyapparent by the third day of life.

(B) Affected infants have unusually smallfontanelles.

(C) Affected infants appear thin.(D) Prolonged hyperbilirubinemia is common.(E) Affected infants usually have a goiter

present at birth.

70. Iron deficiency is the leading cause of anemia ininfancy and childhood. Which of the followinginfants is at greatest risk of this early on?

(A) a premature infant(B) an infant with ABO incompatibility(C) an infant with physiologic hyperbiliru-

binemia(D) a postmature infant(E) an infant with polycythemia

71. A term infant has respiratory distress and ablood count reveals a hematocrit of 65%. Yourresident states that fetal hemoglobin has anincreased affinity for oxygen as compared withadult hemoglobin. What effect might thisresult in?

(A) an increased mass of fetal hemoglobin(B) thin red cell membrane in the fetus(C) the presence of unconjugated bilirubin

in the red cell(D) decreased binding with 2,3-diphospho-

glycerate (2,3-DPG)(E) intracellular alkalosis

72. A term infant was born with the anomalyshown in Figure 3-2. Which of the following isthe most likely cause of this infant’s masslesion?

Page 58: Questions - uhs.edu.kh You suspect Lyme disease, most likely contracted by which of the following? (A) ingestion of unripe fruit (B) ingestion of spoiled fruit (C) drinking of contaminated

Questions: 66–75 59

(A) scalp abscess(B) posterior hydrocephalus(C) encephalocele(D) myelomeningocele(E) hydrocele

73. An infant born with malformed forearms isnoted to develop severe hemorrhagic manifes-tations. Which of the following would be themost likely cause of the bleeding problem?

(A) vitamin K deficiency(B) factor VIII deficiency(C) thrombocytopenia(D) disseminated intravascular coagulation(E) factor IX deficiency

74. You are a medical student evaluating a jaun-diced infant in clinic. The infant appearshealthy and has been entirely breast-fed. Thetotal bilirubin level is 19 mg/dL, direct biliru-bin is 0.2 mg/dL, and the hemoglobin isnormal. You have diagnosed “breast milk jaun-dice.” Which of the following is the most typicalfeature of breast milk jaundice?

(A) It is an unconjugated hyperbilirubinemia.(B) It usually appears within the first 12 hours

of life.(C) The elevated levels may persist for more

than 60 days.(D) It is generally associated with moderate

anemia and reticulocytosis.(E) It is uncommon in Caucasian infants.

75. You are counseling a diabetic teenager whostates she is trying to get pregnant. She has notbeen compliant with her diabetes managementover the past few years. You are trying to stressthe importance of glucose control prior to con-ception and during pregnancy by explainingthe possible complications associated withinfants born to diabetic mothers. Which of thefollowing is the most common problem ininfants of diabetic mothers?

(A) birth asphyxia(B) sacral agenesis(C) hyponatremia(D) hypoglycemia(E) cardiac septal hypertrophy

Figure 3-2(Courtesy of Clarence Greene Jr., MD)

Figure 3-3(Courtesy of Doug C. Rivard, DO)

Page 59: Questions - uhs.edu.kh You suspect Lyme disease, most likely contracted by which of the following? (A) ingestion of unripe fruit (B) ingestion of spoiled fruit (C) drinking of contaminated

60 3: The Neonate

76. A 12-month-old infant with a cloudy cornea isdiagnosed with primary infantile glaucoma, arare disorder occurring in 1:10,000 live births.You explain to the parents that it can be a majorcause of blindness. Which of the following state-ments regarding infantile glaucoma is true?

(A) It is best treated with medicalmanagement.

(B) It is frequently seen with trisomy 21.(C) It presents with the white pupil sign.(D) It is rarely diagnosed before 6 months of

age.(E) It is associated with congenital rubella

and neurofibromatosis.

77. A premature infant is now 4 hours old and yoususpect respiratory distress syndrome (hyalinemembrane disease). Which of the following isthe least likely first clinical presentation of res-piratory distress syndrome?

(A) tachypnea(B) delayed capillary refill(C) cyanosis(D) wheezing(E) pallor

78. Fetal heart rate monitoring is being used duringthe labor of a 40-year-old primagravida who isnow 1 week post dates. Which of the followingwould most commonly be seen in a healthy baby?

(A) heart rate 60–80 beats per minute(B) accelerations of the heart rate(C) heart rate below 60 beats per minute(D) decreased beat-to-beat variability in

heart rate(E) fixed heart rate

79. You are the resident on call and are calledurgently to the bedside of a newborn. The nurseis concerned that the infant has developed atension pneumothorax. Which of the followingclinical signs most likely represent those seen ina newborn with tension pneumothorax?

(A) cyanosis, apnea, and tachycardia(B) apnea, hypertension, and bradycardia(C) tachypnea, cyanosis, and bradycardia

(D) wheezing, tachycardia, and hypertension(E) wheezing, hypotension, and apnea

80. You are evaluating an infant born at 39 weeksgestation weighing 2000 g. Which of the fol-lowing would be the appropriate classificationof this infant?

(A) low birth weight(B) premature(C) small for gestational age(D) low birth weight and small for gesta-

tional age(E) premature and small for gestational age

81. A term infant is now 10 hours of age and has aseizure. Which of the following diagnoses ismore common among term rather than pre-mature infants?

(A) intraventricular hemorrhage(B) hemorrhagic disease of the newborn(C) sepsis(D) subdural hemorrhage(E) congenital infection

82. A term neonate begins vomiting during thefirst few days of life, and develops a distendedabdomen. The family history is positive forcystic fibrosis. Which of the following condi-tions, if found, would most likely be related tothe family history?

(A) annular pancreas(B) duodenal atresia(C) hypertrophic pyloric stenosis(D) meconium ileus(E) volvulus

83. An infant delivered precipitously to a motherwho had not received prenatal care. The infanthas finding suggestive of congenital syphilis.Maternal and infant testing confirm this diag-nosis. Which of the following is most sugges-tive of early congenital syphilis?

(A) disseminated intravascular coagulation(B) bullous lesions of the palms and soles(C) hepatitis

Page 60: Questions - uhs.edu.kh You suspect Lyme disease, most likely contracted by which of the following? (A) ingestion of unripe fruit (B) ingestion of spoiled fruit (C) drinking of contaminated

Questions: 76–93 61

(D) dermal erythropoiesis(E) pneumonia

84. Prior to the newborn screening program inyour state, you are evaluating an infant with askeletal survey due to concerns of trauma.Which of the following roentgenographic find-ings in a newborn infant is most suggestive ofundiagosed hypothyroidism?

(A) epiphyseal dysgenesis(B) absence of ossification of the hamate bone(C) prominent thymic shadow(D) osteoporosis(E) cardiomegaly

85. A 5-day-old appropriate-for-gestational agemale presents with tachypnea, poor feeding,and lethargy. On examination, the neonateappears in shock with hypotension, pallor, andpoor capillary refill. Among the following,which is the most likely diagnosis?

(A) tetrology of Fallot(B) tricuspid atresia(C) transposition of the great vessels(D) truncus arteriosus(E) hypoplastic left heart syndrome

86. Incontinentia pigmenti (IP) is an X-linkedhereditary disorder and is typically lethal inmales. Skin lesions may be present at birth.Which of the following best describe the initiallesions of incontinentia pigmenti?

(A) hypopigmented(B) scaly(C) waxy papules(D) inflammatory bullae(E) small vesicles

87. A newborn infant is noted to have a peculiarface with low-set ears and widely spaced eyeswith epicanthic folds. The chin is receeding, andthe nose is broad and flat. Oligohydramnios wasnoted prior to delivery. Examination of the pla-cental membranes reveals amnion nodosum.Which of the following diagnoses should besuspected?

(A) bilateral renal agenesis(B) atrioventricular canal defect(C) esophageal atresia(D) group B streptococcal infection(E) transposition of the great vessels

88. Hydrocephalus, chorioretinitis, and diffuse cere-bral calcifications are present in a newborn male.Of the following pets, which is most likely to bethe source of this zoonotic congenital infection?

(A) dog(B) cat(C) horse(D) rabbit(E) gerbil

DIRECTIONS (Questions 89 through 93): Thefollowing group of questions is preceded by a listof lettered answer options. For each question,match the one lettered option that is most closelyassociated with the question. Each lettered optionmay be selected once, multiple times, or not at all.

(A) trisomy 13 (Patau syndrome)(B) trisomy 18 (Edwards syndrome)(C) 5p deletion (cri-du-chat syndrome)(D) trisomy 21 (Down syndrome)(E) 45,X or XO (Turner syndrome)

89. Low-set ears, nail hypoplasia, “rocker-bottom”feet, growth retardation, severe mental retardation

90. Holoprosencephaly, microphthalmia or anoph-thalmia, cleft lip and palate, polydactyly, severemental retardation

91. Hypotonia and hyperextensible joints, clin-odactyly, Brushfield spots, duodenal atresia,flattening of the occiput

92. Microcephaly, mental retardation, hyper-telorism, and cat-like (mewing) cry ininfancy

93. Redundant skin at nape of neck, low posteriorhairline, edema of dorsum of feet in newbornperiod, cardiac and renal anomalies

Page 61: Questions - uhs.edu.kh You suspect Lyme disease, most likely contracted by which of the following? (A) ingestion of unripe fruit (B) ingestion of spoiled fruit (C) drinking of contaminated

62 3: The Neonate

DIRECTIONS (Questions 94 through 104): For eachof the multiple choice questions in this section selectthe one lettered answer that is the best response ineach case.

94. A 42-week gestation neonate is born bycesarean delivery following repeated fetal heartrate decelerations. At rupture of membranes,thick meconium was noted. The infant has norespiratory effort but slight movement and aheart rate of 90 beats per minute. Of the fol-lowing, which is the first intervention?

(A) pass an umbilical artery catheter tomeasure pH and Po2

(B) start bag-and-mask ventilation with100% oxygen

(C) suction the mouth and trachea toremove the meconium

(D) intubate the trachea and begin ventila-tion with 100% oxygen

(E) establish monitoring with ECG andpulse oximeter

95. An ultrasound at 33 weeks gestation reveals asmall-for-gestational-age fetus with no otherabnormalities. At birth at 39 weeks, the 1.6-kgneonate requires no resuscitation. On examina-tion, the infant is noted to be microcephalic andhave a generalized rash consisting of dozens ofblue-purple macules. His abdominal examina-tion reveals hepatosplenomegaly. Which of thefollowing is the most likely diagnosis?

(A) group B streptococcal septicemia(B) isoimmune thrombocytopenia(C) congenital cytomegalovirus infection(D) congenital toxoplasmosis(E) congenital L monocytogenes infection

96. A term infant at 12 hours of age is normalexcept for jaundice. Initial laboratory valuesreveal a total bilirubin of 12 mg/dL and severalspherocytes on the peripheral blood smear.Among the following, which is the most likelydiagnosis?

(A) hereditary elliptocytosis(B) hereditary spherocytosis

(C) glucose-6-phosphate dehydrogenasedeficiency

(D) disseminated intravascular coagulopathy(E) ABO isoimmune hemolytic disease

97. A 3.5-kg term male delivered by uncomplicatedvaginal delivery has a normal physical exami-nation. On the day of discharge the mother tellsyou that an older sibling has just broken out withchicken pox. Which of the following is true?

(A) All newborns are very susceptible tovaricella upon exposure.

(B) This infant is likely already infectedbecause of the maternal exposure to thesick sibling.

(C) Varicella immune globulin shouldbe administered to all infants in thissituation.

(D) If the mother had varicella in the past,the infant is likely protected with pas-sively acquired antibody from themother.

(E) The infant will not develop a typicalvaricella infection, but will likelydevelop varicella zoster.

98. A previously healthy term infant is brought toclinic by his mother at 2 weeks of age becauseof rapid breathing, poor breastfeeding, andexcessive sleeping. He is afebrile. Which of thefollowing is the most common arrhythmia atthis age?

(A) supraventricular tachycardia(B) atrial flutter(C) atrial fibrillation(D) complete atrioventricular block(E) ventricular tachycardia

99. A 1-hour-old, 33-week gestation male neonatedevelops tachypnea, grunting respirations, anddeep intercostal retractions. A chest x-rayreveals a homogeneously grainy appearanceto both lung fields. An arterial blood gas wouldmost likely reveal which of the following?

(A) respiratory alkalosis and hypoxemia(B) respiratory acidosis and metabolic

alkalosis

Page 62: Questions - uhs.edu.kh You suspect Lyme disease, most likely contracted by which of the following? (A) ingestion of unripe fruit (B) ingestion of spoiled fruit (C) drinking of contaminated

Questions: 94–104 63

(C) respiratory acidosis and hypoxemia(D) respiratory and metabolic alkalosis(E) metabolic acidosis and respiratory

alkalosis

100. A college student delivers an infant in thecampus housing area after denying her preg-nancy and experiencing 6 hours of abdominalpain. Paramedics arrive and find an infant theyestimate to be 4–5 lb in weight, crying but slightlydusky. They administer oxygen. The newborn’stemperature is 94°F. Hypothermia in this infantwould produce which of the following?

(A) decreased oxygen consumption(B) hypertriglyceridemia(C) hypercalcemia(D) hypoglycemia(E) metabolic alkalosis

101. A 30-day-old, former 24-weeks gestation, 600 gneonate had a difficult initial respiratory coursecomplicated by a tension pneumothorax. Shehad serial head ultrasound evaluations duringthe first weeks of life. All previous studiesrevealed a normal immature brain. Now, thehead ultrasound reveals an abnormality. Amongthe following, which is most likely?

(A) grade II intraventricular hemorrhage(B) grade IV intraventricular hemorrhage(C) aqueductal stenosis(D) periventricular leukomalacia(E) vein of Galen aneurysm

102. A newborn infant has signs of congestive heartfailure. Physical examination does not reveal asignificant cardiac murmur. Auscultation of thehead reveals a loud cranial bruit. Which of thefollowing is the most likely diagnosis?

(A) polycythemia(B) hyperthyroidism(C) ruptured cerebral aneurysm(D) transposition of the great vessels(E) arteriovenous malformation of the great

vein of Galen

103. A term 4.3-kg infant is delivered vaginally to a33-year-old woman with juvenile-onset dia-betes. The delivery was complicated by severeshoulder dystocia and the infant experienced abrachial plexus injury with limited movementof the right arm. At 72 hours of age, the infantis noted to be tachypneic but he is pink andwell perfused. Which of the following is themost likely explanation for his tachypnea?

(A) respiratory distress syndrome (RDS)(B) diaphragmatic paralysis(C) pulmonary hemorrhage(D) pneumothorax(E) cystic adenomatoid malformation of the

lung

104. A term infant is delivered vaginally to an18-year-old primagravida after an uncompli-cated pregnancy. Mother presented with rup-ture of membranes and a low-grade fever.Labor and delivery proceeded rapidly, maternalantibiotics were not administered prior todelivery of the infant. Maternal screeningduring pregnancy for group B streptococcuswas negative. At 30 hours of age the infant ismottled, not feeding well, and cries whenhandled. A lumbar puncture is performed andthe spinal fluid reveals 660 WBC/mm3 and aCSF protein of 290 mg/dL. Which of the fol-lowing is the most likely agent causing thisinfant’s infection?

(A) group B streptococcus(B) Escherichia coli(C) Listeria monocytogenes(D) Haemophilus influenzae(E) Streptococcus pneumoniae

Page 63: Questions - uhs.edu.kh You suspect Lyme disease, most likely contracted by which of the following? (A) ingestion of unripe fruit (B) ingestion of spoiled fruit (C) drinking of contaminated

64

1. (B) Cystic fibrosis (CF) is inherited in an autoso-mal recessive fashion, creating a 25% recurrencerisk for subsequent pregnancies. Hirschsprungdisease usually is sporadic with a risk of about3%–5% to siblings of an affected case. There arerare families with Hirschsprung disease trans-mitted as an autosomal dominant or recessivepattern of inheritance. Ventricular septal defectsare also multifactorial in inheritance, the recur-rent risk for siblings of a patient with congenitalheart disease is 3%–4%. Trisomies are most typ-ically sporadic events and recurrence variesdepending on the mother’s age. A small pro-portion of trisomies are transmitted by a parentas part of a balanced translocation with higherrecurrence risks in those families. (Behrman 1239;Brodsky, 165)

2. (B) The majority (90%–95%) of newborn infantsinfected with cytomegalovirus are asympto-matic at birth. Asymptomatic congenitalcytomegalovirus infection is a leading cause ofsensorineural hearing loss, which can occur inapproximately 7% of infants. Hearing loss isoften progressive and may not be detected untilthe patient is older. Sudden loss of residual hear-ing at age 4–5 years has been associated withcongenital cytomegalovirus. (Behrman, 1067, 2129)

3. (D) Congenital infections with cytomegalovirus,toxoplasmosis, or syphilis may present withdelayed onset of sensorineural hearing loss.Hyperbilirubinemia at levels requiring exchangetransfusion is also associated with sensorineuralhearing loss. Aminoglycosides put an infant atincreased risk for hearing loss particularlywhen used in combination with other ototoxicdrugs or when levels are not appropriately

monitored with adequate dosing adjustmentsin response to these levels. Bacterial meningitisalso has a strong association with hearingimpairment. Although some cheomotherapeu-tic agents are associated with hearing loss, neu-roblastoma does not have a primary associationwith sensorineural hearing loss. (Behrman, 2130)

4. (A) Omphaloceles are among the most commonof the abdominal wall defects noted in the new-born and pediatricians must know how todiagnose and provide additional care. In addi-tion, they need to understand the associationwith other defects. Cardiovascular anomaliesoccur in 25%–30% of infants with omphaloceles.Omphaloceles also are frequently associatedwith chromosomal anomalies (50%) and other

Answers and Explanations

Figure 3-4(Courtesy of Emily A. Thorell, MD)

Page 64: Questions - uhs.edu.kh You suspect Lyme disease, most likely contracted by which of the following? (A) ingestion of unripe fruit (B) ingestion of spoiled fruit (C) drinking of contaminated

Answers: 1–9 65

syndromes including the Beckwith-Wiedemannsyndrome (omphalocele, severe hypoglycemia,macrosomia, and macroglossia). Gastroschisis isa defect of the abdominal wall which is oftenconfused with omphalocele. It is characterizedby a defect where the intestines are located outsideof the abdomen, and exposed to the air. Unlikeomphalocele, there is no sac over the intestines(see Figure 3-4).

The other lesions listed are not typically asso-ciated with chromosomal, syndromic, or othernongastrointestinal anomalies. (Brodsky, 273)

5. (C) The newborn with cystic fibrosis generallyis asymptomatic. The most common manifes-tation, meconium ileus, occurs in 15%–20% ofCF patients. The ileum is completely obstructedwith meconium, which results in intestinalobstruction. The meconium plug syndromealso is seen with an increased frequency ininfants with cystic fibrosis, but it is less specificthan meconium ileus. Sweat chloride testingin suspected cases of cystic fibrosis has alwaysbeen troublesome in the newborn period butDNA testing is now available and is 90%–95%sensitive. In the United States, some states haveinstituted routine newborn screening for cysticfibrosis. (Behrman, 1437)

6. (C) Breast hypertrophy is common in full-terminfants. The less mature the infant, the smallerthe amount of breast tissue. Long lanugo,translucent skin, and a soft ear pinna that doesnot readily recoil are indicative of prematurity.The Ballard scoring system uses physical find-ings such as these to determine the gestationalage of the newborn. Veins and tributaries overthe abdomen and pitting edema over the tibiaare associated with pathological conditions andare not used in gestational age assessment.(Ballard, 1991)

7. (A) Oral candidiasis (thrush) is a common prob-lem and typically does not indicate a seriousunderlying problem. It is seen frequently in thefirst few weeks of life. The primary means ofinfection in healthy newborns is vertical trans-mission from maternal vaginal moniliasis.It typically requires no immunologic investiga-tion and usually responds rapidly to treatment

with oral nystatin suspension. Treating possiblesources such as the thumb, the breast, bottlenipples, or pacifier is important to avoid recur-rence. If thrush occurs after infancy, or if it isatypically recurrenct and/or resistent, investi-gation of the immune system may be war-ranted. Immunodeficiency diseases that areassociated with T-cell disorders, such as 22q11deletion syndrome or HIV infection, should beconsidered. In the sick, premature infant, espe-cially those with central lines or other invasivedevices, oral thrush is more serious and mayherald systemic infection. (Burg, 150; Behrman,588)

8. (A) Resuscitation of the newborn is a system-atic procedure with a logical progression ofevents, which best ensures a successful out-come. The infant should be placed on awarmer, left uncovered, and positioned in the“sniffing” position. If no meconium is present,secretions should be removed from the mouthand then nose with a bulb syringe in order toclear the airway. Once the airway is clear,drying the infant thoroughly will provide stim-ulation for the infant and will prevent heat loss.Removal of the wet linen is an important stepto prevent further heat loss. Assessment ofheart rate, respirations, and color should not becompleted until these important first stepshave been carried out. Suction of the mouthand trachea may be indicated as a first step ifmeconium is present and the baby is not vigorous.(Kattwinkel, Lesson 2)

9. (B) Unconjugated bilirubin passes easily acrossthe placenta. This is the major mechanism forfetal elimination of bilirubin. Bilverdin reductaseconverts biliverdin to bilirubin. The placentadoes not eliminate biliverdin, thus there is a fetaladvantage to make bilirubin. During fetal life,normal bowel function is not established andthe hepatic conjugating system is somewhatdormant so these sytems do not play a promi-nent role in bilirubin removal. Unconjugatedbilirubin (more fat soluble and less water solu-ble than conjugated bilirubin) easily crosses theplacenta to the maternal circulation for dis-posal by the maternal liver. (Fanaroff, 1425;Brodsky, 301)

Page 65: Questions - uhs.edu.kh You suspect Lyme disease, most likely contracted by which of the following? (A) ingestion of unripe fruit (B) ingestion of spoiled fruit (C) drinking of contaminated

66 3: The Neonate

10. (C) Rh hemolytic disease rarely occurs in thefirst pregnancy, but transfusion of Rh+ bloodinto an Rh– mother sensitizes the mother result-ing initially in production of IgM antibodies,followed by IgG antibody production. Once amother is sensitized, the following pregnan-cies result in hemolytic disease. Sensitization isprevented by injection of anti-D gamma glob-ulin (RhoGAM) at 28 weeks’ gestation andimmediately after delivery. The incidence ofRh incompatibility is 11/10,000 total births.Prior to RhoGAM, it developed in 1% of preg-nancies. Hereditary spherocytosis and con-genital infections, causing hemolytic disease,are distinguished by negative Coombs testingas well as other clinical characteristic examfindings or peripheral smear findings. ABOincompatibility may occur in the first preg-nancy. Subsequent pregnancies are not moreseverely affected. It would not occur when thefetus is blood type O. (Brodsky, 289; Behrman, 601)

11. (A) Erythroblastosis fetalis is caused bytransplacental passage of maternal antibodyactive against RBC (red blood cell) antigens. It ischaracterized by an increased rate of RBCdestruction and is an important cause of new-born anemia and jaundice. The severe form isusually a result of Rh incompatibility. If severeanemia develops in utero this can result in fetalhydrops, fetal distress, and possibly fetal death.In severe cases, diagnosed before birth, intrauter-ine transfusion can improve survival. Thetransfusion is accomplished by injection ofdonor blood (packed red cells) into the fetalumbilical vein under ultrasound guidance.Although spherocytosis can cause neonataljaundice, neither spherocytosis nor sickle celldisease causes serious intrauterine problems.Intrauterine transfusion is not the treatmentfor fetal distress and bradycardia or congenitalheart disease. (Behrman, 601)

12. (C) Current technology, including the use ofultrasound monitoring during the procedure,has made intrauterine transfusion relativelysafe. Premature onset of labor is the major com-plication but fetal death, fetal bleeding, fetalbradycardia, premature rupture of membranes,and chorioamniotis are all risks of the procedure.

Graft-versus-host reaction has been reportedbut is rare, irradiation of the blood to kill thelymphocytes prior to transfusion should avoidthis. Testing of donor blood for human immun-odeficiency virus (HIV) antibody is routine andusually will prevent transfusion transmissionof AIDS. Transfusion reactions have not beenreported to be a problem, blood should becross-matched against mother’s blood to selectthe most compatible blood for transfusion.(Behrman, 601)

13. (C) Kernicterus is a condition of neurologicdamage that is the direct result of the toxiceffects of bilirubin on the central nervoussystem. Since unconjugated bilirubin crossesthe blood–brain barrier, whereas conjugatedbilirubin does not, the risk of kernicterus is mostclosely related to the serum level of unconju-gated bilirubin. The basal ganglia are particu-larly susceptible to damage with high levels ofunconjugated bilirubin. Although kernicterusis more frequently reported when other com-plicating conditions exist (eg, prematurity,sepsis, isoimmunization), it has also beenreported in healthy babies born at or near term.(Burg, 298)

14. (D) Infants are born in a state of relativeextravascular fluid expansion. Through the firstweek of life, dilute urine and relatively lowintake of fluid result in a loss of 5%–10% oftheir total body mass (up to 10%–15% in pre-mature infants). This weight loss is a normalphysiologic loss of fluid from the interstitialfluid space and is not a catabolism of body tis-sues. (Merenstein, 351)

15. (D) In most cases of ABO isoimmune hemolyticdisease of the newborn, the mother is type Oand the infant is type A or B. Mothers with A orB blood types produce mostly IgM antibodieswhich do not cross the placenta, whereas motherswith blood type O produce mostly IgG anti-bodies which cross the placenta. Since the IgGantibodies are “naturally” occurring, diseasemay occur in the first pregnancy. Hemolysiswith ABO incompatibility is more mild thanRh disease because A and B antigens areexpressed in all tissues of the body, including

Page 66: Questions - uhs.edu.kh You suspect Lyme disease, most likely contracted by which of the following? (A) ingestion of unripe fruit (B) ingestion of spoiled fruit (C) drinking of contaminated

Answers: 10–21 67

the placenta, therefore maternal antibody isdiluted and neutralized. Subsequent pregnan-cies with ABO incompatibility are not moreseverely affected. This differs from Rh diseasewhere IgG antibodies are made more swiftlywith repeat exposure and thus more severehemolysis is noted with repeat pregnancies.Late anemia can occur with ABO disease.(Brodsky, 289)

16. (B) Late-onset chronic anemia in ABO isoim-mune hemolytic disease is the most commonlate complication and usually presents as high-output congestive heart failure (CHF). The symp-toms of CHF in neonates are usually tachypneaand failure to feed. The hemoglobin will reach itsnadir at the sixth to twelfth week of life.Postdischarge montoring of hemoglobin andhematocrit is essential and late transfusion maybe needed. Kernicterus and gallstones areuncommon. Bilirubinuria is benign. (Behrman, 605)

17. (B) Unconjugated bilirubin in the serum istightly bound to albumin. The unconjugatedbilirubin molecule is lipid soluble, and thatwhich is not bound to albumin enters the brainreadily. Once the albumin binding sites are sat-urated, there is an increase in the potentiallytoxic, free unconjugated bilirubin. A bilirubin/albumin molar ratio of 1 corresponds to approx-imately 8.5 mg bilirubin per gram of albumin.Factors that are associated with low serumalbumin levels (eg, prematurity) and factorsthat impair bilirubin binding to albumin (eg,acidosis or drugs such as sulfa compounds)can result in kernicterus at lower serum biliru-bin levels. (Merenstein, 548)

18. (A) Prematurity is the strongest predisposingfactor to surfactant deficiency disease or respi-ratory distress syndrome (RDS). While lowbirth weight infants are also at risk, the risk ismore related to the gestational age of the infant.Maternal diabetes does predispose newbornsto RDS by causing delayed surfactant releasecompared to infants of comparable gestationalage, but is not as strong of a risk as prematurityitself. Cesarean delivery predisposes more to acondition known as transient tachypnea of thenewborn, related to excessive fluid retention

within the lungs. Meconium in the amnioticfluid may cause respiratory disease if aspiratedinto the infant’s lungs but the pathophysiologyof the disease differs from RDS. (Brodsky, 71)

19. (D) Males have a higher neonatal mortality ratethan females, both overall and when correctedfor weight and gestational age. African Americanshave a higher rate of prematurity and low birthweight and, therefore, have an overall higherneonatal mortality rate than Caucasians.Among infants of comparable weight and ges-tational age greater than 1500 g, however,African American infants have a lower mor-tality than do Caucasian infants. (Behrman, 52)

20. (E) Pulmonary surfactant is a hydrophobic sub-stance composed mostly of phospholipids(approximately 70%). The majority is DPPC ordipalmitoyl phosphatidylcholine (40%), whichis the most physiologically active component ofsurfactant and most responsible for reductionof surface tension. The total protein contri-bution is about 10% and neutral lipids about10%. The proteins appear to be important instructural lipid organization and, to somedegree, for regulation of surfactant production.(Brodsky, 47)

21. (D) Neonatal mortality is highest during thefirst 24 hours of life. Overall, neonatal mortal-ity accounts for 65% of all infant deaths (deathsbefore 1 year of age) and is higher than anyother period in childhood. Neonatal mortalityhas declined dramatically between 1980 and2000 to about 4.6 per 1000 live births. Improvedobstetric and neonatal management have con-tributed to this decline in mortality. The majorcauses of neonatal mortality are related topreterm birth, low birth weight, and lethal con-genital anomalies. While mortality decreasesas maturity and birth weight increase, infantswho are delivered after 42 weeks gestationhave an increased mortality rate compared totheir term counterparts. African Americannewborns still have about twice the infantmortality of Caucasians infants, mostly fromcomplications of preterm delivery. (Behrman,519, 641)

Page 67: Questions - uhs.edu.kh You suspect Lyme disease, most likely contracted by which of the following? (A) ingestion of unripe fruit (B) ingestion of spoiled fruit (C) drinking of contaminated

68 3: The Neonate

22. (B) Pulmonary surfactant is produced in a com-plex set of structuring steps involving the endo-plasmic reticulum and Golgi apparatus of thetype II alveolar cells of the lung. It is storedwithin these cells in structures known as lamel-lar bodies. Surfactant proteins are expressed inthe third trimester. Phosphatidylcholine andphosphatidylglycerol, the predominant lipidsin surfactant, increase with lung maturity.Typically, after 34–35 weeks gestation, they are atsufficient levels such that respiratory distress syn-drome will not be present, although this variesgreatly for individual patients. (Brodsky, 47)

23. (C) The normally grown 30-week gestationinfant weighs 1100–1800 g (mean +2 S.D.).(Merenstein, 391)

24. (B) Maternal diabetes mellitus during cardio-genesis results in an approximately threefoldincreased risk of congenital anomalies. Thisincludes cardiac malformations and lum-bosacral agenesis. The increased risk is associ-ated with poor maternal glucose control duringembryogenesis. In late gestation, poor maternalglucose control predisposes the infant to asy-metric septal hypertrophy which can lead toaortic outflow obstruction. (Behrman, 613)

25. (D) Blood returning from the placenta, to thefetus, via the umbilical vein courses throughthe ductus venosus. This blood with higheroxygen saturation mixes with the dexoy-genated blood returning from the fetus’ inferiorvena cava, enters the right atrium, and prefer-entially crosses the foramen ovale into the leftatrium. It is then pumped into the aorta by theleft ventricle. After birth, expansion of the lungsand an increased arterial Po2 results in a rapiddecrease in pulmonary vascular resistance.Simultaneously, there is an increase in systemicvascular resistance with removal of the low-resistance placental circulation. These eventsresult in a physiologic cessation of flow acrossthe foramen ovale. (Behrman, 1479)

26. (A) Erythema toxicum is a benign conditionseen in 30%–70% of normal, term infants. It ismuch less common among premature infants,particularly those weighing less than 2500 g.

The skin lesions usually appear within the first3 days of life and are not associated with anysystemic signs or symptoms. They typicallylast less than 7 days. Infants are afebrile, andthe peripheral white blood cell counts arenormal. The rash presents with one or more ofthree types of skin lesions including erythe-matous macules, wheals, and vesiculopustuleson an erythematous base. (Eichenfield, 91)

27. (B) The lecithin–sphingomyelin ratio of amni-otic fluid specimens reflects the compositionof fetal lung fluid, which is effluxed from thelungs into the amniotic sac. It is a useful indi-cator of lung maturity and risk of developmentof respiratory distress syndrome or hyalinemembrane disease. (Brodsky, 49)

28. (E) Some infants with severe respiratory distresssyndrome who require mechanical ventilationand high concentrations of oxygen developchronic pulmonary changes known as bron-chopulmonary dysplasia (BPD). (Behrman, 580)

29. (E) Surfactant replacement therapy is delivereddirectly into the lungs via an endotracheal tube.Multiple studies have documented the improve-ment in survival of infants with respiratory distresssyndrome, who receive surfactant replacementafter delivery. Common complications includetransient hypoxemia and bradycardia associatedwith the dosage procedure and air leak syn-dromes from undetected rapid improvement inpulmonary compliance. (Merenstein, 652)

30. (C) The characteristic chest x-ray findings ininfants with respiratory distress syndromeinclude reduced lung volumes, air bron-chograms, reticulogranularity, and lung opaci-fication. The x-ray appearance is frequentlycharacterized as having a homogenous groundglass appearance. These changes are presumedto represent diffuse alveolar atelectasis whichincreases lung density. Air-filled bronchi (ie, airbronchograms) then stand out against theopaque lung tissue. Lobar densities are infre-quent. Pneumothorax does occur as a complica-tion of the disease or its treatment, but not inassociation with patchy densities. (Merenstein, 650)

Page 68: Questions - uhs.edu.kh You suspect Lyme disease, most likely contracted by which of the following? (A) ingestion of unripe fruit (B) ingestion of spoiled fruit (C) drinking of contaminated

Answers: 22–37 69

31. (D) The purpose of continuous positive airwaypressure (CPAP) is to increase the functionalresidual capacity and improve oxygenation byimproving ventilation and perfusion mismatch.Improvement of ventilation (a decreased arterialCO2) and maintenance of an adequate cardiacoutput is desired when providing CPAP, butthese parameters may be adversely affected ifthe appropriate pressure is not provided for theindividual patient. Pneumothorax is one of thecomplications of treatment with continuous pos-itive airway pressure. There is no evidence thatcontinuous positive airway pressure preventsinfection. To the contrary, the invasive technol-ogy (eg, endotracheal tube) often required canpredispose to infection. (Merenstein, 603)

32. (E) Although the pathophysiologic process inthe development of retinopathy of prematu-rity (ROP) is not completely understood, thereare several factors that have been associatedwith this disease. Although hypoxia mayincrease the risk and severity of ROP, hyperoxiaand oxygen radicals also have an associationwith increased risk for this disease. Arterialoxygenation should be closely monitored inthe premature infant. It is clear that oxygenalone is not the only predisposing factor.Acidosis, apnea of prematurity, anemia, nutri-tional status, and alterations in cerebral perfu-sion are other factors that have been associatedwith ROP in some infants. (Merenstein, 641)

33. (B) A decrease in production of pulmonary sur-factant by alveolar cells has been demonstratedin the lungs of infants and experimental ani-mals with respiratory distress syndrome. Thebiologic function of surfactant is to lower thesurface tension of the alveolar lining and therebystabilize the alveoli. Absence of surfactant leadsto alveolar collapse (atelectasis), which in turncauses the pathophysiologic, clinical, androentgenographic changes characteristic of thisdisorder. (Behrman, 575)

34. (B) In the healthy term male infant, hemophiliaA(factor VIII deficiency) and hemophilia B (factorIX deficiency) are the most common severe inher-ited bleeding disorders. Hemophilia occurs in1:5000 males with factor VIII deficieny being the

most common (85%). Disseminated intravascularcoagulopathy (DIC) is unlikely in a healthyinfant. von Willebrand factor is high just afterbirth, making symptoms uncommon in the new-born. Protein C deficiency is associated withthrombosis not bleeding. (Note: Verification thatthere is no family history of bleeding problemsshould be done prior to any newborn circumci-sion. Once bleeding has begun after a fresh cir-cumcision in a patient with factor VIII deficiency,it is very difficult to control.) (Behrman, 1657–1665)

35. (D) Any opacity of the lens is a cataract. Thedifferential diagnosis for cataracts identified inthe neonatal period is wide and includes devel-opmental disorders, congenital infections, andmetabolic disorders. Galactosemia, a disorderof carbohydrate metabolism, should be a pri-mary consideration in a neonate presentingwith cataracts. (Behrman, 2105)

36. (B) Blue or slate-gray spots (historicallyreferred to as Mongolian spots) are macularlesions. They are most commonly found on thepresacral area, have variably defined margins,and can often be quite large. The incidence islow in Caucasian infants (< 10%), but they arepresent in more than 80% of African-American,Asian, and East Indian infants. They typicallyfade with time and malignant degenerationdoes not occur. Their presence and locationshould be documented as they may be mis-taken for bruising later in infancy. They arethought to be produced from melanocytes thathave arrested in their migration from neuralcrest to epidermis. (Behrman, 2162)

37. (B) Cyanosis in the newborn can be due to manycauses, but by far the most common cause is res-piratory disease. If cyanosis is present withoutrespiratory symptoms, such as tachypnea, onemust classically consider congenital heart dis-ease or the less commonly identified methe-moglobinemia. Cyanosis typically will notnormalize in the case of congenital heart dis-ease or methemoglobinemia. Central nervoussystem diease could also cause cyanosis, butthis is typically due to hypoventilation.(Behrman, 560)

Page 69: Questions - uhs.edu.kh You suspect Lyme disease, most likely contracted by which of the following? (A) ingestion of unripe fruit (B) ingestion of spoiled fruit (C) drinking of contaminated

70 3: The Neonate

38. (E) Beckwith-Wiedemann syndrome encom-passes abdominal wall defects (especiallyomphalocele), macroglossia, and macrosomia. Itsrecognition is critical in the immediate newbornperiod as about one-third to one-half of cases havehypoglycemia. Cases are usually sporadic, thegene is located at 11p15. The characteristic phe-notype occurs due to a variety of different geneticmechanisms, which all result in a dosage imbal-ance of a number of genes clustered at 11p15.Patients with this disorder have an increased riskof abdominal tumors (ie, hepatoblastoma andWilms tumor) during childhood. Regular screen-ing for these tumors during the early part of child-hood is warranted. (Jones, 175–177)

39. (C) The combination of microcephaly, cerebralcalcifications, and blindness is typical of thedamage caused to neural tissues by intrauter-ine infection with either cytomegalovirus ortoxoplasmosis. Subdural bleeding results inenlargement of the head. Cerebral agenesis isnot associated with calcifications, and ery-throblastosis would not explain any of the find-ings listed. Primary microcephaly is not associatedwith either cerebral calcifications or blindness.(Brodsky, 199–201)

40. (A) The findings described are typical of thrush(oral infection with C albicans), which iscommon in young infants. Although E coli,L monocytogenes, and group B streptococcusall are important pathogens in the neonatalperiod, they typically are not associated withpharyngeal infection or oral exudate. Group Astreptococcus is a common cause of exudativetonsillitis in the older child but is an extremelyrare pathogen in the newborn infant likely dueto maternally acquired antibody. Additionally,the exudate noted with group A streptococcalinfection would be in the area of the tonsilsrather than on the buccal mucosa. (Behrman, 870,1392; Brodsky, 204)

41. (A) Neurocutaneous syndromes are character-ized by both skin and central nervous systemabnormalities. They include a heterogenousgroup of disorders. Hypopigmented patchesobserved in the newborn period may be a signof neurofibromatosis or tuberous sclerosis and

would raise great suspicion if seen on the bodyof an infant with a family history of seizures. Theskin lesion of tuberous sclerosis has classicallybeen described as an ash leaf. Neurofibromatosisis likely to have pigmented cafe au lait spotsdevelop over time as well. Salmon patchesand pustular melanosis are normal, benignfindings in newborn infants. A harlequin colorchange, while striking, typically is harmless.(Behrman, 215)

42. (A) The cranium of the child grows at rightangles to several sutures, primarily the sagittal,coronal, lambdoidal, and temporal. Isolated pre-mature closure (craniosynostosis) of the sagittalsuture results in a long and narrow skull(scaphocephaly) with a slightly greater thannormal total circumference. With early fusionof a single suture, there usually are no signs ofincreased intracranial pressure, and the prob-lem is chiefly cosmetic. Craniosynostosis of mul-tiple sutures, however, often is associated withincreased intracranial pressure. (Behrman, 1992)

43. (E) Transient neonatal pustular melanosis is abenign lesion. It initially appears as a pustuleswithout erythema and evolve into ruptured pus-tules with scale surrounding a hyperpigmentedmacule. The hyperpigmented macules can lastup to 3 months. They typically cluster under thechin, forehead, neck, or lower back. Pustules mayhave already ruptured at the time of birth. Theyare somewhat more common in infants withgreater amounts of skin pigment. (Brodsky, 356)

44. (A) The most common congenital defect of theesophagus is atresia of the proximal segmentwith a fistula between the distal segment andthe trachea. This type of esophageal atresiaaccounts for 80%–90% of all cases. Attempts tofeed the infant will result in regurgitation andmay precipitate aspiration. Even prior to feed-ing, aspiration of gastric secretions may occurthrough the distal fistula and cause a damagingpneumonitis. Hypoplastic left heart syndromeand group B streptococcal sepsis are morelikely to present with apnea or cyanosis unre-lated to feeding. Disseminated neonatal herpessimplex infection similarly presents with asepsis-like picture typical after 4–7 days of age.

Page 70: Questions - uhs.edu.kh You suspect Lyme disease, most likely contracted by which of the following? (A) ingestion of unripe fruit (B) ingestion of spoiled fruit (C) drinking of contaminated

Answers: 38–52 71

Pyloric stenosis generally presents after the firstmonth of life with vomiting. (Behrman, 1219)

45. (B) Both cephalohematoma and caput are asso-ciated with prolonged and difficult labor anddelivery. They are also associated with anormal neurologic examination, unless therehas been concomittant intracranial trauma.Caput, however, is diffuse and poorly demar-cated edema of the scalp, whereas a cephalo-hematoma is a subperiosteal collection of blood.Therefore, a cephalohematoma is sharplydemarcated and limited to a single bone.Cephalohematomas may be bilateral, but each isseparate and sharply demarcated. (Brodsky, 143)

46. (B) There is a clear association between advanc-ing maternal age and nondisjunction chromo-some disorders (failure of a chromosome pairto separate). The rate of Down syndrome, themost common chromosome abnormality seenin newborns, increases with advancing mater-nal age from approximately 1/1500 at age 25 tomore than 1/100 by age 40. (Jones, 7–12)

47. (C) While the association of maternal age andchromosome problems is well known, a lesserdiscussed phenomenon is the clear relation-ship of autosomal dominant disorders andpaternal age at conception. The paternal agein disorders such as achondroplasia andMarfan syndrome, for example, is substantiallyhigher than the general population. It isthought that the accumulation of DNA replica-tive errors during spermatogenesis is respon-sible for this phenomenon. (Fanaroff, 122)

48. (C) There is not always a correlation betweenblood glucose levels and classic clinical manifes-tations of hypoglycemia in the neonate. The lackof symptoms does not indicate that the glucoselevel is adequate for brain metabolism. Althougha clear lower limit glucose level in newborninfants has not been determined, many authoritesare recommending that any blood glucosevalue less than 50 mg/dL in the neonate beevaluated and treated. (Behrman, 505)

49. (A) Congenital clubfoot (talipes equinovarus) isusually an isolated congenital anomaly. When

associated with another disorder, the centralnervous system is most commonly involved. Itis commonly associated with myelomeningo-cele. Genitourinary disorders resulting in lowamniotic fluid volumes may result in clubfootif the foot has been held in a deformed positionin utero. (Behrman, 1983, 2256)

50. (E) Hypoplasia of the mandibular area in earlygestation results in a posteriorly located tongueduring development and impaired closure ofposterior palate. The posterior displacement ofthe tongue can cause significant airway obstruc-tion, which is the most urgent problem encoun-tered in these infants with Pierre Robin sequence.While substantial spontaneous growth of themandible occurs in many of these patients withina few months, temporary tracheostomy maybe required in severe cases. Prone or partiallyprone positioning may relieve the obstructionin less severe cases. (Jones, 262; Behrman, 120;Tibesar, 27)

51. (B) So-called pyridoxine dependency is a groupof rare, inborn metabolic errors that causesevere neonatal seizures that respond to phar-macologic (as opposed to physiologic) doses ofpyridoxine. Although the disorder is extremelyrare, it has given rise to the practice of sometimesempirically administering an intravenous doseof pyridoxine to infants with unexplained andotherwise uncontrollable seizures. (Fanaroff, 1617)

52. (E) Mutations in the mitochondrial DNA canproduce specific diseases. In most cases, thefunctional mitochondria of both males andfemales are acquired from the mother. Themitochondrial complement passed in the eggmay not represent the total mitochondrial pop-ulation of the mother, thus there can be greatvariability in disease symptoms within afamily. As would be anticipated, disorders ofenergy metabolism are predominant in thisgroup of disorders. A myopathy or neurologicdisease coming from the maternal side shouldalert the clinician to consider a mitochondrialetiology. Paternal inheritance may occcur, butthis is uncommon. (Behrman, 380)

Page 71: Questions - uhs.edu.kh You suspect Lyme disease, most likely contracted by which of the following? (A) ingestion of unripe fruit (B) ingestion of spoiled fruit (C) drinking of contaminated

72 3: The Neonate

53. (E) Most infants who experience fetal brady-cardia during labor are not severely asphyxi-ated at birth and ultimately have a goodneurologic outcome. Likewise, most infantswho fail to breathe initially or are cyanotic atbirth and have a low 1-minute Apgar scorerecover promptly and do well neurologically.Although a low 5-minute Apgar score gener-ally indicates more prolonged oxygen prob-lems and correlates somewhat better withoutcome than does the 1-minute score, most ofthese infants also will be normal. In contrast, asubstantial population of newborns with lowApgar scores and seizures in the first 36 hoursof life will show neurologic abnormalities onlong-term follow-up. (Volpe, 439-442)

54. (C) The lesions described are typical for neona-tal acne (acne neonatorum), a benign skin erup-tion occurring during the first few weeks oflife, probably secondary to transplacental pas-sage of maternal hormones. Comedones arenot typically present with neonatal acne.Adenoma sebaceum, one of the several cuta-neous manifestations of tuberous sclerosis,typically is not present in the neonatal period.The distribution and decsription of the rashin this patient are not typical of congenitalsyphilis, impetigo, or pustulosis. (Eichenfield, 94,144–151)

55. (D) The maternal serum alpha-fetoprotein(MSAFP) test is used to screen for open neuraltube defects. About 90% of affected pregnanciescan be identified. High MSAFP levels are alsoseen in abdominal wall defects (eg, omphalo-cele) and twin gestation. Low MSAFP levelshave been associated with trisomy 18 and 21,incorrect gestational age assessment, andintrauterine growth restriction. Maternal serumtesting can be used in combination with ultra-sound and amniocentiesis to fine-tune riskassessments for these disorders. (Behrman, 535)

56. (B) Intestinal atresia accounts for 30%–40% ofall neonatal intestinal obstructions of whichduodenal atresia is most common. Bilious vom-iting without abdominal distention is the hall-mark of duodenal obstruction. The doublebubble appearance on abdominal radiograph is

due to a distended gas-filled stomach and prox-imal duodenum. Down syndrome is associatedwith all diseases listed except volvulus. Athoughannular pancreas does result in a duodenalobstruction, it is seen less commonly than duo-denal atresia. Vomiting due to pyloric stenosisand gastric volvulus is nonbilious and radiographdenotes a dilated stomach. Vomiting due topyloric stenosis often does not start until after3 weeks of life. Hirschsprung disease is a lowerintestinal obstruction and associated withdelayed passage of stools. (Behrman, 1229-1239;Jones, 8)

57. (C) Necrotizing enterocolitis (NEC) occurs fre-quently among low birth weight infants whohave had repeated episodes of hypoxia or poorperfusion. The usual signs of NEC are abdom-inal distention, bloody stools, vomiting,hypothermia, and lethargy. Intussusception israre in the neonatal period. Volvulus and con-genital aganglionic megacolon (Hirschsprungdisease) are unrelated to low birth weight orhypoxia and usually are associated with failureto pass stool. Shigella infection is very uncom-mon in the nursery. (Behrman, 590, 1239)

58. (B) Most capillary or cavernous hemangiomasare small or even invisible at birth. During thefirst weeks or months of life, they begin to growand may become very large before finally start-ing to regress after a few years. Spontaneousregression usually (but not invariably) is com-plete or nearly so. Recognition of the naturalhistory of this lesion permits reassurance ofparents and avoidance of unnecessary therapy.The best cosmetic results are achieved by nat-ural regression. Active intervention (surgeryor laser therapy) should be advised only whena complication such as trapping of platelets(Kassabach-Merritt syndrome), erosion oftissue, or impairment of a life-threatening func-tion (eg, vision, breathing) is a problem.(Eichenfield, 336–346)

59. (D) The most immediate task when an infantpresents with vesicles and bullae at birth is torule out infection. Although rare in the new-born period, the infections listed can presentwith severe disease which may be life threatening

Page 72: Questions - uhs.edu.kh You suspect Lyme disease, most likely contracted by which of the following? (A) ingestion of unripe fruit (B) ingestion of spoiled fruit (C) drinking of contaminated

Answers: 53–65 73

if the infant is not adequately treated withappropriate antibacterial or antiviral therapyand supportive therapy for fluid and elec-trolyte issues. Decreased immune defenses anda fragile fluid and electrolyte balance can makethe situation worse in the newborn period.Staphylococcus aureus infection is most likely.Herpes simplex and gram-negative infectionsdo not generally present with bullous lesions.Epidermolysis bullosa is a group of inheriteddiseases in which the skin cannot withstandfriction and thus skin blistering develops, butis very rare. Severe forms of the disease canalso be life threatening. Maternal diabetes isnot associated with these skin findings at birth.(Eichenfield, 137–178)

60. (B) Characteristically, the lesions of erythematoxicum (erythematous macules and papules)are loaded with eosinophils. The cause of thistransient disorder of term newborns is un-known, and the lesions resolve spontaneouslywithin a few days. Cultures of the lesions aresterile. It is important not to confuse this totallybenign condition with the rash of serious dis-orders such as staphylococcal pustulosis or dis-seminated viral infection. (Eichenfield, 91)

61. (B) Infants born to mothers with hyper-parathyroidism often develop transient hypo-parathyroidism, resulting in hypocalcemia andhyperphosphatemia. The mechanism, presum-ably, is suppression of the fetal parathyroidglands by excessive transplacental maternalparathyroid hormone. Occasionally, the mother’scondition may be undiagnosed, and otherwiseunexplained hypocalcemia in an infant canlead to the diagnosis of hyperparathyroidism inthe mother. (Fanaroff, 1511)

62. (B) Hereditary tyrosinemia is disorder of aminoacid metabolism. Tyrosine is ingested from pro-tein and synthesized endogenously from pheny-lalanine. Normally, the body metabolizes excesstyrosine to carbon dixoide and water. Deficienciesalong this metabolic pathway will result inbuildup of toxic metabolites (most commonly,fumarylacetoacetate) and hepatocellular dys-function. The hepatic dysfunction frequentlypresents as conjugated hyperbilirubinemia, also

known as direct hyperbilirubinemia. The otherproblems listed cause indirect (or unconjugated)hyperbilirubinemia. (Behrman, 402, 1310–1311)

63. (A) The infant described in this scenario mostlikely has congenital adrenal hyperplasia ofwhich more than 90% of cases are caused by21-hydroxylase deficiency. Ambiguous genitaliain association with congenital adrenal hyper-plasia usually indicates a female pseudoher-maphrodite (46 XX karyotype with ambiguousexternal genitalia due to excess androgens). Thisdisorder is associated with both cortisol andaldosterone deficiency of which the classic signsand symptoms are progressive weight loss andvomiting, hyponatremia, hyperkalemia, andhypoglycemia. Without treatment, shock, cardiacarryhthmias, and death may occur. In manystates, the newborn screening programs will testfor congenital adrenal hyperplasia (detection ofelevated 17-hydroxy progesterone) in order toidentify and treat patients prior to the devel-opment of symptoms. Although 3-beta-hydroxysteroid dehydrogenase deficiency maypresent with both salt wasting and ambiguousgenitalia, this disorder is very rare. The otherdisorders listed do not present with salt wastingand hypotension. (Brodsky, 314; Behrman, 1909–1915)

64. (B) Infants born to women with active anduntreated Graves disease may be hyperthyroidat birth, presumably as a result of transplacen-tal passage of thyroid-stimulating antibodies.These antbodies can cause an increase in fetalthyroid hormone production. If the mother isreceiving antithyroid medication, this alsocrosses the placenta, and the infant may beeuthyroid or even hypothyroid at birth. Even ifthe infant is asymptomatic at birth, the anti-bodies may persist for several weeks so regularscreeing of the infant’s thryoid function duringthe first 4–6 weeks of life is indicated. (Brodsky, 310)

65. (C) Approximately 75% of neonatal disease isdue to HSV 2 (typically genital herpes).Transmission most commonly occurs at deliv-ery due to virus in the birth canal (95%). Inutero infection from ascending or transplacen-tal passage is also possible (5%). A primarymaternal genital infection is the single greatest

Page 73: Questions - uhs.edu.kh You suspect Lyme disease, most likely contracted by which of the following? (A) ingestion of unripe fruit (B) ingestion of spoiled fruit (C) drinking of contaminated

74 3: The Neonate

risk for disease in the infant due to the presenceof a high viral replication load, longer viralexcretion, and lack of maternal antibody pas-sage to the infant. The risk for disease in theinfant during a primary maternal infection is10–20 times higher than in the presence ofmaternal secondary or recurrent lesions. Inmost neonates with disease, there is no mater-nal clinical history of HSV. Prolonged ruptureof membranes increases the risk of ascendinginfection. Fetal scalp monitoring may break theinfant’s skin barrier and thus is contraindic-taed in the presence of known maternal HSVinfection. Prematurity is also a risk factor likelydue to low transplacental IgG antibodies inpremature infants. (Brodsky, 192)

66. (B) After delivery, the infant is cut off from itscontinuous calcium supply prompting a pre-cipitous fall in serum calcium over the first1–2 days. In normal circumstances, this stimu-lates PTH causing a gradual return to higherserum calcium levels. PTH secretion is inhib-ited in infants of diabetic mothers. Therefore,hypocalcemia is a common problem in infantsof diabetic mothers. (Polin, 87)

67. (E) Passage of meconium usually occurs withinthe first 12 hours of life. Approximately 15%of term deliveries have meconium in the amni-otic fluid, suggesting meconium was passedin utero. Also, 99% of term infants and 95%of preterm infants will have passed theirfirst meconium stool by the age of 48 hours.In a term infant, failure to pass meconium by24–48 hours should prompt evaluation for anunderlying problem such as intestinal obstruc-tion, Hirschsprung disease, or hypothyroidism.(Polin, 144; Behrman 527)

68. (E) The description and location of the lesion(beneath fading forceps marks) are typical ofsubcutaneous fat necrosis, which may followtrauma (forceps). Buccal cellulitis caused byH influenzae frequently has a violaceous colorbut is exceedingly rare in the newborn periodand infants are likely to appear systemically ill.(Eichenfield, 184–185, 420–422)

69. (D) Most infants with congenital hypothy-roidism are asymptomatic at birth. Thyroidhormone is critical for normal brain develop-ment, deficiency of thyroid hormone duringthe first 2–3 years of life could result in irre-versible brain development. Neonatal screen-ing programs assist the clinician in identifyingthese infants after birth, although errors inscreening occur. Awareness of the subtle clini-cal signs and symptoms is imperative. Theinfants may be edematous, have a large poste-rior fontanelle, have difficulty with feeding,and have a history of prolonged jaundice. (Polin,102; Brodsky, 308)

70. (A) Low birth weight, a faster rate of growth,and iatrogenic blood loss are factors that putpremature infants at an increased risk of earlyiron deficiency. They have iron stores adequatefor less than 3 months postnatally. Iron stores ina term infant are sufficient for the first 4 to6 months of life. In cases of hemolysis (such asABO incompatibility) and physiologic hyper-bilirubinemia, iron is not lost from the body.Polycythemia is an elevated hemotocrit com-monly seen in cases of intrauterine hypoxia orplacenta vascular insufficiency or from placen-tal transfusion such as delayed cord clamping.(Merenstein, 521–534).

71. (D) Fetal hemoglobin binds poorly to 2,3-DPG.Because 2,3-DPG binding decreases the affinityof hemoglobin for oxygen, fetal hemoglobin,unbound to 2,3-DPG, has an increased affinityfor oxygen. Prenatally this works to the advan-tage of the fetus in obtaining oxygen from thematernal blood (across the placenta), but post-natally it is to the infant’s disadvantage inreleasing oxygen at the tissue level. (Fanaroff,1089)

72. (C) The mass shown is a typical example of anencephalocele. These lesions consist of hernia-tion of the meninges, with or without braintissue, through a defect in the skull. They occurmost commonly in the occipital region(70%–80% of cases). It is a restricted disorder ofneuralation that involves anterior neural tubeclosure. The precise pathogenesis is unknown.The mass is far too large to be an abscess. There

Page 74: Questions - uhs.edu.kh You suspect Lyme disease, most likely contracted by which of the following? (A) ingestion of unripe fruit (B) ingestion of spoiled fruit (C) drinking of contaminated

Answers: 66–78 75

is no such entity as posterior hydrocephalus.Hydroceles are limited to the scrotum.Myelomeningocele is restricted failure of pos-terior neural tube closure. Approximately 80%of lesions occur in the lumbar area. (Volpe, 7–9)

73. (C) The roentgenogram reveals absence of theradius and suggests the thrombocytopeniawith absent radius (TAR) syndrome. Affectedinfants have early thrombocytopenia, whichmay be associated with bleeding. Infants man-ifest congenital skeletal malformations of thearm and hand. The other disorders listed arecauses of bleeding in the neonatal period butare not typically associated with congenitalabnormalities of skeletal development.(Merenstein, 537)

74. (A) Breast milk jaundice results from anincrease in unconjugated bilirubin. Prolongedand exaggerated jaundice occurs because of aninhibitor or inhibitory substance in the breastmilk that prolongs increased enterohepatic cir-culation. The hyperbilirubinemia becomesexaggerated by day 5 of life and persists for4–14 days followed by a gradual decline.Elevation of bilirubin levels within the first12 hours of life is typically associated withhemolysis. Since breast milk jaundice is notassociated with hemolysis, levels do not esca-late within the first day of life and there is notan association with anemia or reticulocytosis.It is a common problem in infants of all races.(Merenstein, 553)

75. (D) Infants of diabetic mothers present with avariety of problems. The most common ishypoglycemia resulting from hyperinsulinism.Hypoglycemia develops in 25%–50% of infantsof diabetic mothers. The probability of hypo-glycemia after delivery increases when maternalfasting glucose is high. There is a threefoldincrease in the incidence of congenital anomliesin these infant including sacral agenesis, con-genital cardiac malformations, and intestinalatresias. Large birth weights are commonamong infants of diabetic mothers which pre-disposes the babies to birth trauma andasphyxia. Cardiomegaly is common in 30% ofthese infants, asymmetric septal hypertrophy

may occur that manifests similarly to idopathichypertrophic subaortic stenosis. This septalhypertrophy typically resolves with time.Hypocalcemia and associated hypomagne-semia is seen frequently, but infants do not typ-ically develop hyponatremia beyond what isnormally seen in newborn infants. (Behrman, 614)

76. (E) Infantile glaucoma typically presents ininfants less than 6 months of age, and requiressurgical treatment. Signs and symptoms includeenlargement and clouding of the cornea, tear-ing, blepharospasm, and photophobia. It isassociated with many other conditions includ-ing Marfan syndrome, neurofibromatosis 1, andcongenital rubella. It can be associated with anumber of chromosomal syndromes but thereis not a classic association with trisomy 21.Infantile or congenital gluacoma begins withinthe first 3 years of life. The white pupil sign issuggestive of a congenital cataract. (Behrman, 2122)

77. (D) Tachypnea, retractions, cyanosis, andgrunting are common findings in neonatal res-piratory distress syndrome (RDS). Pallor andpoor perfusion may also be present but are isnot the most common initial manifestation.Wheezing is not a feature of RDS. The clinicalfindings relate to the pathogenesis of the dis-ease, which primarily is atelectasis due to col-lapse of the terminal airways caused bysurfactant deficiency. (Merenstein, 650)

78. (B) Uterine contractions can precipitate a fall infetal PAO2. The fetus has a limited ability tocompensate for hypoxemia. Adults increasetheir heart rate to increase their total cardiacoutput while the fetus responds by redistribu-tion of their cardiac output. Blood flow isincreased to the brain and heart and perfusionto less critical organs is reduced. Acute hypox-emia leads to acidosis and fetal bradycardia.The fetal heart rate is usually 110–160 beats perminute with considerable beat-to-beat varia-tiability. Accelerations of the heart rate arenormal and not usually of concern. A rate lessthan 100 beats per minute is unusual and wor-risome. A fixed heart rate or decreased beat-to-beat variability often indicates fetal hypoxia(fetal distress). Decelerations may be of no

Page 75: Questions - uhs.edu.kh You suspect Lyme disease, most likely contracted by which of the following? (A) ingestion of unripe fruit (B) ingestion of spoiled fruit (C) drinking of contaminated

76 3: The Neonate

consequence or may be ominous, dependingon their temporal relationship to uterine con-tractions. Early decelerations are usuallybenign, whereas late or variable decelerationscan indicate fetal distress. (Merenstein, 26)

79. (C) Infants with a symptomatic pneumothoraxtypically develop tachypnea, cyanosis, andincreased work of breathing. Apnea and irri-tability may also be an early sign. The chestmay appear asymetric and breath sounds maybe absent. In a tension pneumothorax, signs ofshock including hypotension and bradycardiamay be noted. The heart apex is shifted awayfrom the affected side. Prompt diagnosis andtreatment is needed. (Behrman, 586)

80. (D) Since the infant in question was more than38 weeks gestational age, she is not premature.She is, however, of low birth weight (< 2500 g)and is small for gestational age. This can be con-firmed by the use of standard intrauterine growthcharts. (Note: Answers (A), (C) and (D) are cor-rect, but (D) is the BEST answer.) (Merenstein, 21, 94)

81. (D) Clinically significant subdural hemorrhagein the neonatal period is typically a traumaticlesion. The majority of the cases have involvedfull-term infants. Intraventricular hemor-rhages occur primarily in premature infants.Hemorrhagic disease of the newborn can occurin either premature or term infants who do notreceive vitamin K prophylaxis, but is not morecommon in term infants. Both congenital infec-tion and neonatal sepsis are more common inpremature than term infants. (Volpe, 485-493)

82. (D) Meconium ileus results from thick, viscidmeconium packed into the terminal ileum. Thesubstance fills the lumen of the small bowelcausing a mechanical bowel obstruction. Thecolon has a resultant small caliber (microcolon)because of lack of use. Progressive abdominaldistention and bilious vomiting are the hall-marks of the clinical presentation. Nearly allneonates presenting with meconium ileus havecystic fibrosis. Of neonates with cystic fibrosis,10%–20% will present with meconium ileus.(Polin, 174)

83. (B) Neither DIC nor dermal erythropoiesis arerecognized manifestations of congenitalsyphilis. There are a variety of highly charac-teristic skin manifestations including bullouslesions on the palms and soles. Cutaneous find-ings of syphilis classically involve the palmar/plantar, perioral and anogenital regions. Mucousmembrane involvement may manifest as snuf-fles, a clear nasal discharge that may be mistakenfor a viral upper respiratory infection. Pneumoniaand hepatitis may occur but also are seen inmany other neonatal infections. (Eichenfeld, 196;Brodsky, 186)

84. (A) Epiphyseal dysgenesis, the developmentof multiple foci of ossification, occurs inpatients with hypothyroidism that are nottreated or are inadequately treated. The devel-opment of ossification centers is also retardedin hypothyroidism. About 60% of infants withcongenital hypothyroidism have x-ray changes,consisent with delayed osseous development,present at birth. The ossification center of thehamate is not normally present at birth, thusx-ray examination of the wrist is of no value inthe newborn period. Roentgenographic demon-stration of absence of the distal femoral epiphysesin a term infant would be suggestive of hypothy-roidism.Although cardiomegaly from myxedemaof the heart can be seen, there are so many other,much more common causes of cardiomegaly inthe newborn that an enlarged heart would notbe suggestive of hypothyroidism. (Behrman,1876)

85. (E) Hypoplastic left heart syndrome typicallypresents within the first few days to weeks oflife with tachypnea, lethargy, and poor feeding.The condition of the neonate worsens dramat-ically with closing of the PDA. Cyanosis maynot be obvious initially, but patients develop agrayish blue color of the skin which resultsfrom a mix of cyanosis and hypoperfusion. Asthe PDA closes, progressive shock will develop.The other cyanotic heart lesions listed are morelikely to present with cyanosis and tachypnearather than shock. (Behrman, 1524–1544)

86. (D) The initial lesions of incontinentia pigmentiare inflammatory bullae that eventually evolve

Page 76: Questions - uhs.edu.kh You suspect Lyme disease, most likely contracted by which of the following? (A) ingestion of unripe fruit (B) ingestion of spoiled fruit (C) drinking of contaminated

Answers: 79–93 77

into hyperpigmented lesions. The bullae aretypically present at birth or during the first fewweeks of life, although neonates may be bornwith or develop the hyperpigmented lesionswithout the preceeding bullous lesions. Mentalretardation, seizures, and several other organsystem anomalies (eg, eye, skeletal system) areoften present. (Eichenfeld, 377)

87. (A) Amnion nodosum (granules on theamnion) suggests oligohydramnios, which, inconjunction with the “Potter facies” described,is characteristic of bilateral renal agenesis.Esophageal atresia often is associated with poly-hydramnios, not oligohydramnios. None of thefeatures of this case are suggestive of congeni-tal heart disease or infection. (Behrman, 547, 1783)

88. (B) Cats are the definitive host for Toxoplasmagondii, an intracellular parasite that can betransmitted from mother to fetus through theplacenta (transplacental transmission). Typically,this is the result of a primary infection in themother. The most severely infected fetusesaquire the infection during the first trimester ofpregnancy. (Note: The table of clinical findingsof infants with congenital infections found inBrodsky’s Neonatology Review on page 201 isworth reviewing!) (Brodsky, 200)

89. (B) Trisomy 18 (Edwards syndrome) is charac-terized by severe retardation of growth andmental development. Most of the patients die inearly infancy, mortality in the first year of life is90%. Characteristic abnormalities include low-set and malformed ears, nail hypoplasia, abnor-mal fisting with index finger overlying thirdfinger, and rocker-bottom feet. The abnormali-ties of the hands and feet are clinically distinc-tive features. Congenital heart disease is usual,most commonly a ventricular septal defect or apatent ductus arteriosus. (Brodsky, 167)

90. (A) The cardinal features of trisomy 13 (Patausyndrome) include cleft lip and palate, holo-prosencephaly, and severe mental retardation.Holoprosencephaly is an incomplete develop-ment of the forebrain, often associated withabsence of the corpus callosum, fusion of the

frontal lobes, and a single ventricle. Other fea-tures of trisomy 13 include ocular abnormali-ties, congenital heart disease, and cutaneousdefects of the scalp (eg, cutis aplasia). Greaterthan 90% of infants die within the first year oflife. (Brodsky, 166)

91. (D) Among live births, Down syndrome is themost common chromosomal abnormality. Itoccurs in about 1 in 800 live births. Fewer than5% of patients have a translocation rather thanan extra (47) chromosome. Hypotonia andhyperextensible joints are characteristic. Mentalretardation is present in all patients, but is lesssevere than in other trisomy syndromes (ie, tri-somies 13 and 18). Clinodactyly, especially ofthe fifth finger, a single plamar crease, anincreased distance between the first and secondtoes, upward-slanting palpebral fissures, epi-canthal folds, flat nasal bridge, and flat occiputare some of the physical findings associatedwith this syndrome. Many of these patientshave serious congenital malformations such ascongenital heart disease or duodenal atresia.Brushfield spots (tiny white spots that form aring in the midzone of the iris) are frequentlypresent. (Brodsky, 168)

92. (C) The cri–du-chat (cry of the cat) syndrome isso named because of the characteristic cry,which is reminiscent of a mewing cat. Thischaracteristic cry is present in infancy but dis-appears as the child grows and usually is goneby the age of 1 or 2 years. The cry is highpitched and distinctive and results from abnor-mal laryngeal development. These patientsalso have microcephaly, severe mental retar-dation, epicanthal folds, downward slant ofpalpebral fissures, hypertelorism, and low-setears. The syndrome is associated with a dele-tion of the short arm of chromosome 5. (Note:The student who understands French has anobvious advantage in answering this question.)(Brodsky, 168)

93. (E) Features of Turner syndrome, 45,X or XO, inthe older child include short stature, lack ofdevelopment of sexual characteristics, primaryamenorrhea, webbing of the neck, cubitus

Page 77: Questions - uhs.edu.kh You suspect Lyme disease, most likely contracted by which of the following? (A) ingestion of unripe fruit (B) ingestion of spoiled fruit (C) drinking of contaminated

78 3: The Neonate

valgus (wide-carrying angle of the arms), andshort fourth metacarpals. In the newbornperiod, redundant skin at the nape of the neck,a low posterior hairline, and edema of thedorsum of the feet are characteristic. Cardiacdefects (especially coarctation of the aorta) andrenal anomalies are common. Intelligence usu-ally is normal, but perceptual difficulties maybe present. (Brodsky, 176)

94. (C) After birth, intubation and tracheal suctiondecrease the risk of developing meconium aspi-ration syndrome in infants who are not vigor-ous at birth. The infant described in this case isnot vigorous (depressed respirations, depressedmuscle tone, and/or a heart rate < 100 beats perminute), thus endotracheal intubation withsuctioning to remove the meconium is indi-cated after the infant is delivered. (Kattwinkel,Lesson 2)

95. (C) While most infants with cytomegalovirus(CMV) infection are asymptomatic at delivery,the combination of growth impairment,hepatosplenomegaly, and the “blueberrymuffin” rash of extramedullary hematopoiesis(specifically dermal erythropoiesis) are char-acteristic of CMV infection (see Figure 3-3).(Brodsky, 199–201)

96. (E) Jaundice in the first day of life nearlyalways is a pathologic process in term infants.Hereditary spherocytosis has an incidence ofapproximately 1/5000 in populations ofNorthern European origin, less in most otherethnic groups. ABO isoimmune hemolyticanemia occurs much more frequently, in about3% of pregnancies, and is associated with early-onset jaundice and spherocytes on the bloodsmear. The spherocytes are generated as splenicdisruption of the red cell membrane occurs.Glucose-6-phosphate dehydrogenase defi-ciency (the most frequently inherited RBCenzyme defect) is associated with normal RBCshape. Patients with disseminated intravascu-lar coagulopathy have bleeding, abnormalcoagulation studies, and thrombocytopenia.(Brodsky, 289–297)

97. (D) Varicella infections in the newborn are fre-quently life threatening. If the mother has hadvaricella in the past, her IgG antibody to vari-cella crosses the placenta and protects theinfant for weeks after delivery. Certainly,infants who face exposure without maternalantibody protection acquire the infection at ahigh rate with serious morbidity, mainly fromvaricella pneumonia. Varicella immune globu-lin is useful in preventing acquisition of infec-tion in susceptible infants. Infants are consideredsusceptible if maternal onset of varicella is5 days or less before delivery. Varicella immuneglobulin is not indicated if the mother haszoster. (Pickering, 711–725)

98. (A) Supraventricular tachycardia (SVT) is themost common tachydysrhythmia in the new-born period. Dual AV nodal pathways, rapidconduction through an accessory bundle, orthe existence of an ectopic atrial pacemaker areall potential mechanisms for development ofSVT. Often predisposing conditions are iden-tified such as Wolff-Parkinson-White syn-drome or structural abnormalities of the heart(eg, Ebstein anomaly). Newborns with SVTgradually develop congestive heart failure if itis not identified. Symptoms of congestive heartfailure in the newborn include anxiousness,restlessness, tachypnea, and poor feeding.(Merenstein, 730)

99. (C) Infants with severe respiratory distress syn-drome (hyaline membrane disease) commonlydevelop hypoxemia and acidosis. Acidosis isusually respiratory (elevation of Pco2 levels to> 50 torr) but a metabolic acidosis componentmay also be present. Diffuse atelectasis andventilation/perfusion mismatch are present inthis disease. (Merenstein, 649–651)

100. (D) In the face of acute cold stress, the infantattempts to conserve heat by vasoconstrictionthus reducing heat loss and initiating thermo-genesis. Glycogen stores are depleted, oxygenconsumption increases. Hypoglycemia and inincreased oxygen need is likely to result buthypercalcemia and hypertriglyceridemia arenot encountered. As hypothermia continues,the infant uses anaerobic metabolism to increase

Page 78: Questions - uhs.edu.kh You suspect Lyme disease, most likely contracted by which of the following? (A) ingestion of unripe fruit (B) ingestion of spoiled fruit (C) drinking of contaminated

Answers: 94–104 79

heat production. This causes an increased lacticacid production with resultant metabolic aci-dosis (not metabolic alkalosis). Acrocyanosis,initially present, will progress to centralcyanosis, apnea, and bradycardia. The large sur-face area to mass ratio of the newborn predis-poses to this problem. (Merenstein, 133–135)

101. (D) Periventricular leukomalacia (PVL) is char-acterized by focal necrotic lesions in theperiventricular white matter. Cranial ultra-sound can detect focal echo denisities and/orcystic lesions surrounding the lateral ventriclesthat are diagnostic of PVL. Other imaging tech-niques may be needed to detect more diffuseinjury. These lesions are rarely found in infantsgreater than 32 weeks gestation. Prematureinfants are predisposed to the development ofPVL due to the complex interaction betweenthe cerebral vasculature and regulation of cere-bral blood flow that is gestatonal age depend-ent. Actively differentiating or myelinatingperiventricular glial cells are also vulnerable toinjury. The findings of PVL may not be evidentuntil 1 month of age or later. Congenital anom-alies of the brain and intraventricular hemor-rhage usually are readily apparent on imagingin the first days of life. (Brodsky, 140; Behrman, 562)

102. (E) Congestive heart failure is a common com-plication of large intracranial arteriovenous fis-tulas. A cranial bruit usually is readily audiblein these patients. The great vein of Galen is a fre-quent location for such arteriovenous malfor-mations. Hyperthyroidism and polycythemiacause congestive heart failure only rarely andare not associated with a cranial bruit; trans-position of the great vessels usually is associ-ated with a cardiac murmur and not with acranial bruit, although a murmer arising fromthe heart may be transmitted to the cranium.Ruptured cerebral aneurysms are exceedinglyrare in the newborn period and present withneurologic findings rather than heart failure orcranial bruits. (Behrman, 1974, 2036)

103. (B) Brachial plexus injury results from stretch-ing of the plexus and nerve roots. The upperroots (C5 and C6) are most vulnerable to injury.Phrenic nerve injury on the same side of the

injury may occur, which results in diaphrag-matic paralysis. This risks are highest forbrachial plexus injury if the infant is large forgestational age and/or if the labor and deliveryis complicated. Respiratory distress syndromeis uncommon in term infants and almostalways presents initially in the first 24 hours oflife. Pulmonary hemorrhage, pneumothorax,and cystic adenomatoid malformation cancause cyanosis and respiratory distress buthave no association with brachial plexus injury.(Brodsky, 144, 71–81)

104. (A) The most common cause of neonatalmeningitis is group B streptococcus (GBS).Escherichia coli generally is the second leadingcause. Listeria is less frequent but not uncom-mon. Haemophilus influenzae and S pneumoniaeare uncommon causes of meningitis during thefirst month of life. GBS is usually verticallytransmitted from the mother either at deliveryor from ascending transmission after rupture ofmembranes. The risk for neonatal infectionincreases in the presence of prematurity, pro-longed rupture of membranes, intrapatumfever, chorior aminionitis, and maternal GBSbacteruria. Vaginal colonization of GBS (orabsence of colonization) may be transient, soscreening cultures are not 100% predictive ofstatus at delivery. (Brodsky, 184–186)

Page 79: Questions - uhs.edu.kh You suspect Lyme disease, most likely contracted by which of the following? (A) ingestion of unripe fruit (B) ingestion of spoiled fruit (C) drinking of contaminated

80 3: The Neonate

Polin RA, Fox WW. Fetal and Neonatal Physiology. 2nd ed.Philadelphia, PA: W.B. Saunders Co.; 1998.

SELECTED READINGS

Page 80: Questions - uhs.edu.kh You suspect Lyme disease, most likely contracted by which of the following? (A) ingestion of unripe fruit (B) ingestion of spoiled fruit (C) drinking of contaminated

CHAPTER 4

Growth and DevelopmentSarah E. Hampl, MD

81

An understanding of pediatric growth and development is essential for those entering primary care. Thisis where it all begins, and wise students will do well to recall their own childhoods as they learn the widearray of physical, developmental, and emotional milestones that today’s infants, toddlers, children, and teensachieve. If one is already in the role of parent, learning or recalling of these milestones is an exciting, pleasur-able task. However, most students have had little prior experience as a caregiver to children, and may find itdifficult to comprehend and remember these milestones and their normal and abnormal variations. A varietyof easily read resources exists to facilitate this learning. Spending time caring for or observing infants, toddlers,and school-age children is a valuable method of naturally learning their activities and behaviors.

Page 81: Questions - uhs.edu.kh You suspect Lyme disease, most likely contracted by which of the following? (A) ingestion of unripe fruit (B) ingestion of spoiled fruit (C) drinking of contaminated

82

DIRECTIONS (Questions 1 through 92): For eachof the multiple choice questions in this sectionselect the one lettered answer that is the bestresponse in each case.

1. Third-year medical students are rotating in thenormal newborn nursery. The students learnthat most neonates actually lose weight afterbirth. One student asks what the average rateof weight gain is following the initial period ofweight loss. After 2 weeks of age, a term neonatewill gain an average of which of the followingincrements of weight?

(A) 15 g/day or 1/2 oz/day(B) 30 g/day or 1 oz/day(C) 45 g/day or 11/2 oz/day(D) 60 g/day or 2 oz/day(E) 120 g/day or 4 oz/day

2. A mother brings her 6-month-old son in for acheckup. She is concerned that when she is play-ing with him and puts a toy behind her back, hedoes not try to find it. Object permanence, theunderstanding that objects continue to existeven when not seen, is a major milestone thatoccurs closest to which of the following ages?

(A) 9–12 months(B) 15 months(C) 18 months(D) 24 months(E) 30 months

3. A 6-month-old girl, comes in for her checkupwith her mother and grandmother. A look in

the girl’s mouth reveals no teeth. What is theusual age for the eruption of the primary teeth?

(A) 2–4 months(B) 4–6 months(C) 6–8 months(D) 8–10 months(E) 10–12 months

4. The girl’s grandmother goes on to report thatshe is sure that her grandaughter will be left-handed, just like her. Consistent use of a dom-inant hand is established at which of thefollowing ages?

(A) 12 months(B) 24 months(C) 4–6 years(D) 7 years(E) 9 years

5. A group of nursing students are learning abouthow vision screening is done in the primarycare office. One student asks at what age visualacuity reaches 20/20. Normal visual acuity typ-ically occurs at which of the following ages?

(A) 1 week(B) 6 months(C) 1 year(D) 4–5 years(E) 7–9 years

6. The father of a 3-year-old boy brings him inwith a 2-week history of purulent rhinorrheaand an occasional headache in the frontal area.He requests that you perform sinus films to

Questions

Page 82: Questions - uhs.edu.kh You suspect Lyme disease, most likely contracted by which of the following? (A) ingestion of unripe fruit (B) ingestion of spoiled fruit (C) drinking of contaminated

Questions: 1–13 83

determine if his son has frontal sinusitis. Whatis the earliest age at which the frontal sinusescan be visualized radiographically?

(A) 3 years(B) 4 years(C) 6 years(D) 8 years(E) 10 years

7. A medical student is rotating with a local pedi-atrician who uses the Parents’ Evaluations ofDevelopmental Status (PEDS) questionnaire.Which of the following characteristics bestdescribes this screening test?

(A) Used to evaluate development of chil-dren 10–14 years

(B) Screens for development in children lessthan 8 years

(C) Utilizes a 100-question format(D) Used to evaluate teen adjustment(E) Less than 50% sensitivity, but high

specificity for diagnosing emotionaldisorders in children

8. A third-year college student reports that shetook a pregnancy test which was positive thismorning. She had moderate alcohol use for thelast 2 months and was worried that develop-ment of the embryo might be affected. Which ofthe following gestational ages describes theembryonic period?

(A) up to 5 weeks(B) up to 9 weeks(C) up to 12 weeks(D) up to 16 weeks(E) up to 20 weeks

9. At 4-weeks gestation, she has not experiencedfetal movements. Which of the following bestdescribes the occurrence of first fetal move-ments, or quickening?

(A) 12 weeks(B) 16 weeks(C) 20 weeks(D) 24 weeks(E) 28 weeks

10. A worried mother brings her 2-year-old to thepediatrician for a well-child check. She is con-cerned because the child still has only 50 wordsin his vocabulary. Which of the following bestdescribes the characteristic age of a toddlerwho speaks 50 words?

(A) 12 months(B) 15 months(C) 18 months(D) 24 months(E) 36 months

11. The mother of a 3-year-old boy reports that heis “on the go” all the time. Typically, childrenstop taking daytime naps at which of the fol-lowing ages?

(A) 18 months(B) 24 months(C) 30 months(D) 36 months(E) 48 months

12. Pediatric residents are performing camp phys-icals for Girl Scouts. A 9-year-old girl asks howshe will know that she is beginning puberty. Ingirls, which of the following is the first visiblesign of puberty?

(A) development of breast buds(B) appearance of early pubic hair(C) menarche(D) enlargement of the clitoris(E) appearance of axillary hair

13. While giving a talk about growing up to theparents of 4-H youth, a father asks the pedia-trician if it is considered normal for a boy tohave signs of puberty at age 8. The median agefor entering puberty in boys is at what age?

(A) 8 years(B) 10 years(C) 12 years(D) 13 years(E) 15 years

Page 83: Questions - uhs.edu.kh You suspect Lyme disease, most likely contracted by which of the following? (A) ingestion of unripe fruit (B) ingestion of spoiled fruit (C) drinking of contaminated

84 4: Growth and Development

14. A 10-year-old female patient, who is at sexualmaturity rating (SMR) stage II is the shortestgirl in her class and her mother wonders whenshe will have her next growth spurt. Maximalheight gain velocity in girls occurs duringwhich of the following SMR?

(A) I(B) II(C) III(D) IV(E) V

15. The grandmother of an 8-year-old boy pres-ents with him for his preschool checkup and isconcerned because he has not grown in heightfor the last 2 years. Which of the following bestapproximates the prepubertal annual increasein height?

(A) 1–2 cm(B) 3–4 cm(C) 5–7 cm(D) 8–10 cm(E) 12–14 cm

16. The causes of childhood obesity are beingreviewed by a group of pediatric residents. Inchildren, obesity is defined as a body massindex (BMI) above which percentile for age andgender?

(A) 50%(B) 75%(C) 95%(D) 120%(E) 150%

17. A mother concerned about her 10-year-old’sweight calls your office to find out how to cal-culate her child’s BMI. Your office nurse tellsher the BMI can be calculated only if one knowswhich of the following values?

(A) Weight(B) Height(C) Weight and height(D) Weight, height, and age(E) Weight, height, gender, and age

18. A pediatric endocrinologist has diagnosed con-stitutional growth delay in a 14-year-old boy.Which of the following are consistent with thisdiagnosis?

(A) bone age is normal compared to chrono-logic age

(B) prognosis for a normal adult height isgood

(C) family history for childhood shortstature

(D) undernourished for the first year of life(E) infant was small for size at birth

19. A 1-week-old infant male presents to his pri-mary care provider’s office for his first visit. Hismother is concerned because he sleeps a lot.Which of the following is the average amountof sleep per day experienced by infants in thefirst month of life?

(A) 12 hours(B) 14 hours(C) 16 hours(D) 18 hours(E) 20 hours

20. A 100 kg, 10-year-old presents for well-childcare. Which of the following history itemsshould be included when evaluating this obesechild?

(A) family history of hyperparathyroidism(B) vigorous physical activities performed

by the child(C) parents’ occupation(D) instances of binge drinking by the parent(E) presence of learning disabilities in siblings

21. A preschool teacher is concerned that a24-month-old boy occasionally repeats wordsspoken to him. She sends a note with mother tohis checkup. The boy’s physical and neurologicexaminations are entirely normal, and thechild’s developmental landmarks are withinnormal limits. Which of the following is the mostlikely description of this child’s symptom?

Page 84: Questions - uhs.edu.kh You suspect Lyme disease, most likely contracted by which of the following? (A) ingestion of unripe fruit (B) ingestion of spoiled fruit (C) drinking of contaminated

Questions: 14–26 85

(A) precocious verbal behavior(B) subtle neurologic abnormality(C) poor language development(D) autism spectrum disorder(E) normal

22. A neonatologist is performing a physical exam-ination with medical students on an infantgreater than 24 hours old. Which of the fol-lowing findings would be considered normalfor the neonate at this age?

(A) has a visual preference for geometricshapes over faces

(B) needs 4–6 hours after birth to suck wellat the breast

(C) is farsighted(D) is unable to hear well(E) learns to differentiate the voice of her

mother from that of other women by4 weeks of age

23. First-time parents of a 1-month-old are anx-ious to hear what they can expect in the nextfew months. Which of the following bestdescribes the usual sequence for attainment ofgross motor milestones in a young infant?

(A) head control, rolling over, handstogether in midline, sits without support

(B) head control, hands together in midline,rolling over, pulls to stand

(C) rolls over, sits without support, handstogether in midline, pulls to stand

(D) sits without support, hands together inmidline, pulls to stand, walks along

(E) pulls to stand, walks along table, sitswithout support, hands together atmidline

24. The parents of a 15 month old are concernedthat their daughter does not seem to be ableto stack cubes into a tower like their son couldat the same age. Her pediatrician reviews thechild’s prior fine motor development with theparents. Which of the following best describesthe correct sequence of fine motor develop-ment one should see in a healthy infant/toddler?

(A) thumb–finger grasp, grasps rattle, trans-fers objects from hand to hand

(B) grasps rattle, thumb–finger grasp,reaches for objects, transfers objectsfrom hand to hand

(C) grasps rattle, transfers objects fromhand to hand, builds tower of twocubes, thumb–finger grasp

(D) reaches for objects, transfers objectsfrom hand to hand, turns pages of book,builds tower of two cubes

(E) grasps rattle, transfers objects fromhand to hand, thumb–finger grasp,builds tower of two cubes

25. A 6-year-old boy is proud to show off the firsttooth he lost but his mother is concerned becausethe tooth was one of his canines and a perma-nent tooth seems to be erupting in this area.Which of the following best describes the correctsequence for eruption of permanent teeth?

(A) central incisors, first premolars, canines,second molars

(B) first molars, central incisors, canines,first premolars

(C) central incisors, lateral incisors, canines,first molars

(D) first molars, lateral incisors, canines,first premolars

(E) lateral incisors, first molars, canines,first premolars

26. The parents of a 6-month-old boy point outthat he has just cut his first tooth, and wonderwhich teeth will come next. Which of the fol-lowing is the correct sequence for eruption ofprimary teeth?

(A) central incisors, first molars, secondmolars, canines

(B) central incisors, lateral incisors, canines,first molars

(C) central incisors, canines, first molars,second molars

(D) central incisors, lateral incisors, firstmolars, canines

(E) canines, central incisors, lateral incisors,first molars

Page 85: Questions - uhs.edu.kh You suspect Lyme disease, most likely contracted by which of the following? (A) ingestion of unripe fruit (B) ingestion of spoiled fruit (C) drinking of contaminated

86 4: Growth and Development

27. A term newborn infant has a normal weightbut height below the 5th percentile. A largeposterior fontanelle is the only other finding.A delayed bone age is noted on skeletal surveybased on the appearance of ossification centersand further testing reveals the diagnosis of con-genital hypothyroidism. In normal neonates,ossification centers are usually radiographi-cally visible at birth in which of the followingsites?

(A) patella(B) lunate (carpal)(C) proximal tibia(D) head of the femur(E) distal tibia

28. The grandfather of a 1-year-old brings him infor a checkup and states that the boy receiveda tricycle for his first birthday but wonderswhen he will be able to pedal to ride it. Whichof the following sequence best demonstratesthe correct attainment of motor milestones?

(A) runs, rides tricycle, skips, hops(B) runs, hops, skips, rides tricycle(C) runs, goes upstairs alternating feet, rides

tricycle, skips(D) hops, skips, rides tricycle, goes upstairs

alternating feet(E) goes upstairs alternating feet, rides tri-

cycle, runs, skips

29. The mother of a 30-month-old brings the pri-mary care provider a note from his parents asteachers (PAT) (early childhood development)specialist. The PAT specialist is concerned bythe child’s lack of self-feeding skills. Which ofthe following social development sequencesdo toddlers follow?

(A) feeds self, helps to undress, washeshands, domestic role-playing

(B) handles spoon well, plays in parallel,dresses and undresses, washes hands

(C) plays in parallel, feeds self, dresses andundresses, washes hands

(D) plays in parallel, dresses and undresses,washes hands, feeds self

(E) washes hands, dresses herself, plays inparallel, handles spoon well

30. A group of student nurses is learning aboutchildrens’ language development. Their instruc-tor explains that language is a critical barometerof both cognitive and emotional development.With which of the following is speech delaymost closely associated?

(A) 22q11 deletion syndrome(B) spina bifida(C) diabetes(D) child abuse(E) asthma

31. The mother of a 2-year-old girl brings her to herpediatrician’s office for a checkup. When askedabout discipline concerns, the mother expressesworry because the child gets mad and throwsherself on the ground screaming. Which of thefollowing statements regarding temper tantrumsis correct?

(A) most often indicate a serious psychoso-cial problem

(B) usually appear at the end of the first year(C) peak prevalence is between 4 and 6 years(D) routinely occur 8–10 times per day(E) usually last between 30 and 45 minutes

32. A group of first-year residents is examining theNational Center for Health Statistics growthcurves for girls and boys. Typically, infants andchildren stay within one or two growth chartchannels. A normal exception to this rule existsfor which of the following?

(A) female infants(B) during preschool years (3–6 years)(C) up to age 2 years(D) male infants(E) during adolescence

33. As part of a routine 18-month check-up, a med-ical student in the outpatient clinic adminis-ters the Denver Developmental Screening Test(DDST) to the child. She reports that the childappeared to function at about a 15–18-monthlevel, but he was noncompliant and difficultto test. Which of the following best explainsthe testing results for this child?

Page 86: Questions - uhs.edu.kh You suspect Lyme disease, most likely contracted by which of the following? (A) ingestion of unripe fruit (B) ingestion of spoiled fruit (C) drinking of contaminated

Questions: 27–39 87

(A) advanced and was stressed by the test(B) delayed and psychologically disturbed(C) delayed and was stressed by the test(D) normal and noncompliance is common

at this age(E) normal but psychologically disturbed

34. The mother of a 6-year-old in first grade reportsthat he is not keeping up with the other kids inany subject and wants testing to be performed.Evaluation of academic failure at school typi-cally includes which of the following?

(A) hearing and visual evaluation(B) personality testing(C) magnetic resonance imaging (MRI) of

head(D) home visit(E) computed tomography (CT) scan of head

35. Vision milestones are being discussed with agroup of student nurses. One student askswhen babies can begin to visually followpeople as they walk across the room. Fixationand tracking through the visual field are welldeveloped by which of the following ages?

(A) at 7 months gestation(B) at birth(C) at 2 months(D) at 6 months(E) at 1 year

36. A tired-appearing mother of a 2-month-old boystates he is feeding and sleeping well but criesfor a total of 3 hours each day. The baby’sexamination is normal. Which of the follow-ing statements about crying is correct?

(A) Crying increases through the entire firstyear of life.

(B) Crying usually is a result of hunger.(C) Any baby crying 3 hours per day, even

in light of a normal examination, war-rants a medical workup.

(D) This amount of crying is normal for thisage.

(E) This degree of crying warrants animmediate skeletal survey.

37. The mother of 9-month-old, who had a normalsleeping pattern in the past, reports he hasstarted awakening and crying once or twice anight. Her primary care physician suggests thatthis may be a manifestation of separation anx-iety. At which of the following ages does sepa-ration anxiety usually first manifest?

(A) 1 week(B) 3–4 months(C) 8–9 months(D) 2–3 years(E) 5–6 years

38. A medical student is checking out a 2-monthwell-child visit to the attending pediatrician.She remarks that the infant is sleeping with herparents. Which of the following statementsregarding cosleeping is true?

(A) Incidence of sudden infant death syn-drome (SIDS) is higher in young infants.

(B) Cosleeping is uncommon in countriesoutside of the United States.

(C) Cosleeping can facilitate transitioning toa child’s own crib or bed.

(D) The primary care provider should referthe child for sleep testing.

(E) The infant’s parents typically reportfeeling well rested.

39. A 7-month-old infant presents for well-childcare and the physician notes a well-developedMoro reflex, palmar grasp reflex, and tonicneck reflexes on examination. The persistenceof neonatal reflexes is an indicator of develop-mental delay, he tells the mother of the infant.At which age should the Moro reflex typicallydisappear?

(A) 3 months(B) 4 months(C) 6–8 months(D) 12–16 months(E) 17–20 months

Page 87: Questions - uhs.edu.kh You suspect Lyme disease, most likely contracted by which of the following? (A) ingestion of unripe fruit (B) ingestion of spoiled fruit (C) drinking of contaminated

88 4: Growth and Development

40. At what age should the tonic neck reflexdisappear?

(A) 3 months(B) 4 months(C) 6–8 months(D) 12–16 months(E) 17–20 months

41. At what age should the palmar grasp reflexdisappear?

(A) 3 months(B) 4 months(C) 6–8 months(D) 12–16 months(E) 17–20 months

42. A 1-year-old girl who has been in a motor vehi-cle accident today is alert, but you note a pos-itive Babinski (upgoing plantar) reflexes. Atwhat age does a Babinski reflex disappear?

(A) 3 months(B) 4 months(C) 6–8 months(D) 15–18 months(E) 20–24 months

43. A mother brings her preteen twins (1 girl,1 boy) in for a checkup and notes that the girl is2 in taller than her twin. She asks the pediatricnurse practitioner if this could be a sign ofpuberty. Pubertal growth spurt in femalesbegins on average at what age?

(A) 8 years(B) 10 years(C) 12 years(D) 14 years(E) 16 years

44. The pubertal growth spurt in males occurs atwhich of the following ages?

(A) precedes the growth spurt in females by2 years

(B) has its onset at the same age as infemales but lasts longer

(C) follows the growth spurt in females by2 years

(D) coincides with attainment of the abilityto ejaculate

(E) begins at Tanner IV stage of sexualmaturity

45. The father of a 13-year-old boy asks if his son’sadult height can be predicted. The peak veloc-ity of growth during adolescence averageswhich of the following?

(A) 1–2 cm/yr(B) 3–4 cm/yr(C) 5–6 cm/yr(D) 7–8 cm/yr(E) 9–10 cm/yr

46. An adolescent male wearing a baggy T-shirtcomes to his pediatrician’s office for evalua-tion of breast development. He is very worriedand embarrassed, and he inquires if there isanything that can be done to treat it. Which ofthe following statements regarding gyneco-mastia in males during adolescence is correct?

(A) is distinctly uncommon(B) usually occurs at age 14–15 years(C) is synonymous with lipomastia(D) is often a continuing problem in adulthood(E) necessitates basic laboratory testing

47. Concerns about childrens’ TV watching are beingdiscussed with a group of parents. The age atwhich a child can reliably distinguish fantasy fromreality occurs at which of the following ages?

(A) 4 years(B) 6 years(C) 7 years(D) 8 years(E) 10 years

48. A 19-year-old G3P3 reports that she had aperiod of depression after the birth of hersecond child. She is concerned this could resur-face with the birth of the third child. Which ofthe following statements is true regarding post-partum depression?

Page 88: Questions - uhs.edu.kh You suspect Lyme disease, most likely contracted by which of the following? (A) ingestion of unripe fruit (B) ingestion of spoiled fruit (C) drinking of contaminated

Questions: 40–54 89

(A) Postpartum “blues” are rare in newmothers.

(B) Other family stress usually lessens inthe postpartum period.

(C) Teenage mothers have a high rate ofdepression.

(D) Psychosis is a common feature.(E) In the majority of cases of postpartum

depression, intervention is necessary forresolution of symptoms.

49. The HEADSS inventory is a useful screeningtool for adolescents. Which of the followingareas are evaluated?

(A) Risk for decreased intelligence quotient(B) Risk for early coronary artery disease(C) Risk for diabetes(D) Risk for normal growth(E) Risk for depression

50. The tired parents of an infant girl ask if thereare any predictive factors for the developmentof colic. Which of the following best describesrisk factors for colic?

(A) Colic usually is associated with infantswho are bottle-fed.

(B) Colic typically begins at 41–42 weeksgestational age regardless of gestationalage at birth.

(C) Colic is most prevalent among cau-casian neonates.

(D) Colic occurs more commonly amongfemales than males.

(E) There are very predictable long-termtemperamental outcomes that emergefrom a colicky infancy.

51. The parents of a 21-month-old boy are concernedbecause he used to say a few words and morerecently, he is just babbling with no recognizablewords. They are worried these may be featuresof autism. Which of the following is a featurethat characterizes autistic spectrum disorders?

(A) Pointing or waving by 18 months(B) Use of 2 word phrases by 18 months(C) Decreased eye contact

(D) Nystagmus(E) Lack of sensitivity to textures or sounds

52. The mother of a 6-month-old girl presents fora well-child check. The baby’s weight andlength are at the 5th percentile, and her headcircumference is at the 25th percentile. Whichof the following is a true statement regardingfailure to thrive in infants?

(A) more common among Hispanic children(B) extremely rare in the United States(C) most often caused by an organic problem(D) more common among female infants

than male infants(E) a major risk factor for later developmen-

tal and behavioral difficulties

53. The parents of a 6-year-old boy bring him to hispediatrician to discuss his behavior patterns.The child’s mother is concerned about hisbehaviors, but his father reports that his sonwill “grow out of it.” Some behavior patternsare considered appropriate at certain develop-mental stages, but are obviously pathologic ifpresent later on in life. Other behavior patternsare considered pathologic at all ages. Which ofthe following is an example of the latter?

(A) temper tantrums(B) lying(C) oppositional behavior(D) truancy(E) rebellion

54. One of this mother ’s concerns is that her6-year-old often physically fights with his sib-lings. Childhood aggression is most commonlyassociated with which of the following?

(A) girls compared to boys(B) children small for their age compared to

children large for their age(C) children from smaller families compared

to children from larger families(D) children diagnosed with attention-

deficit hyperactivity disorder(E) children with borderline personality

disorder

Page 89: Questions - uhs.edu.kh You suspect Lyme disease, most likely contracted by which of the following? (A) ingestion of unripe fruit (B) ingestion of spoiled fruit (C) drinking of contaminated

90 4: Growth and Development

55. Primary care office nurses are designing a ques-tionnaire to screen for toxin poisoning in theirpractice. They wonder how to describe pica sothat a parent can understand it. Pica is theingestion of nonnutrient substances like dirtand chalk. Which of the following statementsregarding pica is true?

(A) The median age of onset is 6 years.(B) It is often a symptom of family disor-

ganization, poor supervision, and affec-tional neglect.

(C) It is caused by iron poisoning and para-sitic infestations.

(D) It usually persists into adolescence.(E) It is caused by viral infections.

56. A father brings his twins (1 boy, 1 girl) in fortheir kindergarten checkup. He is concernedbecause both children wet the bed at night, andhe wonders how long it will last. Which of thefollowing statements regarding nocturnal enuresisis true?

(A) Prevalence at age 10 years ranges from7% to 10%.

(B) A marked familial pattern is oftennoted.

(C) Workup routinely recommended in themanagement includes renal ultrasoundand micturating cystourethrogram torule out kidney disease and vesci-coureteric reflux.

(D) Bed wetting alarms are rarely useful inthe management of some children withenuresis.

(E) Among 5-year-old children, enuresis ismore common in females.

57. A pediatrician has followed a child in fostercare since infancy. This child has experiencedacademic and social success despite difficultenvironmental stresses. Factors which con-tribute to resiliency in childhood include whichof the following?

(A) difficulty with adaptability to changes(B) low level of energy and persistence(C) a desire to investigate and master new

situations

(D) absence of sense of personal autonomy(E) family history of academic success

58. The mother of a kindergartner is concernedthat her child may have a learning disability.Which of the following would be consistentwith a learning disability?

(A) reverses some letters when writing hisname

(B) had delayed speech as toddler(C) diagnosis before age 3 years(D) has a discrepancy between intelligence

and achievement in one or more areas(E) cannot read yet

59. The mother of a 2-year-old girl wonders if herdaughter’s motor skills are age appropriate.Which of the following is a motor skill thatmost 2-year-olds have attained?

(A) stands on one foot for 10 seconds(B) climbs stairs using alternating feet(C) pedals a tricycle(D) copies a circle(E) builds a tower of 8–10 cubes

60. The mother of a 2-year-old girl is concerned thather daughter is resisting efforts to be toilet-trained.Which of the following recommendations fortoilet training are advisable?

(A) Begin training before age 2 years(B) Continue even if the child is extremely

resistant(C) “Time out” for “accidents”(D) Don’t begin if more than five wet dia-

pers in a day(E) Sticker charts are a useful tool to track

successes

61. The normal cognitive development of toddlersis being discussed with a group of medical stu-dents. The developmental thrust of the 2-year-old is best expressed by which of the followingphrases?

(A) “me do it”(B) “show me how”

Page 90: Questions - uhs.edu.kh You suspect Lyme disease, most likely contracted by which of the following? (A) ingestion of unripe fruit (B) ingestion of spoiled fruit (C) drinking of contaminated

Questions: 55–67 91

(C) “that can’t be right”(D) “why”(E) “you do it”

62. The mother of a 9-month-old girl reports that sheis now waking up at night, after sleeping all nightfor several months. Nighttime awakening at thisage can be treated by which of the following?

(A) Vary the bedtime routine(B) Avoid the use of a transitional object

such as a teddy bear(C) Give a bottle of formula or breast milk to

the infant in bed(D) Reassure by rocking the infant to sleep(E) Put infant to bed while drowsy but still

awake

63. In discussing the features of the body massindex (BMI) curve for children to a group ofmedical students, the pediatrician reports thatchildren experience a period of growth calledthe adiposity rebound, where their bodyweight begins increasing to a greater degreethan their height is increasing. Adiposityrebound typically occurs at which of the fol-lowing average ages?

(A) 1 week(B) 1 year(C) 5 years(D) 10–12 years(E) 13–17 years

64. In coordinating pediatric residents to performhead start (early childhood development)physicals and developmental screenings on agroup of preschool children, it is mentionedthat a child who can walk downstairs alternat-ing his or her feet, do a broad jump, and throwa ball overhand, also would be expected to dowhich of the following?

(A) add five and five(B) identify three or four coins(C) name two or three colors(D) multiply three times three(E) write his or her name

65. The mother of a 3-year-old girl reports that herdaughter is afraid to go to bed at night. Whichof the following are recommended methods ofmanaging nighttime fears?

(A) Perform an exhaustive “search” formonsters in the child’s bedroom

(B) Vary the bedtime schedule(C) Vary the regular nighttime routine prior

to bedtime(D) Allow the child to watch TV and videos

for at least one hour prior to bedtime(E) Visit the child briefly to provide reassur-

ance when the child awakens with fear

66. The parents of a child who recently turned5 years old bring him in for a well-child checkand assessment of school readiness. Regardingschool readiness, which of the following is true?

(A) Any concerns about the child’s ability tolearn will probably resolve if the childdelays kindergarten entry for a year.

(B) Previous daycare experience is irrelevant.(C) The ability to follow 1–3 step directions

is an important skill for kindergartensuccess.

(D) Private schools usually have moreaccess to specialized services than dopublic schools.

(E) If the child is short for his age, his par-ents should delay starting him inkindergarten for a year.

67. The pathophysiology of various causes of neona-tal cyanosis is being discussed with a group ofpediatric residents doing their nursery rotation. Inthe term newborn, which of the following state-ments regarding fetal hemoglobin is correct?

(A) Nearly 100% of a neonate’s hemoglobinis fetal hemoglobin.

(B) Fetal hemoglobin binds oxygen lesstightly than adult hemoglobin.

(C) Oxygenated fetal hemoglobin is bluerather than red.

(D) Fetal hemoglobin prevents sickling ofsickle cells.

(E) Fetal hemoglobin hemolyzes easily andis a major cause of neonatal jaundice.

Page 91: Questions - uhs.edu.kh You suspect Lyme disease, most likely contracted by which of the following? (A) ingestion of unripe fruit (B) ingestion of spoiled fruit (C) drinking of contaminated

92 4: Growth and Development

68. A 12-month-old infant presents to her primarycare provider’s office with respiratory distress.The normal respiratory rate of a 1-year-oldchild is which of the following?

(A) over 80 breaths per minute(B) between 60 and 80 breaths per minute(C) between 35 and 50 breaths per minute(D) between 20 and 30 breaths per minute(E) between 10 and 12 breaths per minute

69. A pediatric cardiologist is reviewing the elec-trocardiogram (ECG) of a term infant withtachycardia with a group of pediatric residents.As compared to older children and adults, theECG of an infant normally shows which of thefollowing findings?

(A) a shorter RR and shorter PR interval(B) a shorter RR and longer PR interval(C) a longer RR and shorter PR interval(D) a longer RR and shorter PR interval(E) an equal RR and PR interval

70. A 2-year-old boy coming in for a checkup isfascinated while his blood pressure is taken,and he remains quiet. The average blood pres-sure at 2 years of age is most likely to be whichof the following values?

(A) 50/30 mm Hg(B) 60/30 mm Hg(C) 75/50 mm Hg(D) 95/60 mm Hg(E) 120/80 mm Hg

71. A 4 week male with a temperature of 101°F hasundergone an evaluation for meningitis. Theresults of his cerebrospinal fluid (CSF) testingreturn and while no white blood cells are seenand the glucose is normal, the protein is recordedas high at 90 mg/dL. The concentration of pro-tein in the CSF of infants during the first fewweeks of life normally may be as high as whichof the following?

(A) 20 mg/dL(B) 45 mg/dL(C) 125 mg/dL

(D) 500 mg/dL(E) 1000 mg/dL

72. The parents of a 4-year-old boy are concernedbecause his speech is still not fully understand-able. Which of the following best describes achild’s conversation at 4 years of age?

(A) fully understandable, although mispro-nunciations and grammatical errors arecommon

(B) fully understandable, with few if anymispronunciations and grammaticalerrors

(C) fully understandable to the parent butnot necessarily to others

(D) somewhat understandable althoughgarbled and indistinct

(E) somewhat understandable, with mostlycorrect use of nouns and mostly incor-rect use of verbs

73. A 7-year-old girl comes to her pediatrician’soffice because her teacher is concerned that shemay have attention-deficit hyperactivity dis-order (ADHD). Hallmarks of ADHD includewhich of the following?

(A) Often quiet demeanor(B) History of seizures at birth(C) History of recurrent ear infections(D) Symptom occurrence in more than one

environment(E) Does not appear to be easily distracted

74. The father of 2- and 8-year-old siblings callstheir pediatrician to discuss how he and thechildrens’ mother can prepare them for the par-ents’ upcoming divorce. Which of the followingstatements is true regarding children ofdivorce?

(A) The emotional availability of one orboth parents is often heightened.

(B) On average, girls have a more difficulttime adjusting initially to divorce thanboys.

(C) As boys reach young adulthood, theymay experience anxiety about male-female relationships.

Page 92: Questions - uhs.edu.kh You suspect Lyme disease, most likely contracted by which of the following? (A) ingestion of unripe fruit (B) ingestion of spoiled fruit (C) drinking of contaminated

Questions: 68–81 93

(D) The majority of children have adjustedto the altered family circumstance bythe second year following the divorce.

(E) Preschool-aged children may demon-strate regressive behavior.

75. The mother of an otherwise healthy infant noticedrecently that she could not identify the soft spot inher infant anymore. Based on this piece of his-tory, this child is most likely to be what age?

(A) 3 months(B) between 3 and 9 months(C) between 9 and 18 months(D) between 18 and 24 months(E) between 24 and 36 months

76. The parents of a 3-month-old boy who is grow-ing well come to the pediatrician’s office for awell-child check. Which of the following guid-ance items regarding growth and developmentare appropriate to discuss at this visit?

(A) may leave infant unattended on couchor changing table as long as he is notrolling over yet

(B) caution against leaving small items on thefloor because infant could pick them up

(C) infant still needs to be fed every 2 hoursat night

(D) expect him to coo, smile and laugh(E) may place infant on his abdomen to sleep

77. A 16-year-old girl presents to her primary careprovider’s office requesting a pregnancy testdue to a late period. Which of the followingfactors are associated with an increased risk ofteen pregnancy?

(A) poor academic performance(B) rigid parental support and supervision(C) living in a middle class community(D) history of drug or alcohol abuse in parent(E) late onset of puberty

78. A medical student is describing the growth pat-tern of a child with growth hormone (GH) defi-ciency. Children with isolated GH deficiencyusually have which of the following?

(A) have a normal bone age(B) have associated mild hypothyroidism(C) grow parallel to, but below, the normal

growth curve(D) have an associated (compensating)

hyperthyroidism(E) show deceleration of growth velocity and

fall away from the growth normal curve

79. Parents bring a 10-year-old boy in for evaluationof short stature. The patient is worried abouthis eventual adult height. Children with consti-tutional growth delay (without endocrine abnor-mality) generally can expect to ultimately achievewhich of the following growth descriptions?

(A) very short and obese(B) short but of proportionate weight(C) of normal height and weight(D) very tall but of proportionate weight(E) very tall and obese

80. The parents of a 7-year-old girl report that shewould rather watch TV than play outside. It isestimated that the average school-age Americanchild watches television for about how manyhours a week?

(A) 4–8 h/week(B) 10–14 h/week(C) 15–20 h/week(D) 25–30 h/week(E) 35–40 h/week

81. The inpatient pediatric team is discussing thecauses of failure to thrive (FTT) as they evalu-ate a 9-month-old infant with weight andlength below the 5th percentile. Which of thefollowing should be included in the differentialdiagnosis of infant who is not thriving?

(A) Small atrial septal defect(B) Acute bacterial meningitis(C) Cystic fibrosis(D) Recurrent otitis media(E) Mild intermittent asthma

Page 93: Questions - uhs.edu.kh You suspect Lyme disease, most likely contracted by which of the following? (A) ingestion of unripe fruit (B) ingestion of spoiled fruit (C) drinking of contaminated

94 4: Growth and Development

82. High school teachers are listening to a lectureabout the features of adolescence. Which of the fol-lowing best describes the period of adolescence?

(A) immediately before, during, and afterpuberty

(B) of maximal physical growth(C) of maximal sexual development(D) of physiologic adjustment to maturity(E) of psychosocial transition from child-

hood to adulthood

83. The mother of a 16-year-old girl reports that shehas noticed declining scholastic grades, refusalto participate in family activities, and anunwillingness to communicate with either par-ents or peers. These findings in an adolescentusually indicate which of the following?

(A) a normal stage of development(B) a normal response to peer pressure(C) a normal reaction to overprotective

parents(D) a transient phase of ambivalence(E) an emotional or psychiatric problem

84. The parents of a 2-year-old boy report he isabout to be thrown out of daycare due to bitingother children. For the primary care provider,which of the following facts are important toinquire about regarding biting behavior?

(A) Age when parents introduced solid food(B) Toileting behavior(C) Age at which child’s first tooth erupted(D) History of pica(E) Concerns about the child’s developmen-

tal progress

85. A hemoglobin and hematocrit is obtained froma 12-month-old who came to his pediatrician’soffice for a checkup. Which of the followingbest describes the normal (average) hemoglo-bin concentration at 1 year of age?

(A) 17 g/dL(B) 15 g/dL(C) 12 g/dL(D) 10 g/dL(E) 8 g/dL

86. A group of medical students on the newbornnursery rotation are discussing the sensory andphysical abilities of a newborn. Which of theseabilities is considered normal?

(A) inability to sense pain(B) preference for geometric shapes over

mother’s face(C) 20/20 vision(D) grasping a caregiver’s finger(E) ability to see objects 2–3 feet away

87. The parents of a 4-year-old with recurrentwheezing are both smokers. They inquire aboutthe child’s likelihood of becoming a smokerwhen he reaches adulthood. The phenomenonof children incorporating habits demonstratedby the adults in their lives is known by whichof the following terms?

(A) time in(B) incidental learning(C) conditioned reinforcement(D) fading(E) rapprochement

88. Growth milestones are being discussed with agroup of medical students in the newbornnursery. During the first year of life, how muchweight would an infant who weighs 7.5 lb(3.4 kg) at birth ordinarily gain?

(A) 5 lb (2.3 kg)(B) 10 lb (4.5 kg)(C) 15 lb (6.8 kg)(D) 20 lb (9 kg)(E) 25 lb (11.4 kg)

89. During the second year of life, the averagechild grows how many centimeters?

(A) 12–15 cm(B) 20–25 cm(C) 30–40 cm(D) 40–50 cm(E) over 50 cm

90. The father of a 2-month-old boy wonders whyhis son’s head looks so big. Which of the fol-lowing best describes the head growth duringthe first month of life?

Page 94: Questions - uhs.edu.kh You suspect Lyme disease, most likely contracted by which of the following? (A) ingestion of unripe fruit (B) ingestion of spoiled fruit (C) drinking of contaminated

Questions: 82–100 95

(A) 0.5 cm(B) 1.25 cm(C) 2 cm(D) 5 cm(E) 7.5 cm

91. The mother of an 11-year-old girl who comes toher pediatrician’s office for a checkup notesthat her daughter is beginning to developbreast buds. She asks when her daughter mighthave her first period. Which of the following isa true statement regarding menarche in theadolescent girl?

(A) precedes the spurt in linear growth(B) occurs simultaneously with Tanner

stage II breast development(C) generally occurs when Tanner stage III

breast and pubic hair development havebeen achieved

(D) occurs simultaneously with Tannerstages IV to V pubic hair and breastdevelopment

(E) generally occurs a year or more afterTanner stage V breast and pubic hairdevelopment have been achieved

92. A pediatric psychologist is discussing the con-cept of temperament with a group of residents.Which of the following best describes the termtemperament?

(A) A parent’s response to a tantrum.(B) The emotional bond that a child feels

with his parent.(C) The acts of independence that a child

demonstrates as she enters toddlerhood.(D) A child’s emotional clinginess to a parent

when faced with an unfamiliar situation.(E) Stable, individual modes of responding

to the environment.

DIRECTIONS (Questions 93 through 100): The fol-lowing group of questions is preceded by a list oflettered answer options. For each question, matchthe one lettered option that is most closely associ-ated with the question. Each lettered option may beselected once, multiple times, or not at all.

Questions 93 through 97

(A) Parents’ Evaluations of DevelopmentalStatus (PEDS)

(B) CAGE questionnaire(C) NICHQ Vanderbilt Assessment Scale(D) Pediatric Symptom Checklist(E) Otoacoustic emission test

93. This test measures development of a 1-year-old. What is it?

94. This test measures externalizing and internal-izing behavior in a 10-year-old. What is it?

95. This test detects alcohol misuse in an adoles-cent. What is it?

96. This screens for deafness in a newborn. What isit?

97. This test evaluates potential ADHD in a 7-year-old. What is it?

Questions 98 through 100

(A) 4–6 months(B) 6–9 months(C) 15–18 months (D) 18–24 months(E) 24–36 months

98. A toddler has started toilet training. What is thetypical age of this child?

99. An infant is experiencing stranger anxiety.What is the typical age when this appears?

100. An infant is able to self-feed. What is the usualage?

Page 95: Questions - uhs.edu.kh You suspect Lyme disease, most likely contracted by which of the following? (A) ingestion of unripe fruit (B) ingestion of spoiled fruit (C) drinking of contaminated

96

1. (B) Newborns may lose up to 10% of their birthweight in the first few days of life, but withnormal nutrition birth weight is regained inapproximately 10 days. The infant subse-quently gains approximately 30 g/day for thefirst several months. (Dixon, 224, 229)

2. (A) At 9–12 months, infants gradually developthe concept of object permanence, or the under-standing that objects exist even when they arenot seen. Around the time this milestone isachieved, infants develop frontal activity onthe electroencephalogram. Infants first applythis concept of object permanence to the imageof their mother because of her emotionalimportance; this realization is a critical part ofattachment behavior. (Dixon, 312)

3. (C) Primary or milk teeth are the initial denti-tion of children. The first teeth to erupt are thecentral incisors, at 6–10 months of age. Typically,the mandibular central incisors erupt beforethe maxillary. The lateral incisors (typicallymaxillary, followed by mandibular) come innext at 9–13 months of age. The first molarsfollow at 13–19 months. The canines erupt ataround 16–23 months. The second molars arethe last to come in at 23–33 months of age.(Kliegman, 1529)

4. (C) Handedness frequently is established bythe fourth year. Frustration may result fromattempts to change children’s hand preferenceonce handedness is established. Unusuallyearly appearance of handedness, such as beforethe second year, should raise suspicion of motorweakness of an upper extremity. (Dixon, 283)

5. (D) At birth, the retina is well developed, butthe lens is rather immobile. Fixation and track-ing through the visual field are well developedby 2 months of age. Infants prefer to gaze at ahuman face rather than geometrical designsand they also prefer curved lines, bright colors,and high contrast. Strabismus is common afterbirth but usually resolves by 3 months of age.Although visual acuity is poor at birth, approx-imately 20/400, it improves rapidly in the first6 months of life to 20/40. However, it does notreach 20/20 until about 4 years of age. (Dixon,149-151)

6. (C) Development of the paranasal sinuses con-tinues throughout childhood. The ethmoid andmaxillary sinuses are present from birth; thefrontal and sphenoid sinuses usually appearradiologically around 6 years of age, and fre-quently develop asymmetrically. Frontal sinusesare absent in 1%–5% of adults. (Kliegman, 1749)

7. (B) The Parents’ Evaluation of DevelopmentalStatus (PEDS) is a useful developmental screen-ing tool for the primary care office. It can be usedfrom birth to 8 years, and screens for variouskinds of emotional and developmental problemsand delays. The PEDS consists of 10 questions,and can be administered in 2–5 minutes. Itssensitivity in detecting children with difficultiesis greater than 75%, and specificity in correctlydetecting normal development is 70%–80%.(Dixon, 729)

8. (B) The fetal period begins at the ninth weekof gestation and ends at birth. It is precededby the embryonic period, during which the

Answers and Explanations

Page 96: Questions - uhs.edu.kh You suspect Lyme disease, most likely contracted by which of the following? (A) ingestion of unripe fruit (B) ingestion of spoiled fruit (C) drinking of contaminated

Answers: 1–17 97

rudiments of all major organ systems develop.(Kliegman, 38)

9. (C) By 20 weeks gestation, most second-timemothers and many first-time mothers feel theirbaby’s first movements. This sensation isknown as “quickening.” (Kliegman, 39)

10. (D) Following the realization that words canstand for things, the child’s vocabulary bal-loons from about 10–15 words at 18 months to100 or more at 3 years. After 18 months, thereis a dramatic increase in expressive and recep-tive vocabulary and by the end of the secondyear, a quantum leap occurs in language devel-opment, such that most children presenting fortheir 2-year-old checkup are reported to say atleast 50 words. (Dixon, 392)

11. (E) Even during the neonatal period, up to 10of an infant’s average of 16 hours of sleep perday are concentrated at night. This is reflec-tive of an adaptation process which beganin the third trimester of pregnancy. Daytimesleep progressively declines in infancy; the12-month-old typically takes two 1-hour napsduring the day. By the age of 3, most toddlerssleep 1 hour during the day, and the majorityof 4-year-olds do not take a daytime nap.(Kliegman, 45)

12. (A) Teenagers have been entering puberty atincreasingly earlier ages during the last cen-tury, presumably because of better nutritionand improved socioeconomic conditions. Theage of menarche decreased by about 4 monthsper decade during the last century. In theUnited States, the average age at menarche is12 years, the average weight is 48 kg, and theaverage height is 158.5 cm. Although the firstmeasurable sign of puberty in girls is the begin-ning of the height spurt, the first conspicuoussign usually is development of breast budsbetween the ages of 8 and 11 years. (Dixon, 541)

13. (B) The first sign of puberty in boys, testicularenlargement, begins as early as 9.5 years. Theappearance of pubic hair is an early event inpuberty, but can occur any time betweenages 10 and 15 years. The penis begins to grow

significantly a year or so after the onset of tes-ticular and pubic hair development, usuallybetween the ages of 10 and 13.5 years. Themedian age for entering SMR II or puberty inboys is 10.5 years. However, great variabilityexists in the timing and onset of puberty andgrowth, and psychosocial development doesnot necessarily parallel physical changes.(Dixon, 537-538)

14. (C) Growth acceleration begins in early ado-lescence, although peak growth velocity is notreached until SMR III in girls and SMR IV inboys. Girls reach their peak height velocitybetween 11.5 and 12 years of age. The heightspurt usually ends by age 14. Girls who matureearly will reach their peak height velocitysooner and attain their final height earlier. Girlswho mature late will attain a greater ultimateheight because of the longer period of growthbefore the growth spurt. Final height is relatedto skeletal age at onset of puberty as well asgenetic factors. (Kliegman, 61)

15. (C) Growth is rapid in the first 2 years of life.Infants increase their length by 50% the firstyear and 25% the next. They triple their birthweight in the first year. After 2 years of age, thegrowth velocity curve stabilizes into the rate formid childhood, which is a weight gain of 2–3 kgor 4.5–6.5 lb/yr and a height gain of 5–7 cm or2–3 in/yr. (Kliegman, 72)

16. (D) Growth charts for children ages 2–20 nowinclude body mass index (BMI) charts. Childhoodobesity is the most common form of malnutri-tion in the United States. Other parameters usedto detect and quantify obesity include meas-urement of skinfold thickness in triceps andsubscapular regions and measurement of percentbody fat by bioelectrical impedance (BIA) ordual x-ray energy absorptiometry (DEXA). BMIvaries with age; it typically increases yearlyafter a nadir between the ages of 2 and 5.Obesity is defined as a BMI greater than 95% forage and gender. (Kliegman, 73)

17. (C) Body mass index (BMI) is calculated as(weight in kilograms)/(height in meters)2. Oncethis value is determined, it must be compared

Page 97: Questions - uhs.edu.kh You suspect Lyme disease, most likely contracted by which of the following? (A) ingestion of unripe fruit (B) ingestion of spoiled fruit (C) drinking of contaminated

98 4: Growth and Development

to others of the child’s same gender and age.(Kliegman, 73)

18. (B) Constitutional growth delay has a charac-teristic pattern of having a late growth spurtduring adolescence, therefore in early adoles-cence the child will be short as compared tohis or her peers. However, he or she will con-tinue to grow later in adolescence and reach anormal adult height. Children with constitu-tional growth delay will be of normal size atbirth. Bone age is delayed and is comparable tothe height age. Children with constitutionalgrowth delay must be differentiated from thosewith undernutrition and endocrinologic shortstature who also have delayed bone age.(Kliegman, 2297)

19. (C) The majority of a newborn’s early days arespent sleeping. Total requirements for sleep are16 hr/24 hr in the first month of life, with mostof the sleeping hours concentrated at night.Sleep requirements decline slowly over the firstyear. By the age of 9 months, the average infantsleeps 14 hr/day and by 3 years of age, mosttoddlers sleep 12 hr/day. (Kliegman, 45)

20. (B) The topic of childhood obesity must beapproached with sensitivity by the primarycare provider. Parents often have much guiltsurrounding their child’s size, and may bedefensive when history questions are pre-sented. Key questions to ask include a familyhistory of children with rapid weight gain, thefrequency and types of vigorous physical activ-ities performed by the child, the parents’ owncommitment to physical activity, and the pres-ence of extreme overeating or binging by thechild. Additional pertinent questions regard-ing the child’s and family’s nutrition habitsand the family’s perception of influence thatthe child’s weight has upon them should bepresented. (Parker, 253–260)

21. (E) Some echolalia (echoing spoken words)during toddlerhood is common and usually ofno concern. When severe or persistent beyondtoddlerhood, echolalia may be a sign of dis-turbed language development, mental retarda-tion, or neurologic disease. Although echolalia

is common in infantile autism, most toddlerswho display word repetition are not autistic.(Note: The question did not ask whether autismshould be considered [it should] but whetherthe child probably has autism [he probably doesnot].) (Dixon, 363, 391-393)

22. (E) Normal newborns are endowed with a setof reflexes to facilitate survival, including rootingand sucking reflexes, and remarkable sensoryabilities. The newborn is no longer considered ablank slate. Instead, the newborn is seen ashaving genetic strengths and weaknesses inneurocognitive organization that are reflectedin temperament, adaptability, responsiveness,and general interaction with the environment.Hearing is well developed at birth, and speechsounds are preferred. The infant learns to rec-ognize his mother’s voice and differentiate itfrom other female voices within the first 3 weeksof life. At birth infants prefer to gaze at a humanface rather than geometric designs, and theyalso prefer curved lines, bright colors, and highcontrast. Infants are myopic at birth. They areable to suckle at the breast immediately afterbirth. (Dixon,152)

23. (B) The attainment of gross motor milestonesbegins at approximately 2 months of age withsteady head control. This allows better visualinteraction between infants and parents. Thechild is then able to be pulled to sit without headlag at around 3 months of age. Putting handstogether in the midline at around 3 months ofage allows the infant to examine objects in themidline and manipulate them with both hands.Once the asymmetric Tonic neck reflex is gone,at around 4 months of age, infants are able tobegin to roll over, first from stomach to backand finally around 6 months of age from back tostomach. Infants will sit without support atapproximately 6 months of age and will pull tostand and cruise around furniture by 9 monthsof age. The average age of beginning to walkalone is 12 months. (Dixon, 248, 264, 289, 323)

24. (E) Around the same time as the infant is ableto put hands together in the midline, he is able tograsp objects and examine them. This occurs at anaverage of 3.5 months of age. He can transfer

Page 98: Questions - uhs.edu.kh You suspect Lyme disease, most likely contracted by which of the following? (A) ingestion of unripe fruit (B) ingestion of spoiled fruit (C) drinking of contaminated

Answers: 18–30 99

objects from hand to hand at approximately5.5 months of age, allowing him to compareobjects. He attains thumb–finger grasp atapproximately 8 months of age, at which timehe begins to feed himself finger foods. Ataround 1 year of age he is able to turn pages ina book, and by 13 months of age he has attainedsufficient visual-motor coordination to scrib-ble. By 15 months of age he can build a tower oftwo cubes, using objects in combination, and by22 months of age he can build his tower up tosix cubes, which requires visual, gross, and finemotor coordination. (Dixon, 264,289,315, 317, 377)

25. (C) Growth of the midface and lower face inchildren occurs gradually. Loss of the decidu-ous (baby) teeth is a more dramatic sign of mat-uration, beginning at around 6 years of age,after eruption of the first molars. Replacementwith adult teeth occurs at a rate of about 4 peryear. The mandibular, then maxillary centralincisors are replaced at the same time or justfollowing the eruption of the first molars ataround 6–7 years of age. After this, the lateralincisors erupt, at around 7–8 years of age. Theadult canines replace the baby teeth at around9–11 years of age, and next to erupt are the firstpremolars and then the second premolars. Thesecond molars erupt at around 12 years of agebut the third molars (wisdom teeth) do noterupt until 17–21 years of age. (Kliegman, 1529)

26. (D) The primary teeth form in dental cryptsthat arise from a band of epithelial cells incor-porated into each developing jaw. Organizationof adjacent mesenchyme takes place in eacharea of epithelial growth, and the two elementstogether comprise the beginning of a tooth. Thefirst primary or deciduous teeth to erupt are thecentral incisors at 5–7 months of age, followedby the lateral incisors at around 7–10 months ofage, and the first molars at around 10–16 monthsof age. The canines do not erupt until 16–20 monthsof age. (Kliegman, 15 21)

27. (C) Ossification centers usually present at birthinclude the distal femur and the proximal tibia.Reference standards for bone maturation facil-itate estimation of bone age. In constitutionalgrowth delay, endocrinologic short stature, and

undernutrition, the bone age is low and is com-parable to the height age. In familial shortstature, the bone age is normal (compared tochronological age). The most commonly usedstandards are those of Gruelich and Pyle,which require radiographs of the left hand andwrist; knee films are sometimes added foryounger children. (Kliegman, 27 71)

28. (C) Most children begin to walk independentlyaround the time of their first birthday; some donot walk until 15 months. At 18 months of age,the infant shows improvements in balance andagility, and the ability to run and climb stairswhile holding a parent’s hand emerges. By30 months of age, the child is able to go upstairsby himself, alternating feet. By 3 years of age,the child has attained the motor coordination toride a tricycle. A child is usually hopping by4 years of age and skipping by 6 years of age.(Dixon, 335, 470)

29. (A) At approximately 18 months, several cog-nitive changes come together to mark the con-clusion of the sensorimotor period. At this time,the infant can feed himself, seek help when introuble, and complain when wet or soiled.From this age forward, the child is increasinglyindependent. By 24 months of age, he helps toundress himself and is able to handle a spoonwell. As a 3-year-old, he helps in dressing andundressing, washes his own hands, andengages in “parallel play.” By age 5, he is ableto fully dress and undress himself. At this timehe is likely to engage in domestic role-playing.(Dixon, 378,406,430, 468)

30. (D) Child abuse and neglect are correlated withdelayed language, particularly the ability toconvey emotional states. Such delays may con-tribute to problems of behavior, socialization,and learning. Mental retardation may firstbecome apparent with delayed speech atapproximately 2 years, although earlier signsmay have been overlooked. Hearing loss is acommon cause of speech delay. Consequently,hearing evaluation should be an integral part ofthe management of delayed speech. The otherchoices listed do not have a proclivity forspeech delay. (Dixon, 400–401)

Page 99: Questions - uhs.edu.kh You suspect Lyme disease, most likely contracted by which of the following? (A) ingestion of unripe fruit (B) ingestion of spoiled fruit (C) drinking of contaminated

100 4: Growth and Development

31. (B) As children mature, they learn what behav-iors are acceptable and how much power theyare able to wield by testing limits. Control is acentral issue. Inability to control some aspect ofthe external world, such as how to make a cer-tain toy work or when to leave, often results ina loss of internal control, that is, a tempertantrum. Fear, overtiredness, or physical dis-comfort can also evoke tantrums. When theyare reinforced by intermittent rewards, as whenthe parent occasionally gives in to the child’sdemands, tantrums can also become anentrenched strategy for exerting control by thetoddler. Tantrums lasting more than 15 minutesor happening regularly more than three timesdaily can reflect underlying medical, emotional, orsocial problems. Tantrums normally appear at theend of the first year of life and peak at 2–4 years.Frequent tantrums after 5 years of age usuallypersist throughout childhood. Clearly, this isan undesirable outcome which, in nearly allchildren, is avoidable. (Dixon, 372–373)

32. (C) A newborn’s birth weight correlates withthe size, nutritional state, and general health ofthe mother and represents the influence of uter-ine constraints on ultimate size. After the first6 months of life, genetic factors influencing ulti-mate height begin to exert their effect. Thegrowth percentile, therefore, may shift signifi-cantly in the first 4–18 months of life. This shiftcan be either up or down. An infant who is smallfor gestational age and has a genetic predispo-sition to larger stature usually experiences accel-erated growth in the first 6 months, and by18 months a stable growth percentile is established.A downward shift is seen in large infants whohave a genetic predisposition to short stature.A stable growth percentile should be establishedby 18 months of age. (Dixon, 230–231)

33. (D) About half of normal children during thesecond year of life are noncompliant whenfaced with a task such as the DDST. This is itselfa developmental phenomenon, reflecting thechild’s movement toward independence andhis newfound ability to resist control and dothings himself. The range of normal on-screen-ing tests such as the DDST is broad, and a scorein the 15–18-month range for an 18-month-old

child is not bothersome. Usually, more formaland complete evaluation is indicated for thechild who is functioning more than one-thirdbelow his chronologic age. (Dixon, 366)

34. (A) In evaluation of academic failure at school,neuropsychological and educational testing iscompleted, as well as audiometry and visualtesting by the school or the pediatrician.Evaluation for attention-deficit hyperactivitydisorder (ADHD) may form part of the evalu-ation for academic failure. Neuroimaging typ-ically is not part of this evaluation process.Although a home visit may be particularlyhelpful, it is not commonly carried out by theschool personnel. (Dixon, 480–485)

35. (C) At birth, the retina is well developed but thelens is rather immobile. Fixation and trackingthrough the visual field are well developed by2 months of age. The length of time that aninfant fixates on a paired visual stimulus hasbeen interpreted as visual preference, and hasalso been correlated with later cognitive devel-opment. (Dixon, 247)

36. (D) Crying gradually increases during the first6–12 weeks of age because it is the main modal-ity by which infants express responses to stim-uli, both aversive and nonaversive. Crying canbe a response to a variety of stimuli, includinghunger, a wet diaper, fear, fatigue, and over-stimulation. Crying gradually decreases after12 weeks of age as the infant develops otherresponses, such as smiling or reaching, orbecomes more adept at self-soothing, such assucking the fingers or the thumb. In the firstweeks of life, however, crying can become adistressing problem for the parents, and cryingassociated with irritability is often labeled ascolic. (Dixon, 234–237)

37. (C) The advent of object constancy correspondswith qualitative changes in social and commu-nicative development. The infant looks backand forth between an approaching strangerand a parent, as if to contrast known fromunknown, and may cling or cry anxiously. Thisoccurs around 8–9 months of age. Separationsoften become more difficult, and infants who

Page 100: Questions - uhs.edu.kh You suspect Lyme disease, most likely contracted by which of the following? (A) ingestion of unripe fruit (B) ingestion of spoiled fruit (C) drinking of contaminated

Answers: 31–47 101

have been sleeping through the night formonths begin to awaken regularly and cry, asthough remembering that parents are in thenext room. (Dixon, 294)

38. (A) Cosleeping is an accepted phenomenon inmany ethnic groups, including African American,Hispanic, and Southeast Asian. It has beenimplicated as a risk factor in sudden infantdeath syndrome (SIDS), especially for infants ofa mother who smokes. Prolonged cosleepingmay predispose children to more significantsleep problems as they get older. Parents, espe-cially fathers, may report less restful sleep.Nonetheless, this issue must be approachedwith sensitivity by the primary care provider.(Kliegman, 1738)

39. (C), 40. (C), 41. (D), 42. (C) Reflex movementbegins in fetal development as early as 9 weeksgestation. However, most of the reflexes associ-ated with the newborn develop between 20 and38 weeks gestation. The Moro reflex disappearsby 7 months of age. Palmar grasp usually disap-pears by 6 months of age, facilitating release ofobjects. The Tonic neck reflex, which is elicitedby turning the infant’s head, resulting in exten-sion of the arm and leg on the side towardwhich the head is turned and flexion of theopposite side, disappears by 8 months of age,unless myelinization or brain developmentis pathologic. The Babinski (upgoing toes)sign, which develops in an infant just prior toterm, does not disappear until 16 months ofage, when adequate myelinization has occurred.(Dixon, 158)

43. (B) Growth acceleration begins in early ado-lescence, although peak growth velocities arenot reached until SMR III or SMR IV. Thepubertal growth spurt begins at about 10 yearsin females and about 12.5 years in males.Different areas of the skeleton attain their peakgrowth at different times. This is seen mostdramatically in the feet, which first develop agrowth spurt. This is followed by a rapidincrease in leg length, and subsequently intruncal growth. Facial growth occurs after peakheight velocity. This asymmetric growth givesthe young adolescent a gawky look. (Dixon, 539)

44. (C) The first sign of puberty in the male, usuallybetween the ages of 10 and 12, is scrotal and tes-ticular growth. The first ejaculation is a notableevent and usually begins 1 year after the begin-ning of testicular growth, but its timing is highlyvariable. Ninety percent of boys have this expe-rience between the ages of 11 and 15 years. Thepubertal growth spurt begins at about 10 yearsin females and about 2 years later in males.Boys have just over 2 more years of preadoles-cent growth than girls do; during this time, leggrowth increases more dramatically than trunkgrowth. Girls have a greater spurt in hip widthrelated to stature than boys do, although boysexceed girls in most other areas of bone growth.The growth spurt begins at SMR II and peaks atSMR IV. (Dixon, 537-539; Kliegman 60-63)

45. (E) The peak velocity of growth among ado-lescent girls corresponds with SMR III, andamong boys with SMR IV. This growth aver-ages 9–10 cm/yr, although this growth is some-what lower in girls than in boys. However, theperiod of pubertal development lasts muchlonger in boys and may not be completed untilthe age of 18 years. Thelarche (breast develop-ment) in girls occurs much earlier than theperiod of peak growth velocity; it is the first vis-ible sign of puberty and may begin as early asage 8 years. (Dixon, 537-54;, Kliegman 62-63)

46. (B) Gynecomastia significant enough to beembarrassing for adolescent males occurs inless than 10% of boys, although some degree ofbreast tissue enlargement is seen in 40%–65% ofadolescent boys in the ages of 14–15. Thisresolves within 2 years on its own in the major-ity of cases, and rarely is pathologic. Lipomastiarefers to soft subcutaneous fat often seen inobese boys and sometimes confused withgynecomastia. (Kliegman, 2385-2386)

47. (C) Most children younger than age 7 cannotreliably distinguish fantasy from reality. This isparticularly relevant given that today’s school-age children spend at least 4 hours in front ofthe TV per day. These young children believethat all the characters act and feel as they areshown on TV, and that the characters live in theTV setting between shows. (Dixon, 425)

Page 101: Questions - uhs.edu.kh You suspect Lyme disease, most likely contracted by which of the following? (A) ingestion of unripe fruit (B) ingestion of spoiled fruit (C) drinking of contaminated

102 4: Growth and Development

48. (C) Postpartum depression is a common occur-rence among new mothers, affecting up to 80%.This usually resolves itself with hormonalchanges back to normal homeostasis, but in 10%a major depression occurs. Depressed parentstend to bring their children to the pediatricianmore often. Teenage mothers are particularlyvulnerable to depression, especially if there ispoor social structure. Even after treatment of aparent with major depression, the children tendto have ongoing problems of functioning.Though maternal psychosis is uncommon, theadjustment of children is related not only totheir exposure to psychiatric symptomatologybut also to more common, longer term patternsof dysfunctional parenting behaviors and aber-rant social interactions. (Dixon, 207–210)

49. (E) The HEADSS inventory assists in the psy-chosocial evaluation of adolescents. Less“sensitive” questions are followed by more sen-sitive ones. Examples of home questions mayinclude “Where do you live, how many siblingsdo you have?” Education questions may include,“What is the best/worst part of school for you?”Activities questions may include, “Do you playany sports or exercise?” as well as “What do youlike to do with your friends?” Questions aboutdrugs may include, “Are you/your friends usingtobacco, marijuana, alcohol, or other drugs?”Sexuality questions may include, “Are any ofyour friends dating someone, and are they get-ting involved physically?” Suicide/depressionquestions may include, “Are you happy withthe way you look and feel?” (Dixon, 550–552)

50. (B) Colic typically begins at 41–42 weeks ges-tational age in both term and preterm infants.There are two patterns of colic, the mostcommon being associated with fussiness begin-ning in the early evening hours and occurringin paroxysms of crying. The second pattern ischaracterized by paroxysms of crying through-out the day in a hyperirritable infant. There areno differences in prevalence of colic amongrace, gender, or method of feeding (breast vsbottle). The presence of colic during infancygives no predictive value for long-term behav-ioral, temperamental, or psychologic outcomes.(Dixon, 233–237)

51. (C) Autism is a neurodevelopmental disordercharacterized by impaired socialization, impairedverbal and nonverbal communication, andrestricted, repetitive patterns of behavior. Earlyrecognition of autism, preferably before thesecond year of life, is critical. Referral criteria inthe second year of life include no pointing orwaving by 18 months, a preference for solitaryplay, diminished eye contact, and an oversen-sitivity to textures and noises. Nystagmus ismore likely to be associated with an ocular disor-der. Lack of use of two-word phrases is a concernafter the age of 2 years. (Dixon, 363–365)

52. (E) Failure to thrive is a major risk factor forlater behavioral and developmental problems.It is diagnosed by persistent and significantdeviation from the growth curves along time.Poverty-stricken children are likely to be affected,at 5%–10% prevalence. Although this couldrepresent an organic disorder, by far the mostcommon cause of failure to thrive is psychoso-cial: the so called nonorganic failure to thrive.(Kliegman 184-187)

53. (D) In children less than 6 years of age, oppo-sitional behavior, rebellion, and lying arecommon as children are just learning to followsocietal rules. Temper tantrums may persistfrom preschoolers up until 5 years of age, usu-ally without pathologic origin. Truancy, orintentional school avoidance, is pathologic atany age. (Dixon, 448–449)

54. (D) Several factors contribute to childhoodaggression. Boys almost universally are reportedto be more aggressive than girls. Larger chil-dren often are more aggressive than smallerones. Children from larger families often aremore aggressive than those from smaller fam-ilies. Conduct-disorder behavior often is asso-ciated with psychopathologic conditions. Bothattention-deficit hyperactivity disorder andborderline personality disorder (BPD) corre-late with aggression. However, ADHD is muchmore common than BPD, so (D) is the bestanswer. Of interest is the correlation in boysbetween severe reading retardation and thedevelopment of aggressive conduct disorder.(Dixon, 446–447)

Page 102: Questions - uhs.edu.kh You suspect Lyme disease, most likely contracted by which of the following? (A) ingestion of unripe fruit (B) ingestion of spoiled fruit (C) drinking of contaminated

Answers: 48–63 103

55. (B) Pica is a chronic eating disorder whichinvolves the eating of nonnutritive substancessuch as dirt, clay, and peeling paint. The age ofonset usually is 1–2 years but may be earlier.This disorder usually remits by middle child-hood. Lack of parental nurturing and mentalretardation are predisposing factors. Childrenwith pica are at an increased risk for lead poi-soning and parasitic infections. Differentialdiagnoses for this behavior include autism,schizophrenia, and certain physical disorderssuch as Kleine-Levin syndrome. (Kliegman, 113)

56. (B) Enuresis, the repeated involuntary or vol-untary discharge of the bladder after the age atwhich bladder control should be maintained, isamong the most common problems brought tothe pediatrician. Among 5 year olds, enuresisoccurs in 7% of males and 3% of females. Twinstudies reveal a marked familial pattern. Enuresisis associated with immigration, socioeconomicdisadvantage, and family psychopathology.Organic pathology can be found in only a verysmall number of cases; physical examination andurine analysis are the only routinely indicatedtests in enuresis. (Kliegman, 113-114, 2249-2250)

57. (E) Contributors to resiliency in childhood orig-inate within the child, the caregiving environ-ment, and the larger sociocultural environment.Child-based contributor include an easy adapt-ability, ability to sustain high levels of activityand to be persistent in achieving goals, a desireto approach and master new situations, and asense of personal autonomy. (Levine, 48)

58. (D) The key criterion to the diagnosis of learningdisability is a significant discrepancy betweenthe child’s estimated intelligence and his or herachievement in one or more learning areas.Although reversal of letters is common in thesechildren, it is not present in all patients and is nota criterion for diagnosis. Furthermore, somereversal of letters may occur in normal childrenas they begin to learn to write. Children withlearning disabilities are usually of average orabove average intelligence, but above averageintelligence is not a criterion for diagnosis.Similarly, visual or auditory perceptual defectsare frequently seen in these patients but are not

criteria for diagnosis. Many of these patients dohave emotional, behavioral, or motivationalproblems in addition to, or in reaction to, theirlearning disability. (Dixon, 488)

59. (E) Major gross motor skill milestones of most3–year-olds include going upstairs alternatingfeet, jumping from the bottom step, pedaling atricycle and standing on 1 foot for greater than3 seconds. A fine motor skill hallmark is copy-ing a circle. It is not until the child is older than4 that he or she can stand on one foot forgreater than 10 seconds. A 2-year-old should beable to build the tower of cubes. (Dixon, 470)

60. (E) Toilet training generally begins around2 years of age. Parents should begin when chil-dren start staying dry for at least 2 hours at atime, are having regular bowel movements, areable to follow simple instructions, are uncom-fortable with dirty diapers and want them to bechanged, are asking to use the potty chair, orare asking to wear regular underwear. Childrenshould not be punished for failures. (Kliegman, 54)

61. (A) Toddlerhood is the period of developingautonomy, when the child normally is seekingto establish his or her own identity and toprove his or her own ability. Toddlers want tocontrol and do everything by themselves. Theyare too impatient and immature to seek oraccept explanations. The toddler’s slogan is“me do it.” (Kliegman, 48)

62. (E) While nighttime awakening by infants ofthis age can be viewed as a setback by parents,the phenomenon is actually a sign that theinfant has acquired an important cognitiveskill, that of object permanence. The approachto nighttime awakening is the same as forestablishing good sleep hygeine; namely that aconsistent bedtime, the use of a transitionalobject, a brief visit to provide reassurance, andputting the baby to sleep while drowsy but stillawake. It is not recommended to feed the infantduring these episodes. (Dixon, 305)

63. (C) Body fat comprises about 16% of bodyweight at birth, increases to about 22% at abouta year of age, and then gradually declines.

Page 103: Questions - uhs.edu.kh You suspect Lyme disease, most likely contracted by which of the following? (A) ingestion of unripe fruit (B) ingestion of spoiled fruit (C) drinking of contaminated

104 4: Growth and Development

Velocity of weight gain slows between the agesof 2 and 5 years, and body mass index (BMI)naturally declines. After approximately age 5,the velocity of weight gain compared to that ofheight gain increases, and BMI increases. Thisis the period known as the adiposity rebound.Children who experience early adiposity rebound(before the age of 5) are at higher risk for obesity.(Kliegman, 234)

64. (C) The gross motor tasks described (walkingdownstairs with alternate feet, throwing overhand,broad jump) are accomplished at 3–4 years ofage. A child of this age also should be able toidentify two or three colors. The other abilitieslisted generally come at, or after, 5 years of age.(Note: This format is common in pediatricexamination questions about development. Thestudent should recognize that often, as is thecase in this question, the correct answer canbe surmised even if the reader cannot identifythe age of the child described in the body of thequestion. Logically, in this case, the correct choicemust be the one corresponding to the youngestage. The ability to name colors precedes all theother items listed and therefore must be thecorrect answer. Knowing the sequence of devel-opment without knowing the correspondingages would be sufficient to answer this ques-tion.) (Dixon, 468-470)

65. (E) Daytime experiences or recall of nightmaresmay precipitate nighttime fears in young chil-dren. Parents must remember that the child’sfear is real, even if the object of the fear is not.Thus, it is not advisable to reinforce the child’sfear of monsters in the bedroom with a pre-tend “search” for them. Having a consistentbedtime for the child, a regular pre-bedtimeroutine, avoidance of TV and videos beforebedtime, and providing brief comfort and reas-surance are all recommended techniques formanaging this issue. (Dixon, 421)

66. (C) Many parents ask their child’s physician ifhe or she should be kept back a year frombeginning kindergarten. With a very few excep-tions, the answer to this question is no.Children who may have actual special chal-lenges need evaluation as soon as possible, and

this is usually most appropriately performedwithin the school system. There is no evidenceto suggest that delaying school entry is corre-lated with academic success once the child doesenter school. The child’s previous socializationand learning experiences should be reviewed.Generally, public schools have more access tospecialized educational services than do pri-vate schools. Children short and/or small fortheir age should not delay kindergarten entry.(Dixon, 464–465)

67. (D) At term birth (40 weeks gestation), the per-centage of fetal hemoglobin varies greatly frominfant to infant but usually is between 60% and90%. Fetal hemoglobin binds more tightly tooxygen than does adult hemoglobin, giving thenewborn infant a relative advantage in pickingup oxygen in the lung but a disadvantage inreleasing oxygen to the tissues. The presence ofsignificant amounts of fetal hemoglobin (whichprotects the sickle erythrocyte against sickling) isthe major reason that young infants with sicklecell disease rarely are symptomatic. Oxygenatedfetal hemoglobin is red, just like adult hemoglo-bin. Fetal hemoglobin does not predispose thered cell to hemolysis. (Kliegman, 2001)

68. (D) Both heart rate and respiratory rate aregreater in infants and young children than inadults, reflecting the relatively larger surfacearea and higher metabolic rate of the youngsters.The respiratory rate of most normal 1-year-oldchildren is between 20 and 30 times per minute.This is slower than a neonate and faster than anolder child or adult. (Kliegman, 389 )

69. (A) For the reasons explained in Answer 68above, the infant normally has a more rapidheart rate than an adult. Rates of up to 180 beatsper minute can be normal in the first year oflife. This increased rate is associated with botha shorter respiratory rate and a shorter PRinterval on the electrocardiogram. (Kliegman,1866-1868)

70. (D) The blood pressure of a normal 2-year-oldchild averages about 95/60 mm Hg. There isalmost no change in normal blood pressurevalues between 2 and 6 years, but after 6 years

Page 104: Questions - uhs.edu.kh You suspect Lyme disease, most likely contracted by which of the following? (A) ingestion of unripe fruit (B) ingestion of spoiled fruit (C) drinking of contaminated

Answers: 64–77 105

there is a gradual increase to an average of120/75 mm Hg at 16 years. It is generally rec-ommended that routine measurement of bloodpressure in children commence at 2–3 years ofage; however, blood pressure should be meas-ured in younger children, including neonates,whenever clinically indicated. (Kliegman, 389)

71. (C) The cerebrospinal fluid concentration of pro-tein in the immediate newborn period normallymay be as high as 125 mg/dL. The concentrationgradually falls to “normal values” of less than45 mg/dL by 6–8 weeks. (Kliegman, 2441)

72. (A) The physician needs to distinguish speech(pronunciation, articulation, and fluency) fromlanguage (content, meaning, vocabulary, andgrammar) and to evaluate each separately.Normally, both speech and language are suffi-ciently developed by 4 years so that the child’sverbal communications are fully understandable,even by strangers. Mispronunciations and gram-matical errors, however, remain common untilabout 41/2 years or even 5 years. (Dixon, 438, 461)

73. (D) Approximately 6%–9% of school-age chil-dren seen in primary care have symptoms con-sistent with attention-deficit hyperactivitydisorder (ADHD). These children most com-monly have a constellation of inattentive, hyper-active, and impulsive traits. Among these arefidgeting, excessive talking, apparent lack ofattention when given directions, distractability,and symptoms reproducible in more than oneenvironment (ie, school and home). (Dixon,482–488)

74. (E) Approximately 50% of children will expe-rience the divorce of their parents by midado-lescence. Adverse effects may vary by age andgender, and include regressive behavior,depression, anger, anxiety, declining academicperformance, and other externalizing and inter-nalizing behaviors. At a time when childrenneed their parents the most, parents may beemotionally unavailable to them as they processtheir own anger, grief, and other emotions. Boysin general appear to have a more difficult initialadjustment than do girls. However, as they

reach young adulthood, girls may experience a“sleeper” effect; though their initial adjustmentoccurred years before, they may undergo anxietyabout male-female relationships. In a longitudinalstudy, nearly 40% of children were doing poorly(academic problems, behavior problems at schooland/or home) 5 years after divorce. (Dixon, 628–629)

75. (C) Despite wide variations in size and rate ofclosure, it is generally accepted that the anteriorfontanel closes sometime between 9 and18 months. That is, by this time, it is composedof cartilage and bone and no longer is palpableas a soft spot. Early closure may be indicativeof a disorder such as premature cranial synos-tosis. Late closure may be seen in conditionssuch as hypothyroidism, rickets, hypophosphata-sia, hydrocephalus, or trisomy 18 syndrome.(Kliegman, 2434)

76. (B) An awareness of normal infant develop-mental milestones can assist providers in tai-loring their guidance topics. It is appropriate toaddress rolling off elevated surfaces, as manyinfants begin rolling over at 3 months orslightly earlier. The typical 3-month-old infantis developmentally incapable of picking upsmall objects and putting them in his mouth.Infants who are growing well should beallowed to sleep through the night. Infantsshould be placed on their backs to sleep, as thissleeping position has been associated withfewer cases of sudden infant death syndrome.At this age, typically developing 3-month- oldsare cooing, smiling and laughing spontaneously.(Dixon, 255, 264)

77. (A) Approximately 20% of sexually active teengirls become pregnant each year in the UnitedStates. Slightly more than half of these preg-nancies result in a live birth, one-third areended by abortion, and 15% end by miscar-riage. Several factors have been associated withan increased risk of teen pregnancy, includinglack of parental supervision, living in poverty,and drug or alcohol use. Adolescent girls withpoor school performance are more likely tobecome pregnant than those with high aca-demic performance. (Dixon, 574)

Page 105: Questions - uhs.edu.kh You suspect Lyme disease, most likely contracted by which of the following? (A) ingestion of unripe fruit (B) ingestion of spoiled fruit (C) drinking of contaminated

106 4: Growth and Development

78. (E) About 50% of patients with isolated growthhormone (GH) deficiency grow normallyduring the first year of life. Growth then decel-erates, and both height and weight fall furtherand further away from the normal curves. Boneage generally is delayed. By definition, in iso-lated GH deficiency other endocrine functionsare normal; there is no associated disturbanceof thyroid function. Before concluding that achild has isolated growth hormone deficiency,however, it is necessary to rule out physicaldestruction of the pituitary gland (eg, by a cran-iopharyngioma), or an associated pituitaryabnormality such as ACTH deficiency. Isolatedgrowth hormone deficiency can be sporadic orgenetic. (Note: The astute student will realizethat it is very unlikely that statements (C) and(E) are both correct and will concentrate onthese, even if uncertain about the remainingchoices.) (Kliegman, 2295-2297)

79. (C) Children with constitutional growth delayare initially short, but have a longer thannormal period of growth and a later thannormal adolescent growth spurt. Althoughthere are some exceptions, ultimate height usu-ally is normal, and ultimate weight is normaland proportionate to height. (Kliegman, 2297)

80. (E) Remarkable as it seems, American childrenspend an average of 30–40 h/week in front ofthe television set, whereas, in general, theyspend only 25–30 h/week in school. Televisionhas a major influence on children’s knowledge,attitude, and behavior. The nature and qualityof the program material is, of course, as mucha problem as the volume. There are consider-able concerns and some data that such exces-sive viewing adversely affects children’sattitudes toward violence, gender roles, con-cepts of racial stereotypes, and commercialism.(Dixon, 425)

81. (D) Failure to thrive typically refers to childrenunder the age of 5 whose growth parametersfall below the lower limits of the normal rangeset by the National Center for Health Statistics.There are many causes of failure to thrive,including psychosocial, medical (organic),nutritional, and developmental. Cystic fibrosis

is an important cause of failure to thrive andshould be excluded in such cases. Childrenwith small atrial septal defects, acute bacterialmeningitis, mild intermittent asthma, or recur-rent otitis media generally do not fail to thriveunless they have another underlying condition.(Parker, 183–187)

82. (E) Adolescence is the period of psychosocialtransition from childhood to adulthood.Puberty is a physiologic event. Adolescence isa psychosocial phenomenon, albeit stronglyinfluenced by the child’s reaction to the phys-ical and physiologic changes of puberty.Adolescence cannot be defined by a fixed tem-poral relation to puberty. Often, adolescenceappears to occur some time after the onset ofpuberty. (Dixon, 535–536)

83. (E) Although adolescence is characteristically adifficult period of psychosocial and emotionalgrowth, change, and adjustment, currentunderstanding of this period indicates thatemotional turmoil, disruptive behavior, andfamily crises are not the norm and, when pres-ent, represent significant pathology. Decliningscholastic grades are always worrisome. Suchbehavior warrants investigation and interven-tion rather than acceptance and reassurance.The differential diagnosis includes emotionalproblems, maladjustment, psychosis, drugabuse, and, rarely, organic disease. (Dixon, 545)

84. (E) Virtually all children bite sometime duringtheir first 3 years of life. Boys are more likely tobite than girls, and children tend to bite otherswhen they are in social situations beyond theircoping abilities. In assessing the issue of biting,it is important to determine the age at whichthe biting began, the situations in which itoccurs, how parents and daycare staff havemanaged biting, and if any concerns have beenraised about the child’s developmental progress.The age of first tooth eruption is not helpful.(Parker, 136–138)

85. (C) The average hemoglobin value at birth isabout 17 g/dL. It then falls rapidly over thenext 2–3 months to a low of about 11 g/dL in theterm infant. Hemoglobin values then gradually

Page 106: Questions - uhs.edu.kh You suspect Lyme disease, most likely contracted by which of the following? (A) ingestion of unripe fruit (B) ingestion of spoiled fruit (C) drinking of contaminated

Answers: 78–94 107

rise, although remaining below adult values untilthe early teen years. The mean value at 1 year ofage is about 12 g/dL. (Kliegman, 2003, 2025–2026)

86. (D) Normal newborns have a surprising arrayof sensory abilities. They can sense pain and arealso responsive to changes in their body posi-tion as they are moved by caregivers. Theyprefer their mother’s face and voice over thefaces and voices of other mothers. Newbornsreflexively grasp the finger of an adult whenthe finger is placed in their palm. Althoughnewborns can see objects 8–10 in away and cantrack slow-moving objects, they are near-sighted, and their visual acuity ranges from20/200 to 20/400. The visual acuity of a childdoes not reach 20/20 until age 4. (Davies,120–122; Kliegman, 2569: 44; Dixon, 149)

87. (B) Several techniques have been studied toimprove childrens’ behavior. Time in is “catch-ing a child being good,” providing mainly brief,nonverbal physical contact by the caregiverwhen the child is behaving well. Reinforcementrefers to an item or activity for which a childworks to gain access. Fading refers to graduallyremoving an object or activity from a child’senvironment rather than abruptly changing it.Rapprochement is defined as the normal tran-sition from independent play to a period ofclinging to the parent when the 18–24-month-old is around other children and adults. Incidentallearning occurs when a child is in the presenceof individuals who repeatedly practice the samebehavior, such as smoking. Children of smokersare more likely to smoke themselves than chil-dren of nonsmokers. (Parker, 55–57; Dixon, 367)

88. (C) Although there is great variation within thenormal range, the average infant roughlytriples his or her birth weight by the first birth-day. This means an increase from an average of7 lb (3.25 kg) at birth to 22 lb (10 kg) at 1 year.Thus, the weight gain during the first year oflife is about 15 lb (6 kg). (Kliegman, 46)

89. (A) As with weight gain, gain in height duringthe second year of life is considerably less thanduring the first year. During the first year, the

child grows about 25 cm (10 in). Between thefirst and second birthdays, the youngster growsan average of only 13 cm (about 5 in), abouthalf that of the first year. In the 4–6-year-old, theaverage height gain is 3 cm/yr. (Kliegman, 54)

90. (C) Head circumference increases relativelyrapidly after birth, growing about 2 cm (1 in)the first month. Of course, this is related to therapid growth of the brain that occurs duringinfancy. The rate of growth in head circumfer-ence in the first 2 months is 1 cm every 2 weeks.In the third through sixth month of life, therate of growth is 1 cm/month. (Dixon, 248)

91. (D) Menarche usually occurs as, or shortlyafter, breast and pubic hair development reachSMR IV. Menarche generally follows ratherthan precedes the adolescent growth spurt. Infact, for most girls, menarche heralds the end ofthe adolescent growth spurt, and there is littlefurther increase in height following menarche.(Dixon, 538-53; Kliegman, 61, 64)

92. (E) A child’s temperament is an important con-cept to consider when evaluating parental con-cerns. It is not the parent’s response to a tantrum,but a child’s stable, individual modes of respond-ing to his or her environment. Temperamenttypes can be used to explain current and forecastfuture behavior. The emotional parent-childbond is known as attachment. A toddler exertsautonomy when acting independently. A tod-dler’s emotional clinginess to the parent whenfaced with other children or adults is known asrapprochement. (Dixon, 37, 356–357)

93. (A) The Parents’ Evaluations of DevelopmentalStatus (PEDS) consists of 10 questions for par-ents of newborns to 8-year-olds. The PEDS iden-tifies levels of risk for various emotional anddevelopmental problems and delays. (Dixon, 729)

94. (D) The Pediatric Symptom Checklist consistsof 35 brief statements of problem behavior,including both externalizing (conduct) andinternalizing (depression) behavior. It takesabout 7 minutes to administer and has a highsensitivity rate. (Dixon, 730).

Page 107: Questions - uhs.edu.kh You suspect Lyme disease, most likely contracted by which of the following? (A) ingestion of unripe fruit (B) ingestion of spoiled fruit (C) drinking of contaminated

Hagan JF, Shaw JS, Duncan P, eds. Bright Futures:Guidelines for Health Supervision of Infants, Children andAdolescents. 3rd ed. Arlington, VA: National Center forEducation in Maternal and Child Health; 2008.

108 4: Growth and Development

95. (B) One of the most widely used tools for detec-tion of substance misuse in the pediatric officesetting is the CAGE questionnaire. C standsfor “Have you ever felt the need to CUT downon your alcohol (or drug) use?” A stands for“Have you ever felt ANNOYED by criticismabout your use?” G stands for “Have you everfelt GUILTY about your use, or GUILTY aboutsomething you said or did while you wereusing?” Finally, E stands for “Have you everneeded an EYE-OPENER in the morning aftera night of heavy use?” (Dixon, 615-616)

96. (E) In the United States, it is recommended thatbirth hospitals within all states perform a hear-ing screen before neonates are discharged. Theotoacoustic emission test (OAE) is quick, easyto administer, inexpensive, and provides a sen-sitive indication of the presence of hearing loss.(Kliegman, 2623)

97. (C) The Vanderbilt Assessment Scale consists ofquestionnaires for teachers and parents as wellas a primary care provider diagnostic tool.Teachers are prompted to observe and assessADHD symptoms and the child’s school andbehavioral performance. The parent tools

assesses 55 different behaviors and their occur-rence over the past 6 months. (Parker, 118)

98. (E) Signs that a child is ready for toilet trainingusually appear between 18 and 24 months, andthe process of becoming fully trained may takeup to a year beyond initial signs of readiness.These clues include increased periods of day-time dryness and signs of interest in using thetoilet. (Dixon, 360)

99. (B) Separation protest or “anxiety” typicallysurfaces between 6 and 9 months, and ismarked by an infant’s crying when his parentor caregiver leaves his presence. Most separa-tion anxiety resolves naturally, as the infantdevelops cognitively and learns that absencesare followed by reunions when parents return.(Levine, 27–28)

100. (C) Self-feeding, although messy and disorgan-ized, represents a milestone in independencefor the young toddler. Fine motor skills, hand-eye coordination, and learning to select differ-ent food textures and colors are several of thedevelopmental skills enhanced by self-feeding.(Dixon, 360–361)

SELECTED READINGS

Page 108: Questions - uhs.edu.kh You suspect Lyme disease, most likely contracted by which of the following? (A) ingestion of unripe fruit (B) ingestion of spoiled fruit (C) drinking of contaminated

CHAPTER 5

Feeding and NutritionSara Viessman, MD

Mary Stahl Levick, MD

William J. Klish, MD

109

Historically, as well as currently, the topic of feeding and nutrition has special importance in pediatricsbecause of the rapid growth and development of the pediatric patient, especially the infant. This rapid growthand development results in both quantitatively and qualitatively different nutritional needs than exist for theadult. Quantitatively, for example, newborns and young infants require more calories and more proteinrelative to body size than do older individuals. In a qualitative sense, for example, certain amino acids areessential for low-birth-weight and preterm infants but not for children or adults or even for normal terminfants. Certain fats are essential for brain growth during the first few years of life but not thereafter.

Breastfeeding clearly is the healthiest choice for most babies. We currently are aware of many nutritional,psychologic, and immunologic advantages of breastfeeding. Further advantages likely will be uncovered inyears to come. It is estimated that if all babies were breastfed for the first 12 weeks of life, the savings recog-nized by the United States would be about $3 billion per year. If all mothers in the WIC program breastfedtheir babies for only 4 weeks, an estimated $30 million would be saved. If all babies worldwide were breastfedfor the first six months of life, hundreds of thousands of babies lives would be saved each year.

Malnutrition, secondary to medical or socioeconomic conditions, remains an important problem ininfants and children in the United States. Self-inflicted malnutrition seen with eating disorders (or disordered eat-ing) is increasingly prevalent. Obesity now is at epidemic proportions among children in the United States.At both ends of this spectrum, children are at risk for serious long-term medical and psychologic problems.

Although specific nutritional deficiencies, except for iron deficiency, are uncommon in developed coun-tries, deficiencies of vitamins or protein remain a major problem among children in developing countries.

Page 109: Questions - uhs.edu.kh You suspect Lyme disease, most likely contracted by which of the following? (A) ingestion of unripe fruit (B) ingestion of spoiled fruit (C) drinking of contaminated

110

DIRECTIONS (Questions 1 through 34): For eachof the multiple choice questions in this sectionselect the one lettered answer that is the bestresponse in each case.

1. A 3-year-old male child from South America isbrought in for an adoption physical examina-tion. He has a 4–5 mm, foamy, mildly vascular,conjuctival lesion next to the cornea on themedial side of both eyes. Which nutrient defi-ciency is most likely to cause this lesion?

(A) vitamin A(B) vitamin B6

(C) thiamine(D) copper(E) riboflavin

2. Parents appear in your office to discuss theirconsiderations for adopting their 11-month-oldfoster son. They state he has a history of failureto thrive at 6 months of age and they are con-cerned about the possibility of long-term prob-lems. Undernutrition in the first year of life haswhich of the following effects?

(A) has no permanent effect on physicalgrowth or development of intelligence

(B) can have permanent effects on physicalgrowth but not on development ofintelligence

(C) can have permanent effects on develop-ment of intelligence but not on physicalgrowth

(D) can have permanent effects on bothphysical growth and development ofintelligence

(E) can have permanent effects on bothphysical growth and development ofintelligence, but only if coupled withpsychosocial deprivation

3. At a 4-month well child visit, a mother is dis-tressed because her baby’s grandmother believesshe should have started feeding her baby solidfood at 2 months of age. It is generally recom-mended that beikost (infant foods other thanmilk) be introduced into the infant’s diet atabout what age?

(A) 3 weeks(B) 6 weeks(C) 3 months(D) 6 months(E) 1 year

4. A 23-month-old female is hospitalized forsevere malnutrition. Which of the followingproblems is most likely to be the result of inap-propriately rapid treatment of the severely mal-nourished child?

(A) hyperkalemia(B) hyponatremia(C) hyperglycemia(D) congestive heart failure(E) renal failure

5. A 10-week-old child weighing 5 kg is beingfed commercial infant formula. The mother isconcerned she is underfeeding her baby. You tellher that, to satisfy both his fluid and caloricrequirements, the daily intake ought to be at leasthow many ounces?

Questions

Page 110: Questions - uhs.edu.kh You suspect Lyme disease, most likely contracted by which of the following? (A) ingestion of unripe fruit (B) ingestion of spoiled fruit (C) drinking of contaminated

Questions: 1–11 111

(A) 12 oz(B) 18 oz(C) 26 oz(D) 36 oz(E) 48 oz

6. A pregnant African-American woman plans tobreastfeed her baby. She has read of the nutri-tional, psychologic, and immunologic advan-tages, but recalled breastfed babies might stillrequire vitamin supplementation. The most cur-rent nutritional recommendations indicate sheshould supplement the baby’s nutrition withwhich vitamin?

(A) vitamin A(B) vitamin B(C) vitamin C(D) vitamin B1

(E) vitamin D

7. You are in charge of the nutrition support serv-ice in the hospital where you are practicing.The company that makes the vitamin/mineralmixture for total parenteral nutrition (TPN) hasgone out of business and for the past 2 monthsyou have been trying to fortify the parenteralnutrition mixtures through other sources. An8-month-old infant with short gut, who hasbeen on TPN for the past 4 months, presentswith a curious erythematous desquamatingrash around his mouth, buttocks, and a smallamount on his hands and feet. Which nutrientdeficiency is the most likely cause?

(A) vitamin A(B) thiamin(C) riboflavin(D) copper(E) zinc

8. Aphysical examination of a 10-year-old Hispanicmale reveals BMI greater than 95th per-centile. Laboratory test results include fastingserum glucose of 98 mg/dL, fasting seruminsulin of 30 µU/mL, an ALT of 60 U/L, totalcholesterol of 128 mg/dL, and triglycerides of130 mg/dL. You have great concern that hemay have which of the following?

(A) diabetes(B) prediabetes(C) nonalcoholic steatohepatitis(D) metabolic syndrome(E) hyperlipidemia

Questions 9 through 11

At 1 year of age, a boy was at the 50th percentile forheight and weight. At 2 years of age, he is at the 25thand 10th percentiles respectively. Review of systemsreveals fussiness, loose stools, and possibly stomachaches all beginning after the mother stopped breast-feeding the boy and introduced table foods. Motherhas consumed milk products lifelong, but her sondoes not drink cow milk.

9. Which is the most likely cause of these symptoms?

(A) toddler’s diarrhea(B) lactose intolerance(C) celiac disease(D) cow milk protein allergy(E) chronic giardiasis

10. The most definitive test for this condition iswhich of the following?

(A) breath hydrogen test(B) abdominal ultrasound(C) serum antigliadin antibodies(D) giardia antigen(E) small bowel biopsy

11. Patients with this condition should avoidwhich of the following?

(A) corn(B) barley and rye(C) rice and legumes(D) oats(E) potatoes and legumes

Page 111: Questions - uhs.edu.kh You suspect Lyme disease, most likely contracted by which of the following? (A) ingestion of unripe fruit (B) ingestion of spoiled fruit (C) drinking of contaminated

112 5: Feeding and Nutrition

Questions 12 and 13

A 5-year-old female presents with a 3-day history ofcrampy abdominal pain and a rash on her lowerlegs. On examination, she is cranky but nontoxic.Her abdomen is mildly diffusely tender with normalbowel sounds. A purpuric rash is located predomi-nately on the extensor surface of lower legs. A com-plete blood count and urine analysis are normal.

12. Which of the following is the best next step?

(A) intravenous corticosteroid therapy(B) intravenous antibiotics(C) hematology-oncology consult(D) sickle cell screening(E) colonoscopy

13. While hospitalized, the child’s abdominal painincreases. She becomes pale and draws up herlegs. On examination, distension of the rightupper quadrant with palpable fullness isfound. Which of the following is true of thiscomplication?

(A) recurrence occurs in at least 15% of children

(B) rupture of the appendix may be an asso-ciated complication

(C) usually starts with stool containingblood-stained mucus

(D) ileoileal involvement is typical(E) highest occurrence is in this age group

14. Because her son prefers other beverages overmilk, a school-age boy’s mother is concerned hemay not be getting enough calcium in his diet.As you recommend calcium-rich foods, youmention the calcium requirement for a school-age child is which of the following?

(A) 0.1 g/day(B) 1.3 g/day(C) 10.5 g/day(D) 50.0 g/day(E) 100 g/day

15. During a preschool physical, the mother of a5-year-old girl states the family follows a strictvegan diet (a diet excluding all animal products

including eggs, milk, and milk products, as wellas meats and fish). In which of the followingvitamins is this child likely to be deficient?

(A) vitamin C(B) vitamin E(C) vitamin B1

(D) vitamin B12

(E) vitamin A

16. At a 2-month well child visit, a mother statesher family has a history of cow milk proteinallergies and wonders if soy formula would bebetter for her baby. Which of the followingstatements regarding soy formula is correct?

(A) Infants fed exclusively with soy formuladisplay growth comparable to infantsfed cow milk protein formula.

(B) The protein in soy formulas is essen-tially nonallergenic, and clinically signif-icant soy protein hypersensitivity isextremely rare.

(C) Soy formula is most useful in childrenwith well-documented, severe, gastroin-testinal allergic reactions to cow milkprotein.

(D) Soy formula should not be used bypatients with a family history of celiacdisease.

(E) Infants fed soy formula should receivesupplemental dietary calcium.

17. A 5-month-old male presents with poor weightgain. His diet consists only of goat milk. Onexamination, he appears tired and is mildlytachycardic. The laboratory value most likely tobe elevated is which of the following?

(A) serum albumin(B) red blood cell mean corpuscular volume(C) hemoglobin(D) serum vitamin B12 level(E) serum sodium

18. A 12-year-old female presents for her well-childvisit with no complaints. However, she is con-cerned about her friend who counts caloriesand seems to be losing too much weight.During a discussion about this issue, you tell

Page 112: Questions - uhs.edu.kh You suspect Lyme disease, most likely contracted by which of the following? (A) ingestion of unripe fruit (B) ingestion of spoiled fruit (C) drinking of contaminated

Questions: 12–22 113

her that the recommended energy intake for a12-year-old female is which of the following?

(A) 1500 kcal/day(B) 1800 kcal/day(C) 2200 kcal/day(D) 2500 kcal/day(E) 3000 kcal/day

19. An 8-month-old female is hospitalized with fail-ure to thrive. She has a 1-day history of fever andcough. On physical examination, you observe avery thin, well-hydrated alert infant in no acutedistress. You obtain a complete blood count andorder a chest x-ray for further evaluation of aone day history of fever and cough. You note thelung fields to be normal but also note an absenceof a thymic shadow. Which of the followingshould be your next step?

(A) obtain an immunology consult(B) order genetic karyotyping(C) repeat the chest x-ray in 24 hours(D) order a cardiac ultrasound(E) feed the infant as tolerated, following

weight and intake closely

20. A single, 35-year-old pregnant female has notbeen tested previously for HIV or hepatitis.While awaiting prenatal screening, she requestsinformation regarding breastfeeding. You wouldmost accurately tell her in the United Statesbreastfeeding of infants is contraindicated inwhich of the following?

(A) an infant whose mother is seropositivefor HIV antibody

(B) an infant whose mother is seropositivefor hepatitis B surface antigen

(C) an infant whose mother is seropositivefor hepatitis C antibody

(D) an infant whose mother was recentlyimmunized with tetanus, diphtheria,acellular pertussis vaccine

(E) an infant whose mother was recentlyimmunized with hepatitis B vaccine

21. A new mother at her baby’s 2-week well visit isconcerned that she may not be able to continuebreastfeeding because her nipples are very soreand cracked, and have even bled. Inquiringabout her feeding techniques, you discover shenurses her baby about 20–25 minutes on eachbreast each feeding. Which of the followingoffers the best supportive advice?

(A) Lengthen the time interval betweenevery nursing by at least an hour.

(B) Nurse for a minimum of 30 minutes oneach breast each feeding.

(C) Offer 1 oz of formula from a bottlebefore each breastfeeding so that herinfant will not be as ravenous.

(D) Limit nursing to 5–10 minutes per breastper feeding.

(E) Substitute a bottle of formula for everyother nursing just until her nipples arehealed.

22. A mother is concerned because her 20-month-old son prefers to eat with his fingers ratherthan use a small spoon. Which of the followingstatements is correct?

(A) At 18 months of age, most toddlers pre-fer to feed themselves with a spoon.

(B) Most children learn to feed themselvesindependently during the second yearof life.

(C) Self-spoon-feeding usually begins at10 months of age when well-definedwrist rotation develops.

(D) Most children can feed themselves witha cup by 6 months of age.

(E) Most children prefer to feed themselveswith a spoon by 12 months of age.

Page 113: Questions - uhs.edu.kh You suspect Lyme disease, most likely contracted by which of the following? (A) ingestion of unripe fruit (B) ingestion of spoiled fruit (C) drinking of contaminated

114 5: Feeding and Nutrition

23. You are following a child with ADHD. Histeacher requests information on diet andADHD. In particular, she is interested in therole of ingestion of sugar in this condition.Which of the following statements is trueregarding controlled studies of the effects ofsucrose ingestion in children?

(A) Sucrose ingestion has been linked toproblem behavior in children withADHD.

(B) Sucrose ingestion has been linked toincreased motor activity in childrenwith ADHD.

(C) Sucrose ingestion is associated with anincrease in both motor activity andproblem behavior in children withADHD.

(D) Sucrose ingestion is not associated withproblem behavior or increased motoractivity in children.

(E) Sucrose ingestion is associated withdecreased motor activity in children.

24. You are in a health clinic in Africa. An 18-month-old child is brought in for your help. On inspectionyou note that the child is edematous, has a darkdesquamating skin rash over most of the pres-sure points on the body, has very thin hair,and what hair is left is reddish in color. Of thefollowing, what laboratory test is most likely tobe present?

(A) serum albumin of 3.7 g/dL(B) serum sodium of 143 meq/L(C) serum albumin of 2.3 g/dL(D) serum potassium of 2 meq/L(E) serum prealbumin of 25 mg/dL

25. Recent concerns about the prevention of ather-osclerotic heart disease have led to recommen-dations for reducing the fat content, andparticularly the cholesterol content, of the dietearly in life. This can include the use of skimmilk. It is recommended that the use of skimmilk begin no earlier what age?

(A) 4 months(B) 6 months

(C) 1 year(D) 2 years(E) 5 years

26. Because of her own success with a low carbo-hydrate diet, the mother of an overweight 9-year-old asks about a low carbohydrate dietfor her son. As you discuss this with her, youpoint out that the greatest percentage of calo-ries in the normal diet of the school-age childcomes from which of the following?

(A) carbohydrate(B) protein(C) fat(D) cholesterol(E) whey

27. A 3-month-old Caucasian female is broughtto you for cold symptoms. On examination,you note her head circumference is at the 50thpercentile, her length at the 25th percentile, andher weight less than the 5th percentile. Sheappears alert but very thin. She has skin foldson her arms, thighs, and buttocks. Parents stateshe drinks four 8 oz bottles of premixed for-mula each day. You feed the baby and shequickly drinks 7 oz. You decide to admit theinfant. How many calories will she likely needfor weight gain?

(A) 60 kcal/kg/day(B) 80 kcal/kg/day(C) 100 kcal/kg/day(D) 150 kcal/kg/day(E) 200 kcal/kg/day

28. The mother of a headstrong 2-year-old is con-cerned that he insists on drinking six bottles ofapple juice per day. Which of the following rep-resents a serious nutritional concern in thissituation?

(A) development of obesity(B) development of diabetes mellitus(C) vitamin C deficiency(D) vitamin A toxicity(E) development of constipation

Page 114: Questions - uhs.edu.kh You suspect Lyme disease, most likely contracted by which of the following? (A) ingestion of unripe fruit (B) ingestion of spoiled fruit (C) drinking of contaminated

Questions: 23–34 115

29. A 3-year-old male presents to your clinic for thefirst time for a well-child visit. You note macro-cephaly, short fingers, and only six teeth. Whatadditional finding is likely?

(A) absent thumb(B) flat facial bones(C) hepatosplenomegaly(D) absent clavicles(E) absent radius

30. An 18-month-old Hispanic male presents for awell-child check. His parents report he drinksseven to eight bottles of cow milk each day anddoes not like solid foods. His weight is greaterthan the 95th percentile while height and headcircumference are at the 25th percentile. Hisheart rate is 180 beats per minute and respira-tory rate is 28 breaths per minute. On exami-nation, you find a chubby, happy, pale, andotherwise normal baby. Which of the followingtests will most likely be abnormal?

(A) sweat test(B) serum sodium(C) eosinophil count(D) serum bicarbonate(E) hemoglobin

31. A 7-year-old male presents for a well-childvisit. You note his height is at 75th percentileand weight is at greater than 95th percentilefor age. His examination is otherwise normal.His father is overweight but his mother is thin.Which of the following is the most appropriatenext step for you to take?

(A) obtain T4 and TSH levels(B) order serum chromosomes(C) refer to an endocrinologist(D) place on a diet of 75% of caloric needs

for height(E) counsel the family on increasing exer-

cise and eliminating target foods fromdiet

32. The most accurate method for assessing adi-posity in the office setting is which of the fol-lowing?

(A) measurement of weight(B) calculation of percent above ideal body

weight for height using age/sex norms(C) calculation of body mass index

(weight/height2)(D) measurement of subcutaneous fat

thickness(E) use of densitometry

33. A 2-year-old Caucasian male presents with fail-ure to thrive, chronic diarrhea, and recurrentpneumonia. Though his family history is neg-ative for cystic fibrosis, a sweat test reveals asodium concentration of 120 mg/dL. Which ofthe following is the appropriate next step incaring for this infant?

(A) iron(B) vitamin B12 and folic acid(C) pancreatic enzyme supplementation(D) copper and magnesium(E) parenteral diuretics

34. A 15-year-old Caucasian female presents withthe complaint that she is always cold. She hasno history of illness, but she has not had aperiod for 4 months. Her mother states she isworried because although her daughter inter-mittently eats a large amount of food, sheseems to be losing weight. On examination, thepatient is extremely thin for her age. Which ofthe following is most likely?

(A) She feels she is too thin, but likes it thatway.

(B) She feels she is too thin, and would liketo gain weight.

(C) She feels she is too fat, and wants to loseweight.

(D) She feels she is too fat, but likes it thatway.

(E) She feels she is normal.

Page 115: Questions - uhs.edu.kh You suspect Lyme disease, most likely contracted by which of the following? (A) ingestion of unripe fruit (B) ingestion of spoiled fruit (C) drinking of contaminated

116 5: Feeding and Nutrition

DIRECTIONS (Questions 35 through 39): The fol-lowing group of questions is preceded by a list oflettered answer options. For each question, matchthe one lettered option that is most closely associ-ated with the question. Each lettered option maybe selected once, multiple times, or not at all.

Questions 35 through 37

(A) carbohydrate(B) protein(C) fat(D) cholesterol(E) whey

35. The greatest percentage of calories in humanbreast milk

36. The greatest percentage of calories in unmodi-fied cow milk

37. The greatest percentage of calories in skim milk

Questions 38 and 39

(A) 2%(B) 10%(C) 15%(D) 25%(E) 40%

38. Percentage of all child deaths related to mal-nutrition in developing countries

39. Percentage of children in the world who aremoderately to severely underweight

Page 116: Questions - uhs.edu.kh You suspect Lyme disease, most likely contracted by which of the following? (A) ingestion of unripe fruit (B) ingestion of spoiled fruit (C) drinking of contaminated

117

1. (A) Vitamin A deficiency remains the leadingcause of preventable childhood blindnessworldwide. Though rare in the United Statesand other developed countries, this condition isprevalent in developing countries, especially inthose with the highest burden of deaths amongchildren under 5 years of age. The lesiondescribed, a Bitot spot, is one of the early clas-sic signs of vitamin A deficiency. As vitamin Adeficiency symptoms progress, the eyes becomedry and itchy (conjunctivitis sicca). Cornealulcers subsequently can form. In the worst casesthese ulcers can rupture allowing eye contentsto avulse, resulting in permanent blindness.

Though most health care providers recognizethat vitamin A deficiency can lead to blindness,most are unaware of the full impact of vitamin Adeficiency on children’s health worldwide.Before developing blindness, vitamin A defi-cient children face a 23% greater risk of dyingfrom infectious diseases such as measles, malaria,and diarrhea. An estimated 100–140 millionchildren live with vitamin A deficiency world-wide. UNICEF and other organizations are work-ing to reduce this number as they work towardmeeting the fourth Millennium DevelopmentGoal which is a two-thirds reduction in under-five mortality by 2015. (McMillan, 113; UNICEFStatistics, http://www.childinfo.org/areas/ vitamina;UNICEF, The State of the World’s Children)

2. (D) Although somewhat controversial, seriousundernutrition during the first year of life is feltto have deleterious and permanent effects onboth mental and physical growth and devel-opment. This appears to be an organic effectindependent of psychosocial deprivation,although exogenous undernutrition often is

compounded by such deprivation. In follow-up,a high frequency of behavior problems andlearning difficulties is found, despite adequateweight gain. The recognition of possible per-manent effects of malnutrition during infancyand early childhood has led to increased effortsto avoid such undernutrition both in otherwisehealthy children and those with chronic disease.(Chase, 282:933–939, 1970; McMillan, 905)

3. (D) It is recommended that the introduction ofnonmilk foods (beikost) be delayed until theage of 4–6 months. Pancreatic enzymes, such asamylase and lipase, are low early in life butseem adequate to digest most nonmilk foods.The most compelling reason to delay intro-duction of solids relates to developmentalreadiness such as disappearance of tonguethrust (extrusor reflex), acquisition of head con-trol, and ability to sit with support. Morerecently, the avoidance of overfeeding andoverweight has become an argument for delay-ing the introduction of solids until after 4–6 monthsof age. Admittedly, it often is difficult to con-vince parents to refrain from introducing solidsat an earlier age. (AAP, 103–115)

4. (D) Congestive heart failure is very easy to pre-cipitate in a severly malnourished child partic-ulary if colloid (blood) or crystalloid (sodium)is infused aggressively. Because the heart is mal-nourished, contractility and therefore cardiacoutput is already impaired. By increasingplasma volume which would occur by infusingeither of the above substances, the preload tothe heart would increase in the face of animpaired pump, resulting in congestive heartfailure. During refeeding of severe malnutrition,

Answers and Explanations

Page 117: Questions - uhs.edu.kh You suspect Lyme disease, most likely contracted by which of the following? (A) ingestion of unripe fruit (B) ingestion of spoiled fruit (C) drinking of contaminated

118 5: Feeding and Nutrition

potassium and phosphate are rapidly takenup by the expanding body cell mass, resultingin low potassium and/or phosphate if notsupplemented. Total body water increases inmalnutrition resulting in a dilutional hypona-tremia which slowly corrects with refeeding.Glycogen stores are depleted in severe malnu-trition and hypoglycemia can easily occur if themalnourished child is stressed. (Tsang, 93–95)

5. (C) An infant’s fluid requirements are 120–140 mL/kg/day. Yet, to get sufficient calories for growth,an infant must take in 150–180 mL/ kg/day.It generally is accepted that the caloric require-ments of a newborn are about 120 cal/ kg/dayfalling to 110 cal/kg/day by 12 months of age.The most recent Dietary Reference Intakes pub-lished by the National Academy of Sciences hasdecreased this slightly and has proposed age,weight, and physical activity based estimatedenergy requirements (EER) derived from regres-sion equations. For this age group (0–3 months)the equation is EER = (89 × weight in kg − 100) +175 kcal. This 10-week-old, 5-kg infant has acaloric requirement of about 500–600 cal/day.Most milks (human and cow) and preparedformulas contain 20 cal/oz. Therefore, it wouldtake 25–30 oz of milk or formula to provide the500–600 calories required each day. (AAP, 241–246)

6. (E) To the best of our current knowledge, for-mulas are fortified with the necessary vitaminsand minerals. Breast milk contains quantita-tively small amounts of vitamin D. However,this is sufficient for many babies. Exclusivelybreastfed infants who are at risk for vitamin Ddeficiency include infants with inadequatematernal intake of vitamin D (eg, mothers whoare strict vegans), infants with inadequate sun-light exposure (eg, dark skin pigmentation, coldclimates, urban environments, clothing prac-tices, and more recently the overuse of sun-blocking agents), and older infants stillexclusively breastfed. Therefore, vitamin D sup-plementation is recommended for most babieswho are exclusively breastfed. Too much vita-min D can be toxic and just as dangerous astoo little. The major target organ of vitamin Dpoisoning is the kidney. (McMillan, 112–116;AAP, 71)

7. (E) The rash described is called acrodermatitiswhich means dermatitis of the acral or outerpoints of the body. This is the classic rash asso-ciated with zinc deficiency. This rash is alsoassociated with a rare genetic disease calledacrodermatitis enteropathica. Infants with acro-dermatitis enteropathica cannot absorb zincwell. They typically present in the first 2 yearsof life with this skin rash, diarrhea, and failureto thrive. Treatment is zinc supplementation.(McMillan, 856, 1999)

8. (C) This obese child has an elevated ALT at60 IU/L (normal <40 IU/L) and is at risk of non-alcoholic steatohepatits (NASH), a form of non-alcoholic fatty liver disease. Male Hispanics ofMexican origin seem to be most susceptable tothis disease in childhood. If the ALT is greaterthan 150% normal, the chance of an obese childto have steatohepatitis is about 85%. Thoughpatients with NASH generally are asympto-matic, their condition may lead to fibrosis andeven cirrhosis. Treatment of NASH in an obesechild primarily includes weight loss and exercise.

Other important comorbidities for obesityin childhood include type 2 diabetes, hyper-lipidemia and the metabolic syndrome, sleepapnea, hypertension, skeletal and psychologi-cal issues. The screening test for diabetes is afasting serum glucose. A serum glucose greaterthan 100 mg/dL is diagnostic of prediabetesand greater than 127 mg/dL is diagnostic ofdiabetes. A fasting serum insulin greater than17 µU/mL is consistent with insulin resistanceand the potential for developing diabetes in thefuture. (McMillan, 2020–2021)

9. (C) Of the choices listed, all may cause diarrheaand abdominal pain in this age and context,but celiac disease and cow milk protein allergyare most likely to cause failure to thrive. Celiacdisease, or gluten-sensitive enteropathy, typi-cally presents at 6 months to 2 years of age,after the introduction of gluten into the diet.The estimated incidence of celiac disease in theUnited States is about 1:250, and can range ashigh as 1:50 in selected populations. A signifi-cant cow milk protein allergy would have man-ifested earlier in life with exposure throughbreast milk. Cow milk allergy usually resolves

Page 118: Questions - uhs.edu.kh You suspect Lyme disease, most likely contracted by which of the following? (A) ingestion of unripe fruit (B) ingestion of spoiled fruit (C) drinking of contaminated

Answers: 5–16 119

by 1–3 years of age. Toddler’s diarrhea, due toexcessive juice/fluid intake, is the mostcommon cause of diarrhea in toddlers under3 years of age, and is due to the osmotic effect ofsorbitol and fructose found in fruit juices andthe limited reabsorptive capacity of the colon.Although common, lactose intolerance andchronic giardiasis (in an immunocompetenthost) should not cause growth deceleration.(McMillan, 1992–1993, 2700–2701)

10. (E) The most definitive test for celiac disease issmall bowel biopsy, although the sensitivity ofantiendomysial antibodies (IgA) approaches100% and the specificity 98%. The tissue trans-glutaminase (IgA and IgG) assay, which iseasier to standardize, has a sensitivity of92%–94% and a specificity of 95%–98%.Antigliadin antibodies (IgA and IgG) can bepresent in cow milk protein enteropathy, Crohndisease, tropical sprue, IgA nephropathy,eosinophilic enteritis, and dermatitis herpeti-formis, accounting for its lower specificity of86% (IgG) and 95.5% (IgA). An important lim-itation of all the IgA antibody assays is the factthat 2%–3% of patients with celiac disease areIgA deficient. (McMillan, 1992–1993)

11. (B) Treatment of celiac disease involves strict life-long avoidance of gluten, found in wheat, barley,and rye. Oat products may be contaminated withgluten during processing, but the allergenicity ofoat itself is still debated. Corn, rice, potatoes, andlegumes are not implicated in gluten-associatedenteropathy. (McMillan, 1992–1993)

12. (A) Although controversial, the use of a shortcourse of 1–2 mg/kg/day prednisone mayhasten resoultion of the abdominal pain inHenoch-Schöenlein Purpura (HSP). An esti-mated 75% of children with HSP develop gas-trointestinal involvement, often as the firstmanifestation. Vomiting is reported in 25% ofcases. The abdominal pain typically resolveswithin 3–7 days. (McMillan, 2562)

13. (D) Acute worsening of the abdominal pain ina child with HSP warrants close attention. Thispatient displays signs and symptoms charac-teristic of intussusception which occurs in 3% of

patients with HSP. Intussusception often resultsin pooling of fluid in the gut with subsequenthypovolemic shock. Among patients with HSP,intussusception occurs most frequently in theolder age group and most frequently (65%) isileoileal. Remember, sporadic cases of intus-susception most commonly are ileocolic occur-ring in children younger than 3 years of age.Intussusception recurs in approximately 5% ofpatients. (McMillan, 2559–2562; Lissauer, 264)

14. (B) The multiplicity of factors affecting calciummetabolism, not the least of which is the effect ofother dietary factors, makes it difficult to give anexact figure for the daily requirement of cal-cium. For a school-age child it is estimated to beabout 1.3 g/day. (Contrast this with the amountof protein required, which for this age group isnearly 1 g per kg of body weight per day!) Thisamount of calcium is easily provided by mostordinary diets, and calcium supplementationusually is not necessary, even for the child whodrinks little or no milk. Eggs, molasses, nuts,and many fish and vegetables are good sourcesof calcium. Avariety of products (specific brandsof cereal, bread, orange juice, and even icecream) are fortified with calcium. (McMillan, 110)

15. (D) A vegan or strict vegetarian diet providesalmost no vitamin B12 as well as marginal levelsof calcium, vitamin D, and iron. The relativelylow caloric density of vegetables also means thata large bulk of food must be ingested to provideadequate calories. However, the content of vita-mins other than B12 and D is apt to be adequate.(Shinwell, 70:582–586, 1982; AAP, 191–208)

16. (A) Soy protein based commercial infant for-mulas presently available provide adequatenutrition (including calcium), and infants fedthese formulas exhibit normal growth. Soy for-mulas often are prescribed for infants with per-sonal or family history of allergy in the hope ofavoiding the development of milk-proteinallergy. Unfortunately, severe gastrointestinalallergic reactions to soy protein in infants arewell recognized and are not rare. For thisreason, soy formula is not recommended forinfants or children already demonstrating sig-nificant gastrointestinal hypersensitivity to cow

Page 119: Questions - uhs.edu.kh You suspect Lyme disease, most likely contracted by which of the following? (A) ingestion of unripe fruit (B) ingestion of spoiled fruit (C) drinking of contaminated

120 5: Feeding and Nutrition

milk protein. These patients are best prescribeda protein hydrolysate formula. Soy-based for-mulas are not recommended for prematureinfants. (AAP Committee on Nutrition, 72:359–363,1983; AAP, 92–93)

17. (B) Goat milk contains little folate. Infants fedprimarily goat milk are at risk to develop amegaloblastic anemia from folate deficiency.Therefore, the mean corpuscular volume (MCV)will be elevated. The metabolism of folic acidand vitamin B12 is interrelated. Hematologicproblems caused by a deficiency in one can beimproved with supplementation of either. Theneurologic complications of vitamin B12 defi-ciency, however, will not be improved withtreatment with folic acid. Therefore, it is impor-tant to carefully sort the child’s deficiencies.Large doses of folic acid should not be givenuntil vitamin B12 deficiency has been excluded.(AAP, 938, McMillan, 1694–1695)

18. (D) Based on the new dietary reference intakes(DRIs), the estimated energy requirement (EER)of an average weight and height, mildly active12-year-old girl would be about 2500 calories.This is based on the newly published regressionequations by the National Academy of Sciences.For girls 9–18 years the equation is EER = 135.3 −[30.8 × age in years] + [PA× (10.0 × weight in kg) +(934 × height in meters)] + 25 kcal. PA is a factorfor physical activity where sedentary is 1, lowactivity is 1.13, average activity is 1.31, and veryactive is 1.56 and taking into account individualages, weights, heights, and physical activities,these new guidelines are more accurate. Forthis girl the equation would read: EER = 135.3 −[30.8 × 12] + [1.31 × (10 × 44)] + [934 × 1.52] + 25or EER = 2524. (Dietary Reference Intakes at Instituteof Medicine website: http://www.iom.edu/CMS/3788/4574.aspx)

19. (E) The thymus dramatically decreases in sizeduring periods of undernutrition. Therefore, it isnot uncommon to see loss of thymic shadow onthe chest x-ray of infants admitted for failure tothrive. Involution of the thymus during starva-tion accounts for some of the increased suscep-tibility to infection in these children. Thecombination of thymic dysplasia and failure to

thrive could bring to mind DiGeorge anomaly(or 22q11 deletion syndrome). These infants, how-ever, typically present with cardiac disease orsymptoms related to abnormal calcium metabo-lism during early infancy. (Katz, 59:490–494, 1977)

20. (A) The human immunodeficiency virus can betransmitted from mother to baby via breast-feeding. In the United States, where othermeans of infant nutrition are readily available,HIV-infected women should be advised not tofeed their breast milk to babies. According toWHO recommendations, in developing coun-tries, the risk of HIV transmission from breast-feeding must be weighed against the risk ofcontaminated or nutritionally incomplete replace-ment feeding, therefore at the time of this writing,it is still recommended that HIV-positive womenfrom developing countries breastfeed theirbabies until 6 months of age. UNICEF reportsonly 37% of babies in developing countries arebeing breastfed. Suboptimal breastfeeding isresponsible for an estimated 1.4 million childdeaths worldwide.

Although hepatitis B surface antigen hasbeen detected in breast milk, studies indicate thisdoes not significantly increase the risk of trans-mission of hepatitis among breastfed infants.Transmission of hepatitis C virus via breast-feeding is theoretically possible but not clinicallydocumented in anti-HCV-positive, HIV-negativewomen. (AAP:Red Book 2006: 126)

21. (D) Normal infants will ingest most of theavailable milk from a breast within 3–5 minutes.Additional sucking stimulates future milkproduction, which is necessary for continuedsuccessful breastfeeding. Prolonged nursing,especially during the first few weeks, can resultin extremely sore or cracked nipples, whichcan result in early cessation of breastfeeding.Early introduction of bottles can undermineattempts to breastfeed. (McMillan, 116)

22. (B) Children do learn to feed themselves inde-pendently during the second year of life.However, this will not necessarily include theuse of utensils. Holding and manipulating aspoon to scoop food and then bring it to themouth requires complex development. Most

Page 120: Questions - uhs.edu.kh You suspect Lyme disease, most likely contracted by which of the following? (A) ingestion of unripe fruit (B) ingestion of spoiled fruit (C) drinking of contaminated

Answers: 17–29 121

2-year-old children are able to use a spoon tofeed themselves, but many still prefer to usetheir fingers. (McMillan, 596–597)

23. (D) Despite public belief, sucrose alone has notbeen determined to be a cause of problembehavior or increasing hyperactivity in chil-dren. Controlled, double-blind studies have notshown a difference in behavior measures, motoractivity, or cognitive performance of childrenreceiving sucrose compared to those receivinga control substance. (Wolraich, 330:301–307, 1994)

24. (C) The child described has many of the classicsigns of Kwashiorkor or protein calorie mal-nutrition. By definition, the serum albumin hasto be less than 2.5 mg/dL to make this diagno-sis therefore C is most correct. Patients withKwashiorkor have a dilutional hyponatremia withserum sodium usually around 130–133 meq/L.Serum potassium is usually normal during theacute phase, but can become low during reha-bilitation if adequate potassium is not in thediet. Prealbumin is an index of protein intakeand functionally is thyroxin-binding protein ortransthyretin. Since it has a very short half-lifeof about 3.5 days, it is a fairly sensitive indica-tor of protein intake. The lower limits of normalis 20 mg/dL. (Tsang, 93–95)

25. (D) Skim milk is essentially free of all fat. If it isused at a time when the infant receives most or allof his calories from milk (as seen in most children< 12 months of age and many children < 2 yearsof age), this could lead to essential fatty acid defi-ciency. Fats, especially saturated fats, are essentialfor myelination and brain growth. Additionally,since skim milk contains only 10 cal/oz, the infantwould need to consume twice the volume in orderto obtain sufficient calories for growth. (Note:There is a difference between skim milk and low-fat milk. The former has zero fat, whereas thelatter has 1/2%–2% fat. Two percent fat milk wouldbe permissible at an earlier age than skim milk.The question specified skim milk.) (AAP Committeeon Nutrition, 72: 253–255, 1983; AAP, 265–268)

26. (A) The average school-age child ingests about30%–35% of their calories from fat, 15% of theircalories as protein, and the rest as carbohydrate.

Carbohydrates are the major source of caloriesin the normal diet throughout life except in thenewborn period, when more than half ofingested calories are derived from fat. Excessivecarbohydrate intake as well as excessive fatintake can result in obesity because of the result-ing increase in caloric intake. Inadequate intakeof carbohydrate (<5% of the calories) not onlyresults in caloric deprivation and weight lossbut also the excessive combustion of fat, a risein fatty acid and ketone bodies in the blood,and occasionally acidosis. This is not conduciveto healthy growth. This child would be betterencouraged to increase activity levels and eat abalanced diet. (McMillan, 109–118)

27. (D) In healthy infants, the nutritional require-ments average 100 kcal/kg/day. However,infants who are failing to thrive most often willneed more than 100 kcal/kg/day to gain weight.Their intake requirements typically are as muchas 50% higher than normal, or 150 kcal/kg/day.(McMillan, 904)

28. (A) Fruit juice is almost pure carbohydrate,mostly sugar rather than starch. Less than 5%of the calories are derived from protein andfat. Excessive intake of fruit juices by a younginfant increases the caloric intake and can resultin obesity. Another problem associated withexcessive intake of juice is the development ofchronic diarrhea due to the relative malab-sorption of sorbitol or fructose, both naturallyfound in many juices. Inadequate proteinintake is also possible despite adequate totalcalories if protein is not provided by othersources such as milk. (AAP, 131 and 1134)

29. (D) Dental eruption correlates with bone mat-uration. This process begins at about 4–6 monthsof age. Eruption usually is symmetric andtends to occur in the mandibular arch justahead of the maxillary arch. Twenty primaryteeth usually erupt in children during the first3 years of life. Although lack of dentition by12 months of age is defined as delayed, this find-ing is a normal variation is a small percentageof children. In this scenario, macrocephaly,short fingers, and the history of dental eruptiondelay strongly suggest an inherited disorder

Page 121: Questions - uhs.edu.kh You suspect Lyme disease, most likely contracted by which of the following? (A) ingestion of unripe fruit (B) ingestion of spoiled fruit (C) drinking of contaminated

122 5: Feeding and Nutrition

such as cleidocranial dysostosis. Absent orhypoplastic clavicles are a common finding inthis autosomal dominant disease. (McMillan,782–783, 2523)

30. (E) Milk contains very little iron, only about1.2 mg/L. The average infant requires 1–1.5 mg/kgof iron daily. A diet predominantly consisting ofmilk is a common cause of iron deficiencyanemia in the older infant and the toddler. Thisis especially true when the milk is unmodifiedcow milk. The iron in cow milk is less wellabsorbed than that in breast milk. Also, a largeintake of unmodified cow milk often is associ-ated with microscopic gastrointestinal bloodloss in infants less than 12 months of age. Cowmilk, of course, contains adequate amounts ofprotein and calories, so the child describedwould grow well. (McMillan, 1692–1693)

31. (E) Childhood obesity is a growing problem inthe United States. It is estimated that around20% of children in the United States are obesewith obesity rates as high as 30% in someregions. Obesity places these children at risk forlife-long health problems, psychologic issues,and discrimination. There will be an underlyingdefinable cause in less than 5% of children withobesity. This child’s adequate linear growth andfamily history along with an otherwise normalphysical examination suggest no further workupis necessary. It generally is not recommendedor very successful to severely restrict calories ina child who is overweight. The best outcomefor an obese child includes a plan that involvesthe entire family’s eating habits and physicalactivity. It is important to increase the physicalactivity of the child. This needs to be tailored tothe child’s interests and should take into accountthe activities readily available. Clearly, this requirescommitment on the part of the family, child, andpractitioner. (AAP, 551–592; Rudolph, 12–16)

32. (C) Although all of the mentioned methodshave roles in the assessment of obesity, the bestand what is now considered standard for theoffice-based measurement of adiposity is tocalculate the BMI and compare it to the pub-lished reference values. A BMI greater than the85th percentile on the CDC reference charts is

considered overweight and greater than the95th percentile is considered obese. The use ofskin-fold measurements although useful are veryhard to do accurately in the obese. Measuringweight alone does not allow for various-sizebody frames. Using weight for height is usefuland is the standard for children less than 3 yearsof age. Weight for height standards do not existfor children greater than 12. Densitometry is stillthe gold standard for body composition meas-urements, that is, body fatness, but requires totalsubmersion in water and therefore is impracticalfor obvious reasons. (AAP, 407–423)

33. (C) A sweat chloride greater than 40 is consis-tent with the diagnosis of cystic fibrosis (CF).This child presents with CF—the most commongenetically transmitted lethal disease amongCaucasians (1/3300). This multisystem diseaseleads to pancreatic insufficiency in about 85% ofpatients. This causes fat malabsorption with result-ant malabsorption of the fat-soluble vitamins(A, D, E, K). Therefore, all CF patients shouldreceive pancreatic enzyme supplementation. CFpatients also are at risk for iron deficiency anemia.Only when anemia or low serum ferritin levels aredocumented should they receive iron supple-mentation. (McMillan, 1431; Rudolph, 717–719)

34. (C) Adolescents with anorexia nervosa typi-cally perceive themselves as too fat and hope tolose weight, even in the face of emaciation. Thedegree of distortion of their body image corre-sponds to the difficulty encountered in treatingthe patients. This adolescent with anorexia ner-vosa seems, by history, to have difficulty withbingeing. Because her weight is low for herage, she is likely purging with self-inducedvomiting. Therefore, she would be classifiedas anorexia nervosa, bulimic type or what iscalled bulimia nervosa. (McMillan, 657)

35. (C) Since fat yields about 9 cal/g, whereas pro-tein and carbohydrate yield only 4 cal/g, morethan 50% of the calories in human breast milkare in the form of fat even though carbohy-drate is more abundant by weight in humanmilk. Of all mammals known, seals have therichest fat content in their milk, which is theconsistency of mayonnaise. The milk of seals

Page 122: Questions - uhs.edu.kh You suspect Lyme disease, most likely contracted by which of the following? (A) ingestion of unripe fruit (B) ingestion of spoiled fruit (C) drinking of contaminated

Answers: 30–38 123

contains 12–20 times more fat than human milkand does not contain lactose. This allows shortlactation times and high pup growth rates.While human infants will double their birthweight by 4 months of age, seal pups quadru-ple their birth weight by 2 weeks of age.(McMillan, 115; Miller, 40)

36. (C) Fat also accounts for about half of the calo-ries in unmodified cow milk. Cow milk con-tains significantly higher amounts of proteinthan human milk. Protein accounts for 22% ofcalories in cow milk as compared to 8% of calo-ries in human breast milk. Of note, the proteinnitrogen concentration in the ileocecal contentsof 1–5-month-old infants fed cow milk formulais three times higher than that found in infantsfed breast milk. (McMillan, 115; AAP, 87–97)

37. (A) Skim milk is essentially free of fat. About60% of the calories are provided as carbohydrate,

and the other 40% as protein. (AAP Committee onNutrition, 72:253–255, 1983; McMillan, 115)

38. (E), 39. (B) Worldwide, undernutrition is thesingle biggest factor for nearly half of all deathsof children under the age of 5 years. UNICEF iscalling for an increased effort in addressing thehealth-related Millennium Development Goals,most of which directly or indirectly call forimprovement in nutrition of children and theirmothers. In order to reach a target indicator, thenumber of under-five deaths in the world mustdecline from 9.7 million in 2006 to less than5 million in 2015. UNICEF states, “It is clear thatmeeting all of these goals will require politicalwill, resources, and sound strategies on anunprecedented scale.” It seems obvious thisshould be a priority for all nations. Jonas Salksaid, “Children are the message we send to afuture we will never see.” (UNICEF, The State of theWorld’s Children; http://www.un.org/millenniumgoals)

Bhatia J, Greer F, and the Committee on Nutrition. Use ofsoy protein-based formulas in infant feeding. Pediatrics.2008;121:1062–1068.

Fomon SJ, Filer LJ, Anderson TA, et al. Recommendationsfor feeding normal infants. Pediatrics. 1979;63:52–59.

SELECTED READINGS

Page 123: Questions - uhs.edu.kh You suspect Lyme disease, most likely contracted by which of the following? (A) ingestion of unripe fruit (B) ingestion of spoiled fruit (C) drinking of contaminated

This page intentionally left blank

Page 124: Questions - uhs.edu.kh You suspect Lyme disease, most likely contracted by which of the following? (A) ingestion of unripe fruit (B) ingestion of spoiled fruit (C) drinking of contaminated

CHAPTER 6

Fluids, Electrolytes,and Metabolic Disorders

Joseph T. Cernich, MD

Angela L. Turpin, MD

125

This chapter covers very common abnormalities as well as very rare abnormalities. Disorders of fluids andelectrolytes often occur with common illnesses such as vomiting and diarrhea and are encountered frequently.Knowledge of fluid and electrolyte requirements for all ages of pediatric patients is vital. Specific metabolicderangements are more commonly associated with certain age groups. For instance, neonates frequently experi-ence hypoglycemia, hyponatremia, and hypocalcemia nonspecifically with illnesses. Older infants and youngchildren commonly develop dehydration and/or electrolyte abnormalities from gastroenteritis.

In contrast to fluid and electrolyte abnormalities, genetic inborn errors of metabolism are rare. Inbornerrors of metabolism are relatively unique to pediatrics in their presentation and management. It is importantto be aware of common presentations of these uncommon disorders as early recognition and treatment can sig-nificantly improve the patient’s outcome. Because of this, these metabolic abnormalities are well reviewed inmany texts and are well tested on examinations. This section will attempt to focus the material and to reviewthe more commonly encountered defects in each area of metabolism (eg, carbohydrates, amino acids, lipids).

Page 125: Questions - uhs.edu.kh You suspect Lyme disease, most likely contracted by which of the following? (A) ingestion of unripe fruit (B) ingestion of spoiled fruit (C) drinking of contaminated

126

DIRECTIONS (Questions 1 through 36): For eachof the multiple choice questions in this sectionselect the one lettered answer that is the bestresponse in each case.

1. A new mother asks you how much formulaher 3 kg infant should be consuming daily. Togive her an idea on how much fluid the childshould be taking, you calculate out the approx-imate daily fluid requirement. Which of thefollowing most closely resembles this child’sdaily fluid requirements?

(A) 900 mL(B) 700 mL(C) 500 mL(D) 300 mL(E) 100 mL

2. A 30 kg child you are taking care of in the hos-pital is receiving IV fluids in preparation forsurgery. You are trying to determine if thefluids are running at an appropriate rate fordaily maintenance. What do you determine isthe approximate daily fluid requirement forthis child?

(A) 3000 mL(B) 2400 mL(C) 1700 mL(D) 1200 mL(E) 1000 mL

3. A 50 kg child you are taking care of has justreceived replacement for her fluid deficit. Youare now trying to determine the hourly fluidrate needed to supply maintenance fluid

requirements. Which number below most accu-rately reflects the correct mL/h?

(A) 125 mL/h(B) 115 mL/h(C) 100 mL/h(D) 90 mL/h(E) 80 mL/h

4. An infant presents with a weight of 8 kg and a3-day history of diarrhea and vomiting. Heappears severely dehydrated with decreasedsensorium, sunken fontanelle, poor skin turgor,and decreased urine output. Which of the fol-lowing most closely estimates the fluid deficitof this child?

(A) 1900–2000 mL(B) 1300–1500 mL(C) 900–1100 mL(D) 400–500 mL(E) 200–250 mL

5. A child with diabetic ketoacidosis has the fol-lowing serum values: glucose 500 mg/dL,Na+ 126 meq/L, K+ 4 meq/L, Cl− 80 meq/L,BUN 16 mg/dL, and HCO3− 6 meq/L. Whichof the following most approximates this patient’sserum osmolality?

(A) 285 mOsm/L(B) 295 mOsm/L(C) 310 mOsm/L(D) 270 mOsm/L(E) 260 mOsm/L

Questions

Page 126: Questions - uhs.edu.kh You suspect Lyme disease, most likely contracted by which of the following? (A) ingestion of unripe fruit (B) ingestion of spoiled fruit (C) drinking of contaminated

Questions: 1–10 127

6. An infant with diarrhea is 10% dehydrated.Before the onset of illness, the weight was 5 kgand the surface area 0.3 m2. The child’s serumsodium concentration is normal. Assuming thechild will not be fed and the diarrhea ceases,what is the approximate total amount of fluidyou should deliver to meet maintenance needsand restore the child to a state of normal hydra-tion in the first 24 hours?

(A) 750 mL(B) 1000 mL(C) 1250 mL(D) 1500 mL(E) 1750 mL

7. A 6-year-old girl has vomiting, headache, andirritability. She does not appear dehydrated.But when reviewing her vitals you notice herweight is up 3 kg from just 3 weeks ago.Laboratory findings are: Na+ 112 meq/L,K+ 4.0 meq/L, Cl− 75 meq/L, HCO3

− 19 meq/L,BUN 10 mg/dL, and creatinine 0.4 mg/dL.Aspot urine sodium concentration is 100 meq/L.Which of the following is the most likely causeof these findings?

(A) decreased glucocorticoid production(B) decreased mineralcorticoid production(C) increased oral intake of water(D) decreased antidiuretic hormone secretion(E) increased antidiuretic hormone secretion

8. A 14-year-old with type 1 diabetes is admittedwith diabetic ketoacidosis. Initial laboratoryvalues are as follows: glucose 563 mg/dL,sodium 136 meq/L, potassium 4.3 meq/L,chloride 107 meq/L, CO2 9 meq/L, BUN18 mg/dL, creatinine 0.6 mg/dL, and calcium9.7 mg/dL. She receives a 10 cc/kg bolus ofnormal saline followed by IV fluids consistingof 1/2 normal saline, as well as IV insulin. Eighthours into therapy, she develops muscle weak-ness. In addition, her electrocardiogram showsflat T waves as well as U waves. What is themost likely cause of her symptoms?

(A) cerebral edema(B) hyponatremia

(C) hypoglycemia(D) hypokalemia(E) hypocalcemia

9. A 10-month-old male infant had multipleepisodes of vomiting and diarrhea over the last24 hours. The infant now has slightly sunkeneyes, mildly decreased activity, and dry skin.Vital signs are stable. Which of the following isthe generally preferred method for rehydra-tion for this patient?

(A) oral administration of a solution containing75 meq/L of sodium and 5 g/dL ofglucose

(B) oral administration of a solution containing50 meq/L of sodium and 2 g/dL ofglucose

(C) oral administration of a solution containing35 meq/L of sodium and 10 g/dL ofglucose

(D) intravenous administration of a solutioncontaining 154 meq/L of sodium and5 g/dL of glucose

(E) intravenous administration of a solutioncontaining 35 meq/L of sodium and10 g/dL of glucose

10. A 16-year-old male is brought to the emergencyroom by his friends. They relate they weredrinking alcohol and that their friend “passedout” about 2 hours ago and is increasingly dif-ficult to arouse. Which of the following is mostuseful in your immediate management of thispatient?

(A) serum sodium(B) serum glucose(C) blood alcohol level(D) serum calcium(E) serum drug screen

Page 127: Questions - uhs.edu.kh You suspect Lyme disease, most likely contracted by which of the following? (A) ingestion of unripe fruit (B) ingestion of spoiled fruit (C) drinking of contaminated

128 6: Fluids, Electrolytes, and Metabolic Disorders

11. You are discussing the family history with thefather of a boy who is new to your practice.During your discussion the father reports thatthe family had a daughter who died at age9 months from complications associated withglycogen storage disease. That child had muscleweakness, hypotonia, and severe cardiomegaly.She did not have significant hyperlipidemia,gout, skeletal involvement, or frequent bacterialinfections. Which glycogen storage disease wasmost likely in the deceased child?

(A) type Ia (von Gierke disease)(B) type Ib (glucose-6-phosphatase microsomal

transport defect)(C) type II (Pompe disease)(D) type III (debranching enzyme deficiency)(E) type IV (branch enzyme deficiency)

12. A 3-day-old infant with severe jaundice, hypo-glycemia, and anemia is further noted to haveliver nodules and cirrhosis. Serum tyrosineand methionine levels are markedly elevated.What enzyme defect is causing this child’sliver failure?

(A) tyrosine aminotransferase(B) phenylalanine hydroxylase(C) fumarylacetoacetate hydrolase(D) malylacetoacetate isomerase(E) carbinolamine dehydratase

13. In counselling a family regarding the diagnosisof alcaptonuria, you state that is a metabolic dis-ease caused by a defect in or lack of homogen-tisic acid oxidase. The excess homogentisic acidleads to which clinical finding?

(A) kinky hair(B) absent patella(C) black urine and darkly pigmented sclera,

cornea, and ears(D) blue sclera(E) absent radii

14. A 4-year-old female presents to the emergencydepartment complaining of pain and weak-ness in her extremities. She has a cabbage-likeodor. Her mother reports that her daughter’s

symptoms developed acutely today despite thechild taking nitisinone and complying with a dietfree of phenylalanine, tyrosine, and methionine.Which of the following is most likely to causefuture death in this child?

(A) renal failure(B) hepatic failure(C) diabetes(D) respiratory failure(E) infection

15. A child with mental retardation is also noted tohave severe myopia. On examination, sublux-ation of the ocular lens (ectopia lentis) is found.This generally occurs after 3 years of age inchildren with which of the following?

(A) hawkinsinuria(B) tyrosinemia(C) phenylketonuria(D) isovaleric acidemia(E) homocystinuria

16. An 8-month-old Caucasian male is broughtwith concerns of a 4-day history of fever anddecreased food intake. The mother states that hehas slept most of the day. On examination, thepatient is febrile with otherwise normal vitalsigns. He does not open his eyes during theexamination. The liver is noted to be slightlyenlarged. His laboratory values show normalelectrolytes without acidosis. The serum glu-cose is 33 mg/dL. Liver enzymes as well asammonia are slightly elevated. Serum insulin isundetectable. A simultaneous urine sample isnegative for ketones. Which of the followingconditions is the most likely diagnosis?

(A) sulfonylurea ingestion(B) glycogen storage disease type I(C) accelerated starvation(D) primary adrenal insufficiency(E) medium-chain acyl-CoA dehydrogenase

deficiency

17. A febrile infant with poor feeding and moderaterespiratory distress due to pneumonia is beingadmitted to your inpatient service. With regard

Page 128: Questions - uhs.edu.kh You suspect Lyme disease, most likely contracted by which of the following? (A) ingestion of unripe fruit (B) ingestion of spoiled fruit (C) drinking of contaminated

Questions: 11–22 129

to sodium content, which of the following intra-venous fluids would be the maintenance fluidof choice of a 12 kg infant?

(A) 0.9% NaCl(B) 0.2% NaCl(C) 0.45% NaCl(D) plasmalyte(E) Ringer lactate

18. A 15-kg toddler with group A coxsackievirusinfection is refusing to drink. His serum potas-sium is normal. For maintenance of intra-venous fluids, what concentration of KClshould be added?

(A) 5 meq KCl/L(B) 10 meq KCl/L(C) 25 meq KCl/L(D) 40 meq KCl/L(E) 50 meq KCl/L

19. A 15-month-old male presents with moderatedehydration. Resuscitation proceeds with assess-ment of airway, breathing, and circulation. Anisotonic intravenous fluid bolus of 20 cc/kg isadministered. Laboratory studies reveal a serumsodium of 165 meq/L and a normal serum potas-sium. Which of the following is the most appro-priate plan for rehydration?

(A) correct the hypernatremia over 8 hourswith 0.45% NS without maintenancepotassium

(B) correct the hypernatremia over 8 hourswith D5W with maintenance potassium

(C) correct the hypernatremia over24–48 hours with D5 0.45% NS withmaintenance potassium

(D) correct the hypernatremia over 12 hourswith D5 0.45% NS with maintenancepotassium

(E) correct the hypernatremia over24–48 hours with D10W without mainte-nance potassium

20. A 12-year-old male is brought to your office forconcerns regarding excessive drinking. The familystates that the excessive drinking is also caus-ing him to urinate frequently. His weight hasdecreased 6 kg in the past 5 months. His currentmedications include albuterol as needed as well aslithium. A urine analysis shows no glucose,ketones, or evidence of a urinary tract infec-tion. The specific gravity is less than 1.005.Further work-up shows the serum sodium to be163 meq/L, with a normal potassium and glu-cose. BUN is mildly elevated. Urine osmolality isless than serum osmolality. The urine concentratesonly minimally when parenteral DDAVP isadministered. What is the most helpful treatmentfor the patient’s condition?

(A) fludrocortisone(B) fluid restriction(C) intranasal DDAVP(D) hydrochlorothiazide(E) insulin

21. One of your patients recently underwentsplenectomy due to an autosomal recessive gly-colytic enzyme deficiency. Which of the fol-lowing is a common clinical sign in neonatalpresentation of this disorder?

(A) sepsis(B) jaundice(C) hepatomegaly(D) hypertonia(E) seizures

22. What is the most likely clinical finding in an8-year-old with Wilson disease?

(A) hepatitis-like illness(B) acral cyanosis(C) polyuria(D) behavioral changes(E) poor visual acuity

Page 129: Questions - uhs.edu.kh You suspect Lyme disease, most likely contracted by which of the following? (A) ingestion of unripe fruit (B) ingestion of spoiled fruit (C) drinking of contaminated

130 6: Fluids, Electrolytes, and Metabolic Disorders

23. A 2-month-old infant is noted to have bilateralcataracts. The remainder of the infant’s exam-ination is normal; and developmentally she ison target. What early dietary adjustment couldhave prevented the development of cataracts inthis child?

(A) eliminating gluten(B) providing low iron formula(C) eliminating milk and milk products(D) providing increased amounts of milk

and milk products(E) providing increased amounts of folate

24. A 10-month-old male infant with a history ofumbilical hernia repair is noted to have coarsefacial features and is diagnosed with Hurler dis-ease. This condition is the most severe of whichof the following groups of inherited diseases?

(A) glycogen storage diseases(B) glycoproteinoses(C) mucopolysaccharidoses(D) sphingolipidoses(E) mucolipidoses

25. You are obtaining the history of a 7-year-old male,who presents to your office with a several-weekhistory of ataxia and visual changes. Previouslyhe has been treated for attention deficit disorder.As you are about to examine the child he beginsto seize. An X-linked peroxisomal disease is sus-pected. Plasma elevation of which of the follow-ing substances confirms your suspicions?

(A) lead(B) iron(C) short chain fatty acids(D) long chain fatty acids(E) very long chain fatty acids

26. An 8-month-old Caucasian male is brought intoyour pediatric clinic for a routine check up.Upon examination you note severely bowedlegs and flaring of both wrists. Feeding historyreveals he is formula fed and receives a dailymultivitamin supplement. Laboratory resultsdemonstrate the following: high 25(OH)2D,low 1,25(OH)2D, low calcium, and normal

phosphorous. What diagnosis is most consistentwith these findings?

(A) vitamin D refractory rickets type I(B) hypophosphatemic rickets(C) vitamin D deficiency rickets(D) 22q11 deletion syndrome(E) cystic fibrosis

27. Pyruvate dehydrogenase complex deficiencyis an autosomal recessive disease and leads tothe development of an anion gap acidosis.Which of the following accumulates to result inan elevated anion gap?

(A) beta-hydroxybutyric acid(B) acetoacetic acid(C) lactic acid(D) hydrochloric acid(E) fumaric acid

28. You are evaluating a 10-month-old infant forrecurrent fractures following relatively minortrauma. You note deep blue sclera and bowingof the lower extremities. X-ray examinationreveals generalized osteopenia. Which of thefollowing is the most likely diagnosis?

(A) achondroplasia(B) histiocytosis X(C) osteogenesis imperfecta(D) osteopetrosis(E) rickets

29. A 2-week-old infant has a 12-hour history ofrecurrent episodes of bilious vomiting. A firm,tender abdomen is noted on physical exami-nation. Flat plate of the abdomen demonstratesa large dilated loop of distended bowel consis-tent with midgut volvulus. Which of the fol-lowing best corresponds with this patient’scondition?

Page 130: Questions - uhs.edu.kh You suspect Lyme disease, most likely contracted by which of the following? (A) ingestion of unripe fruit (B) ingestion of spoiled fruit (C) drinking of contaminated

Questions: 23–34 131

30. A 4-month-old male with failure to thriveingests greater than 175 cal/kg/day during72 hours of hospitalization, but does notgain weight. Laboratory values are serumNa+ 138 meq/L, K+ 3.5 meq/L, Cl− 111 meq/L,HCO3

− 12 meq/L, BUN 2 mg/dL, Cr 0.2 mg/dL,glucose 112 mg/dL, serum pH 7.30, and urinepH 8.0 (under oil). The serum phosphate levelis 2.4 meq/L. Which of the following is themost likely condition of this infant?

(A) nonorganic failure to thrive(B) congenital adrenal hyperplasia(C) cystic fibrosis(D) renal tubular acidosis(E) diabetes insipidus

31. A 5-week-old male presents with poor feed-ing, poor growth, a peculiar odor, hypertonia,and hyperactive reflexes. He is afebrile. Historyreveals no problems with labor and deliveryand early hospital discharge at 24 hours of age.He has not seen a physician since that time.Which of the following is the most likely etiol-ogy of this infant’s condition?

(A) sepsis(B) pyloric stenosis(C) overfeeding(D) phenylketonuria (PKU)(E) hypothyroidism

32. The most important part of management of theinfant described above is to restrict consump-tion of which of the following?

(A) iron(B) complex carbohydrates

(C) short chain fatty acids(D) phenylalanine(E) all proteins

33. A 2-week-old female is admitted to the hospitalfor jaundice, hypoglycemia, and acidosis. Shehas also recently been febrile and blood culturesare positive for Escherichia coli. The patient ismost likely deficient in which enzyme?

(A) galactokinase(B) galactose-1-phosphate uridyl transferase(C) pyruvate dehydrogenase(D) galactose-1-phosphate dehydrogenase(E) uridyl diphosphogalactose-4-epimerase

34. You are seeing a 10-month-old Caucasian femalewhose mother brought her in for wheezing. Thisis the child’s third visit in the past 2 months forthe same complaint. Today you also notice thather weight is less than the 3rd percentile whileher length is at the 25th percentile. What is thefirst choice for diagnosing the suspected disease?

(A) pancreatic biopsy(B) molecular genetics(C) evaluate stool sample for fecal fat(D) sweat chloride test(E) obtain blood sample for endomysial

antibodies

Serum pH Serum Electrolytes

[Na+] [K+] [Cl-] [HCO3-] [BUN] [Cr](meq/L) (meq/L) (meq/L) (meq/L) (mg/dL) (mg/dL)

(A) 7.28 128 5.8 88 16 25 0.2(B) 7.35 130 2.8 90 21 18 0.1(C) 7.50 130 3.6 88 34 18 0.1(D) 7.45 140 4.0 100 22 9 0.1(E) 7.32 140 3.0 112 18 13 0.2

Page 131: Questions - uhs.edu.kh You suspect Lyme disease, most likely contracted by which of the following? (A) ingestion of unripe fruit (B) ingestion of spoiled fruit (C) drinking of contaminated

132 6: Fluids, Electrolytes, and Metabolic Disorders

35. A6-week-old presents with a history of frequent,nonbilious vomiting, which the parents state“shoots clear across the room.” Examinationreveals a thin, alert baby. Gastric peristalticwaves are present as well as a small epigastricmass. Which of the following sets of electrolytes,with values in meq/L, is most consistent withthis infant’s probable condition?

(A) Na+ 142, K+ 4.0, Cl− 110, HCO3− 22

(B) Na+ 140, K+ 3.8, Cl− 96, HCO3− 22

(C) Na+ 138, K+ 3.5, Cl− 88, HCO3− 38

(D) Na+ 145, K+ 2.5, Cl− 110, HCO3− 30

(E) Na+ 128, K+ 6.0, Cl− 110, HCO3− 18

36. The parents of a 3-day-old male infant presentto the emergency department with their child.They report that he was born without complica-tion and was previously doing well. Today, hebecame very lethargic and started vomiting feeds.Mother reports that she had a brother who diedsuddenly as a neonate. An ammonia level wasdrawn and was noted to be markedly elevated.You suspect ornithine transcarbamylase defi-ciency. What is the probability of these parentshaving future offspring effected by this disease?

(A) 50% of males would be severely affected.(B) 25% of males and 25% of females would

be affected.(C) All offspring would be affected.(D) 50% of males and 50% of females would

be severely affected.(E) No increased risk compared to general

population.

Questions 37 through 39

A 5-month-old male infant, previously well, isadmitted to the hospital following 2 days of severediarrhea. There is no history of fever or vomiting.The infant has been fed unmodified cow milk,orange juice, tea, rice water, and plain water. Thechild is lethargic and dehydrated with sunken eyes,depressed fontanelle, dry mucous membranes, andpoor skin turgor. Pulses are “thready,” and capillaryrefill time is 4 seconds. Blood pressure is 70/30 mm Hg,heart rate is 190 beats per minute, T is 102°F, weightis 6.3 kg. The remainder of the examination is withinnormal limits.

37. The child should immediately be given a rapidintravenous infusion (bolus) of what amount ofnormal saline?

(A) 1 mL/kg(B) 5 mL/kg(C) 20 mL/kg(D) 50 mL/kg(E) 100 mL/kg

38. The initial intravenous fluid bolus given to thischild should contain which of the following?

(A) 140 meq/L of sodium(B) 100 meq/L of sodium(C) 75 meq/L of sodium, D5

(D) 35 meq/L of sodium, D5

(E) no sodium, D25W

39. This initial intravenous bolus should also con-tain which of the following?

(A) 60 meq/L of potassium(B) 40 meq/L of potassium(C) 20 meq/L of potassium(D) 10 meq/L of potassium(E) no potassium

Questions 40 through 42

A 1-week-old female infant was admitted becauseof vomiting, weight loss, and poor feeding. Theinfant weighed 2.8 kg at birth. Vomiting and poorfeeding started the fourth day of life and loosestools were a problem since birth. The infant wasfed a standard prepared cow milk formula. Onexamination, the child was found to be poorly nour-ished and mildly dehydrated. Weight was 2.1 kg.The clitoris was large and the posterior aspects ofthe labia majora were fused. The remainder of thephysical examination was normal. Serum electrolyteswere Na+ 110 meq/L, Cl_ 82 meq/L, K+ 7.2 meq/L,BUN 31 mg/dL, and glucose 56 mg/dL.

40. The most likely cause of this clinical picture iswhich of the following?

(A) viral gastroenteritis(B) obstructive uropathy

Page 132: Questions - uhs.edu.kh You suspect Lyme disease, most likely contracted by which of the following? (A) ingestion of unripe fruit (B) ingestion of spoiled fruit (C) drinking of contaminated

Questions: 35–47 133

(C) adrenal insufficiency(D) inappropriate feeding(E) inappropriate secretion of ADH

41. This patient’s chromosomal analysis would mostlikely show which of the following karyotypes?

(A) XX(B) XY(C) XO(D) XXY(E) XYY

42. Therapy for this child should include which ofthe following?

(A) peritoneal dialysis(B) low protein diet(C) fluid restriction(D) low salt diet(E) glucocorticoid and mineralcorticoid

replacement

Questions 43 through 45

An 8-day-old male infant is brought to clinic for aroutine postdelivery evaluation. On examination,the child weighs 6 kg. Multiple pits are present onthe posterior helix of the ears. The neonate is exces-sively jittery. Pregnancy history, labor, and vaginaldelivery were unremarkable. The mother states thebaby is feeding well.

43. Evaluation of which of the following would bemost beneficial in an infant with these physicalfindings and history?

(A) calcium(B) thyroid(C) magnesium(D) glucose(E) sodium

44. Additional family history is obtained and thesiblings examined. There is a history of mentalretardation in the family. Some of the siblingsare large for their age and have the same linesor pits seen on the posterior helix of their ears.Which of the following genetic disorders doesthis family most likely have?

(A) Prader-Willi syndrome(B) neurofibromatosis(C) 22q11 deletion syndrome(D) Sotos syndrome(E) Beckwith-Wiedemann syndrome

45. Based on your diagnosis, the most likely etiol-ogy of this child’s jitteriness and abnormal lab-oratory study is which of the following?

(A) hyperinsulinism(B) thyrotoxicosis(C) SIADH(D) adrenal insufficiency(E) excess growth hormone

Questions 46 and 47

A 3-year-old, formerly healthy male toddler is seenin clinic with an acute onset of vomiting. On physi-cal examination the child is tachypneic (respirationsof 60 breaths per minute), febrile (temperature of102°F), sleepy, and difficult to arouse. The parentsexplain they are visiting the child’s grandparents.Serum laboratory results are Na+ 150 meq/L, K+ 2.9,Cl− 99, HCO3

− 18 meq/L, glucose 45 mg/dL, aniongap 26, BUN 16 mg/dL, creatinine 0.3 mg/dL.Blood gas pH 7.25, Pco2 15, Po2 88, BE −18.0.

46. This child most likely ingested which of thefollowing?

(A) ethanol(B) ibuprofen(C) acetaminophen(D) organophosphate(E) oil of wintergreen

47. Treatment beyond acute supportive care shouldfocus on which of the following?

(A) alkalization of the urine(B) administration of a cathartic(C) administration of N-acetylcystiene(D) peritoneal dialysis(E) antibiotic administration

Page 133: Questions - uhs.edu.kh You suspect Lyme disease, most likely contracted by which of the following? (A) ingestion of unripe fruit (B) ingestion of spoiled fruit (C) drinking of contaminated

134 6: Fluids, Electrolytes, and Metabolic Disorders

DIRECTIONS (Questions 48 through 56): The fol-lowing group of questions is preceded by a list oflettered answer options. For each question, matchthe one lettered option that is most closely associ-ated with the question. Each lettered option maybe selected once, multiple times, or not at all.

Questions 48 through 52

(A) citrullinemia(B) porphyria(C) abetalipoproteinemia(D) familial hypercholesterolemia(E) hypertrophic pyloric stenosis(F) Gaucher disease (MPS II)(G) Wilson disease

48. A 3-year-old presents to your office for evalua-tion of recurrent rash with sun exposure. Thissun sensitivity seems to be worsening with age.Examination of his skin reveals areas of resolvingdamage and new areas of exposure which arered, swollen, and excoriated. Additional physicalfindings are splenomegaly and brownish teeth.

49. A 4-month-old child presents with diarrheaand poor growth since birth. The child has ade-quate oral intake but recurrent large malodor-ous stools. He has been evaluated for cysticfibrosis but tests were negative. Stool studiesreveal an increased amount of fecal fat. Celiacsprue is considered and the child undergoescolonoscopy. Intestinal biopsy, however, demon-strates normal-appearing villi.

50. A 16-year-old male comes to clinic for a routinesports physical examination. His physicalexamination is unremarkable except for the skinfindings of multiple pale masses which appearto be xanthomas. Additional family history isremarkable for the sudden early deaths of twopaternal uncles with myocardial infarction.

Questions 51 through 54

(A) familial hypercholesterolemia(B) Tay-Sachs disease(C) Wilson disease(D) Menkes syndrome

51. Kayser-Fleischer rings

52. cherry red macular spots

53. kinky hair

54. Arcus juvenilis

Questions 55 and 56

(A) hypercalcemia(B) hypocalcemia(C) hypercholesterolemia(D) hypomagnesemia

55. An 11-month-old male infant presents withabdominal distention and vomiting. His oralintake is three times his daily maintenancerequirement of formula and his urine output is10 mL/kg/h. The child is small for age andhas abnormal “elfin” facies. Physical examina-tion is remarkable for a loud grade IV/VI sys-tolic ejection murmur at the left sternal border.Echocardiography reveals supravalvular aorticstenosis.

56. A 3-month-old female with a history of seizuresdevelops cyanosis and dyspnea. A grade IV/VIsystolic murmur is present. A chest x-ray showsa boot-shaped heart. No thymic shadow isnoted on the radiograph.

Page 134: Questions - uhs.edu.kh You suspect Lyme disease, most likely contracted by which of the following? (A) ingestion of unripe fruit (B) ingestion of spoiled fruit (C) drinking of contaminated

135

1. (D) Infants weighing less than 10 kg expend anaverage of 100 cal/kg of body weight. A 3-kginfant has a basal caloric requirement of300 calories. The infant needs approximately100 mL of water per 100 calories metabolized.This translates to a fluid requirement of 300 ml.This concept is an important one to remembersince fluid and caloric calculations for adultsare different from those for children. (McMillan,67; Behrman, 242–245)

2. (C) There is a simple method for calculatingthe fluid (and caloric) requirements for aninfant or child: 100 mL/kg for the first 10 kg ofbody weight, 50 mL/kg for the second 10 kgof body weight (between 11 and 20 kg), and20 mL/kg for each kg of body weight greaterthan 20 kg. Using this method, the fluidrequirements for this child are calculated asfollows:

For the first 10 kg

100 mL/kg × 10 kg = 1000 mL

For the second 10 kg

50 mL/kg × 10 kg = 500 mL

For the remaining 10 kg

20 mL/kg × 10 kg = 200 mL

This gives the child a total fluid require-ment of 1000 mL + 500 mL + 200 mL = 1700 mL.Notice that this child does not require 10 timesas much fluid as the 3-kg child. (McMillan, 67–68;Behrman, 242–245)

3. (D) Calculate the daily fluid requirements ofthe patient.

For the first 10 kg

10 kg × 100 mL/kg = 1000 mL

For the next 10 kg

10 kg × 50 mL/kg = 500 mL

For the remainder of the body weight

30 kg × 20 mL/kg = 600 mL

Adding these together (1000 mL + 500 mL +600 mL) gives a daily total of 2100 mL. Thisdaily maintenance requirement divided over24 hours (2100 mL/24 h) gives you 88 mL/h,or approximately 90 mL/h. (Behrman, 242–245)

Answers and Explanations

Page 135: Questions - uhs.edu.kh You suspect Lyme disease, most likely contracted by which of the following? (A) ingestion of unripe fruit (B) ingestion of spoiled fruit (C) drinking of contaminated

136 6: Fluids, Electrolytes, and Metabolic Disorders

4. (B) The infant described is probably about 15%(severely) dehydrated. In classifying infantsand young children, dehydration is dividedinto mild (5%), moderate (10%), and severe(15%) based on clinical examination. (For anolder child and adult, comparable figureswould be 3%, 6%, and 9%.) An infant that ismildly (5%) dehydrated has a normal toslightly increased heart rate, slightly drymucous membranes, poor tear production, andslightly decreased urine output. An infantmoderately (10%) dehydrated demonstratesworsening of the previous signs along withdecreased skin turgor and sunken anteriorfontanelle. An infant with severe (15%) dehy-dration presents with symptoms of hypov-olemic shock with decreased blood pressure,delayed cap refill, Kussmaul respiration, anddepressed sensorium. Many students andphysicians would calculate a rough estimateof total body fluid deficit as 8 kg × 15% = 1200 mLdeficit. However, this calculation is not entirelycorrect, or accurate, as dehydration and percentdehydration are related to a premorbid weight(wtpre)—in this case, the infant’s weight beforehe became dehydrated. Current weight = wtpre ×(1 − percent dehydrated/100).

Therefore, the deficit is 15% of 9.4 kg, or1400 mL. (Note: Total fluid deficit is not thesame as total fluid requirement. Daily totalfluid requirements in a dehydrated childincludes deficit fluid replacement in additionto routine daily fluid requirements and anyabnormal ongoing losses such as continueddiarrhea.) (Behrman, 245–249)

5. (A) Total serum osmolality in this patient isnormal (285–295 mOsm/L). The elevatedserum glucose increases osmolality, but this iscompensated by the lowered sodium concen-tration which decreases osmolality. By usingthe calculation 2(Na) + BUN/2.8 + glucose/18,the estimated osmolality is figured as 2(126) +16/2.8 + 500/18, or 285 mOsm/L. The actualmeasured osmolality might be a little lowerthan this calculated figure because some of theionic material is protein bound and thereforedoes not actually contribute to osmolality.When managing a patient in diabetic ketoaci-dosis the abnormalities in osmolarity are

important, and warrant slow correction ofserum glucose and careful monitoring of serumelectrolytes and mental status changes. (Note:It is important to recognize that a patient is notnecessarily hyperosmolar with a blood glucoseof 500 mg/dL or hypoosmolar with a lowserum sodium.) (Behrman, 193)

6. (B) A child’s maintenance requirements mightbe calculated either on a caloric (weight) basis(5 kg × 100 mL(calories)/kg = 500 mL) or on a sur-face area basis (0.30 m2 × 1600 mL/m2 = 480 mL).For children weighing less than 10 kg, the pre-ferred method is the use of the caloric basis, orthe Holliday-Segar method. The deficit, how-ever, must be calculated on the basis of weight(not surface area): 5 kg × 100 mL/kg = 500 mL.Adding deficit and maintenance: 500 mL +500 mL = 1000 mL. (Robertson, 283–285; Behrman,245–249).

7. (E) The diagnosis of the syndrome of inappro-priate antidiuretic hormone secretion (SIADH) ismade in this patient with a low serum sodium,normal renal function, and inappropriately ele-vated urine sodium concentration. This increasein the extracellular fluid volume occurs from theaction of antidiuretic hormone or vasopressinon the renal collecting tubule, resulting in reten-tion of water and loss of sodium into the urine.Management involves fluid restriction and treat-ing the underlying cause of the SIADH whichincludes, among others, central nervous systemlesions, pulmonary diseases, and medications.

All of the listed answers are possible causesof hyponatremia, defined as a serum sodiumless than 130 meq/L. Symptomatic hypona-tremia (eg, weight gain, nausea, confusion) isnot seen until serum sodium concentration isnear 120 meq/L. Seizures and coma occurwhen the serum sodium value drops quickly orbecomes less than 110 meq/L. Adrenal insuf-ficiency, with deficiency of glucocorticoid/mineralcorticoid, as a cause of hyponatremiaoften will be accompanied by acidosis and hyper-kalemia. Psychogenic polydypsia can cause adramatically low serum sodium, but urine isdiluted with a low sodium level. Dependingon the laboratory method used, hypertriglyc-eridemia can result in a fictitiously low sodium;

Page 136: Questions - uhs.edu.kh You suspect Lyme disease, most likely contracted by which of the following? (A) ingestion of unripe fruit (B) ingestion of spoiled fruit (C) drinking of contaminated

Answers: 4–10 137

it is a rare cause of hyponatremia in children.SIADH is the most likely etiology of this child’shyponatremia. (Behrman, 199–202, 2043–2044)

8. (D) Deficiency of insulin resulting in diabeticketoacidosis (DKA) can be associated with sev-eral metabolic derangements. Insulin acts todrive potassium intracellularly and deficiencyof insulin decreases this movement of potas-sium from the extracellular space to the intra-cellular space. In addition, as acidosis developssecondary to excessive ketone production,potassium is further shifted extracellularly inexchange for a hydrogen ion which moves inthe opposite direction. With hyperglycemia,osmotic diuresis ensues and extracellular potas-sium is lost in the urine. Therefore, patientswith DKA are depleted of potassium even ifserum levels are normal or elevated. Patientsare at risk for developing hypokalemia as treat-ment is initiated. Both insulin administrationand correction of acidosis shift potassium backto the intracellular compartment, thereforedropping serum levels. Symptoms and signs ofhypokalemia include muscle weakness thatmay progress to paralysis, cardiac dysrhyth-mias, as well as flat or absent T waves and thepresence of a U wave on ECG. Potassium levelsshould be monitored closely and potassiumreplacement started as long as hyperkalemia isnot present and the patient has voided.

Cerebral edema is the most common causeof morbidity related to DKA and most oftenoccurs after treatment is initiated. It is notassociated with weakness or ECG changes.Hyponatremia can also complicate therapy forDKA. Hyponatremia is not associated with thepatient’s symptoms or ECG changes. It shouldbe noted that for every 100 mg/dL increase inserum glucose, the serum sodium willdecrease by roughly 1.6 meq/L. Therefore, thepatient’s original sodium would correct tonearly 142 meq/dL if the hyperglycemia isconsidered. Hypocalcemia is associated withneuromuscular excitability and prolongedQT interval. Hypoglycemia is another pos-sible complication of DKA and blood sugarsmust be monitored closely. (McMillan, 67–69,2110–2112)

9. (A) Infants that present with these physicalfindings are mildly to moderately dehydratedand are candidates for oral rehydration ther-apy. Consistent administration of small vol-umes is effective in correcting dehydrationeven with continued diarrhea. Composition ofthe oral replacement fluid should be designed tooptimize repletion of the ECF space and pro-vide some carbohydrate to give minimal caloriesin order to avoid catabolism. The relationship ofsodium concentration to carbohydrate concen-tration is important for appropriate absorptionof sodium without worsening the fluid lossesfrom an osmotic diarrhea caused by overad-ministered and/or inappropriate carbohydratesolutions. Most household liquids (tea andkool-aid) are inappropriate because of theirhypotonic (low sodium) content. However,commercially available rehydration formulas(ie, Pedialyte) are useful. Intravenous therapyis necessary only when the child refuses fluidsorally or has severe dehydration with impend-ing circulatory collapse. (Behrman, 246–250)

10. (B) In an obtunded or comatose patient, regard-less of suspected etiology, a serum glucosemeasurement is useful in directing acute man-agement efforts. The change in sensorium canbe a result of the direct effects of alcohol or aresult of hypoglycemia which occurs secondaryto the effect of alcohol on gluconeogenesis.Gluconeogenesis is impaired in two ways. First,ethanol is an acid in the blood stream and cre-ates a metabolic acidosis which directly reducesthe activity of the metabolic pathway responsi-ble for glucose mobilization. Second, the deple-tion of NADH in the system by the metabolismof alcohol to acetaldehyde (via reduction ofNAD by alcohol dehydrogenase) indirectlystops glucose production, as NADH depletionhalts gluconeogenesis. The other pathway ofalcohol metabolism, which utilizes the micro-somal system, results in the abnormal clearanceof other drugs metabolized by this same path-way in the body. The preferential metabolism ofethanol by this system often causes the clear-ance of other drugs to be slowed and theirblood levels to rise, resulting in their additivetoxic effects to ethanol toxicity. (Behrman, 515)

Page 137: Questions - uhs.edu.kh You suspect Lyme disease, most likely contracted by which of the following? (A) ingestion of unripe fruit (B) ingestion of spoiled fruit (C) drinking of contaminated

138 6: Fluids, Electrolytes, and Metabolic Disorders

11. (C) Pompe disease is an abnormality of carbohy-drate metabolism with autosomal recessive inher-itance. Clinically normal at birth, the infants havea deficiency of acid maltase that leads to a build-up of glycogen products in cells. This in turnleads to marked muscle weakness, hypotonia,and severe cardiomegaly. Children with Pompedisease often die within the first 2 years of lifefrom cardiac or respiratory failure. (Behrman, 474)

12. (C) A deficiency in the enzyme fumarylacetoac-etate hydrolase results in moderate elevations oftyrosine. This is thought to lead to accumulationof intermediate metabolites, primarily succiny-lacetone, leading to damage of the liver, kidney,and central nervous system. (Behrman, 347–349)

13. (C) The defect seen in alcaptonuria leads toincreased amounts of homogentisic acid whichis excreted in the urine. Alkaline urine appearsblack secondary to oxidation and polymeriza-tion of the homogentisic acid. This unusuallycolored urine helps establish the diagnosis.Additionally, the sclera, cornea, and ear carti-lage of patients with alcaptonuria becomedarkly pigmented (ochronosis) secondary tothe black polymer of homogentisic acid. Laterin life they develop arthritis, which is the onlydisabling aspect of the illness. (Behrman, 347–349)

14. (B) Tyrosinemia primarily affects the liver,peripheral nerves, and kidneys. Early deathmost often is a result of hepatic failure. Affectedinfants may become symptomatic as early as2 weeks of age, or may appear normal during thefirst year of life. Commonly, affected infants willfirst present with an acute hepatic crisis precip-itated by an intercurrent illness and resultantcatabolic state. Chronic renal failure, respiratoryfailure, and paralysis can also occur, but are lesslikely to be the cause of death. Diabetes andinfection are not associated. (Behrman, 402–405)

15. (E) Subluxation of the ocular lens known asectopia lentis occurs generally after 3 years ofage in homocystinuria, leading to severe myopiaand iridodonesis (quivering of the iris). Thesepatients lack the enzyme cystathionine synthaseand have multiple problems including severemental retardation, increased thromboembolic

events, psychiatric disorders, skeletal abnormal-ities resembling Marfan disease, generalizedosteoporosis, and seizures. None of the otherchoices is associated with suluxation of the lens.(McMillan, 1833–1834)

16. (E) The patient has hypoketotic hypoglycemiawithout acidosis. Of the possible diagnoses,medium-chain acyl-CoA dehydrogenase defi-ciency (MCADD) is most likely. MCADD is themost common disorder of fatty acid oxidation.Deficiency of the enzyme results in decreasedability to convert medium-chain fatty acids intoacetyl-CoA in mitochondria. During hypo-glycemia, acetyl-CoA is used to produce ketonebodies and is used as a source of energy via thetricarboxylic acid cycle. Therefore patients withMCADD present with hypoketotic hypo-glycemia. Patients usually present between theages of 5–24 months and symptoms often occurin conjunction with an intercurrent illness.Clinical presentation can vary from inciden-tally discovered hypoglycemia to encephalopa-thy. Some patients may be misdiagnosed withReye syndrome. Treatment consists of avoidingprolonged fasting. l-Carnitine can be used insymptomatic patients.

Adrenal insufficiency, accelerated starva-tion, and glycogen storage disease type I resultin hyperketotic hypoglycemia, which is notconsistent with the patient’s presentation.Accidental ingestion needs to be considered incases of hypoglycemia. Sulfonylureas act toincrease insulin secretion by the beta cells ofthe pancreas. Insulin acts to decrease lipolysiswhich then decreases production of fatty acids.This in turn, decreases ketone body (as ketonesare synthesized from fatty acid breakdown)formation resulting in hypoketotic hypo-glycemia. The patient’s insulin level is appro-priately low in this case, making sulfonylureaingestion unlikely. (McMillan, 2176–2178)

17. (B) Sodium and chloride requirements areapproximately 3 meq/kg/day, or 36 meq NaClin a 12 kg infant. Requirements for fluids in a12 kg infant are approximately 1100 mL/day.Therefore, 36 meq of NaCl must be provided in1100 mL of fluid each day to meet requirements(36/1100 × 1000 = 32 meq NaCl/L IVF). Of the

Page 138: Questions - uhs.edu.kh You suspect Lyme disease, most likely contracted by which of the following? (A) ingestion of unripe fruit (B) ingestion of spoiled fruit (C) drinking of contaminated

Answers: 11–22 139

choices listed, 0.9% NaCl contains 155 meqNaCl/L, 0.45% NaCl contains 77 meq NaCl/L,0.2% NaCl contains 35 meq NaCl/L, LR contains130 meq NaCl/L, and plasmalyte contains140 meq NaCl/L. Thus, the best choice forintravenous fluids in this child is 0.2% NS.(McMillan, 68; Robertson, 284)

18. (C) Potassium requirement generally is calcu-lated at 2–2.5 meq KCl/kg/day. For this 15 kgchild that would be 30–37 meq KCl/day. Usingthe Holliday-Segar method, the child’s mainte-nance fluid requirement is 1250 cc/day; 30 meqKCl/1.250 L = 24 meq KCl/L, or approximately25 meq KCl/L. (McMillan, 68; Robertson, 284)

19. (C) Evaluation of dehydration must alwaysinclude the “ABCs” to evaluate for hypov-olemic shock. Following initial fluid resuscita-tion, the serum sodium should be correctedover 24–48 hours. The duration of the hyper-natremic state determines the body’s level ofcompensation of osmolality. The brain has beenfound to make “idiogenic osmoles” in order topreserve cerebral oncotic pressure and to pre-vent neuronal shrinkage. Administering toomuch free water (such as D5W) too quickly canlead to cerebral edema and seizures, thus thehypernatremic dehydration must be correctedslowly. (Rudolph, 1651–1652)

20. (D) The patient is presenting with diabetesinsipidus (DI). This condition can be divided intotwo forms: central or nephrogenic. Central DI isdue to lack of antidiuretic hormone from the pitu-itary gland and responds to DDAVP. NephrogenicDI is due to renal unresponsiveness to antidiuretichormone. As this patient’s urine did not concen-trate upon administration of DDAVP, nephro-genic DI is the diagnosis. Nephrogenic DI can betreated with hydrochlorothiazide. Although itmay seem paradoxical to treat polyuria with adiuretic, depletion of sodium (due to diuretictherapy) results in increased reabsorption ofsodium and water in the proximal tubule. Thisresults in a decrease in the water, lost due to thedefect of ADH action, on the collecting ducts.Nephrogenic DI can be congenital or acquired.The congenital form is x-linked and presents ininfancy in males. Acquired nephrogenic DI can

be due to renal failure, obstructive lesions of theurinary tract, vesicoureteral reflux, nephrocal-cinosis, hypokalemia, hypercalcemia, or drugssuch as lithium or amphotericin.

Fludrocortisone is a mineralocorticoidagonist used to treat deficiency of aldosterone.Aldosterone deficiency presents with hypona-tremia with hyperkalemia. Fluid restriction isdangerous in the face of diabetes insipidus asit can lead to severe dehydration in a shortperiod of time. Nephrogenic DI will not respondto DDAVP, whether it is given parenterally,orally, or nasally. Insulin has no benefit in thispatient, although diabetes mellitus usually alsopresents with polyuria, polydipsia, and weightloss. (McMillan, 1901–1904)

21. (B) Pyruvate kinase deficiency causes a con-genital nonspherocytic hemolytic anemia withresultant hyperbilirubinemia. The breakdownmay be so severe as to put the neonate at risk forbilirubin encephalopathy and require exchangetransfusion. Blood transfusion may be indicatedfor the anemia. Splenectomy improves theanemia but does not cure the underlying defect.Sepsis is not a common initial presentation.Gallstones may occur causing hepatomegaly inthose with pyruvate kinase deficiency. However,this typically occurs in adults rather than neonates.Seizures and hypertonia are not associated.(Rudolph, 1542; McMillan, 1455)

22. (A) Wilson disease is an autosomal recessivedisorder associated with progressive accumu-lation of intracellular copper in the liver, brain,kidneys, and eyes. This accumulation of copperresults from impaired incorporation of copperinto ceruloplasmin and decreased biliarycopper excretion. The gene for Wilson diseaseis located on the long arm of chromosome 13and is designated ATP7B. Typically, the firstorgan to demonstrate signs of dysfunction fromcopper deposition is the liver. Undiagnosedyoung adults usually present with neurologicfindings. Deposition of copper in Descemetmembrane, known as Kayser-Fleischer rings,may be absent in young children. Polyuria isnot a common symptom, but renal dysfunc-tion can occur causing Fanconi syndrome.(McMillan, 2021–2022; Rudolph, 1491–1492)

Page 139: Questions - uhs.edu.kh You suspect Lyme disease, most likely contracted by which of the following? (A) ingestion of unripe fruit (B) ingestion of spoiled fruit (C) drinking of contaminated

140 6: Fluids, Electrolytes, and Metabolic Disorders

23. (C) Strict elimination of mammalian milk, whichis the natural source of lactose (and thereforegalactose) is the treatment for galactokinase defi-ciency. Avoidance of milk products and otherproducts, such as some medications or candythat contain lactose, is important. If started earlyin the neonatal period, this diet can prevent theformation of nuclear cataracts, usually the solemanifestation of this disease. Once formed, thecataracts are not reversible. The disease is auto-somal recessive with gene coding located onchromosome 17q24. Eliminating gluten wouldbe the treatment for celiac disease, while pro-viding increased amounts of folate may be help-ful in pyruvate kinase deficiency. (Rudolph, 641)

24. (C) The mucopolysaccharidoses (MPS) form agroup of diseases associated with lysosomalaccumulation of partially degraded acidmucopolysaccharides. Hurler disease, the mostsevere of the MPS disorders, is autosomalrecessive and occurs in about 1:100,000 births.It is associated with coarse facial features,clouding of the corneas, deafness, airwayobstruction, hydrocephalus, thickening of thecardiac valve leaflets, cardiomyopathy, con-gestive heart failure, poor growth and devel-opment, mental retardation, and early deathoften secondary to cardiopulmonary problems.(McMillan, 2201–2202; Rudolph, 657–658)

25. (E) ALD is an X-linked disease characterized byprogressive neurologic degeneration and adre-nal insufficiency. Patients with ALD have ele-vated plasma levels of very long chain fattyacids (VLCFAs), which accumulate in cerebralwhite matter and adrenal cortex. Treatmentincludes hormone replacement for adrenalinsufficiency, restriction of dietary VLCFAs,provision of C18:1 and C22:1 fatty acids, andbone marrow transplant. The dietary supple-ments and bone marrow therapy seem to bemore successful when instituted prior to onsetof neurologic symptoms. Death generallyoccurs 3–5 years after onset of symptoms. Highplasma levels of short or long chain fatty acidsare typically associated with hyperinsulinismand hypoglycemia, not ALD. Elevated plasmalead and iron levels are not associated withALD. (Rudolph, 2324)

26. (A) Vitamin D refractory rickets type I is arare cause of rickets. It results from impaired1-hydroxylation of 25(OH)2D to 1,25(OH)2D(the active form of vitamin D) and causes lowcalcium levels in the face of normal vitamin Dintake and absorption. Phosphate levels may benormal or slightly decreased in this disorder.Levels of 25(OH)2D are usually normal orelevated while 1,25(OH)2D levels are low.Hypophosphatemic rickets is characterized bylow serum phosphate caused by abnormalrenal phosphate loss. It is an X-linked autoso-mal dominant disorder. Vitamin D deficiencyrickets results from poor absorption and/orpoor intake of vitamin D. Both forms of vitamin Dwould be low in such children. Dark-skinned,breastfed individuals born in the winter are atparticular risk for vitamin D deficiency rickets.Breast milk is known to have insufficient vitaminD; and supplementing breastfed children is rec-ommended. Sunlight exposure causes produc-tion of vitamin D in the skin. Therefore inadequateexposure to light may compound this problem.Children with cystic fibrosis may also havevitamin D deficiency rickets caused by poorvitamin D absorption due to pancreatic insuffi-ciency. Finally, children with 22q11 deletion syn-drome may have hypocalcemia, but this istypically due to the congenital absence ofparathyroid glands. Vitamin D levels are normal.(Rudolph, 798, 2156–2160)

27. (C) Pyruvate is converted to acetyl-CoA duringglycolysis via the pyruvate dehydrogenasecomplex, which consists of five enzymes.Babies with this defect may have a severe lacticacidosis with resultant tachypnea, hypotonia,lethargy, and coma in the first few days of life,or the disease may not manifest itself formonths to years until the child is stressed byinfection, prolonged fasting, or other condi-tions requiring increased gluconeogenesis.Treatment centers on reducing gluconeogen-esis (the major pathway is through pyruvate)by providing a high-fat, low-carbohydratediet, normalizing blood sugars, and admin-istering sodium bicarbonate and carnitine.Beta-hydroxybutyrate and acetoacetate areketone bodies and play a prominent role in dia-betic ketoacidosis. (McMillan, 2334)

Page 140: Questions - uhs.edu.kh You suspect Lyme disease, most likely contracted by which of the following? (A) ingestion of unripe fruit (B) ingestion of spoiled fruit (C) drinking of contaminated

Answers: 23–32 141

28. (C) The child described probably has osteo-genesis imperfecta, a disorder characterizedby osteoporotic bones that fracture easily. Bluesclera, present in infancy, is a common fea-ture. Some patients also have opalescentdentin (dentinogenesis imperfecta), and manydevelop conductive hearing loss in adoles-cence. At least four types have been described,with differing modes of inheritance and vary-ing degrees of severity. Osteopenia is not seenwith achondroplasia. Phenotypically, peoplewith achondroplasia have rhizomelic shortstature, flat nasal bridge, and prominent fore-head. Those with histiocytosis X demonstratesharply demarcated osteolytic lesions on x-rayrather than osteopenia. Osteopetrosis radi-ographs show extreme bone density. Childrenwith rickets may have leg bowing and/orflared wrists noted on x-ray in addition toosteopenia. Osteogenesis imperfecta is the onlychoice that has blue sclera associated with it.(Rudolph, 2161)

29. (B) Bilious vomitus consists of gastric andduodenal (pancreatic secretions and bile) con-tents. Pancreatic secretions contain sodium at120–140 meq/L, potassium at 5–15 meq/L,chloride at 100–150 meq/L, bicarbonateat 100 meq/L; bile contains sodium at120–140 meq/L, potassium at 5–15 meq/L,chloride at 80–120 meq/L, and bicarbonate at40 meq/L. Knowledge of body fluid composi-tion is useful in understanding the metabolicderangement seen. Typically, bilious vomitingresults in isotonic dehydration and either aneutral pH or acidic pH. This contrasts withpure gastric losses, such as those seen in pyloricstenosis, where the dehydration typicallyresults in a hypochloremic, metabolic alkalosis.(Rudolph, 1400–1402; Robertson, 288)

30. (D) This patient most likely has renal tubularacidosis (RTA), described as Fanconi or aFanconi-like syndrome. This is a proximal RTAheralded by urinary losses of bicarbonate andphosphate. Classic findings are aminoaciduria,phosphaturia, and glycosuria with resultanthyperchloremic metabolic acidosis with normalanion gap. There are multiple causes of this dis-order including inborn errors of metabolism

(cystinosis, galactosemia, tyrosinemia, andWilson disease), and acquired forms from heavymetal toxicity and drug toxicity (ie, gentamicin,valproic acid, cisplatin). The metabolic acidosisand phosphate loss seen in this disorder lead togrowth failure and vitamin D-resistant rickets.Nonorganic failure to thrive would not beexpected with this calorie intake. Congenital adre-nal hyperplasia typically demonstrates hyper-kalemia and hypernatremia. Diabetes insipidususually exhibits hypernatremia. (Rudolph, 1708–1709)

31. (D) Classic phenylketonuria results from thelack of, or near complete lack of, phenylala-nine hydroxylase. Impaired metabolism of thecommon essential amino acid phenylalanineto tyrosine leads to build-up of phenylpyruvicacid and phenylethylamine. These products aswell as excess phenylalanine lead to centralnervous system damage. Newborns are rou-tinely screened for PKU. The screening is per-formed by collecting blood spots on filter paperat the time of discharge from the hospital tohome and before 7 days of life. Infants dis-charged at less than 24 hours of age shouldhave the screening test repeated prior to14 days of life. Unfortunately, this baby didnot have adequate newborn screening. For PKU,presymptomatic diagnosis allows control ofthe disease through control of the diet of theinfant. Though further CNS damage can beprevented in this case with dietary control, thesigns and symptoms this baby displays are, forthe most part, irreversible. The other choicesare not associated with this constellation ofsymptoms. (Rudolph CD, 579, 609–611)

32. (D) Treatment of phenylketonuria consists of avery strict diet low in phenylalanine. Reducingphenylalanine in the diet will then reduce theserum levels and help reduce the consequencesof the disease, which include brain damage,mental retardation, seizures, athetosis, hyper-activity, and behavioral problems. The diet isdifficult to adhere to, and creates increasedstress on the child and the family, which addsto the complexity of the disease. (Behrman,344–346)

Page 141: Questions - uhs.edu.kh You suspect Lyme disease, most likely contracted by which of the following? (A) ingestion of unripe fruit (B) ingestion of spoiled fruit (C) drinking of contaminated

142 6: Fluids, Electrolytes, and Metabolic Disorders

33. (B) Classic galactosemia occurs in 1 in 60,000newborns. Defective or total lack of galactose-1-phosphate uridyl transferase causes an inabil-ity to convert galactose into glucose. Normallygalactose converts to galactose-1-phosphateby galactokinase. Galactose-1-phosphate isthen converted to glucose-1-phosphate by theenzyme in question. Without the enzyme,galactose-1-phosphate accumulates, causingparenchymal damage to the kidneys, liver, andbrain. Indirect hyperbilirubinemia, acidosis,and urosepsis (classically due to E coli), fre-quently are seen at presentation. Other mani-festations include hypoglycemia, hepatomegaly,vomiting, seizures, and cataracts. With time,cirrhosis and mental retardation can develop.(Rudolph, 640–641)

34. (D) The incidence of cystic fibrosis (CF) inCaucasians is about 1 in 3,300 live births; inci-dence in African-Americans and Asians is sig-nificantly lower. CF is an autosomal recessivedisease causing an abnormality in the cysticfibrosis transmembrane conductance regulatorresulting in abnormal transport of sodium andchloride across epithelial cell membranes.CF affects primarily the pulmonary and gas-trointestinal systems with varying degrees.Pulmonary symptoms include chronic coughand sputum production, persistent radi-ographic changes (hyperinflation, atelectasis),wheezing, nasal polyps, and digital clubbing.GI symptoms include meconium ileus, pro-longed jaundice, hepatic disease due to biliarycirrhosis, and failure to thrive (deficiency offat-soluble vitamins and protein). Patients sus-pected of having CF are first evaluated by asweat chloride test. A sweat chloride concen-tration of 60 mmol/L or greater is consistentwith a diagnosis of CF. Concentrations of40 mmol/L or greater in infants aged 3 monthsor younger is highly suggestive of CF. DNAanalysis can detect the majority of mutationsthat cause CF and may be done for further con-firmation or when results of a sweat chloridetest are borderline. There are capabilities to donewborn screening, but this is not in wide-spread use at this time. Children with CF mayhave increased fecal fat due to pancreatic insuf-ficiency. However, measuring fecal fat alone

is not diagnostic. Pancreatic biopsy is notconsidered a diagnostic measure. Endomysialantibodies are a useful screen for celiac dis-ease, but not helpful in CF. (Rudolph, 1967–1973;McMillan, 1992)

35. (C) Pyloric stenosis with persistent vomitingleads to loss of hydrochloric acid. The resultanthypochloremic metabolic alkalosis can be pro-found, and must be corrected prior to surgery.The incidence is approximately 1/150 maleinfants and 1/750 female infants and the risks aregreatly increased in male infants born to motherswho had pyloric stenosis. (Rudolph, 722:1402)

36. (A) Ornithine transcarbamylase (OTC) defi-ciency is the most common of the urea cycle dis-orders. It is an X-linked trait with affected malestypically presenting as neonates with extremehyperammonemia while citrulline levels are sig-nificantly reduced. Therapy includes a restricteddietary protein intake and special foods withonly essential amino acids, l-citrulline, andsodium phenylbutyrate. Many individuals dieduring the neonatal period even with aggressivetherapy. Because the disorder is X-linked, maleoffspring have a 50% chance of having severe OTCdeficiency, though female carriers can have vary-ing degrees of usually much milder symptoms.(McMillan, 2162–2163)

37. (C) In this child, immediate treatment indicatedis a 20 mL/kg rapid infusion of fluid to quicklyexpand the extracellular fluid volume. Suchaggressive fluid resuscitation is the mainstay ofpreventing and treating shock. Only isotonicfluids should be used. Use of hypotonic fluidsresults in loss of fluid intracellularly, cell lysis,and potentially life-threatening cerebral edema.Other fluids such as albumin and blood can beused for volume expansion, but usually are notas readily available as crystalloid solutions inemergent rehydration situations. Dependingon the degree, severity, and nature of thevolume loss, two to three isotonic fluid bolusesof normal saline (154 meq/L of sodium andchloride) may be required. (Rudolph, 1644–1646)

38. (A) The initial hydrating solution should con-tain 135–154 meq/L of sodium for two reasons.

Page 142: Questions - uhs.edu.kh You suspect Lyme disease, most likely contracted by which of the following? (A) ingestion of unripe fruit (B) ingestion of spoiled fruit (C) drinking of contaminated

Answers: 33–43 143

One reason is that sodium will effectively stayin the intravascular space and help to expandthe intravascular volume because it does notreadily cross intracellularly. The second reasonis the record of safety in administering isotonicfluid as opposed to hypertonic or hypotonicfluid. Even in cases of severe hypernatremic orhyponatremic dehydration, the additionalsodium and free water provided with even twoto three boluses of the isotonic fluid is negligi-ble in its impact on the serum sodium level,especially when repleting the intravascularspace of a patient in shock or near shock.(Rudolph, 650–653)

39. (E) Because of the concern about renal impair-ment/damage from the dehydration andhypoperfusion of the kidneys, potassiumshould be withheld from the rehydration solu-tions until the patient has voided or the serumpotassium level is known. In addition, potas-sium chloride should be administered as abolus only in specific situations and in a care-fully controlled manner because of the possi-bility of precipitating EKG abnormalities,arrythmias, and muscle weakness. (Rudolph,1645–1646)

40. (C) The history of vomiting and diarrhea, theweight loss greater than 10% of birth weight,low serum sodium, and elevated serum potas-sium are highly suggestive of adrenal insuffi-ciency. The findings on physical examination ofa prominent clitoris and labial fusion are sug-gestive of virilization, and further support thediagnosis of adrenal insufficiency. Congenitaladrenal hyperplasia (CAH) could explain allof these findings. CAH is a condition resultingfrom an inherited defect in cortisol synthesis.Ninety to ninety-five percent of cases arecaused by 21-hydroxylase deficiency. Mostforms of this disorder are associated with vir-ilization and/or salt wasting. Although a viralgastroenteritis could explain the dehydration inan older infant or child, it would be less likelyin a neonate of this age. Those affected by inap-propriate secretion of ADH also have lowsodium, but often experience abnormal increasesin weight secondary to fluid retention. (Rudolph,2038–2041)

41. (A) Congenital adrenal hyperplasia is an auto-somal recessive genetic disorder associatedwith either XX or XY genotype. However,because of the virilization associated withCAH, many affected XX females are evaluatedfor ambiguous genitalia in the newborn period.Males and females are affected equally, thoughdiagnosis in males is often delayed outside thenewborn period until they experience a “salt-losing” crisis (usually in the second week oflife after maternal mineralocorticoids aremetabolized) or show evidence of increasedmasculinization. Many states have added17-hydroxyprogesterone levels to the new-born screening in order to detect those maleinfants without ambiguous genitalia. Prenataldiagnosis is available for at risk families.Screening methods include HLA typing, DNAprobe hybridization, and measurements of17-hydroxyprogesterone in the amniotic fluid.(Behrman, 1909–1913)

42. (E) Therapy for the salt-losing form of adrenalhyperplasia requires replacement of both min-eralcorticoids and glucocorticoids. In stressedchildren (ie, those experiencing illness or salt-losing crisis) a “stress” dosage should be admin-istered of the glucocorticoid that is two to threetimes the daily calculated replacement. A highsalt intake is also useful depending on the degreeof salt wasting. For infants, NaCl is added to theformula (1 g NaCl/10 kg of weight). (Behrman, 1735)

43. (D) Although jitteriness in neonates can resultfrom abnormalities of each of the possibleanswers, this infant has qualities that make hypo-glycemia the most likely cause. The size of theinfant is well above the 90th percentile for new-born males. Many causes of macrosomia (eg,previous large-for-gestational age infants andmaternal diabetes) are associated with abnormalglucose metabolism. The additional findings ofear anomalies could implicate a calcium abnor-mality (eg, DiGeorge anomaly/sequence), butthe child has no heart murmur and is not failingto thrive at this point, making this a less likelydiagnosis. (McMillan, 346–347)

Page 143: Questions - uhs.edu.kh You suspect Lyme disease, most likely contracted by which of the following? (A) ingestion of unripe fruit (B) ingestion of spoiled fruit (C) drinking of contaminated

144 6: Fluids, Electrolytes, and Metabolic Disorders

44. (E) There is a well-known association ofBeckwith-Wiedemann syndrome (BWS) withhypoglycemia affecting up to 50% of childrenwith the disorder. Physical findings are charac-terized by macrosomia, microcephaly, vis-ceromegaly, and macroglossia. Other associatedanomalies are an increased incidence of hemi-hypertrophy, omphaloceles, cryptorchidism,and renal tumors. The ear pits or creases arenot always present in affected patients but arehighly characteristic of BWS. Transmission isoften from a sporadic mutation on chromosome11p15, although autosomal dominant inheri-tance is also seen. The frequency of occurrenceis 1:15,000, with variable expression. Mild tomoderate mental deficiency has been reportedin this disorder and is thought to be relatedto neonatal hypoglycemia. The other syn-dromes are not associated with hypoglycemia.(McMillan, 2641)

45. (A) The exact etiology of hypoglycemia in aneonate can be difficult to determine andinvolves a differential that is very different fromthat seen in an older infant, child, or adult.Small–for-gestational-age infants and prema-ture infants both have increased incidence ofsymptomatic hypoglycemia. A majority of theirglucose problems are related to deficient glyco-gen stores, muscle protein, and body fat neededfor metabolization to meet energy requirements.Infants born to diabetic mothers also experi-ence an increased incidence of hypoglycemia.However, hypoglycemia in infants of diabeticmothers is not due to insufficient stores, but isdue to hyperinsulinemia and low glucagonlevels. Beckwith-Wiedemann syndrome infantsalso experience hyperinsulinemia which causeshypoglycemia. Their increased insulin secre-tion is caused by pancreatic islet cell hypertro-phy. Treatment for these infants is the same asfor other causes of hyperinsulinism; supportiveadministration of intravenous glucose at a rateof 6–8 mg/kg/min. At times, more aggressivetreatments are warranted (eg, increased ratesof glucose administration and supplementationof regulatory hormones by injections of steroidsand growth hormone). (McMillan, 346–347)

46. (E) Oil of wintergreen is a solution containingmethylsalicylate, a form of salicylic acid. Wheningested, a small quantity can cause a rathertoxic ingestion because one teaspoon (5 cc) ofthis liquid contains an amount of salicylateequivalent of 22 adult aspirin. Salicylate inges-tions are impressive in the unique metabolicderangements comprised of a mixed respira-tory alkalosis with metabolic acidosis. Initially,the salicylate is absorbed rapidly and directlystimulates the respiratory center causing tachyp-nea and a respiratory alkalosis. Eventually, lacticacid levels, along with other metabolic acidlevels, begin to rise in the serum resulting in asevere acidosis. Oxidative phosphorylation isuncoupled by the salicylate and the patient’sacidosis worsens. Symptoms of hyperpnea,tachycardia, lethargy, vomiting, dehydration,and coma can occur. Treatment is directed atprimarily supportive care until the toxicity hasbeen corrected. Organophosphate poisoningleads to cholinergic symptoms. Acetaminophenoverdose leads to initial nonspecific symptoms,followed by liver dysfunction. Acidosis is rarein ibuprofen ingestion as this usuallly presentswith GI complaints. Ethanol ingestion is notassociated with a respiratory alkalosis but isassociated with a metabolic acidosis. (Behrman,2367–2368; McMillan, 755–758)

47. (A) The mainstay of management with anyingestion is (1) preventing further absorption ofthe toxic material, (2) facilitating removal ofthe toxin from the system, and (3) administer-ing an antidote to the toxin/poison if one isavailable. These management steps along withgood supportive care will minimize the dura-tion of toxicity and facilitate rapid recovery ofthe patient with the least amount of adverseeffects.

Patients with salicylate toxicity typicallyare approximately 5%–10% dehydrated andhave experienced losses of potassium andhydrogen via the urine. Therapy includes bothaggressive rehydration and alkalinization ofthe urine with administration of sodium bicar-bonate. Unfortunately, no antidote is availablefor salicylate poisoning. In severe ingestions,hemodialysis may be necessary if the quantity

Page 144: Questions - uhs.edu.kh You suspect Lyme disease, most likely contracted by which of the following? (A) ingestion of unripe fruit (B) ingestion of spoiled fruit (C) drinking of contaminated

Answers: 44–54 145

of ingestion, level of salicylates, and observedtoxicity are not responding to conventionalmeasures. (Behrman, 2367–2368; McMillan, 757–758)

48. (B) This child is suffering from one of the por-phyrias, a group of relatively rare inherited dis-orders that result in defects in heme biosynthesisand a build-up of heme precursors. These pre-cursors exist in elevated amounts in the skincreating the reactions to ultraviolet light. Someforms cause repetitive attacks of abdominal pain(neurovisceral attacks). (McMillan, 1999–2003)

49. (C) This patient’s history is most suggestive ofabetalipoproteinemia, or Bassen-Kornsweigdisease. This disease is an autosomal recessivedisorder resulting in absence of the betal-ipoproteins in the plasma. The resultant fea-tures are fat malabsorption (due to failure toform chylomicrons in the intestine), failure tothrive, cerebellar ataxia, retinitis pigmentosa,and changes in red blood cell morphology (acan-thocytosis). Management includes supplemen-tation of fat-soluble vitamins (ADEK) andmaintenance of a low-fat diet. (McMillan, 1998)

50. (D) This patient’s history is most suggestive offamilial hypercholesterolemia, an autosomaldominant disorder with a prevalence of 1/500individuals. Additional evaluation of a fastingserum cholesterol level (with HDL and LDLlevels), triglyceride levels, and 12-lead ECG isindicated. Dietary reduction of cholesterol canbe effective treatment; however, cholesterol-lowering agents such as cholysteramine (ionexchange resin) or lovastatin (HMG-CoAreductase inhibitor) are usually necessary.(McMillan, 1865–1866)

51. (C) Wilson disease is an autosomal recessivedisorder associated with increased depositionof copper into tissues, primarily the liver ini-tially, then other parts of the body. The basicdefect appears to be an inability to excretehepatic copper into bile and an inability toincorporate copper into apoceruloplasmin.Storage of copper in the cornea may result inbrownish green granular copper deposits in

Descemet membrane, given the name Kayser-Fleischer rings. The presence of these often cor-relates with neurologic involvement. (McMillan,2355–2357)

52. (B) Tay-Sachs disease is one of the sphingolipi-doses and produces disease secondary toabnormal lysosomal accumulation of gly-cosphingolipids, gangliosides, and sphin-gomyelin. Faulty degradation due to absent ordeficient lysosomal acid hydrolases is thedefect. Tay-Sachs is associated with storage ofGM2 ganglioside in the nervous system. Cherryred spots represent a normal red macular areaof the eye surrounded by a white area of stor-age material. Later, the spots will turn darker asmacular degeneration progresses. The diseaseis associated with progressive CNS degenera-tion, seizures, blindness, respiratory problems,and death, usually by 3 or 4 years of age.(McMillan, 1875)

53. (D) Menkes syndrome is an X-linked recessivedisease associated with faulty copper trans-port. In part, the basic defect includes animpaired ability to incorporate copper into cer-tain enzymes that need it as a cofactor. Childrenwith the disease develop progressive neuro-logic deterioration, seizures, and eventualdeath (primarily from infection). The mostcharacteristic aspect of the disease is the “kinkyhair” which is brittle, depigmented, dull, short,and brush-like. (McMillan, 2355; Jones, 216–217)

54. (A) Arcus juvenilis is the appearance of anopaque ring close to the periphery of the corneaassociated with hypercholesterolemia andhyperlipidemia. Other ocular findings are xan-thomas of the eyelids and pale deposits in theretina (lipemia retinalis). Familial hypercho-lesterolemia results in accelerated cholesterolsynthesis, leading to increased risk of athero-genesis which results in peripheral vasculardisease, heart attack, or stroke. The disease isalso associated with the more commonly knownxanthomas of the skin. (McMillan, 1865–1866)

Page 145: Questions - uhs.edu.kh You suspect Lyme disease, most likely contracted by which of the following? (A) ingestion of unripe fruit (B) ingestion of spoiled fruit (C) drinking of contaminated

146 6: Fluids, Electrolytes, and Metabolic Disorders

55. (A) This patient shows symptoms of hypercal-cemia (increased drinking and voiding) com-monly associated with Williams syndrome.Williams syndrome is an autosomal dominantgenetic disorder that commonly results from spo-radic mutations on chromosome 7 (a microdele-tion of the elastin gene). The syndrome ischaracterized by its unique facial features ofprominent lips and thin philtrum, as well ascardiac anomalies (eg, supravalvular aorticstenosis). Infantile hypercalcemia often is notedand can lead to problems of nephrocalci-nosis, constipation, and mental status changes.

The hypercalcemia rarely persists into adult-hood. (Rudolph, 2153; Robertson, 301; Jones, 120–123)

56. (B) The patient presents with findings of tetralogyof Fallot. The absence of a thymic shadow indi-cates thymic aplasia. These findings are consistentwith 22q11 deletion syndrome. This condition isdue to developmental defects in the 3rd and 4thpharyngeal arches and is associated withconotruncal heart defects, thymic aplasia (leadingto immunodeficiency), and hypoparathyroidism.Initial presentation is often due to hypocalcemicseizures. (McMillan, 1547–1548, 2075–2076, 2462–2463)

Clayton PT. Diagnosis of inherited disorders of livermetabolism. J Inherit Metab Dis. 2003;26:135–146.

Kliegman RM, Behrman RE, Jenson HB, Stanton BF.Nelson Textbook of Pediatrics. 18th ed. Philadelphia, PA:Saunders Elsevier; 2007: 267–319.

McMillan JA, Feigin RD, DeAngelis CD, Jones MD. et al.Oski’s Pediatrics: Principles and Practice. 4th ed.Philadelphia, PA: Lippincott Williams & Wilkins; 2006:2145–2175, 2181–2217.

Nassogne MC. Urea cycle defects: Management and out-comes. J Inhert Metab Dis. 2005;28:407–414.

Prietsch V, Lindner M, Zschocke J, Nyhan WL, Hoffmann GF.Emergency management of inherited metabolic diseases.J Inherit Metab Dis. 2002;25:531–546.

SELECTED READINGS

Page 146: Questions - uhs.edu.kh You suspect Lyme disease, most likely contracted by which of the following? (A) ingestion of unripe fruit (B) ingestion of spoiled fruit (C) drinking of contaminated

CHAPTER 7

Infectious DiseasesEmily A. Thorell, MD

Angela Myers, MD, MPH

As many as 80% of child health visits relate to infectious disease. Public health programs directed towardimprovements in water, nutrition, and immunization against communicable diseases has resulted insignificant declines in infant mortality of up to 200-fold between the mid-1800s and late 1990s. Still half of allpostneonatal deaths in the United States are caused by potentially preventable causes including infectionsand injuries. The advent of Haemophilus influenzae type b vaccine in the late 1980s virtually eradicated themost important pathogen of sepsis and meningitis in the infant population. However, the persistence of suchpathogens as tuberculosis, HIV, and malaria and the emergence of new infectious agents offset the gains ofthe last century and continue to underscore the importance of the study of infectious diseases, particularlyfor those involved in the care of children. The following questions address appropriate knowledge ofpediatric infections with focus on specific etiologic agents, epidemiology of disease, clinical manifestations,diagnostic tests, treatment, and control measures.

147

Page 147: Questions - uhs.edu.kh You suspect Lyme disease, most likely contracted by which of the following? (A) ingestion of unripe fruit (B) ingestion of spoiled fruit (C) drinking of contaminated

148

DIRECTIONS (Questions 1 through 101): For eachof the multiple choice questions in this sectionselect the one lettered answer that is the bestresponse in each case.

1. A 13-year-old female presents to the emergencydepartment with a 3-day history of fever above104°F (40°C); vomiting; diarrhea; and diffuse,erythematous rash. She is found to have ortho-static hypotension. Laboratory evaluation revealsdecreased platelets and elevated liver and renalfunction tests. The mother of the child informsyou that the child is currently menstruating. Yoususpect that this child has a toxin mediated infec-tion. What is the most likely etiology of this toxinmediated infection?

(A) Streptococcus pyogenes(B) Staphylococcus aureus(C) Neisseria gonorrhoeae(D) Streptococcus agalactiae(E) Shiga-toxin-producing Escherichia coli

2. What are additional features commonly associ-ated with the toxin-mediated process describedin Question 1?

(A) pneumonia and pleural effusion(B) arthritis and myositis(C) exudative tonsillitis and cervical adenitis(D) thrombocytopenia and conjunctival

hyperemia(E) intraabdominal abscess and ascites

3. Hilar lymphadenopathy is noticed on the chestx-ray obtained for a 6-year-old male known tohave mild intermittent asthma. The child is from

rural Ohio and lives on a farm with chickens.Tuberculin skin testing is negative and the familydenies history of exposure to tuberculosis. Yoususpect histoplasmosis infection. What is themost likely mode of transmission of the spores?

(A) ingestion(B) direct contact from another person(C) inhalation(D) skin inoculation(E) contaminated fomites

4. You are seeing a healthy, full-term infant justafter delivery. The mother plans on breast-feedingthe baby and asks you how long immunitytransferred to the baby from the pregnancywould be present. When does passively trans-ferred maternal IgG antibody reach a nadir?

(A) 1–2 months(B) 3–6 months(C) 12–18 months(D) 18–24 months(E) 24 months

5. The mother from Question 3 also tells you thatshe has read that breast-feeding immunity isdifferent than that acquired in utero. What isthe major source of immunity conferred inbreast milk?

(A) T-cell mediated(B) complement mediated(C) IgM(D) IgA(E) IgD

Questions

Page 148: Questions - uhs.edu.kh You suspect Lyme disease, most likely contracted by which of the following? (A) ingestion of unripe fruit (B) ingestion of spoiled fruit (C) drinking of contaminated

Questions: 1–12 149

6. The mother of a 3-year-old child in your officefor a well-child visit mentions that her child hasbeen scratching his bottom lately. The motherreports to you that on a few occasions she hasseen what appears to be rice in his diaper. Whichof the following is the most likely explanation?

(A) Enterobius vermicularis infection.(B) The child has malabsorption disorder.(C) The child eats rice frequently.(D) Strongyloides stercoralis infection.(E) Taenia solium infection.

7. You were called to the neonatal nursery to seean infant with the following features: growthretardation, rash, and absent red reflexes. Onyour examination, the baby is very small, withbluish purpuric skin lesions, and obviouscataracts. The baby also has a heart murmur.You suspect congenital infection. What is themost likely etiology?

(A) HIV(B) toxoplasmosis(C) CMV(D) parvovirus(E) rubella

8. A 6-year-old unimmunized child has fever of104°F (40°C) and cropped vesicles on the trunkwith scattered scabbed lesions. Which of thefollowing infections is the likely diagnosis?

(A) measles(B) smallpox(C) HHV-6(D) herpes simplex virus(E) varicella

9. The mother of a 6-month-old child seeing youfor a well visit, requests immunization againstmeasles as she has heard it is present in thecommunity. At what age should the first doseof live attenuated measles vaccine (as MMR) beroutinely administered in the absence of a com-munity measles outbreak?

(A) 6 months of age(B) 9 months of age

(C) 12–15 months of age(D) 18–24 months of age(E) At the time of school entry

10. A 12-month-old child presents to your office fora well-child visit. The mother informs you thatat the 6 month visit the child had fever andlocalized arm swelling after diphtheria-tetanus-acellular pertussis (DTaP) vaccine. She is won-dering if this vaccine should be held. Which ofthe following is a contraindication to receivingDTaP immunization?

(A) encephalopathy within 7 days of previ-ous dose

(B) family history of seizures(C) for a preterm infant who is 2 months of

age(D) an internationally adopted child(E) a prior dose associated with temperature

> 103.1°F (39.5°C)

11. An 8-year-old child presents to the emergencydepartment with a 3-day history of fever and newaltered mental status. Her parents report that theyhave been camping several times and she hassustained a lot of mosquito bites. Mosquitoes arerecognized as vectors in the transmission ofencephalitis of which of the following viruses?

(A) arbovirus(B) coxsackievirus(C) enterovirus(D) influenza virus(E) mumps virus

12. A malnourished 3-year-old child from Mexicohas a positive tuberculin skin test (TST > 10 mmfor this age). Which of the following would bemost concerning for extra-pulmonary disease?

(A) fever(B) hilar lymphadenopathy on roentgeno-

graph(C) night sweats(D) weight loss(E) hepatosplenomegaly

Page 149: Questions - uhs.edu.kh You suspect Lyme disease, most likely contracted by which of the following? (A) ingestion of unripe fruit (B) ingestion of spoiled fruit (C) drinking of contaminated

150 7: Infectious Diseases

13. A 15-year-old female comes to your office with4-day history of fever and cough. Chest x-rayfindings include bilateral patchy infiltrates.Oxygen saturation is normal. What is the mostlikely etiology of her pneumonia?

(A) Streptococcus pneumoniae(B) Staphylococcus aureus(C) Mycoplasma pneumoniae(D) Chlamydophila psittaci(E) Pneumocystis jiroveci

14. A 3-month-old infant has had upper respira-tory symptoms for a few days and presents tothe emergency department with respiratorydistress, wheezing, and hypoxia. You diagnosebronchiolitis. What is the most common causeof acute bronchiolitis in infants?

(A) respiratory syncytial virus(B) parainfluenza virus(C) cytomegalovirus(D) influenza virus(E) human metapneumovirus

15. On tuberculin skin testing, a well 5-year-oldwho traveled last year to Mexico is found tohave 8 mm of induration to 5 tuberculin units(TU) of purified protein. There is no historysuggestive of contact with tuberculosis. Whatdoes this reaction likely indicate?

(A) sensitivity to the diluent(B) subcutaneous rather than intradermal

injection of test material(C) cross-sensitivity to nontuberculous

mycobacteria(D) tuberculosis infection(E) tuberculosis disease

16. A 4-year-old child presents with fever, nightsweats, weight loss, and cough. A diagnosis ofpulmonary tuberculosis infection was madebased on chest x-ray and tuberculin skin test-ing. The mother of the child asks when herchild may return to day care. When can a childdiagnosed with pulmonary tuberculosis infec-tion return to day care or school?

(A) as soon as effective therapy has beeninstituted

(B) 2 weeks after therapy is initiated(C) once negative sputum smears are

confirmed(D) once therapy is completed(E) whether they are receiving therapy or not

17. A 7-year-old child who recently traveled withher family to India presents to your office witha 2-day history of fever, diarrhea, and tenesmus.Stool examination reveals blood and leukocytes.You suspect infection with Salmonella typhi asthe cause of her symptoms. What additionalsigns/symptoms are typically found with thisinfection?

(A) rectal prolapse(B) hepatosplenomegaly and abdominal

pain(C) intensely pruritic skin rash(D) cough and lymphadenopathy(E) toxic megacolon and perforation

18. You are seeing an 8-month-old child in youroffice for recurrent thrush, candida diaper der-matitis, and multiple recent skin pustules. Inaddition, he was recently hospitalized with ahepatic abscess. You suspect that this child mayhave chronic granulomatous disease. Which ofthe following defense mechanisms is defectiveand the cause for recurrent infections in chronicgranulomatous disease?

(A) leukocyte migration(B) synthesis of collagen(C) capillary permeability(D) tissue repair following injury(E) phagocyte oxidative burst

19. A mother brings her 11-month-old infant alongwith her 3- and 9-year-old children into youroffice for a several day history of pruritic rash.You note the rash is similar in the two olderchildren with burrows characteristic of scabiesin between the fingers, and on the flexor sur-faces of the wrists. Which of the following rep-resents the typical manifestation in infants?

Page 150: Questions - uhs.edu.kh You suspect Lyme disease, most likely contracted by which of the following? (A) ingestion of unripe fruit (B) ingestion of spoiled fruit (C) drinking of contaminated

Questions: 13–25 151

(A) a maculopapular rash

(B) concentration of burrows in the anterioraxillary folds

(C) characteristic sparing of the face(D) initial fever(E) a papulovesicular rash

20. You are seeing a 4-year-old child in the emergencydepartment with abdominal pain, and acute onsetof bloody diarrhea. Due to a recent outbreak in thecommunity related to a popular fast food restau-rant, you suspect E coli 0157:H7. Which of the fol-lowing is seen most commonly as a complicationof Shiga-toxin-producing E coli (formerly knownas enterohemorrhagic E coli) diarrhea?

(A) meningitis(B) hemolytic-uremic syndrome(C) chronic diarrhea(D) endocarditis(E) pneumonia

21. A full-term infant is born to a mother withoutprevious prenatal screening. Laboratory eval-uation obtained at delivery reveals a positiveRPR at 1:32. You carefully examine the infantafter birth to evaluate for cutaneous manifes-tations of congenital syphilis. Which of the fol-lowing cutaneous findings would be mostcharacteristic of congenital syphilis?

(A) that the lesions are sterile(B) that lesions are most numerous on the

trunk(C) a fleeting pink macular rash(D) a papular purpuric eruption on the legs

and buttocks(E) vesiculobullous lesions of the palms and

soles

22. The parents of a 1-month-old female infantbring their daughter to the emergency depart-ment for febrile illness. The parents deny anylocalizing symptoms, but report fussiness. Youproceed with laboratory evaluation includingcomplete blood count, urinalysis with culture,blood culture, and CSF examination. The mostcommon serious bacterial infection encoun-tered in a febrile 1-month-old infant is

(A) Escherichia coli urinary tract infection(B) Salmonella enteritis(C) group A streptococcal bacteremia(D) meningococcemia(E) Haemophilus influenzae type b meningitis

23. A 10-year-old female presents to the emergencydepartment for forensic examination after sexualabuse. Cultures are obtained for Neisseria gonor-rhoeae and Chlamydia trachomatis. What is themost common manifestation of infection withN gonorrhoeae in a prepubertal girl?

(A) arthritis(B) conjunctivitis(C) peritonitis(D) salpingitis(E) vaginitis

24. A 16-year-old female presents to your officewith complaints of vague abdominal discom-fort and vaginal discharge. Pelvic examinationis performed and N gonorrhoeae grows on choco-late agar. What is the most common clinicalpresentation for primary symptomatic gono-coccal disease in the adolescent female?

(A) hematuria(B) cervicitis(C) fever and shaking chills(D) arthritis(E) painless chancre

25. In reviewing for your microbiology final exam-ination, you learn that staphylococcal food poi-soning is caused by a heat-stable, preformedenterotoxin. What are the clinical characteristicsassociated with this disease process?

(A) high fever(B) onset of symptoms within minutes of

ingestion of the toxin(C) concurrent staphylococcal bacteremia(D) vomiting and abdominal cramps, fol-

lowed by diarrhea(E) frequently accompanied by a rash

Page 151: Questions - uhs.edu.kh You suspect Lyme disease, most likely contracted by which of the following? (A) ingestion of unripe fruit (B) ingestion of spoiled fruit (C) drinking of contaminated

152 7: Infectious Diseases

26. You are seeing a school-age child in your office forfever and sore throat. You diagnose streptococcalpharyngitis based on a positive rapid strepto-coccal antigen test. The mother is concerned thather 1-year-old child may also be infected. Youinform her that group Astreptococcal infection inthis age group is likely to present as which of thefollowing?

(A) meningitis(B) scarlet fever(C) fever and peritonsillar abscess(D) acute rheumatic fever(E) fever and serous rhinitis

27. A 12-year-old presents to the emergencydepartment in August with fever 102.2°F (39°C)and intense headache. A lumbar puncture (LP)is performed, and your suspicion of asepticmeningitis is confirmed. Which of the follow-ing CSF findings is most likely 48 hours into thecourse of enteroviral meningitis?

(A) 5,000 WBC, 90% polymorphonuclearleukocytes

(B) 100 WBC, 90% eosinophils(C) 150 WBC, 80% lymphocytes(D) 50 WBC, 90% polymorphonuclear

leukocytes(E) 50 WBC, 70% monocytes

28. It is the middle of the winter and you haveseen many children of different ages in yourclinic with upper respiratory symptoms. Whichof the following children is most likely to havegroup A streptococcal infection?

(A) exudative pharyngitis in a 1-year-old(B) tonsillitis, rash, and fever in a 5-year-old(C) cough and pharyngitis in a 15-year-old(D) “slapped cheek” appearance in a

5-year-old(E) fever, congestion, cough, and pharyngi-

tis in a 3-year-old

29. A 4 -year-old child presents to your office withfever, and increased work of breathing mani-fested by tachypnea and retractions. Achest x-rayconfirms lobar pneumonia. What is the mostlikely etiology of pneumonia in this child?

(A) Mycoplasma pneumoniae(B) Streptococcus pyogenes(C) Chlamydophila pneumoniae(D) Streptococcus pneumoniae(E) Staphylococcus epidermidis

30. A 2-year-old presents to the emergency depart-ment with temperature of 104°F (40°C) andgeneralized erythema that is exquisitely tender.On close examination you notice large superfi-cial bullae. You suspect a toxin-mediated infec-tion. Which of the following toxins is the causeof these manifestations?

(A) exfoliatoxin A and B(B) toxic shock syndrome toxin-1(C) Shiga toxin(D) lipopolysaccharide(E) enterotoxin A

31. A 3-year-old child is admitted to the hospital inFebruary with temperature of 101.3°C (38.5°C),vomiting, and diarrhea. The mother describesthe stools as watery, green, and very foul smelling.The most common organism causing this pres-entation is

(A) adenovirus(B) rotavirus(C) Salmonella enteriditis(D) Shigella sonnei(E) enterovirus

32. A 12-year-old male who just returned from BoyScout camp comes to your office after 2 days offever, vomiting, headache, and myalgias. Younote a petechial rash predominately on hisextremities including his palms and soles (seeFigure 7-1). You suspect Rocky Mountain spottedfever. Which of the following laboratory find-ings would be consistent with that diagnosis?

Page 152: Questions - uhs.edu.kh You suspect Lyme disease, most likely contracted by which of the following? (A) ingestion of unripe fruit (B) ingestion of spoiled fruit (C) drinking of contaminated

Questions: 26–37 153

(A) leukocytosis, anemia, and thrombocytosis(B) leukopenia, hypernatremia, and throm-

bocytopenia(C) leukopenia, hyponatremia, and throm-

bocytosis(D) leukopenia, hyponatremia, and throm-

bocytopenia(E) leukocytosis, erythrocytosis, and throm-

bocytosis

33. There has been a recent increase in cases of per-tussis in your community. When counseling afamily who declines immunization, you shouldinform them of the common manifestations ofpertussis. Which of the following is a typicalpresentation of pertussis infection?

(A) 4 or 5 days of high fever followed bycroupy cough

(B) sudden onset of fever and cough(C) gradual onset of cough, followed by

abrupt onset of fever and whooping(D) mild upper respiratory symptoms fol-

lowed by paroxysmal cough (E) protracted fever and paroxysmal cough

34. The mother of the family in the previous ques-tion develops paroxysmal cough a few weekslater. Her infant is subsequently admitted tothe hospital with apneic episodes. Which ofthe following white blood cell counts is mostsuggestive of pertussis?

(A) 3,000/mm3 with 75% lymphocytes(B) 20,000/mm3 with 65% lymphocytes(C) 7,000/mm3 with 65% polymorphonu-

clear leukocytes(D) 25,000/mm3 with 65% polymorphonu-

clear leukocytes(E) 12,000/mm3 with 20% eosinophils

35. A 12-year-old child developed fever about 1week after visiting relatives in India. The feverhas persisted for about 10 days. Diarrhea waspresent for a few days, and then cleared. Thechild is now constipated. The child appearsmoderately acutely ill. The liver and spleen areenlarged. There are palpable, small (2–4 mm)erythematous spots on the trunk only. What isthe most likely cause of this child’s infection?

(A) measles(B) typhoid fever(C) Neisseria meningitidis bacteremia(D) rat-bite fever(E) leptospirosis

36. You are working in a clinic in rural Mexico andexamine an 8-year-old boy who has a rectaltemperature of 100°F (38°C), bilateral tenderparotid swelling, and pain when you flex hisneck. He has been complaining of a headache.His immunization history is unknown. What isthe most likely cause of this child’s infection?

(A) brucellosis(B) cysticercosis(C) Epstein-Barr virus infection(D) mumps(E) leukemia

37. A previously well 3-year-old child has a tem-perature of 104°F (40°C), headache, and stiffneck. He has a 2-day history of upper respira-tory tract symptoms and is now vomiting. Thisclinical presentation most likley representswhich of the following diagnoses?

(A) brain tumor(B) optic neuritis(C) subarachnoid hemorrhage(D) meningitis(E) bacterial sepsis

Figure 7-1(Courtesy of Emily A. Thorell, MD)

Page 153: Questions - uhs.edu.kh You suspect Lyme disease, most likely contracted by which of the following? (A) ingestion of unripe fruit (B) ingestion of spoiled fruit (C) drinking of contaminated

154 7: Infectious Diseases

38. The child in the previous question has a CSFevaluation which revealed 4000 WBCs whichare 95% polymorphonuclear leukocytes. Culturegrows S pneumoniae and he is treated with ceftri-axone to which the organism is susceptible. He isnoted to have subdural effusions on CT scan ofhis head. Which of the following is true concern-ing subdural effusions with bacterial meningitis?

(A) They are the result of inadequatetreatment.

(B) They are indicative of a bleedingdisorder.

(C) Finding them is a poor prognosticindicator.

(D) They are caused by incidental traumarather than the infection itself.

(E) They are a common occurrence.

39. A 16-year-old adolescent presents with tem-perature of 103.1°F (39.5°C) and purpuric skinlesions. The child is found to have a blood pres-sure of 60/30 and heart rate of 180 with bound-ing pulses and a respiratory rate of 40. Whatinfectious agent is the most likely to cause theabove symptoms?

(A) Neisseria meningitidis(B) Haemophilus influenzae(C) Streptococcus pneumoniae(D) Staphylococcus aureus(E) Group B beta-hemolytic streptococcus

40. Of the choices below, what is the first priorityin management of the child in question #39?

(A) intravenous immune globulin infusion(B) CSF examination(C) intravenous fluid resuscitation(D) taking an exposure history(E) complete blood count and blood culture

41. A 3-year-old child presents with fever for 8 days,lymphadenopathy, splenomegaly, and numer-ous reactive or atypical lymphocytes on periph-eral blood smear. The monospot test is negative.A likely cause of this clinical picture is infectionwith which of the following?

(A) adenovirus(B) respiratory syncytial virus(C) herpesvirus(D) Epstein-Barr virus(E) rubella virus

42. Which of the following drug regimens is themost appropriate chemoprophylaxis for adulthousehold contacts of a child with meningo-coccal meningitis?

(A) single-dose ciprofloxacin(B) penicillin for 2 days(C) ceftriaxone IM(D) trimethoprim-sulfamethoxazole for

7 days(E) penicillin and rifampin for 2 days

43. You are counseling a mother who is 37 weeksinto her second pregnancy. Group B strepto-coccal screening was negative with her priorpregnancy 2 years ago, but she was positiveon testing this time. She understands that shewill receive antibiotics during labor, but is con-cerned about possible risk of infection in hernewborn. Which of the following is true regard-ing early-onset group B streptococcal infectionin the neonate?

(A) It is more common in infants born atless than 37 weeks gestation.

(B) It usually presents with meningitis.(C) It is associated with good prognosis if

treated promptly.(D) It generally presents with osteomyelitis.(E) It is frequently associated with recurrent

disease.

44. A 24-year-old primigravid woman presents tolabor and delivery with fever, malaise, abdominalpain, and back pain. She is found to be in laborand subsequently delivers a full-term infant withtachypnea, grunting, and tachycardia. Blood cul-ture evaluation of the infant reveals Listeria mono-cytogenes. Which maternal exposure is considereda risk factor for development of listeriosis?

Page 154: Questions - uhs.edu.kh You suspect Lyme disease, most likely contracted by which of the following? (A) ingestion of unripe fruit (B) ingestion of spoiled fruit (C) drinking of contaminated

Questions: 38–51 155

(A) ingestion of washed raw vegetables(B) exposure to cat feces(C) ingestion of unpasteurized dairy or

prepared meat(D) ingestion of shellfish(E) exposure to bird droppings

45. A 17-month-old nonimmunized child has hadfever for 4 days and now has a maculopapularrash. She is seen in clinic and diagnosed ashaving measles. There is a 4-month-old siblingat home. What is the appropriate managementfor the sibling?

(A) immediate immunization with liveattenuated measles vaccine

(B) immediate immunization with killedmeasles vaccine

(C) a single dose of immune globulin (IG)(D) IG plus live attenuated measles vaccine(E) no treatment necessary

46. A young child with fever, cough,hepatosplenomegaly, and eosinophilia has3 negative examinations of the stool for ova andparasites. What is the most likely parasite tocause this combination of symptoms?

(A) Ascaris lumbricoides(B) Toxocara canis(C) Dracunculus medinensis(D) Enterobius vermicularis(E) Trichuris trichiura

47. You are working in a hospital in Africa on aninternational elective. A 2-year-old child pres-ents with vomiting and a distended abdomen.Abdominal radiographs reveal acute obstruction ofthe small intestine with air fluid levels. Eosinophiliais found on complete blood count. Of the fol-lowing parasitic infections, which is most likely topresent with intestinal obstruction?

(A) Enterobius vermicularis(B) Necator americanus(C) Ascaris lumbricoides(D) Strongyloides stercoralis(E) Trichuris trichiura

48. A 3-year-old child presents with 5 days of fever;conjunctivitis; red, cracked lips; polymorphousrash; and an isolated 2 cm cervical lymph node.You diagnose Kawasaki syndrome based onclinical findings. Which of the following seque-lae of Kawasaki syndrome is most common?

(A) fulminant hepatitis(B) coronary artery aneurysm(C) recurrent pericarditis(D) cerebral edema(E) renal failure

49. A 15-month-old child presents to your officewith a high fever, and an intense, red rash onthe cheeks with circumoral pallor. What is themost likely etiology of this febrile exanthem?

(A) enterovirus 71(B) adenovirus(C) parvovirus B19(D) rubeola virus(E) coxsackievirus A16

50. A 2-week-old infant presents to the pediatricintensive care unit in shock with vesicular rash,pneumonitis, hepatitis, and coagulopathy. Theprenatal and birth history were uneventful andthe mother was healthy. What is the most likelyetiology of this illness?

(A) group B streptococcus(B) herpes simplex virus(C) varicella virus(D) Treponema pallidum(E) human immunodeficiency virus

51. Which of the following tests should be includedin the initial workup of the infant in the previousquestion?

(A) long bone x-ray(B) liver biopsy(C) MRI head(D) CSF evaluation(E) abdominal ultrasound

Page 155: Questions - uhs.edu.kh You suspect Lyme disease, most likely contracted by which of the following? (A) ingestion of unripe fruit (B) ingestion of spoiled fruit (C) drinking of contaminated

156 7: Infectious Diseases

52. You are working in the local health departmentand receive a phone call regarding a hepatitis Aoutbreak in a day care center. The teachers askyou if they will be able to identify infected chil-dren. What percentages of infected childrenless than 6-years-old are symptomatic?

(A) <1%(B) 15%(C) 30%(D) 50% (E) 75%

53. A family has recently moved to your area fromChina. The mother tells you that her hepatitis Bscreen was positive on arrival to the UnitedStates. She has a 2-month-old child and asksyou what signs of infection would be in thechild. What is the most common presentationof congenital hepatitis B?

(A) jaundice(B) arthritis(C) asymptomatic infection(D) papular acrodermatitis (Gianotti-Crosti

syndrome)(E) vomiting and diarrhea

54. In regard to the mother of the child in the previousquestion, the presence of which of the followingserologic markers for hepatitis B represents anincreased risk of transmitting infection?

(A) HBsAg(B) HBeAg(C) IgM anti-HBc(D) antibody to HBc(E) antibody to HBs

55. A 10-month-old child has a temperature of104°F (40°C) for 4 days without other signs.On the fourth day a rose pink, maculopapularrash appears and the temperature returns tonormal. What is the most likely diagnosis?

(A) echovirus(B) human herpes virus 6(C) measles virus

(D) group A streptococcus(E) typhus

56. An 18-month-old child presents to your officewith a 2-day history of fever. He is not eatingwell and the mother tells you that she thinkshis mouth hurts. On examination you see 3 mmvesicles on erythematous bases on the softpalate and tonsils. What is the most likely eti-ology of this infection?

(A) streptococcal pharyngitis(B) herpangina(C) herpes simplex virus(D) human herpes virus 6(E) candida pharyngitis

57. The child in the above question also has smallvesicular lesions on his palms and soles. Yoususpect hand, foot, and mouth syndrome.What virus is the most likely causative agent ofthis disease?

(A) adenovirus(B) group A streptococcus(C) Arcanobacterium haemolyticum(D) coxsackievirus A16(E) herpes simplex virus

58. You are taking care of a 26-month-old childwith bacterial meningitis. Blood and spinalfluid cultures are positive for N meningitidis.The child’s household consists of the parents, onegrandparent, a 6-month-old sibling, a 5-year-oldsibling, and a 14-year-old exchange studentwho has been living with the family for about1 month. Which members of the householdshould receive chemoprophylaxis?

(A) everyone in the household(B) everyone except the grandparent(C) everyone except the grandparent and

the visiting student(D) the siblings only(E) the 6-month-old sibling only

Page 156: Questions - uhs.edu.kh You suspect Lyme disease, most likely contracted by which of the following? (A) ingestion of unripe fruit (B) ingestion of spoiled fruit (C) drinking of contaminated

Questions: 52–65 157

59. A 4-year-old child presents with temperature of101.3°C (38.5°C) and a sore throat. On examina-tion, you notice exudative pharyngitis and bilat-eral, anterior cervical lymphadenopathy. Rapidstreptococcal antigen testing is negative. What isthe most likely etiology of this presentation?

(A) Kawasaki syndrome(B) Behçet syndrome(C) adenovirus(D) parainfluenza virus(E) respiratory syncytial virus

60. A previously well 12-year-old presents withfever, sore throat, and general malaise. Onphysical examination, you notice generalizedlymphadenopathy, exudative pharyngitis, andsplenomegaly. Laboratory evaluation revealselevated serum transaminase levels. What isthe organism most likely responsible for thesefindings?

(A) Treponema pallidum(B) hepatitis B virus(C) rubella virus(D) Epstein-Barr virus(E) toxoplasmosis

61. A 15-year-old presents with pain, photopho-bia, and blurred vision in one eye. Examinationreveals chemosis of the affected eye and smallvesicular lesions below the eye and on the nose.Application of fluorescein dye reveals branch-ing, dendritic lesions on the cornea. What isthe most likely cause of this clinical picture?

(A) cytomegalovirus(B) adenoviral infection(C) rubella virus(D) herpes simplex infection(E) trauma

62. A 4-month-old infant born with perinatallyacquired HIV infection presents with fever,grunting, and hypoxia. Influenza and RSV test-ing are negative. Which of the following is thelikely opportunistic infection in this infant withperinatally acquired HIV infection?

(A) Pneumocystis pneumonia(B) disseminated cytomegalovirus infection(C) disseminated toxoplasmosis(D) cryptococcal meningitis(E) giardiasis

63. A newborn infant has microcephaly, periven-tricular calcifications, jaundice, and thrombo-cytopenia. You suspect congenital infection.Which of the following is most likely?

(A) Epstein-Barr virus(B) cytomegalovirus(C) coxsackievirus B(D) human immunodeficiency virus(E) human parvovirus B-19

64. A 12-year-old who went camping 2 weeks ago inOklahoma now presents with fever and headache.Laboratory studies demonstrate leukopenia,thrombocytopenia, elevated liver enzymes, andhyponatremia. Of the following, what is themost likely causative agent?

(A) Rickettsia prowazekii(B) Rickettsia typhi(C) Ehrlichia chaffeensis(D) Coxiella burnetii(E) Borrelia burgdorferi

65. A 12-year-old child presents to the pediatricintensive care unit in shock with evidence ofdisseminated intravascular coagulation. Bloodculture reveals lancet-shaped, gram-positivecocci in pairs. You suspect S pneumoniae infec-tion. Which of the following underlying dis-eases places them at increase risk for invasivepneumococcal infection in terms of incidenceand severity?

(A) underlying liver disease(B) sickle cell disease(C) valvular cardiac disease(D) cystic fibrosis(E) end stage renal disease

Page 157: Questions - uhs.edu.kh You suspect Lyme disease, most likely contracted by which of the following? (A) ingestion of unripe fruit (B) ingestion of spoiled fruit (C) drinking of contaminated

158 7: Infectious Diseases

66. A 7-year-old unimmunized child presents withfever and vesicular rash. You notice about 300lesions, some of which were crusted. You sus-pect varicella infection. Which of the followingis a TRUE statement about varicella?

(A) It has an incubation period of 5–7 days.(B) The rash is confluent, centrifugal, and

pustular.(C) It is associated with Koplik spots.(D) There is a high risk of shingles.(E) It can cause visceral dissemination in

the immunocompromised host.

67. You are counseling a primigravid woman whohas been found to be rubella nonimmune onprenatal laboratory evaluation. She asks you ifher fetus is at risk for malformations. When ismaternal infection with rubella virus mostcommonly associated with congenital defects?

(A) in the first 4 weeks of gestation(B) in the second month(C) in the third or fourth month(D) in the last trimester(E) anytime during pregnancy

68. A 15-year-old has exudative tonsillitis, cervicaladenitis, and splenomegaly. His monospot testis positive. Which of the following is the mostcommon complication encountered?

(A) chronic fatigue lasting greater than6 months

(B) hemorrhage(C) pneumonia(D) encephalitis(E) airway obstruction

69. An 8-year-old male presents to the emergencydepartment with an erythematous, swollenhand. He sustained a dog bite to the hand theday prior which broke the skin and was treatedwith irrigation. Purulent material drains fromthe bite site with palpation. What is the mostlikely organism causing this infection?

(A) Pasteurella multocida(B) Eikenella corrodens

(C) Streptobacillus moniliformis(D) group A streptococcus(E) Staphylococcus aureus

70. A 15-year-old presents with fever, acute pharyn-gitis, and rash. Rapid streptococcal antigen test-ing is negative. Three days later the laboratoryinforms you that A haemolyticum is growing onculture. Which of the following is the mostaccurate description of the rash associated withthis infection?

(A) It has a rough, sandpaper-like texture.(B) It has a tendency to involve the palms

and soles.(C) It has a tendency to become vesicular

within 24 hours.(D) It is pustular.(E) It has a predilection for the face.

71. A 3-year-old child travels with his family toMexico, where he dines on the local cuisine. Onemonth after return, he develops fever, anorexia,myalgias, and abdominal pain. Serologic testingis positive for Brucella infection. Which of thefollowing is a common manifestation of bru-cellosis in children?

(A) hepatosplenomegaly(B) glaucoma(C) meningitis(D) endocarditis(E) osteomyelitis

72. A 3-year-old child in day care develops abdom-inal pain with diarrhea and malaise. His classrecently had a field trip to the local petting zoo.Stool culture subsequently grows Campylobacterjejuni. Which of the following is the most commonclinical manifestation that follows infection withCampylobacter?

(A) Guillain-Barré syndrome(B) polyarticular arthritis(C) encephalitis(D) inguinal lymphadenitis(E) anterior uveitis

Page 158: Questions - uhs.edu.kh You suspect Lyme disease, most likely contracted by which of the following? (A) ingestion of unripe fruit (B) ingestion of spoiled fruit (C) drinking of contaminated

Questions: 66–78 159

73. You see a 5-year-old child in your office with apalpable lymph node in her right axilla that isprogressing in size over the last 3 days. It hasbecome erythematous and warm. On exami-nation, the node is indurated. You also notemany scratches on the child’s arms. On furtherquestioning, you discover that the family hasrecently adopted a new kitten. What is the mostlikely etiology of the child’s adenopathy?

(A) Coxiella burnetii(B) Streptobacillus moniliformis(C) Bartonella henselae(D) Mycobacterium avium complex(E) Mycobacterium tuberculosis

74. A 3-month-old infant is admitted to the hospitalwith respiratory distress. He has had cough andcongestion for the last 2 days and is now breath-ing too fast to eat effectively. You suspect bron-chiolitis and respiratory syncytial virus (RSV)antigen testing is positive. Which of the followingroentgenographic findings is most commonlyseen in infants with bronchiolitis caused by RSV?

(A) hyperinflation(B) hilar adenopathy (C) multilobar infiltrate(D) lower lobe infiltrates(E) pleural effusion

75. An 8-year-old was hiking with his Boy Scouttroop a few days ago. He now presents to youroffice with 2 days of abdominal pain. On furtherhistory, you realize he drank from the streamon his hike. You suspect Giardia intestinalis infec-tion. What additional symptom would you seemost commonly in a child with giardiasis?

(A) fever(B) watery diarrhea(C) bloody diarrhea(D) failure to thrive(E) vomiting

76. After confirmation of giardiasis by antigen test-ing on the stool, you initiate therapy on thechild in the previous question. What is the mostappropriate therapy for giardiasis?

(A) trimethoprim-sulfamethoxazole(B) ketoconazole(C) mebendazole(D) nitrofurantoin(E) metronidazole

77. A 3-year-old child from Somalia presents to theemergency department with a 2-week historyof low-grade fever, headache, and irritability.This morning he was difficult to arouse andsubsequently had a seizure. On examination,he has a temperature of 102.2°F (39°C), nuchalrigidity, and somnolence. You discover that heand his parents had positive tuberculin skintests on arrival to the United States and suspecttuberculous meningitis. Which of the followingis characteristic of the cerebrospinal fluid intuberculous meningitis?

(A) The color is usually blood tinged.(B) Protein is normal.(C) Culture reveals tuberculous organisms

within 1 week.(D) Glucose is low.(E) Neutrophils predominate.

78. A3-month-old African American infant presentsto the emergency department on Thanksgivingweekend with fever and hematochezia. Fromthe history, you discover that the family pre-pared chitterlings for the Thanksgiving meal.Fecal leukocytes are present. Stool cultures arepositive. What is the most likely etiology ofthis infant’s infection?

(A) Salmonella enteritidis(B) Salmonella typhi(C) Escherichia coli(D) Shigella flexneri(E) Yersinia enterocolitica

Page 159: Questions - uhs.edu.kh You suspect Lyme disease, most likely contracted by which of the following? (A) ingestion of unripe fruit (B) ingestion of spoiled fruit (C) drinking of contaminated

160 7: Infectious Diseases

79. A 16-year-old sexually active male presents toyour office with a painful ulcer on his penisand fluctuant, tender lymph nodes in his leftinguinal area. You aspirate the most fluctuantnode and Gram stained smear is negative. Whatis the most likely etiology of this presentation?

(A) lymphogranuloma venereum(B) herpes simplex virus(C) syphilis(D) chancroid(E) granuloma inguinale

80. The mother of a newborn infant brings her 2-week-old child in for his first checkup in November.The mother has heard reports on the local newsof influenza in the community and that younginfants are at very high risk if they contract thedisease. She asks you if her child can be immu-nized. At what age can a child first receiveinfluenza vaccine?

(A) birth(B) 2 months(C) 4 months(D) 6 months(E) 12 months

81. A 13-year-old male is taking inhaled steroidsfor his moderate intermittent asthma. Onexamination, you find white plaques on hisbuccal mucosa. You suspect thrush caused byCandida albicans. Which of the following wouldbe most appropriate for treatment?

(A) itraconazole(B) amphotericin B(C) metronidazole(D) nystatin(E) mebendazole

82. A 3-year-old female with perinatally acquiredhuman immunodeficiency virus has history ofmedical nonadherence to therapy. Her mostrecent CD4 count was less than 100. She presentsto your clinic with fatigue, abdominal cramping,vomiting, and diarrhea. She has not been eatingand has had significant weight loss since her lastvisit. Upon further questioning, you realize that

she has had diarrhea for over a month. Stoolexamination reveals 4–6 µm spherical cysts. Whatis the most likely etiology of this presentation?

(A) Giardia lamblia(B) Entamoeba histolytica(C) Cryptosporidium parvum(D) Isospora belli(E) Cryptococcus neoformans

83. A9-month-old presents to your office with 2-dayhistory of temperature of 101.6°F (38.7°C) andfussiness. Mother reports that the child hasbeen pulling at her ear. On physical examina-tion, you note an erythematous, bulging, righttympanic membrane with purulent effusion.You diagnose otitis media. What organism isthe most likely cause of this infection?

(A) Streptococcus pyogenes(B) Streptococcus pneumoniae(C) Haemophilus influenzae type B(D) Staphylococcus aureus(E) Pseudomonas aeruginosa

84. A14-year-old presents to your office with 2 weekhistory of cough and purulent rhinorrhea. Henow has 2-day history of fever and frontalheadache. On examination he is tender to pal-pation over his maxillary sinuses and his mothertells you that he appears swollen over that area.You make the clinical diagnosis of acute sinusi-tis. He has no allergies to medication. What isthe first-line therapy for this child?

(A) ceftriaxone(B) cephalexin(C) clindamycin(D) amoxicillin(E) azithromycin

85. A 15-year-old male just returned from a hunt-ing trip with his father. He has an abrupt onsetof fever, chills, myalgia, and headache. Hedevelops painful anterior cervical chain lym-phadenopathy. On examination, you see anulcer under his arm. His father tells you thatthey removed a tick from that spot a few days

Page 160: Questions - uhs.edu.kh You suspect Lyme disease, most likely contracted by which of the following? (A) ingestion of unripe fruit (B) ingestion of spoiled fruit (C) drinking of contaminated

Questions: 79–91 161

prior. What is the most likely diagnosis for thispatient?

(A) tularemia(B) Lyme disease(C) Rocky Mountain spotted fever(D) ehrlichiosis(E) Q fever

86. A 7-year-old child was grabbing at a neighbordog’s tail and was bitten on the arm. He pres-ents to the emergency department for woundmanagement. The dog’s behavior is normaland it is in custody of animal control, but theowner’s do not have record of the last rabiesvaccine. The health department has not hadreports of rabies in the county where the childwas bitten. Besides local wound care, what isthe most appropriate intervention regardingpossible rabies exposure?

(A) Euthanize the dog and send brain tissuefor rabies antigen testing.

(B) Give the child the rabies vaccine seriesas well as rabies immune globulin andobserve the dog for 10 days for signs ofrabies.

(C) Give the child rabies vaccine alone andobserve the dog for 10 days for signs ofrabies.

(D) Give the child rabies immune globulinalone and observe the dog for 10 daysfor signs of rabies.

(E) Observe the dog for 10 days for signs ofrabies.

87. A 5-year-old unimmunized child fell whileplaying in an old barn and sustained a lacera-tion to his leg. After local wound care, whatwould be the most appropriate managementregarding tetanus prophylaxis?

(A) give Td only(B) give DTaP only(C) give Td and tetanus immune globulin(D) give DTaP and tetanus immune globulin(E) give tetanus immune globulin only

88. A family is visiting their summer home in NewEngland. Their 8-year-old child presents toyour office with a rash over his back. The rashstarted as a small, erythematous macular areaand is expanding to form an annular, erythe-matous lesion with a distinct border that ismore than 10 cm in diameter. What is thecausative agent of this child’s infection?

(A) Rickettsia rickettsiae(B) Borrelia burgdorferi(C) Anaplasma phagocytophilia(D) Rickettsia typhi(E) Rickettsia prowazekii

89. What is the term used to describe the exan-them in the patient in the above question?

(A) erythema nodosum(B) erythema marginatum(C) erythema migrans(D) erythema toxicum(E) erythema multiforme

90. A 2-year-old African American child presents toyour office with areas of alopecia. On examina-tion you note a boggy, inflammatory mass withsome purulent drainage over the scalp and occip-ital lymphadenopathy. Cultures are obtained viaskin scraping and grow Trichophyton tonsurans.What is the best initial treatment for tinea capitis?

(A) topical nystatin(B) oral griseofulvin(C) topical miconazole(D) oral itraconazole(E) ketoconazole shampoo

91. You are attending in the newborn nursery andare informed that a baby was born to a motherwith human immunodeficiency virus (HIV).What is the best test to diagnose HIV infectionin this infant?

(A) Enzyme immunoassay (ELISA)(B) RNA PCR(C) Western blot(D) DNA PCR(E) HIV p24 antigen

Page 161: Questions - uhs.edu.kh You suspect Lyme disease, most likely contracted by which of the following? (A) ingestion of unripe fruit (B) ingestion of spoiled fruit (C) drinking of contaminated

162 7: Infectious Diseases

92. A 5-month-old child is admitted with severehypoxia and respiratory failure to the pediatricintensive care unit. A diagnosis of P jirovecipneumonia (PCP) is made on silver staining ofbronchoalveolar lavage fluid. You suspectimmunodeficiency in this patient, and HIVtesting is negative. What is the most likelyimmunodeficiency?

(A) Bruton agammaglobulinemia(B) adenosine deaminase deficiency(C) common variable immunodeficiency(D) Job syndrome(E) asplenia

93. A9-year-old child is hospitalized with her secondepisode of meningococcemia. Immunodeficiencyis suspected. What is the most likely immunedeficiency which would predispose this child torecurrent N meningitidis infection?

(A) X-linked agammaglobulinemia(B) Chediak-Higashi syndrome(C) human immunodeficiency virus(D) Wiskott-Aldrich syndrome(E) terminal complement deficiency

94. You are seeing a 6-month-old for well-child visitand immunizations. Anticipatory guidance isgiven. What immunizations would you pre-scribe if the child were up to date at that point?

(A) hepatitis B, DTaP, HIB, IPV, PCV(B) hepatitis A, hepatitis B, DTaP, HIB, PCV(C) DTaP, HIB, IPV, PCV, MMR(D) hepatitis A, DTaP, HIB, IPV, PCV(E) DTaP, HIB, MMR, varicella, PCV

95. A 6-year-old child has recently emigrated fromsub-Saharan Africa. For the last 2 weeks, he hashad fever every other day along with nausea,vomiting, arthralgia, and abdominal pain. Onfurther questioning, the mother tells you thatthe child has rigors with fever and sweats whenthe fever breaks. You suspect malaria. What is thebest diagnostic method for this illness?

(A) aerobic and anaerobic blood culture(B) serologic testing(C) thick and thin blood smear preparations

(D) PCR on serum(E) liver biopsy

96. You diagnose group A streptococcal pharyngi-tis in an 8-year-old child and treat appropri-ately. The mother of the child asks how longshe must stay home from school. When can thechild return to school?

(A) as soon as therapy is initiated(B) 24 hours after the initiation of therapy(C) 48 hours after the initiation of therapy(D) 72 hours after the initiation of therapy(E) the full length of therapy

97. An 18-year-old male comes home from collegefor winter break and comes to your office com-plaining of malaise, knee pain, and rash. Onexamination, you find generalized, polymor-phic, maculopapular rash including the palmsand soles, generalized lymphadenopathy, andsplenomegaly. When questioned about hissexual history, he tells you he has had multiplepartners. Of the following, what is the mostlikely etiology of his illness?

(A) primary syphilis(B) secondary syphilis(C) chancroid(D) lymphogranuloma venereum(E) gonococcal infection

98. In regard to the patient in the above question,what is the most appropriate antimicrobialtherapy for his treatment?

(A) penicillin(B) ceftriaxone(C) cephalexin(D) azithromycin(E) ciprofloxacin

99. A 3-year-old child is in your office for the secondtime with otitis media that is not responding toantibiotics. He has been on standard-dose amox-icillin for the last 10 days and his tympanicmembrane is still bulging with purulent effusion.You perform a tympanocentesis and recoverS pneumoniae. The parents ask you why the initial

Page 162: Questions - uhs.edu.kh You suspect Lyme disease, most likely contracted by which of the following? (A) ingestion of unripe fruit (B) ingestion of spoiled fruit (C) drinking of contaminated

Questions: 92–101 163

antibiotic did not work. What is the most likelycause of pneumococcal resistance to amoxicillin?

(A) mutation in mef (membrane effluxpump) gene

(B) mutation in erm (erythromycin ribosomemethylation) gene

(C) beta-lactamase(D) point mutation in DNA gyrase(E) penicillin-binding protein mutation

100. You are seeing a 5-year-old child with recurrentfurunculosis secondary to methicillin-resistantS aureus (MRSA). Susceptibility testing con-firms the isolate is clindamycin susceptible andthat D-zone testing is negative. You elect totreat with clindamycin. What is the mechanismof action of clindamycin?

(A) inhibitor of cell wall synthesis(B) inhibitor of protein synthesis(C) inhibitor of bacterial DNA synthesis

(D) inhibitor of bacterial enzyme(E) interferes with cell membrane perme-

ability

101. An 18-month-old child presents to the emer-gency department with fever. The parentsinform you that they think the child has beenlimping on the right leg for the last week. Onexamination, the only focal finding is that theright knee seems swollen and a little warm.Arthrocentesis is performed and synovial fluidshows 75,000 WBC with 90% neutrophils. Gramstain is negative. After 5 days, the culture growsa gram-negative coccobacillus. What is the mostlikely etiology of the septic arthritis in this child?

(A) Staphylococcus aureus(B) Streptococcus pyogenes(C) Kingella kingae(D) Salmonella enteritidis(E) Fusobacterium necrophorum

Page 163: Questions - uhs.edu.kh You suspect Lyme disease, most likely contracted by which of the following? (A) ingestion of unripe fruit (B) ingestion of spoiled fruit (C) drinking of contaminated

164

1. (B) Manifestations of staphylococcal toxic shocksyndrome, which include fever, mental statuschanges, conjunctivitis, diffuse macular erythro-derma, and multiple organ failure, are caused bya toxin elaborated by the staphylococci ratherthan by tissue invasion of the organism. Thename of the toxin is toxic shock syndrome toxin-1(TSST-1), and is produced by S aureus. The organ-ism usually can be cultured from skin or mucousmembrane and only rarely from the blood. It hasbeen recovered from the vagina and has beenassociated with the use of tampons, especiallythose designed to be changed infrequently.Streptococcal toxic shock syndrome is cause by Spyogenes (group A streptococcus). The incidenceis highest among young children, particularlythose with concomitant varicella. The organismcan be isolated from blood about 50% of the time.Both N gonorrhoeae and S agalactiae can be foundin the genital tract (the former as a pathogen andthe latter as normal flora); neither is associatedwith toxin-mediated disease. Shiga toxin pro-duced by E coli causes diarrhea and can be asso-ciated with hemolytic uremic syndrome. (Long,110–112; American Academy of Pediatrics, 660–661)

2. (D) The case definition of staphylococcal toxicshock syndrome includes five to six of the fol-lowing findings: fever; diffuse rash; desqua-mation 1–2 weeks after onset; hypotension; ifobtained, negative results for blood (exceptS aureus approximately 5% of the time), throat,and CSF cultures, and negative serologic test-ing for Rocky Mountain spotted fever, measles,or leptospirosis; and multisystem organ failure.Organ involvement must include three or moreof the following: gastrointestinal symptoms,

muscular symptoms such as myalgia, mucousmembrane involvement such as conjunctivalhyperemia, vaginal or oropharyngeal involve-ment, hepatic dysfunction, renal dysfunction,platelet count less than 100,000/µL, or centralnervous system dysfunction such as alteredmental status. (American Academy of Pediatrics, 662)

3. (C) Histoplasma capsulatum is the most commonprimary systemic mycosis in the United States,and most often occurs in the Ohio and Mississippiriver valleys. The organism grows in moist soiland is facilitated by bird droppings. As inadults, the respiratory tract is the portal of entryfor histoplasmosis in essentially all cases in chil-dren. Inoculation other than by inhalation isexceedingly rare, and person to person transmis-sion does not occur. (Long, 11198; American Academyof Pediatrics, 371–372)

4. (B) Passively transferred maternal IgG decreasescommensurate with the half-life, which is approx-imately 30 days. Therefore the nadir occurs ininfants at the age of 3–6 months. (Long, 92)

5. (D) Human milk provides optimal nutritionfor the growing infant. It contains lactose andother carbohydrates that are substrates for pro-tective microflora, such as Bifidobacterium andLactobacillus spp. The major immunoglobulinpresent in breast milk is secretory IgA, and isavailable to act at the mucosal surface of thesmall intestine. Immunity is conferred againstmany specific enteropathogens and toxins.It also has been shown to inhibit bindingof H influenzae and S pneumoniae to pharyngealcells. (Nelson, 158)

Answers and Explanations

Page 164: Questions - uhs.edu.kh You suspect Lyme disease, most likely contracted by which of the following? (A) ingestion of unripe fruit (B) ingestion of spoiled fruit (C) drinking of contaminated

Answers: 1–12 165

6. (A) Enterobius vermicularis infection (Pinworms)occurs from ingestion of embryonated eggswhich hatch intraluminally and mature as theydescend the GI tract. An adult female depositseggs on the perianal skin at night causing pru-ritus ani. It is considered the most commonhelminthiasis in the United States and otherdeveloped countries. Most infections occur inchildren with documented transmission in childcare centers and elementary schools. The mostreliable diagnosis is made via transparent tapepreparation. Malabsorption disorders and inges-tion of rice do not cause pruritus. Strongyloidesstercoralis is a round worm and T solium is thepork tape worm. Both may cause abdominal pain,vomiting, and diarrhea, but typically not pruritusani. (Long, 1301–1302; American Academy of Pediatrics,520–522, 629–630, 644–645)

7. (E) Infants congenitally infected with rubellaexcrete the virus in urine and pharyngeal secre-tions and can be infectious for many weeks ormonths. Such infants pose a definite hazard tononimmune family members, caretakers, as wellas medical and nursing personnel. Pregnantfemales who are not certain of their rubella immu-nity status should not handle these infants or theirsecretions. Infants with perinatally acquired HIVare generally asymptomatic at birth. An infantwith congenital CMV and toxoplasmosis mayhave a similar presentation to congenital rubella,although cataracts and congenital heart diseaseare not typical manifestations. In addition, patientswith toxoplasmosis may have scattered cerebralcalcifications, while patients with CMV may haveperiventricular calcifications. (American Academy ofPediatrics, 574–579, 666, 273–274)

8. (E) Of the so-called common childhood diseaseslisted in this question, primary varicella infec-tion (or chickenpox) is most likely to presentwith a generalized, pruritic vesicular rash andmild fever. The typical exanthem appears firston the scalp, face, or trunk. New crops of lesionsdevelop over a 1–7 day period. Progressionfrom vesicle to pustule to crusted scab occursquickly such that lesions of all stages are pres-ent after the first 48 hours. (American Academy ofPediatrics, 711; Long, 1021–1022).

9. (C) Ordinarily, the first dose of live attenuatedmeasles vaccine (combined with mumps andrubella as the MMR vaccine) should be admin-istered between 12 and 15 months of age tomaximize the likelihood that transplacentallyacquired antibodies have disappeared completelyfrom the child’s blood. Otherwise, these antibod-ies may blunt the infant’s immunologic responseto the vaccine. On the other hand, administrationof the vaccine should not be delayed muchbeyond 15 months because of the seriousness ofmeasles infection. It is recommended that asecond dose of the vaccine be given when thechild enters elementary school. In areas where,or at times when, measles is epidemic, it is rec-ommended that an initial dose of monovalentmeasles vaccine be given as early as 6 months ofage, followed by the two MMRs as noted.(American Academy of Pediatrics, 447, 448; Long, 1126)

10. (A) The use of a combined diphtheria-tetanus-acellular pertussis (DTaP) vaccine is routineand is the method of choice for the primaryimmunization of infants and young children.There are two contraindications to pertussisimmunization: an immediate anaphylactic reac-tion or encephalopathy within 7 days of a priordose manifested by major alterations in con-sciousness or protracted generalized or focalseizures without recovery within 24 hours.(American Academy of Pediatrics, 40, 510–513)

11. (A) Mumps virus, enterovirus, coxsackievirus,and influenza virus are spread primarily by directcontact (eg, respiratory droplets, hands) and notthrough an arthropod vector. Anumber of viruses,notably the arboviruses, are spread by mosquitovectors, as well as ticks and sandflies. Examples inNorth America include eastern and westernequine encephalitis, St. Louis encephalitis, andLaCrosse and West Nile encephalitis. (Long, 1084)

12. (E) Fever, cough, hilar lymphadenopathy, andelevated sedimentation rate are seen commonlyin uncomplicated primary pulmonary tubercu-losis. Hepatosplenomegaly is generally not seenin uncomplicated primary pulmonary tubercu-losis, but does occur in more than 50% of childrenwith disseminated disease. (Long, 776)

Page 165: Questions - uhs.edu.kh You suspect Lyme disease, most likely contracted by which of the following? (A) ingestion of unripe fruit (B) ingestion of spoiled fruit (C) drinking of contaminated

166 7: Infectious Diseases

13. (C) Mycoplasma pneumoniae has long been knownto cause pneumonia, bronchitis, otitis media,myringitis bullosa, and nonspecific upper respi-ratory infection. More recently, this organism alsohas been recognized as a cause of various non-respiratory manifestations such as polymorphousmucocutaneous eruptions including Stevens-Johnson syndrome, encephalitis, and meningitis.Other neurologic manifestations reported withM pneumoniae infection include transverse myelitis,psychosis, poliomyelitis-like syndrome, andGuillain-Barré syndrome. Streptococcus pneumo-niae and S aureus typically cause lobar pneumonia,oftentimes with empyema. While chest radi-ograph findings in M pneumoniae infection ofteninclude patchy alveolar infiltrates with smallpleural effusion, the accompanying chest radi-ograph which shows a white out of one lungwould be most likely associated with empyema.While C psittaci may have the same clinical pres-entation as mycoplasma, there will typically bea history of bird exposure. Pneumocystis jiroveci isconsidered a pathogen in the immunocompro-mised host (see Figure 7-2). (Long, 979–981;American Academy of Pediatrics, 251, 468)

14. (A) Infants with bronchiolitis can have varyingpresentations, from mild tachypnea to severecough, retractions, tachypnea, and wheezing.

RSV is the most common cause, and infection isuniversal in the first 2 years of life. Parainfluenza,influenza, and human metapneumovirus areall recognized causes of bronchiolitis. However,they are not as common as RSV. CMV is not acause of bronchiolitis. (Long, 1113; American Academyof Pediatrics, 560–561)

15. (C) In healthy children over age 4 years, reac-tions of less than 5-mm induration to 5 TU areconsidered negative. Reactions between 5 and15 mm are considered doubtful and usually rep-resent infection with nontuberculous mycobac-teria. Reactions of 15 mm or more indurationgenerally are considered positive and indicativeof infection with M tuberculosis but do not nec-essarily mean clinically evident disease. Thepoint to remember is that the threshold forinterpreting a tuberculin skin test as positive islower for children with signs or symptoms sug-gestive of tuberculosis or with a history of con-tact with tuberculosis. (American Academy ofPediatrics, 660; Long, 772)

16. (A) In general, children with uncomplicated pri-mary pulmonary tuberculosis are noninfectiousand do not require isolation. This is believed torelate to the scanty sputum production, lack ofexpectoration, and the small number of organ-isms that can be recovered from sputum or gas-tric culture. In contrast, children with laryngealinvolvement, with cavitary pulmonary tuber-culosis, extensive pulmonary infection, positivesputum AFB smears, or suspected congenitaltuberculosis should be considered contagiousand appropriate precautions should be taken.Children with tuberculosis disease can attendschool or child care if they are receiving therapy.(American Academy of Pediatrics, 696)

17. (B) In enteric Salmonella infections, fever ischaracteristic; diarrhea may or may not be present.Some features highly suggestive of Salmonellainfection include leukopenia and a relative brady-cardia; that is, the heart rate is slower thanwould be anticipated for the degree of feverpresent, though this is seen more commonly inadults than in children. Clinical manifestationssuch as fever, abdominal pain, hepatomegaly,splenomegaly, rose spots, and changes in

Figure 7-2

Page 166: Questions - uhs.edu.kh You suspect Lyme disease, most likely contracted by which of the following? (A) ingestion of unripe fruit (B) ingestion of spoiled fruit (C) drinking of contaminated

Answers: 13–24 167

mental status are reported. (American Academy ofPediatrics, 579–580; Long, 815)

18. (E) The defect in chronic granulomatous dis-ease (CGD) is the inability of polymorphonu-clear and mononuclear leukocytes to effectintracellular killing of catalase positive phago-cytosed bacteria. The neutrophils can ingestbacteria normally, but cannot produce the intra-cellular hydrogen peroxide needed to kill theorganisms. Common manifestations includerecurrent skin and sinopulmonary infectionsas well as candidal infections. Hepatic abscessesare a common cause of fever and abdominalpain in these patients. CGD is inherited pri-marily as a sex-linked recessive trait, althoughsome females with the disorder have beenreported. Leukocyte adhesion deficiency rep-resents an impairment of leukocyte migration.Children with Ehlers-Danlos syndrome havea defect in collagen synthesis. Increased capil-lary permeability and impaired wound healinghave a variety of causes, including DIC andsepsis, as well as hepatic and renal dysfunction.(Long, 618–619, 621; Berhman, 715–716)

19. (E) The clinical feature of scabies infestationthat is typical for the infant is an intensely pru-ritic rash with involvement of the entire skin sur-face including the scalp, palms, and soles withlesions that are often pustulovesicular. In contrast,older children and adults are generally infestedbetween fingers and toes. The eruption is causedby a hypersensitivity reaction to the proteins of thescabies mite, Sarcoptes scabiei. Diagnosis is made bymicroscopic evaluation of skin scraping. (AmericanAcademy of Pediatrics, 584–585)

20. (B) Formerly known as enterohemorrhagicE coli, Shiga-toxin-producing strains of E coli(STEC) are associated with a range of gas-trointestinal symptoms from self-limited non-bloody diarrhea to hemorrhagic colitis. A triadof microangiopathic hemolytic anemia, throm-bocytopenia, and acute renal dysfunction istermed hemolytic-uremic syndrome and mayfollow the diarrheal illness within a week in2%–20% of infected children. E coli is one ofthe most common causes of meningitis in theneonate. Chronic diarrhea, pneumonia, and

endocarditis are unlikely manifestations ofdiarrheal illness related to E coli. (Long, 796–798;American Academy of Pediatrics, 291–293)

21. (E) In general, T pallidum can be demonstratedin any mucous membrane or cutaneous lesionof congenital syphilis, especially a moist one.All of these lesions are capable of spreadingthe organism and are highly infectious. Theclassical cutaneous manifestations of congeni-tal syphilis are a copper-colored maculopapu-lar rash and vesiculobullous lesions of thepalms and soles, often referred to as pemphi-gus syphiliticus. (Long, 932)

22. (A) Infants less than 2 months of age who pres-ent with undifferentiated fever are at high riskfor serious bacterial infection. The standard ofcare for such infants is a meticulous historyand physical examination with laboratory eval-uation to detect urinary tract, bloodstream, orCNS infection. The most common serious bac-terial infection encountered in this young infantpopulation is urinary tract infection with 90%of infections caused by E coli. While seen spo-radically, H influenzae type B meningitis haslargely been eradicated due to widespread useof HIB vaccine. Salmonella enteritis, group Astreptococcal bacteremia, and meningococ-cemia are unlikely in an infant. (Long, 124, 533)

23. (E) Vaginitis is the most common form of gono-coccal infection in the prepubertal female. Theunestrogenized, alkaline vaginal mucosa of theprepubertal girl is especially vulnerable to col-onization and infection with N gonorrhoeae. Onestudy confirmed the diagnosis of gonorrhea in9% of prepubertal children presenting with achief complaint of vaginal discharge; sexualabuse must be considered in such patients.Ascending infection (salpingitis, peritonitis)occurs, but only rarely in prepubertal children.Arthritis is uncommon, and conjunctivitis isessentially restricted to the newborn period.(Long, 361; American Academy of Pediatrics, 302)

24. (B) Gonococcal infection in the female adoles-cent is often asymptomatic; clinical infectionusually manifests as cervicitis, urethritis, andsalpingitis. The incubation period is 2–10 days.

Page 167: Questions - uhs.edu.kh You suspect Lyme disease, most likely contracted by which of the following? (A) ingestion of unripe fruit (B) ingestion of spoiled fruit (C) drinking of contaminated

168 7: Infectious Diseases

Complications can include bartholinitis, PID,and perihepatitis (Fitz-Hugh-Curtis syndrome).Hematogenous spread can involve skin andjoints (arthritis-dermatitis syndrome) and canoccur in approximately 3% of untreated people.This is most common when infected within1 week of menstruation. Hematuria can occur,but is not the most common manifestation.Fever and shaking chills would be associatedwith PID, a secondary complication. Chancresare not associated with gonococcal infections.(American Academy of Pediatrics, 302; Long, 367)

25. (D) Staphylococcal food poisoning is causedby a toxin elaborated in spoiled food before inges-tion. Bacteremia does not occur, since this is notan infection. Fever is uncommon. Symptoms—vomiting, abdominal cramps, and diarrhea—usually begin within 2–4 hours of ingestion ofthe toxin and are generally limited to the gas-trointestinal system. Rash, high fever, and bac-teremia are features of staphylococcal toxicshock syndrome. (American Academy of Pediatrics,597; Long, 111)

26. (E) Group A streptococcal infection in youngchildren is commonly manifested by persistentfever and mucoserous nasal discharge. This syn-drome has been referred to as “streptococcosis.”Localized pharyngeal involvement is uncom-mon in the first year of life. Meningitis causedby this organism is very uncommon at all agesand is generally associated with foreign body.The clinical picture of scarlet fever is rarely seenin the first 3 years of life and acute rheumaticfever is rare before 4 or 5 years of age. (Long, 172)

27. (C) In temperate climates, summer and late falloutbreaks of enteroviral infection are commonwith many thousands of cases of meningitisreported each year, the vast majority caused byenteroviruses. Fever, headache, and photopho-bia are commonly reported in children withenteroviral meningitis. CSF pleocytosis is usu-ally noted with white blood cell counts typicallybetween 100–1000. While neutrophil (PMN) pre-dominance can be seen early on, lymphocytesbecome predominant between 8 and 48 hours ofonset. Definitive diagnosis is made by culture;

although PCR detection is faster, more sensi-tive, and 100% specific. (Long, 306)

28. (B) Pharyngotonsillitis is the typical clinicalmanifestation of group A streptococcal infec-tion. Scarlet fever is a syndrome of tonsillitis,fever, and rash caused by an erythrogenic toxin-producing streptococcus in a patient lackingantitoxin immunity. Streptococcal respiratorytract infections generally peak in children aged5–11 years and winter predominance is gener-ally noted. Patients younger than 3 years of agewith exudative pharyngitis are more likely tohave viral disease, as are older children whopresent with sore throat, cough, and/or rhin-orrhea. Slapped cheek appearance is character-istic of parvovirus infection. (American Academy ofPediatrics, 610; Long, 442)

29. (D) Pneumococcal pneumonia is the mostcommon acute bacterial pneumonia of infants(excluding the neonate) and children. It usuallyoccurs in association with, or as a complication of,a viral upper respiratory illness. The onset of clin-ical manifestations is usually rapid and abrupt,and although fever is a prominent feature, respi-ratory signs and symptoms are common and usu-ally are present early. Staphylococcus epidermidis isa known cause of bacteremia in the immunocom-promised host or those with indwelling centralcatheter. Mycoplasma pneumoniae and Chlamydophilpneumoniae typically cause patchy infiltrates seenon chest x-ray in the older age child. Streptococcuspyogenes is a less likely cause of pneumonia, butcan cause invasive infection with bacteremia,multiple organ involvement, and pneumonia.(Long, 247, 249, 693, 876, 983)

30. (A) Staphylococcal scalded skin syndrome ismediated by exfoliative toxins A and B. Infantsare most commonly affected and clinicallypresent with tender, generalized erythrodermafollowed by generalized appearance of flaccidbullae, which rupture. Lipopolysaccharide isan endotoxin in the cell wall of gram-negativebacteria. Enterotoxin A and toxic shock syn-drome toxin-1 are produced by S aureus, theformer is associated with food poisoning and thelatter is associated with toxic shock syndrome.Shiga toxin may be produced by shigella and

Page 168: Questions - uhs.edu.kh You suspect Lyme disease, most likely contracted by which of the following? (A) ingestion of unripe fruit (B) ingestion of spoiled fruit (C) drinking of contaminated

Answers: 25–37 169

various strains of E coli. It is associated withdiarrheal illness and HUS. (Long, 102, 688–689;American Academy of Pediatrics,:291, 597–599)

31. (B) Rotaviruses are recognized as the majorcause of severe gastroenteritis in children. Mostcases in the United States occur during the wintermonths. The clinical manifestations of rotavirusinfection include nonbloody diarrhea often pre-ceded by fever and vomiting. Approximately10% of infected infants are hospitalized to correctdehydration. Adenovirus and enterovirus arenot typical causes of diarrheal infection in thewinter months. While salmonella and shigellacause diarrheal illness, there is no predilection forwinter months. Shigella outbreaks have beendocumented in day care settings. (Long, 1079–1080;American Academy of Pediatrics, 202–203, 590, 284–285)

32. (D) Rocky Mountain spotted fever (Rickettsiarickettsii) infection causes a vasculitis. Electrolyteand hematologic abnormalities seen with thisdisease include: hyponatremia, thrombocytope-nia, leucopenia, and anemia. (American Academy ofPediatrics, 570; Long, 916–917)

33. (D) Pertussis classically begins with a catarrhalstage indistinguishable from a common coldand lasts up to a week. This is followed by grad-ual worsening of the cough, finally reachingthe paroxysmal stage, characterized by thick,tenacious secretions and fits of forceful cough-ing that often end in a whoop as the child isfinally able to take a full breath. Fever is usuallymild or absent. (American Academy of Pediatrics,498; Long, 861–862)

34. (B) Pertussis usually is associated with a markedabsolute lymphocytosis. Peripheral white bloodcell counts in the range of 20,000–30,000/mm3

with a majority of lymphocytes are characteris-tic of this infection. This is a useful and impor-tant diagnostic feature. (Long, 863)

35. (B) Typhoid fever (infection with S typhi) ischaracterized by an incubation period of 6–21days, followed by fever, malaise, and, in somecases, rose spots. These are small, palpable ery-thematous lesions on the trunk. (Petechiae aresmaller and not palpable, see Figure 7-3.)

Diarrhea, if present, usually clears and often isfollowed by constipation. Measles is charac-terized by cough, conjunctivitis, coryza, andan erythematous maculopapular rash. Neisseriameningitidis infection causes meningitis and/ormeningococcemia, the latter of which is anacute febrile illness manifested by malaise,prostration, and rash that can evolve into septicshock with DIC and death. Rat bite fever andleptospirosis do not have diarrhea as a part oftheir constellation of symptoms. (Long, 815;American Academy of Pediatrics, 424, 441, 452, 559,579–580)

36. (D) Mumps causes subclinical infection inone-third of cases. The most common mani-festation is unilateral parotid swelling thatbecomes bilateral with a prodrome of headache,fever, abdominal pain, and anorexia. EBVinfection typically manifests as exudativepharyngitis and cervical chain adenopathy.Brucellosis is a systemic illness that typicallycauses fever, joint pain, malaise, and anorexia.Leukemia may vary in presentation, but oftenpresents with fever, hematologic abnormality,petechial rash, and nonspecific complaints.The initial sign of cysticercosis in children isoftentimes a seizure in an otherwise healthychild. (Long, 856, 1038, 1109–1110; Behrman,1694–1695)

37. (D) In this patient, the acute onset in associa-tion with fever, headache, and stiff neck favorthe diagnosis of meningitis. Brain tumor isunlikely to cause fever, and typically either

Figure 7-3(Courtesy of Mary Anne Jackson, MD)

Page 169: Questions - uhs.edu.kh You suspect Lyme disease, most likely contracted by which of the following? (A) ingestion of unripe fruit (B) ingestion of spoiled fruit (C) drinking of contaminated

170 7: Infectious Diseases

presents with signs/symptoms of increased ICPor focal neurologic signs. Subarachnoid hemor-rhage does not have fever or respiratory symp-toms as part of the clinical findings. Bacterialsepsis, without meningitis, should not cause stiffneck. Optic neuritis is characterized by inflam-mation, degeneration, or demyelination of theoptic nerve. Meningitis is confirmed by exami-nation of the cerebrospinal fluid and the accom-panying Figure 7-4 shows the typical findings ofbacteria seen on Gram-stained smear. (Long,285–286, 307–308; Behrman, 849, 1703, 2041, 2121)

38. (E) Subdural fluid collections (effusions)develop in 10%–30% of patients, and are espe-cially common in infants. The exact pathogen-esis is unknown, but they are usually sterileand not related either to inadequate therapyor to a bleeding disorder. They are asympto-matic in 85%–90% of patients, and are rarelyfatal. The symptomatology that is associatedwith effusions is headache, vomiting, seizures,and fever. However, these are also commonsymptoms of meningitis. (Behrman, 2043–2044,Snedeker, 86:163–170)

39. (A) Although the combination of fever, hem-orrhagic eruption, and shock can be seen withbacteremic infection caused by a wide variety oforganisms including H influenzae, S pneumoniae,S aureus, and beta-hemolytic streptococcus, thiscombination of findings is most commonly seen

with meningococcal (N meningitidis) infection.The hemorrhagic skin lesions (purpura fulmi-nans) suggest disseminated intravascular coag-ulation, an ominous prognostic sign. (Long,738–739)

40. (C) While all of the answers are important inclinical evaluation and treatment of the childwith meningococcemia, starting with the ABCsof basic life support still applies. As airway man-agement choices are not available, addressingthe circulatory status with intravenous fluidswould be the most appropriate initial step.Antibiotic administration should occur rapidlyand preferably after blood cultures have beendrawn. Initial CSF evaluation including cultureis helpful, but is not always feasible dependingon the degree of illness of the child. It shouldbe completed prior to antibiotics when possi-ble; however it can still be useful after antibi-otic administration to evaluate for pleocytosis.(Long, 740.)

41. (D) The monospot tests for heterophile anti-bodies, which are rarely produced by childrenyounger than 4 years. A negative result doesnot exclude EBV infection. Patients present witha typical clinical picture of lymphadenopathy,splenomegaly, and atypical (reactive) lympho-cytes on peripheral blood smear. Respiratorysyncytial virus, herpes infection, and rubellainfection are unlikely to cause atypical lym-phocytosis or splenomegaly. Although bothHSV and noncongenitally acquired rubella maypresent with lymphadenopathy, adenovirustypically causes an upper respiratory tract infec-tion and pharyngoconjunctival fever. (AmericanAcademy of Pediatrics, 286–487, 361–362, 560–561, 574)

42. (A) Household contacts of those with meningo-coccal disease are at 500–1000 times greater riskfor acquisition of meningococcal infection thanthe general population. Although several drugregimens for prophylaxis of contacts of meningo-coccal disease are acceptable, the primaryregimen recommended at this time is rifampin,10 mg/kg every 12 hours (q12h) for a total offour doses for children and a single dose ofciprofloxacin for those older than 18 years.(American Academy of Pediatrics, 455–456; Long, 741–742)

Figure 7-4(Courtesy of Mary Anne Jackson, MD)

Page 170: Questions - uhs.edu.kh You suspect Lyme disease, most likely contracted by which of the following? (A) ingestion of unripe fruit (B) ingestion of spoiled fruit (C) drinking of contaminated

Answers: 38–49 171

43. (A) Group B streptococcus is the leading bac-terial cause of neonatal infection in most med-ical centers in the United States. Two clinicalsyndromes, early onset and late onset, havebeen described, although some features dooverlap both groups. The early-onset picture ismost frequent in high-risk infants (eg, prema-ture, prolonged rupture of membranes) andusually presents as a severe, rapidly progres-sive illness in the first day or even hours of life.Pneumonia and bacteremia are the most commonmanifestations, but meningitis occurs in aboutone-third of cases. Manifestations of late-onsetinfection are often indolent and include bac-teremia and meningitis, as well as other focalinfections such as osteomyelitis, septic arthritis,omphalitis, and breast abscess. (American Academyof Pediatrics, 620–621; Long, 712–713)

44. (C) Listeria monocytogenes meningitis is uncom-mon in childhood except for the newbornperiod, where in some series it accounts for upto 10% of cases of bacterial meningitis. A fewpediatric cases occur in immunodeficient chil-dren, and a rare case in the otherwise normalchild. Risk factors for the development of lis-teriosis in a newborn infant is maternal ingestionof unpasteurized dairy products or preparedmeats (hotdogs, deli meat) and ingestion ofunwashed raw vegetables. Exposure to cat fecesincreases the risk of toxoplasmosis, which mayalso be transmitted transplacentally. Exposureto bird droppings is associated with histoplas-mosis and C psittaci pneumonia. Ingestion ofshellfish is not associated with listeriosis orneonatal infection.(American Academy of Pediatrics,251, 426, 666, 728; Long, 763–764)

45. (C) Active immunization with live measles vac-cine at the time of contact is too late to assureprotection. Additionally, children should notreceive the vaccine before the age of 6 monthsto assure an adequate immunologic response.The recommended management of susceptiblesiblings of a contact case is the use of a pre-ventive dose of gamma globulin (0.25 mg/kg)and later immunization with the live-virus vac-cine. In the past, when most women had naturalmeasles infection, maternal IgG levels of measlesantibody were very high, and transplacental IgG

afforded the young infant relatively strong pro-tection. Cases of measles infection in this age-group were often milder than in older children.Such is not the case now. Many mothers havelow levels of measles antibody (from immu-nization rather than natural infection), andtheir infants are susceptible to severe measlesinfection. (American Academy of Pediatrics, 443–444)

46. (B) The dog ascarid, T canis, cannot complete itslife cycle in human beings, therefore eggs orworms are not discharged in the stool. The clin-ical picture of toxocariasis (often referred to asvisceral larva migrans) is characterized byfever, hepatomegaly, and eosinophilia. Less fre-quently, there may be involvement of the lung,central nervous system, heart, or retina. Noneof the other parasites listed characteristicallycauses hepatomegaly. (American Academy of Pediatrics,665; Long, 1304–1306)

47. (C) Ascaris lumbricoides is the largest intestinalroundworm, and occasionally intestinal obstruc-tion may result from heavy infection. The inci-dence of this complication has been estimated at2 per 1,000 infected children per year. Intestinalobstruction has not been observed with any ofthe other parasites listed. (American Academy ofPediatrics, 218; Long, 1297-1298)

48. (B) Kawasaki syndrome is a multisystem vas-culitis which typically manifests with high,spiking fever for 5 or more days along withconjunctival infection, mucositis, polymorphousrash, changes in peripheral extremities, andsingle cervical lymph node swelling. Treatmentwith aspirin and intravenous immune globulinis indicated; without treatment, 20% of childrendevelop coronary artery aneurysms. Fulminanthepatitis, recurrent pericarditis, cerebral edema,and renal failure are not features of Kawasakisyndrome. (American Academy of Pediatrics, 412–413;Long, 989–991)

49. (C) The clinical manifestations of human par-vovirus B-19 include erythema infectiosum(healthy child), polyarthropathy syndrome(adults especially women), chronic anemia/purered cell aplasia (immunocompromised hosts),transient aplastic crisis (sickle cell patients), and

Page 171: Questions - uhs.edu.kh You suspect Lyme disease, most likely contracted by which of the following? (A) ingestion of unripe fruit (B) ingestion of spoiled fruit (C) drinking of contaminated

172 7: Infectious Diseases

hydrops fetalis/congenital anemia (fetus).Erythema infectiosum is most commonly diag-nosed and easily recognized. A distinctive rashfeaturing a “slapped cheek” appearance isnoted that is often associated with circumoralpallor. Coxsackievirus A16 and enterovirus 71are causes of hand-foot-and-mouth syndrome.Enterovirus 71 is also associated with encephali-tis. Adenovirus is not associated with a char-acteristic exanthem, however it may causeconjunctivitis. Rubeola virus is the causativeagent of measles, and generally presents witha triad of cough, coryza, and conjunctivitis, aswell as Koplik spots and rash. (American Academyof Pediatrics, 202, 284, 484–485)

50. (B) Neonatal herpes simplex virus (HSV) infec-tion is typically transmitted in infants whenmothers experience primary infection at thetime of delivery and typically manifest in thefirst 1–4 weeks of life. There are three forms ofdisease, though clinical overlap is often noted.Disease may be localized to the skin, eyes, andmouth, it may be disseminated with promi-nent involvement of lungs and liver or local-ized central nervous system involvement mayoccur. Perinatally acquired HIV infection is typ-ically asymptomatic initially. Congenital syphilismay present with vesicular rash, but is notassociated with sepsis. Group B streptococcalinfection does not present with rash. Congenitalvaricella occurs when maternal infection pres-ents within 5 days before or 2 days after deliv-ery. (Long, 1016–1017; American Academy of Pediatrics,361, 637, 711)

51. (D) A full sepsis evaluation should be per-formed, as well as viral cultures of CSF, lesions,eye, and mucous membranes (NP and rectal).HSV DNA can be detected by polymerasechain reaction assay in the CSF of patients withcentral nervous system disease and is the diag-nostic test of choice for such patients. (AmericanAcademy of Pediatrics, 363–364)

52. (C) Hepatitis A virus infection is generally anacute self-limited illness. Symptoms includefever, malaise, jaundice, anorexia, and nausea.Only 30% of infected children fewer than 6 years

of age are symptomatic. Few of these childrendevelop jaundice. (American Academy of Pediatrics,326–327)

53. (C) Symptomatology with hepatitis B infectionis age dependent, and infants are typicallyasymptomatic. Between 5% and 15% of childrenbetween 1 and 5 years of age develop typicalsymptoms such as anorexia, nausea, malaise,and jaundice, while 30%–50% of adolescentsand adults will develop acute symptoms. (Long,1066–1067, American Academy of Pediatrics, 335–336)

54. (B) Chronic infection with hepatitis B virus (HBV)occurs in over 90% of perinatally infectedinfants, 25%–50% of those infected are betweenages 1 and 5 years and between 6% and 10% ofolder children and adults. Those patients withdetectable HBeAg are more likely to transmitinfection as they have higher blood concentra-tions of HBV DNA. (American Academy of Pediatrics,336)

55. (B) High fever without other signs and clearingof the fever on appearance of a rash are char-acteristic of roseola and is the defining clinicalexpression of primary infection with humanherpes virus 6. In measles infection, fever con-tinues for several days after the appearance ofthe rash. The same is true for the several typesof typhus. In infection to echovirus, the rashand fever usually appear together. The rash ofscarlet fever often appears at the time of tem-perature elevation. (American Academy of Pediatrics,375–376)

56. (B) Herpangina is a syndrome characterizedby small vesicles or punched-out ulcers on thetonsils and fauces, uvula, pharynx, and edge ofsoft palate. The remainder of the mouth andthroat usually appear normal on examination.Infections caused by group A streptococcus andcandida do not present with vesicular lesions.The vesicular lesions seen in HSV infection aregenerally limited to the anterior portion of themouth in immunocompetent individuals.HHV-6 is the causative agent of roseola, whichdoes not generally cause an enanthem. (Long,201, 1151)

Page 172: Questions - uhs.edu.kh You suspect Lyme disease, most likely contracted by which of the following? (A) ingestion of unripe fruit (B) ingestion of spoiled fruit (C) drinking of contaminated

Answers: 50–64 173

57. (D) Hand-foot-and-mouth syndrome is a spe-cific syndrome that can be caused by a variety ofviral agents. It was originally described in asso-ciation with coxsackievirus A16, but enterovirus71 can cause an identical clinical picture. Group Astreptococcus and Arcanobacterium haemolyticumcause similar manifestations of exudative pharyn-gitis. However the latter is a much less commoncause of pharyngitis. Adenovirus does not typ-ically cause enanthem. While HSV infection cancause oral as well as digital lesions, this is anatypical presentation with occurrence on boththe palms and soles. (American Academy of Pediatrics,284; Long, 1152)

58. (A) Meningococcal meningitis is one of the mostcommon causes of bacterial meningitis in chil-dren along with S pneumoniae. Chemoprophylaxisof all contacts who live in the household is routineas the risk of secondary transmission is 100–1000times that of the general population. (AmericanAcademy of Pediatrics, 455)

59. (C) Most cases of pharyngitis are caused byviruses, though the finding of an exudativepharyngitis often leads the practitioner to con-sider the diagnosis of group A streptococcalinfection. However, there are a number ofpathogens which may produce exudativechanges in children with pharyngeal infection,including adenovirus, Epstein-Barr virus,Corynebacterium diphtheriae, and N gonorrhoeae.Kawasaki syndrome, Behçet syndrome, parain-fluenza, and respiratory syncytial virus do notcause exudative pharyngitis. (American Academyof Pediatrics, 202)

60. (D) The clinical and laboratory findingsdescribed are most suggestive of infectiousmononucleosis from Epstein-Barr virus. Hepaticinvolvement evidenced by elevated serumtransaminase levels is common. Syphilis, HBV,rubella, and toxoplasmosis do not present withthis constellation of symptoms. (American Academyof Pediatrics, 286)

61. (D) The clinical picture described is typical ofherpes simplex keratoconjunctivitis. The asso-ciated vesicular lesions on the skin and the den-dritic corneal ulcers are almost pathognomonic

of this infection. It is important to diagnoseherpes keratitis correctly because infection canbe recurrent and lead to loss of vision, andbecause therapy is available. The other choicesdo not cause dendritic ulcers. (Long, 1014–1015)

62. (A) Infants with perinatally acquired HIV infectionoften present with nonspecific symptoms and signsincluding lymphadenopathy, hepatosplenomegaly,failure to thrive, oral candidiasis, and recurrentdiarrhea. Among the diseases listed, Pneumocystisjiroveci pneumonia (PCP) is the most com-monly encountered opportunistic infection ininfants with perinatally acquired infection.(American Academy of Pediatrics, 378)

63. (B) Infants with congenitally acquiredcytomegalovirus infection are generally asymp-tomatic though 10% present with a syndrome thatincludes intrauterine growth retardation, thrombo-cytopenia, jaundice, hepatosplenomegaly, micro-cephaly, intracerebral calcifications, and retinitis.EBV infection is not associated with congenitalinfection. Perinatal coxsackievirus infectionmay manifest with myocarditis and encephali-tis, human parvovirus B-19 as congenital anemiaor hydrops fetalis, and perinatally HIV-infectedinfants are generally asymptomatic at birth.(American Academy of Pediatrics, 274, 284, 385, 484)

64. (C) Human monocytotrophic ehrlichiosis infec-tion is an acute systemic febrile illness seenmost commonly in the south central, south-eastern United States with infection caused byEhrlichia chaffeensis and transmitted via the biteof the Lone star tick (Amblyomma americanum).Anaplasma phagocytophilum causes human gran-ulocytotrophic anaplasmosis (formerly ehrli-chiosis) with a similar clinical picture, butpredominantly in the north central and north-eastern United States and transmitted by thedeer tick (Ixodes scapularis). The clinical pictureis similar to Rocky Mountain spotted fever(though more often without rash) is reportedwith prominent CNS and gastrointestinalsymptoms; hyponatremia, leukopenia, anemia,and elevated liver transaminases are commonlaboratory manifestations. Doxycycline is thedrug of choice and should be used even inyoung patients as severe and fatal disease has

Page 173: Questions - uhs.edu.kh You suspect Lyme disease, most likely contracted by which of the following? (A) ingestion of unripe fruit (B) ingestion of spoiled fruit (C) drinking of contaminated

174 7: Infectious Diseases

been noted. Rickettsia typhi and R prowazekiicause endemic and epidemic typhus respec-tively. Coxiella burnetii is the causative agent ofQ fever which generally does not cause theselaboratory findings. The organism B burgdorfericauses Lyme disease which has the character-istic finding of erythema migrans. (AmericanAcademy of Pediatrics, 281–282, 428, 550, 567, 570;Long, 888–889)

65. (B) In the asplenic patient, there is an increasedrisk for fulminant life-threatening infectioncaused by S pneumoniae. Infection occurs in thosewith posttraumatic asplenia, HIV infection, con-genital asplenia, polysplenia syndromes, orthose who undergo splenectomy in the course oftreatment for malignancy as well as patientswith sickle cell disease. (Long, 635–636; AmericanAcademy of Pediatrics, 526–527)

66. (E) The rash of varicella follows an incubationperiod of 10–21 days. There is the onset of avery pruritic rash, with crops of lesions thatbegin as papules and progress to vesicles andfinally crusted scabs. Typically, all three stagesof skin lesions are identified on clinical exami-nation. Fever usually is mild to moderate. Whilea generally self-limited and benign course isnoted, severe disease may occur occasionallyin the otherwise healthy host, especially ado-lescents and adults. A progressive and severedisease with visceral dissemination is seen in30%–50% of children with lymphoproliferativemalignancies, solid tumors, or posttransplan-tation with the development of hepatitis,encephalitis, and pneumonia. Fatal disease hasalso been reported in those treated with high-dose corticosteroids and in those with otherdefects of T-cell function. (Long, 1022–1024)

67. (A) Congenital malformations, stillborns, andabortions all have been reported with rubellainfection during pregnancy. The congenitalrubella syndrome consists of ophthalmologic,cardiac, auditory, and neurologic abnormali-ties with rates as high as 85% if infection occursin the first 4 weeks of gestation, decreasing to20%–30% during the second month, and 5%during the third or fourth month. These infantsmay continue to excrete rubella virus for 1 year

or more after birth and pose a risk of infectionfor susceptible hosts. (American Academy ofPediatrics, 574)

68. (E) Infectious mononucleosis syndrome gen-erally presents with exudative pharyngitis thatmay be associated with petechiae or erythe-matous macules on the palate. A rash occursoccasionally and splenomegaly may be noted.Airway obstruction secondary to markedlyenlarged tonsils is an infrequent but importantcomplication. (Long, 1040)

69. (A) The most common cause of Pasteurella infec-tion in children is a dog or cat bite. The organ-ism is present in the oropharyngeal flora of70%–95% of cats, and 25%–50% of dogs. Theinfection typically occurs within 24 hours, andcan include regional lymphadenopathy, chills,and fever, as well as swelling, erythema, andpurulent discharge at the site. Eikenella corrodensinfections generally occur after human bites.Streptobacillus moniliformis is the causative agentof rat bite fever. Infections with Group A strep-tococcus or Staphylococcus aureus are possible, butmuch less common than Pasteurella in patients withanimal bites. (American Academy of Pediatrics, 487)

70. (A) Pharyngitis caused by A haemolyticum infec-tion is hard to differentiate from that of strep-tococcal infection. The rash associated withArcanobacterium is scarlatiniform, similar tothat of group A streptococcus, and infectionshould be suspected in the adolescent whopresents with pharyngeal exudates and rashafter streptococcal infection has been excluded.(American Academy of Pediatrics, 217)

71. (A) Brucellosis is a zoonotic disease in whichchildren are an accidental host, most commonlycontracting infection by ingesting unpasteurizedmilk. Onset of illness can be acute or insidious.Symptoms of disease are usually nonspecific andinclude fever, malaise, anorexia, joint pains,headache, and night sweats. Physical findingsinclude lymphadenopathy, hepatosplenomegaly,and occasionally arthritis. Meningitis, endo-carditis, and osteomyelitis are rare complicationsof disease. Glaucoma is not reported. (AmericanAcademy of Pediatrics, 235, Long, 856–857)

Page 174: Questions - uhs.edu.kh You suspect Lyme disease, most likely contracted by which of the following? (A) ingestion of unripe fruit (B) ingestion of spoiled fruit (C) drinking of contaminated

Answers: 65–78 175

72. (A) Campylobacter infection is characterizedby watery diarrhea possibly followed by blood-streaked stools, fever, abdominal pain, andmalaise. It is usually self-limited, but may haveimmunoreactive complications (Guillain-Barrésyndrome, reactive arthritis [monoarticular],Reiter syndrome, and erythema nodosum)occur during convalescence. It is estimated thatin 30%–40% of cases of Guillain-Barré syndromeoccurred within 2 weeks of Campylobacter infec-tion. Anterior uveitis, inguinal lymphadenitis,and encephalitis are not sequelae of Campylobacterinfection. (American Academy of Pediatrics, 240–241,Long, 871)

73. (C) Cat-scratch disease (CSD) is caused byB henselae. The most common manifestation isunilateral regional lymphadenopathy. Smallerythematous papules can occur at the inocu-lation site and precede the lymphadenopathyby 1–2 weeks. The typical patient lacks consti-tutional symptoms, but 30% may have mildfever, malaise, headache, and anorexia. SystemicCSD is rare. Infections due to nontuberculousmycobacterium, such as M avium, are typicallysubacute and nontender. Streptobacillus monili-formis is the causative agent of rat bite fever.Coxiella burnetii is the causative agent of Q fever.Mycobacterium tuberculosis infection is less likelyin a child with a history of kitten exposure, whois not from a TB-endemic area. (American Academyof Pediatrics, 246, Long, 851–854)

74. (A) Respiratory syncytial virus (RSV) is the mostimportant cause of bronchiolitis in infants andyoung children and causes acute respiratory tractillness in patients of all ages. Typical roentgeno-graphic findings include hyperinflation, peri-bronchial thickening, and atelectasis especiallyinvolving the right upper lobe. Infiltrates are typ-ically associated with bacterial pneumonia.Pleural effusions and hilar adenopathy are find-ings not typically associated with bronchiolitis.(Long, 1115–1116)

75. (B) Giardia lamblia infection can cause a broadspectrum of clinical manifestations. Findingsfrom asymptomatic secretion of organismsto chronic diarrhea and failure to thrive areseen. Most commonly, children either are

asymptomatic or have an acute episode ofwatery diarrhea. Low-grade fever, abdominalpain, nausea, and anorexia may occur with theacute process. Protracted illness is usually asso-ciated with passage of foul-smelling stools,abdominal distention, and malabsorption.These can lead to failure to thrive. Bloodydiarrhea and cough are not associated withgiardiasis. (American Academy of Pediatrics, 283, Long,1242–1243)

76. (E) In addition to correcting dehydration andelectrolyte abnormalities, metronidazole is thedrug of choice. A 5–7-day course of therapyhas a cure rate of 85%–90%. Relapse is commonin immunocompromised patients, who mayrequire prolonged treatment. Treatment of asymp-tomatic carriers is generally not recommended.(American Academy of Pediatrics, 300)

77. (D) Tuberculous meningitis is the most seriouscomplication of tuberculosis in children. It usu-ally occurs within 2–6 months of initial infec-tion. It is difficult to diagnose rapidly as skintesting may be negative and chest films may benormal. Culture results can take anywherefrom 1–10 weeks to yield a positive result. CSFfindings can give clues of the diagnosis. CSFwhite cell counts range from 10 to 500 withusual lymphocytic predomination. Glucose istypically 20–40. Protein concentration is ele-vated, often markedly. Blood-tinged CSF is notassociated with tuberculous meningitis. (Long,776–777, American Academy of Pediatrics, 678)

78. (E) Children less than 1 year of age are at great-est risk for infection. Patients with increasedavailability of free iron such as those with sicklecell anemia, B-thalassemia, G6PD, and otherhemoglobinopathies, are also at increased riskof infection with Y. enterocolitica bacteremia.This is thought to be the result of bacterial ironscavenging systems which increase virulence.The most common manifestation of Y enteroco-litica is enterocolitis with fever and bloody,mucous-filled diarrhea, not bacteremia. Patientswith sickle cell anemia also have functionalasplenia and are more susceptible to infectionscaused by encapsulated organisms. (AmericanAcademy of Pediatrics, 821; Long, 732)

Page 175: Questions - uhs.edu.kh You suspect Lyme disease, most likely contracted by which of the following? (A) ingestion of unripe fruit (B) ingestion of spoiled fruit (C) drinking of contaminated

176 7: Infectious Diseases

79. (A) Lymphogranuloma venereum is an inva-sive lymphatic infection caused by Chlamydiatrachomatis. It starts as an ulcerative lesion onthe genitalia along with tender suppurativeinguinal and/or femoral adenopathy. It is mostoften unilateral. HSV can cause a painful ulcerand nonfluctuant adenopathy. Syphilis pro-duces a nontender chancre. Chancroid is causedby Haemophilus ducreyi and has an identicalpresentation to LGV. However, the diagnosis ofchancroid is strongly suggested if gram-negativecoccobacilli are seen. (American Academy of Pediatrics,248, 252)

80. (D) The recommended age range of childrenfor annual influenza immunization has beenexpanded to include all children 6 monthsthrough 18 years of age. This targets all school-aged children and will in turn reduce trans-mission of influenza to household contacts andcommunity members. The recommendationalso includes anyone who wants avoidinfluenza infection, and caregivers of infantsunder 6 months of age. Additionally, All health-care professionals, pregnant women, peopleover age 50, and people with a chronic illnessshould also receive annual vaccine. (MMWR.2008:57(RR-07):1–60)

81. (D) Topical nystatin is traditionally used totreat thrush and candidal diaper dermatitis.For uncomplicated infections, topical therapy isrecommended. However, candidal esophagi-tis and systemic candidiasis may be treatedwith systemic therapy, with fluconazole beinga reasonable option in infections with knownsusceptibility to that therapy. Amphotericin B anditraconazole are effective therapies, but morebroad spectrum, and not necessary for this patient.Metronidazole and mebendazole are not antifun-gal agents. (American Academy of Pediatrics, 244)

82. (C) Cryptosporidiosis usually causes self-limitedinfection in the immunocompetent host. Inimmunocompromised hosts, such as those withHIV, it can cause severe, debilitating disease.Transmission is generally from infected ani-mals, person to person, or contaminated water.Outbreaks have been associated with pettingzoos, day care centers, and local swimming

pools. Diagnosis is made by identifying oocyststhat are 4–6 µm in diameter on stool examination.The presentations of giardiasis, entamebiasis,and isosporiasis can be similar. The oocysts ofE histolytica and G lamblia are generally 10–15 µmin diameter and those of Isospora belli are 23–33 µm.Giardiasis is generally recognized by visualizationof the flagellated trophozoites in stool. Cryptococcusneoformans is a fungi not a protozoan and does notcause gastrointestinal disease. (American Academy ofPediatrics, 271, Long, 1236, 1241, 1245–1246).

83. (B) Pneumococcus is the most common causeof acute otitis media and of invasive bacterialinfections in children. Nontypeable H influenzaeis also a common cause, but since the advent ofHib vaccine H influenzae type B is rarely seen.Staphylococcus aureus and P aeruginosa causecomplicated otitis media in patients with tym-panostomy tubes. Pseudomonas aeruginosa is themost common cause of otitis externa.Streptococcus pyogenes can cause otitis media,however less frequently. (American Academy ofPediatrics, 525–526; Block, 57)

84. (D) Amoxicillin is the first-line therapy for otitismedia and acute sinusitis in children withoutpenicillin allergy. Initial therapy for acute sinusitisshould be with an agent of the narrowest spectrumthat is active against the likely pathogens, includ-ing M catarrhalis, H influenzae, and S pneumoniae.(American Academy of Pediatrics, 531)

85. (A) The most common presentation of tularemiais ulceroglandular syndrome, which is charac-terized by a painful maculopapular lesion atthe portal of bacterial entry that subsequentlyulcerates. Painful, regional lymphadenopathy isassociated, that may eventually drain. Wild mam-mals, including rabbits, are the hosts. Infection canbe spread by arthropods, or water and soil that arecontaminated by infected animals. Ingestion ofinadequately cooked meat or inhalation of organ-isms while preparing hides can also cause infec-tion. Lyme disease, RMSF, ehrlichiosis, and Q feverare tick-borne illnesses that do not cause ulcerog-landular syndrome. (American Academy of Pediatrics,704; Long, 148, 891)

Page 176: Questions - uhs.edu.kh You suspect Lyme disease, most likely contracted by which of the following? (A) ingestion of unripe fruit (B) ingestion of spoiled fruit (C) drinking of contaminated

Answers: 79–92 177

86. (E) Adog, cat, or ferret suspected of having rabiesshould be captured, confined, and observed for10 days by a veterinarian. If signs of rabies develop,the animal should be euthanized and the brainshould be examined for rabies by a qualified lab-oratory. If wild mammals can be captured afteran unprovoked bite they should be euthanizedand examined, as clinical manifestations in wildanimals can not be interpreted reliably. (AmericanAcademy of Pediatrics, 554–555)

87. (D) If a child is unimmunized, or immuniza-tion is incomplete for tetanus, a dose of theappropriate vaccine for age should be given,along with tetanus immune globulin (TIG) ifthe wound is considered dirty. As this child is5 years old, DTaP would be the best choiceaccording to the childhood immunization sched-ule. (American Academy of Pediatrics, 650–651)

88. (B) Rash is the first clinical manifestation ofLyme disease and is characteristic of earlylocalized disease. It occurs at the site of the tickbite and first appears 7–14 days later. The rashgradually expands without treatment and canappear as a target lesion with central clearing.The rash associated with RMSF and ehrlichio-sis is generally petechial and widespread. Therash that accompanies R typhi (endemictyphus) is maculopapular and can occur overany area, typically in more than one area.Rickettsia prowazekii (epidemic typhus) confersa rash that begins on the trunk and spreads tothe limbs with concentration in the axillae.(Long, 941; American Academy of Pediatrics, 428–429)

89. (C) The rash of Lyme disease is termed ery-thema migrans. Erythema nodosum is mostcommonly associated with group A strepto-coccal pharyngitis and is represented bypainful nodules on the shins. It is also associ-ated with many other organisms. Erythemamarginatum is associated with acute rheumaticfever, which occurs in fewer than 3% of patientswith rheumatic fever. Erythema toxicum is acommon newborn rash with eosinophilic pus-tules. Erythema multiforme has numerous man-ifestations; from macular to vesicular to urticarial.Diagnosis is established by finding donut-shapedlesions with an erythematous border and central

clearing. It is typically associated with viral infec-tion or drug reaction. (Long, 451, 707, 941–942)

90. (B) Trichophyton tonsurans is the causative agentin greater than 90% of cases in the UnitedStates. It is common in school-age children, andcan be associated with scaling, hair loss, orkerion, which is a boggy inflammatory mass.Local lymphadenopathy occurs with kerionformation. Topical antifungal medications arenot effective. Griseofulvin is the drug of choice,and 4–6 weeks of treatment is generally required.Treatment with oral itraconazole or fluconazoleis effective, but licensed in the United States forthis indication. (American Academy of Pediatrics, 655)

91. (D) Detecting HIV in an infant or child lessthan 18 months of age is complicated as thesechildren can carry maternal antibody to HIV,found on ELISA testing, and not be infected.DNA PCR assay is the preferred diagnostic test.Infants born to HIV-infected mothers should betested by HIV DNA PCR in the first 48 hoursof life. These infants need follow-up tests at1–2 months of age and again at 2–4 months ofage. Infection is confirmed with two separatepositive tests. Infection is excluded if two PCRtests performed within the first few months ofage and a third performed at 4 months of ageor greater are all negative. RNA PCR is utilizedin HIV-infected patients to monitor viral load.Western blot is used as a confirmatory test inpatients who are older than 18 months. HIVp24 antigen is known to give false-positiveresults in the first month of life. (American Academyof Pediatrics, 385–386)

92. (B) Adenosine deaminase deficiency is an auto-somal recessive form of severe combinedimmune deficiency (SCID). It causes increasedthymocyte and immature B-cell death fromaccumulation of purine metabolites. Severecombined immune deficiency (SCID) is aninherited deficiency of T- and B-lymphocytefunction that occurs in about 1/50,000 livebirths. Fungal infections, such as P jirovecipneumonia (PCP), are suggestive of severeCD4 lymphocyte (T cell) deficiency. PCP is acommon pathogen associated with SCID.Bruton agammaglobulinemia, common variable

Page 177: Questions - uhs.edu.kh You suspect Lyme disease, most likely contracted by which of the following? (A) ingestion of unripe fruit (B) ingestion of spoiled fruit (C) drinking of contaminated

178 7: Infectious Diseases

immunodeficiency, Job syndrome, and aspleniaare not associated with T-cell defects. (Long, 603,627–628)

93. (E) Patients with terminal compliment defi-ciencies (C5–C9) are incapable of forming themembrane attack complex, which is responsi-ble for bactericidal activity of neisserial infec-tions. These patients are therefore predisposedto disseminated meningococcal and gonococcalinfections. X-linked agammaglobulinemia is aB-cell defect. Patients with Chediak-Higashi syn-drome have a defect in phagocytosis. Patientswith HIV have poor T-cell function, and patientswith Wiskott-Aldrich syndrome have defects inboth T- and B-cell function. (Long, 301, 605)

94. (A) DTaP vaccine should be given as a primaryseries at 2, 4, and 6 months of age, followed bya dose at 12–15 months and 4–6 years. MMRvaccine should be given at 12–15 months ofage followed by a dose at 4–6 years. Primary vari-cella vaccine should be given at 12–15 months ofage, with a second dose at 4–6 years of age.Pneumococcal conjugate vaccine (PCV) shouldbe given as a primary series at 2, 4, and 6 monthsof age followed by a fourth dose at 12–15 months.Hepatitis A vaccine series should be given atage 12 months, followed by a second dose in 6months. The primary series for polio vaccine(IPV) should be performed at 2 and 4 months,followed by a third dose at 6–18 months, and abooster at 4–6 years. Haemophilus influenzae(Hib) vaccine should be administered at 2, 4, and6 months, followed by a dose at 12–15 months.Hepatitis B vaccine should be given at birthfollowed by two more immunizations with thesecond being at least 1 month after first, and thethird 6 months after the first. (American Academyof Pediatrics, 26)

95. (C) Definitive diagnosis of malaria relies onparasite identification on stained blood films.Both thick and thin films should be examined,as small numbers may be seen on the thickfilm. The thin film is useful for species identi-fication as well. Parasites will not grow on cul-ture. Serologic testing is generally not helpful.PCR is currently being developed, but is not

commercially available. Liver biopsy is unnec-essary. (American Academy of Pediatrics, 437)

96. (B) Children with streptococcal pharyngitisshould not return to school for 24 hours afterbeginning appropriate therapy. (American Academyof Pediatrics, 617)

97. (B) Secondary syphilis is a systemic multiorgandisease that begins 6–12 weeks after infection.Mucocutaneous lesions are common. Skin lesionsbegin on the trunk and eventually involve mostof the body, including the palms and soles.Generalized lymphadenopathy is found in 85%of patients. Arthritis, osteitis, gastritis, hepatitis,splenomegaly, and nephritic syndrome can occur.Systemic symptoms do not occur in primarysyphilis, which begins as a primary chancre.Chancroid, caused by H ducreyi, and lymphogran-uloma venereum (LGV) have an identical pres-entation of a painful chancre with suppurativeenlargement of local lymph nodes. Gonococcalinfection does not manifest as rash on the palmsand soles, or splenomegaly. (Long, 931; AmericanAcademy of Pediatrics, 631)

98. (A) Syphilis has been successfully treated withpenicillin for more than 45 years. Parenteralpenicillin remains the preferred treatment.Recommendations for duration of therapyvary depending on clinical manifestations.Desensitization in a hospitalized setting shouldbe undertaken in the penicillin-allergic patient.(American Academy of Pediatrics, 636–637)

99. (E) The resistance of S pneumoniae to penicillinsis due to resistant novel penicillin-binding pro-teins. S pneumoniae lacks beta-lactamases. Asthe name implies, the presence of the erm geneleads to macrolide resistance. Presence of themef gene produces a membrane efflux pumpwhich causes macrolides to actively be pumpedfrom the cell. S pneumoniae does not have pres-ence of the mef gene. Increasing the antibioticdose of amoxicillin from standard dose to highdose overwhelms the penicillin-binding pro-tein sites. (Long, 728–729)

Page 178: Questions - uhs.edu.kh You suspect Lyme disease, most likely contracted by which of the following? (A) ingestion of unripe fruit (B) ingestion of spoiled fruit (C) drinking of contaminated

Answers: 93–101 179

100. (B) Clindamycin is a lincosamide which are notstructurally related to macrolides, but have sim-ilar mechanism of action. They inhibit RNA-dependent protein synthesis by binding to the50S ribosomal subunits, inhibiting peptide bondformation in susceptible organisms. Clindamycinis most active against gram-positive organismsand anaerobes. The other choices availableare mechanisms of other classes of antibiotics.(Long, 690)

101. (C) Kingella infections generally occur in chil-dren under 5 years of age, with the mostcommon manifestations being suppurative

arthritis and osteomyelitis. The knee is the mostcommonly involved joint. Kingella organismsare fastidious, gram-negative coccobacilli,which are difficult to isolate in culture. It is bestgrown in anaerobic conditions with higherrecovery when joint fluid is inoculated into ablood culture bottle, and held for at least 7 days.S aureus and S pyogenes more commonly causeseptic arthritis, however they are both gram-positive cocci. Salmonella is a common cause ofosteomyelitis in sickle cell patients. Fusobacteriumis a rare cause of osteomyelitis. (American Academyof Pediatrics, 416)

Esper F, Boucher D, Weibel C, Martinello RA, Kahn JS.Human metapneumovirus infection in the UnitedStates: Clinical manifestations associated with a newlyemerging respiratory infection in children. Pediatrics.2003 Jun;111(6 Pt 1):1407–1410.

SELECTED READINGS

Page 179: Questions - uhs.edu.kh You suspect Lyme disease, most likely contracted by which of the following? (A) ingestion of unripe fruit (B) ingestion of spoiled fruit (C) drinking of contaminated

This page intentionally left blank

Page 180: Questions - uhs.edu.kh You suspect Lyme disease, most likely contracted by which of the following? (A) ingestion of unripe fruit (B) ingestion of spoiled fruit (C) drinking of contaminated

CHAPTER 8

Injuries, Poisoning,and Substance Abuse

Gary S. Wasserman, DO

Jennifer A. Lowry, MD

Richard J. Mazzaccaro, PhD, MD

Unintentional injuries are the leading cause of death for children aged 1–19 years. Among unintentionalinjuries, motor vehicle accidents account for nearly two-thirds of deaths. In addition, drownings and poisoningsremain common causes of morbidity and mortality in the pediatric population. The rates of firearm-relatedinjuries and deaths are manyfold higher in the United States than in other developed countries. Poisonings,substance abuse, and child abuse often lead to lifelong medical and social consequences. Pediatricians aretrained to provide anticipatory guidance to prevent injuries, poisonings, and substance abuse and in practice,age-based advice is well outlined.

181

Page 181: Questions - uhs.edu.kh You suspect Lyme disease, most likely contracted by which of the following? (A) ingestion of unripe fruit (B) ingestion of spoiled fruit (C) drinking of contaminated

182

DIRECTIONS (Questions 1 through 42): For eachof the multiple choice questions in this sectionselect the one lettered answer that is the bestresponse in each case.

1. A 15-year-old girl was at an overnight rave24 hours ago and was given Ecstasy by afriend. This morning, she is found comatoseand resuscitation is unsuccessful. Most deathsrelated to Ecstasy have been linked to which ofthe following?

(A) cerebrovascular accident(B) myocardial infarction(C) hyponatremia or hyperthermia(D) metabolic alkalosis with compensatory

respiratory acidosis(E) hyperkalemia or hypothermia

2. A2-year-old is found with an opened empty bottleof acetaminophen tablets and has pill fragments inhis mouth. The major cause of morbidity and mor-tality in acute poisoning with acetaminophen iswhich of the following?

(A) hepatic injury(B) gastric bleeding(C) metabolic acidosis(D) methemoglobinemia(E) hypoglycemia

3. A 4-year-old boy has just fallen off the top rungof the ladder while climbing up to the 3-mdiving board at the local swimming pool. Helands on the pavement and is motionless whenthe lifeguard arrives there in less than a minute.Following closed head injury, which of the fol-lowing would be most ominous?

(A) irritability(B) vomiting(C) dilated, fixed pupils(D) amnesia for the event(E) drowsiness

4. Yearly, over 20,000 individuals younger than21 years sustain traumatic head injuries whilebicycling. Helmet use while bicycling preventswhat percent of serious brain injury?

(A) less than 10%(B) 20%–25%(C) 50%(D) 80%–90%(E) 99%

5. An 18-month-old presents to the emergencydepartment with a 2-day history of lethargy.Blue and yellow bruising over the buttocks andthighs are noted and retinal hemmorhages areseen on fundoscopic examination. Parents denyany history of bleeding or bruising other thanwhat you see. Laboratory studies including aurine analysis, platelet count, bleeding time,and PT/PTT reveal normal values. Which ofthe following is the most likely diagnosis toexplain this child’s findings?

(A) von Willebrand disease(B) acute lymphoblastic leukemia(C) Henoch-Schöenlein purpura(D) idiopathic thrombocytopenic purpura.(E) child abuse

6. Most cases of serious physical child abuseinvolving children occur at what age?

Questions

Page 182: Questions - uhs.edu.kh You suspect Lyme disease, most likely contracted by which of the following? (A) ingestion of unripe fruit (B) ingestion of spoiled fruit (C) drinking of contaminated

Questions: 1–11 183

(A) less than 1-month old(B) between 1 month and 4 years old(C) between 5 and 12 years old(D) between 13 and 16 years old(E) who are neurologically impaired

7. A 24-month-old is seen for a well-child visit.The family resides in a 45-year-old home andthe parent notes that the child has developed ahabit of eating paint which has chipped off ofthe window sills. You are concerned that thechild could be at risk for lead toxicity. In theUnited States, the appropriate recommenda-tions for routine lead screening is at which ofthe following ages?

(A) at the 6 months check up(B) at the 6 and 9 months check up(C) at the 1- and 2-year check up(D) at 3- and 5-year check up(E) at the 12–15-year check up

8. Which of the following combinations of signsand symptoms is most suggestive of chroniclead poisoning?

(A) ataxia, fever, diarrhea, and polycythemia(B) lethargy, vomiting, hallucinations, and

vesicular rash(C) anemia, leukopenia, thrombocytopenia,

and hepatomegaly(D) lethargy, abdominal cramps, constipation,

and anemia(E) hypertension, rash, cough, and

leukocytosis

9. A 3-year-old child is brought to the emergencyroom 3 hours after he was found playing withan open kerosene bottle. The parents state thechild initially had some gagging and coughingand within an hour developed labored breath-ing. At the time of your evaluation though, thechild’s examination is completely normal. Whatis the most appropriate action at this time?

(A) discharge to home, advise parents toreturn if they note any problems

(B) obtain a chest x-ray(C) observe child in emergency room for

1 hour(D) induce emesis with syrup of ipecac(E) admit to the hospital for 24 hours of

observation

10. A 4-year-old child is brought to the urgent careby parents who were concerned because thechild suddenly developed unusual posture andmovements. Examination reveals an alert childwho holds the head in a tilted position and hasuncontrolled, writhing movements of the handsand arms. The examination is otherwise normal.When questioned regarding medications athome, parents state they do have routine coldmedicine, acetaminophen, and some kind ofantiemetic medication in an unlocked medicinecabinet. At this time you should do which ofthe following?

(A) perform a head computerizedtomography (CT) scan

(B) perform a lumbar puncture(C) obtain an electroencephalogram(D) administer naloxone intravenously(E) administer diphenhydramine

intravenously

11. A 4-year-old boy was found playing with anopen bottle of drain cleaner about 1 hour ago.His mother reports that he now refuses to drinkand talk but appears alert though anxious. Youshould advise the mother to do which of thefollowing?

(A) administer syrup of ipecac(B) closely observe the child and bring to

the emergency room if conditionworsens

(C) administer milk of magnesia(D) give the child cold frozen fruit popsicles(E) immediately bring the child to the emer-

gency room for evaluation

Page 183: Questions - uhs.edu.kh You suspect Lyme disease, most likely contracted by which of the following? (A) ingestion of unripe fruit (B) ingestion of spoiled fruit (C) drinking of contaminated

184 8: Injuries, Poisoning, and Substance Abuse

12. Shellfish poisoning, which is caused by eatingshellfish that have ingested toxic dinoflagel-lates (red tide), is characterized by which of thefollowing?

(A) blindness(B) vomiting and diarrhea(C) seizures and coma(D) weakness and paralysis(E) rash and fever

13. A toddler presents with a known ingestion ofiron tablets. By parental count of pills remain-ing in the bottle it appears he ingested morethan 60 mg/kg of elemental iron. Upon admis-sion to the pediatric intensive care unit, he isvomiting. Which of the following chelatingagents should be administered?

(A) deferoxamine mesylate(B) ethylene diamine tetraacetic acid (EDTA)(C) British anti-Lewisite (BAL)(D) hemoglobin(E) penicillamine

14. A 3-year-old child who is unresponsive pres-ents with weakness, excessive salivation, brady-cardia, and constricted pupils. The parents areso distraught, it is difficult to get information.However, an astute emergency room physicianrealizes the most likely drug or toxin to causethese signs is which of the following?

(A) diphenhydramine(B) phenobarbital(C) ethyl alcohol(D) a hydrocarbon(E) an organophosphate

15. Following stabilization of the patient in theabove question, the emergency room physicianadministers a test dose of a drug and thepatient clinically improves. What drug did thisphysician most likely administer?

(A) naloxone(B) diphenhydramine(C) atropine(D) N-acetylcysteine(E) phenobarbital

16. The mother of a high school senior varsity foot-ball player is concerned that her son may betaking some type of performance-enhancingdrug. Among 18-year-old males, approximatelywhat percent have used anabolic steroids asperformance enhancers?

(A) 1%(B) 5%–10%(C) 15%–20%(D) 30%–40%(E) 60%

17. Three symptomatic adolescents with a historyof ingesting seeds of jimsonweed are arrivingvia ambulance from a referring hospital emer-gency room. Which of the following best describesthe expected signs and symptoms of jimson-weed poisoning?

(A) agitation/hallucinations, dilated pupils(B) coma, pinpoint pupils(C) hallucinations, bradycardia(D) coma, bradycardia(E) pallor, pinpoint pupil

18. Ingestion of LSD will most likely result inwhich of the following?

(A) convulsions(B) euphoria(C) hallucinations(D) sedation(E) tremors

19. A teenager who sniffs spot remover and thenengages in stressful physical activity is at risk for

(A) convulsions(B) hypertension(C) rhabdomyolysis(D) severe headache(E) sudden death

20. Which of the following are the most commonlyused “date-rape” drug?

(A) amphetamines and LSD(B) gamma-hydroxybutyrate and

flunitrazepam

Page 184: Questions - uhs.edu.kh You suspect Lyme disease, most likely contracted by which of the following? (A) ingestion of unripe fruit (B) ingestion of spoiled fruit (C) drinking of contaminated

Questions: 12–27 185

(C) cocaine and phenobarbital(D) ephedra and codeine(E) phenobarbital and gamma-

hydroxybutryate

21. After being lifted up by one hand, a young tod-dler refuses to use that arm and holds it againsther trunk flexed at the elbow with the forearmmidway between pronation and supination.The child most likely has which of the following?

(A) a shoulder dislocation(B) a radial head subluxation(C) a fracture of a carpal bone(D) avulsion of the ulnar nerve(E) a fracture of the radius

22. A 4-year-old child falls on an outstretchedarm. The child is likely to sustain which of thefollowing?

(A) fracture displacement of the radialepiphysis

(B) Colles fracture(C) comminuted radial and ulnar fracture(D) shoulder dislocation(E) humeral fracture

23. Which of the following sets of blood gas valuesis most compatible with acute aspirin poison-ing in a 16-month-old child?

(A) pH 7.60, PCO2 40, HCO3 40(B) pH 7.5, PCO2 40, HCO3 30(C) pH 7.25, PCO2 20, HCO3 8(D) pH 7.20, PCO2 45, HCO3 20(E) pH 7.00, PCO2 35, HCO3 8

24. Hyperventilation due to salicylate poisoning

(A) is apparent on physical examinationwithin minutes of ingestion

(B) is characterized by an increase in rateand depth of ventilation

(C) is characterized by an increase in depthof ventilation only

(D) is characterized by an increase in rate ofventilation only

(E) does not occur in young children

25. Which of the following findings would be mostsuggestive of the form of child abuse referredto as the abusive head trauma syndrome(formally called shaken babt syndrome)

(A) ecchymosis over the mastoid area(B) retinal hemorrhages(C) ecchymoses and petechiae over the

upper arms and upper trunk(D) circumferential ecchymosis on extremities(E) cervical spine dislocation

26. A 9-year-old is injured while sledding. Onadmission, the child appears in shock and iscomplaining of pain in the left shoulder. Ofimmediate concern is the likely diagnosisof

(A) rupture of the descending aorta(B) dislocation of the left shoulder(C) rupture of the spleen(D) rupture of the left diaphragm(E) fracture of the humerus

27. Which of the following is the most commonunintentional fatal injury to children 1–19 yearsof age

(A) motor vehicles(B) swimming pools(C) firearms(D) bicycles(E) fireworks

Page 185: Questions - uhs.edu.kh You suspect Lyme disease, most likely contracted by which of the following? (A) ingestion of unripe fruit (B) ingestion of spoiled fruit (C) drinking of contaminated

186 8: Injuries, Poisoning, and Substance Abuse

28. A 1-year-old child is brought to the emergencyroom because of a swollen left thigh. The parents,who appear very concerned, state they left thechild in the care of a newly hired housekeeperearly that morning, and when they returnedhome in the evening they noted the swelling.Other than tender swelling of the thigh, physicalexamination is entirely normal. X-ray examinationdiscloses a displaced fracture of the shaft of thefemur; skeletal survey reveals no other fractures orabnormalities. The grandparents, who live withthe parents and who had accompanied them ontheir trip, corroborate the parents’ story. Afterfirst admitting the child for treatment of the frac-ture, what is the most appropriate next step?

(A) order a computerized tomography (CT)scan of the head

(B) order a complete coagulation profile(C) order magnetic resonance imaging

(MRI) of the leg(D) order calcium, phosphate, and uric acid

laboratory tests(E) report the incident to a child protection

agency

29. Syrup of ipecac should be administered in thehome to which of the following children?

(A) 3-year-old child who ingested lye (sodiumhydroxide) 5 minutes prior

(B) 4-year-old child found obtunded,suspected of narcotic poisoning

(C) 2-year-old child who is having briefseizures, suspected of ingesting sibling’sphenytoin

(D) 2-year-old child who ingested toiletbowl cleaner

(E) none of the above

30. A 4-year-old child is playing in the basement.The child suddenly comes upstairs, screamingof being bitten by a spider. There is an erythe-matous 2 cm macule on the child’s face whichappears to have a central tiny puncture site.Over the next few hours the lesion becomeslarger, more painful, and in 24 hours it appearsdarker. The most likely complication in thischild would be which of the following?

(A) necrosis at the site of the bite(B) renal failure(C) hepatic failure(D) muscle cramps and seizures(E) convulsions

31. Management with multiple-dose activated char-coal may be indicated in the overdose of whichof the following?

(A) iron(B) cyanide(C) carbamazepine(D) tricyclic antidepressants(E) methanol

32. A 2-year-old child is retrieved from a near-drowning episode in a pool. The child is apneicon retrieval, but is quickly and successfully resus-citated. On arrival in the emergency room, abnor-malities which of the following are most likelyto be present and require immediate attention?

(A) hyponatremia and hypokalemia(B) hyponatremia and hyperkalemia(C) hyperkalemia and acidosis(D) acidosis and hypoxemia(E) hypoxemia and hemolysis

33. Manifestations of the first stage of severe acuteiron poisoning include

(A) lethargy and gastrointestinal irritation(B) metabolic alkalosis and hypertension(C) hemolysis and neutropenia(D) renal, hepatic, and cardiac failure(E) hypoglycemia and hepatic injury

34. Which of the following statements regardingautomobile safety for children is correct?

(A) Children, beyond the age of 1 year, or20 lb, may ride either facing the front orrear of the car.

(B) Children over 25 lb may use adult-typerestraints.

(C) A 1-year-old child held in the lap of aseat-belted adult is almost as safe as inan infant restraint device.

Page 186: Questions - uhs.edu.kh You suspect Lyme disease, most likely contracted by which of the following? (A) ingestion of unripe fruit (B) ingestion of spoiled fruit (C) drinking of contaminated

Questions: 28–40 187

(D) Safety restraints are not needed for infantsless than 3 months of age or 10 lb of bodyweight.

(E) Infants under the age of 1 year shouldride in restraint devices facing the rearof the car.

35. Which of the following statements regardingdrowning and near-drowning is correct?

(A) Four-sided fencing around pools has notdecreased the incidence of drownings.

(B) Children under the age of 5 years whodrown in home pools most often enterthe pool by climbing over a fence.

(C) The incidence of drowning peaks in ele-mentary school-age children.

(D) Approximately 50% of those who dieare declared dead at the scene.

(E) Among children who survive, neurologicimpairment is generally uncommon.

36. Concerning child sexual abuse, which of thefollowing is correct?

(A) Boys and girls are at equal risk.(B) In the majority of cases, the abuser is

well known to the child.(C) Physical contact is necessary to fulfill

diagnostic criteria for child sexual abuse.(D) An estimated 10% of children are sexually

abused each year in the United States.(E) There must be evidence of sexual inter-

course for successful prosecution.

37. Which of the following statements regardingfirearm-related injuries in children less than19 years of age is correct?

(A) Intentional firearm-related injuries faroutnumber unintentional firearm-relatedinjuries.

(B) Half of firearm-related deaths are theresult of suicide.

(C) Firearms account for the majority of allinjury deaths in this age group.

(D) Firearm-related injuries result in moredeaths than do motor vehicle accidentseach year.

(E) Most firearm-related deaths occur afterprolonged hospital course.

38. Which of the following is most suggestive ofunintentional (nonabuse) injuries in children?

(A) hand print bruise on the face of a child(B) belt marks on the buttocks of a 3-year-

old child(C) bruises on upper thighs reported to

result from “spanking”(D) fractured femur of a 1-month-old baby

from rolling off the bed(E) multicolored bruises along the shins of a

4-year-old child

39. Injuries remain the leading cause of deathamong all children of age 1–19 years. For those15–19 years of age, which of the following liststhe categories of injuries from most common toleast common?

(A) suicide, homicide, motor vehicle accidents(MVA)

(B) homicide, suicide, MVA(C) MVA, suicide, homicide(D) MVA, homicide, suicide(E) homicide, MVA, suicide

40. Overdose of which of the following is mostlikely to be complicated by hypoglycemia?

(A) salicylates(B) lead(C) tricyclic antidepressants(D) opioids(E) organophosphates

Page 187: Questions - uhs.edu.kh You suspect Lyme disease, most likely contracted by which of the following? (A) ingestion of unripe fruit (B) ingestion of spoiled fruit (C) drinking of contaminated

188 8: Injuries, Poisoning, and Substance Abuse

41. Last week, a 14 year old boy is caught smoking acigarette by his mother and the teen is now beingseen in your pediatric clinic. His mother is con-cerned about short- and long-term effects of smok-ing tobacco at such a young age. Which of thefollowing statements concerning tobacco/cigarettesmoking in preadolescent and adolescent years ismost accurate?

(A) The average smoker in the United Statesstarts at age 16 years.

(B) 50% of adolescent smokers becomeadult smokers.

(C) Adolescent smokers become nicotinedependent after smoking fewer ciga-rettes than the adult smoker.

(D) The use of cigars and smokeless tobaccois rare in adolescence.

(E) Cigarette use cannot be detected byevaluating a urine drug screen.

42. A12-month-old infant who has been walking wellfor 3–4 weeks, now limps on the right leg. Nospecific injury can be recalled by the parents. Thechild is afebrile. Examination reveals tendernesswith gentle twisting of the lower right leg. An x-rayreveals an oblique nondisplaced fracture of thedistal tibia. This condition is referred to as whichof the following?

(A) toddler fracture(B) Monteggia fracture(C) nursemaid ankle(D) Cozen fracture(E) Osgood-Schlatter disease

DIRECTIONS (Questions 43 through 52): The fol-lowing group of questions is preceded by a list oflettered answer options. For each question, matchthe one lettered option that is most closely associ-ated with the question. Each lettered option maybe selected once, multiple times, or not at all.

Questions 43 through 52

(A) naloxone(B) flumazenil(C) oxygen(D) ethanol or 4-methylpyrazole(E) deferoxamine(F) sodium bicarbonate(G) atropine(H) sodium nitrite/sodium thiosulfate or

hydroxocobalamin(I) methylene blue

43. organophosphates

44. cyanide

45. tricyclic antidepressants

46. benzodiazepines

47. ethylene glycol

48. carbon monoxide

49. methemoglobinemic agents

50. opioids

51. methanol

52. iron

Page 188: Questions - uhs.edu.kh You suspect Lyme disease, most likely contracted by which of the following? (A) ingestion of unripe fruit (B) ingestion of spoiled fruit (C) drinking of contaminated

189

1. (C) Raves, or underground all-night parties, typ-ically are attended by adolescents and youngadults. Probably the most popular club drug usedat raves is Ecstasy, or methlyenedioxymetham-phetamine (MDMA). The use of Ecstasy, anamphetamine with hallucinogenic and stimu-lant properties, results in CNS agitation, tachycar-dia, hypertension, and diaphoresis. Serotonergiceffects of Ecstasy also include enhanced sensualperceptions and blunted perceptions of hungerand thirst. Most Ecstasy-related deaths have beenlinked to hyperthermia (increased physical activ-ity in enclosed space) or hyponatremia (waterintoxication or SIADH). (Poirier, 2002;18:216–218,www.usdoj.gov/ndic)

2. (A) Ingestion of acetaminophen in therapeuticdoses typically does not result in side effects.However, ingestion of potentially toxic doses(> 10 g in adults and > 200 mg/kg in children)may result in hepatotoxicity. When present, hepa-totoxicity peaks 48–96 hours after ingestion. Theuse of antidotal therapy with N-acetylcysteine isguided by the serum acetaminophen leveldrawn no less than 4 hours following ingestion.(Acetaminophen: Consensus Guideline for out of hospitalmanagement; Rudolph, 2002;409–411)

3. (C) Following head trauma, eye changes such asfixed, dilated pupils usually are indicative ofincreasing intracranial pressure or focal neurologicdamage. Ahistory of unconsciousness, irritabilityand lethargy, amnesia for the event, and/or vom-iting are seen commonly in the absence of majorintracranial injury. (Rudolph, 348, 2244; Singer,62:819, 1978)

4. (D) An astounding 88% of serious brain injurysustained while bicycling is prevented withhelmet use. Additionally, helmet use prevents anestimated 65% of injuries to the mid and upperface. Two factors have been identified as havinga strong association with bicycle helmet use byyoung children. These are helmet use by anaccompanying parent and a state mandatoryhelmet use law or local ordinance. Communitieshave successfully raised the rate of helmet usewith a variety of programs. (AAP Committee onInjury and Poison Prevention 2001;108:1030–1032)

5. (E) Physical abuse is the leading cause of seri-ous head injury among infants, and studiessuggest 95% of serious head injuries in infantsunder 1 year of age are related to child abuse.The entity “abusive head trauma syndrome(formally referred to as shaken baby syn-drome)” typically occurs in infants less than2 years and was first described as a constella-tion of findings which included retinal hem-orrhages, subdural and/or subarachnoidhemorrhages, and little or no evidence ofexternal cranial trauma caused by “extremerotational cranial acceleration induced by vio-lent shaking.” Evidence of other injuries, suchas bruises, rib fractures, long-bone fractures,and abdominal injuries are often not present. Inup to 90% of cases, unilateral or bilateral retinalhemorrhages are present. In cases of leukemiaand idiopathic thrombocytopenic purpura,hematologic abnormalities generally are presentand suggestive of the diagnosis. Henoch-Schöenlein purpura is a systemic vasculitistypically manifest by a triad of arthritis,abdominal pain (related to small boweledema and inflammation and occasionally to

Answers and Explanations

Page 189: Questions - uhs.edu.kh You suspect Lyme disease, most likely contracted by which of the following? (A) ingestion of unripe fruit (B) ingestion of spoiled fruit (C) drinking of contaminated

190 8: Injuries, Poisoning, and Substance Abuse

intussusception), and a classic purpuric rashwhich appears on the buttocks and legs of thechild; neurologic manifestations may occur butare not characterized by retinal hemorrhage.(AAP Committee on Injury and Poison Prevention, July2001;108(1):206–210; Kliegman, 2007)

6. (B) Most cases of physical child abuse andalmost all deaths from abuse occur in the agegroup less than 4 years, especially less than 2 or3 years, before the child can communicate effec-tively with others. In approximately 10% of allemergency room visits for injuries in childrenyounger than 5 years, abuse is the etiology ofthose injuries. Although neurologically impairedchildren are at increased risk for abuse, mostvictims are previously healthy and have no pre-existing neurologic deficits. (Behrman, 111)

7. (C) Cases of lead poisoning in the United Statesare seen chiefly in the toddler age group. This isunderstandable, considering the mechanism ofpoisoning. The major source of lead poisoning inchildren living in urban areas is old, flaking leadpaint, found in pre-1950 buildings; it is esti-mated that 25% of US children reside in build-ings with lead-based paint. Some children willchronically ingest the paint flakes. Although it isno longer legal to use lead paints indoors, oldbuildings still may have layers of paint withhigh lead content beneath the more recent coatsof “lead-free” paint. Children become exposedto lead as older homes are renovated. As severe,overt lead poisoning has become less common,there has been an increased concern aboutthe long-term neurodevelopmental effects ofsubclinical lead poisoning, and guidelines forlead screening are issued by the CDC. Currentguidelines from the American Academy ofPediatrics state that routine screening shouldoccur at 1 and 2 years of age. (McMillan, 599–632;AAP Committee on Environmental Health, October2005;116(4):1036–1046 [doi:10.1542/peds.2005-1947])

8. (D) Common signs of lead poisoning includelethargy, abdominal cramps, constipation, andanemia. Vomiting also is common. Ataxia is seenoccasionally. The other items listed—fever, diar-rhea, rash, hallucinations, hypertension, throm-bocytopenia, and cough—are not associated with

lead poisoning. (McMillan, 632; AAP Committee onEnvironmental Health, October 2005;116(4):1036–1046[doi:10.1542/peds.2005-1947])

9. (B) Up to 28,000 children (usually < 5 years ofage) ingest hydrocarbons each year. Most chil-dren will remain asymptomatic. Those whodevelop respiratory symptoms, even if transient,should be evaluated by a physician. Achest radi-ograph should be part of the initial evaluation ofall those children who developed respiratorysymptoms, such as wheezing, coughing,gagging, or dyspnea, even if symptoms weretransient. Pneumonitis is most commonlyencountered in children with respiratory symp-toms; radiographs can be normal initially. Thedecision to hospitalize a child is not based onchest radiograph alone but based on clinicalsymptoms and need for oxygen or other sup-portive care. Syrup of ipecac should never begiven as vomiting is associated with an increaserisk for aspiration and subsequent respiratorycomplications. (Fleisher, 914–915; Flomenbaum,1435–1442)

10. (E) The child described has developed extrapyra-midal symptoms typical of phenothiazine toxic-ity. These findings may occur with therapeutic aswell as excessive dosage and are common in chil-dren. Akinesia, trismus, opisthotonos, torticollis,chorea, dystonia, and oculogyric crises may beseen. These symptoms usually respond dramati-cally to intravenous diphenhydramine (Benadryl)or benztropine mesylate (Cogentin), althoughrelapses are frequent. The clinical picture is soclassic that it can be suspected even in theabsence of a history of ingestion. Aside frommanaging this ingestion and screening for othermedications this child may have ingested, youmust work with these parents to prevent futureepisodes of ingestion. (Flomenbaum, 1043–1044)

11. (E) This child should be evaluated immediatelyby a physician. Ingestion of caustic agents, alka-line or acidic, results in dermal and mucosalinjury from contact. Refusal to drink may indicatetissue damage and an assessment should be per-formed to determine the extent of this damage.The physical examination should include closeexamination of the oropharynx. Clinical or

Page 190: Questions - uhs.edu.kh You suspect Lyme disease, most likely contracted by which of the following? (A) ingestion of unripe fruit (B) ingestion of spoiled fruit (C) drinking of contaminated

Answers: 6–18 191

radiographic evaluation can demonstrate signs ofesophageal or tracheal perforation. This includessubcutaneous air and crepitus, pneumothorax,pneumomediastinum, or pneumoperitoneum. Ifendoscopic evaluation is indicated, it should beperformed within the first 24 hours because ofincreased risk of iatrogenic perforation with thisprocedure after 48 hours following ingestion.Long-term effects of caustic ingestions includeesophageal stricture formation. (Flomenbaum,1410–1413)

12. (D) Shellfish poisoning is characterized by pares-thesia and numbness of the mouth and face, gen-eralized weakness, and paralysis. The incubationperiod is brief, lasting from minutes to hours.Treatment is supportive; in severe cases, mechan-ical ventilatory assistance may be required.Presumably the flagellates ingested by the shell-fish produce a neurotoxin, which is, in turn,ingested when the shellfish are eaten (typicallyoysters, clams, mussels, or scallops), accountingfor the symptoms. Larger numbers of dinoflagel-lates in the water where the shellfish are harvestedcan impart a red or reddish-brown color to thewater, the so-called red tide. (Flomenbaum, 703–704)

13. (A) Iron poisoning in children generally doesnot occur with ingestion of multivitamins, butmore commonly it is associated with prenatalvitamins or stand-alone iron-containing tablets.Deferoxamine it is the chelating agent of choicefor iron poisoning. It combines with iron toform ferrioxamine, which is excreted in theurine. Parenteral administration (intravenous)is given for children with severe poisoning(typically defined by clinical symptoms/toxicityand/or a measurement of iron greater than500 mg/dL). (Flomenbaum, 634)

14. (E) The signs described—coma, weakness, exces-sive salivation, bradycardia, and constrictedpupils—are classic for organophosphate poi-soning. Organophosphates are potent and irre-versible inhibitors of acetylcholinesterase andcan bind to muscarinic and nicotinic receptors.Excessive salivation is not seen in phenobarbital,diphenhydramine, ethyl alcohol, or hydrocar-bon poisoning. (Flomenbaum, 1500–1502)

15. (C) Atropine is the major antidote in the treatmentof organophosphate poisoning. Administeredintravenously, it can result in rapid improvementin signs and symptoms. This improvement, how-ever, typically is short-lived, necessitating repeatdosing. (Flomenbaum, 1505–1506)

16. (B) It is estimated that 6.6% of high school seniorshave used anabolic steroids and one-third ofthem were not even participating in organizedhigh school sports. Other performance-enhancingdrugs include substances classified as supplements(such as caffeine, creatine, or androstenedione), orprescription drugs (such as beta-blockers, ordiuretics). Some have been banned by governingboards or made illegal by law. These are classifiedas illicit or banned substances and include, amongothers, narcotics, human growth hormone, ana-bolic steroids, and gamma-hydroxybutyrate.A common triad consisting of acne, striae, andgynecomastia is reported in men. Multiorgan tox-icity is possible and involvement of the heart(myocardial infarction), liver (hematomas, hep-atitis), skin (acne, hirsutism in women), endocrine(gynecomastia, testicular atrophy, amenorrhea,and breast atrophy in women), and neurologic(depression, mania, mood lability, and aggres-siveness) systems may be seen. (Flomenbaum, 1688)

17. (A) Jimsonweed seeds are abused because oftheir profound hallucinogenic effect. This, asnamed, is a weed with widespread growththrough the continental United States and manyother countries. Though some data suggest theuse of this substance by adolescents is declin-ing, fatalities associated with ingestion of jim-sonweed continue to be reported. Typicallypatients present as agitated adolescents withsigns of anticholinergic toxicity such as tachycar-dia, dilated pupils, and flushed skin. Convulsionsare infrequent, but can be severe and difficult tocontrol. (Flomenbaum, 220)

18. (C) Hallucinations, especially visual, are themost common and most striking effect of LSD.Sensations are magnified and distorted. Thepatient may imagine seeing odors or hearingcolors. The emotional response can be eitherpositive and pleasurable or negative and fright-ening. (Fleisher, 934)

Page 191: Questions - uhs.edu.kh You suspect Lyme disease, most likely contracted by which of the following? (A) ingestion of unripe fruit (B) ingestion of spoiled fruit (C) drinking of contaminated

192 8: Injuries, Poisoning, and Substance Abuse

19. (E) Sudden death is not infrequent in thosewho sniff organic solvents. The risk appears tobe especially great if the inhalation involves ahalogenated hydrocarbon (frequently used assolvents or spot removers) and is followed byexercise or other vigorous physical activity. Ithas been postulated that this may be related tosensitization of the myocardium by the volatilehydrocarbons. A lethal arrhythmia then is pre-cipitated by the catecholamine release occa-sioned by the exercise. (Knight, 1987;34:335,337)

20. (B) Both flunitrazepam (Rophynol®; streetnames: roachies, roofies, the forget pill, roofenol)and gamma-hydroxybutyrate (GHB; street names:easy lay, grievous body harm) are used as agentsin date-rape. Flunitrazepam, a benzodiazepinenot licensed for use in the United States butreadily available from dealers for less than $5per tablet, quickly dissolves in liquids as a col-orless and odorless sedative/hypnotic agent.Symptoms of dizziness, disorientation, and/ornausea begin within 15–20 minutes followingingestion, and peak with unconsciousness within1–2 hours. These, along with the amnestic prop-erties of flunitrazepam, have enabled sexualpredators to render their victims helpless, andhave made prosecution for sexual assault crimeseven more challenging. GHB, commonly usedas a euphoriant or aphrodisiac at parties orraves, also can be unknowingly ingested by avictim who is subsequently sexually assaultedor raped. This compound can be obtained insmall quantities at large parties or in liter bottlesfrom Canadian web sites. GHB also is popularamong body builders because it is reputed toincrease the production of growth hormonewhich increases muscle mass. (Fliesher, 939,www.rapecrisiscenter.com; Poirier, 2002;18:53–59)

21. (B) When a young child is lifted off the ground,dragged, or swung by one arm, the radius maypartially escape from the annular ligament at theelbow. This subluxation of the radial head is acommon injury, also commonly called nursemaidinjury or nursemaid elbow. The child holds theinjured arm flexed at the elbow and refuses tomove it. The subluxation usually is easily reducedby supination of the arm. Recurrent radial headsubluxations are seen in 33% of patients. Therefore,

caretakers should be informed of this risk and ofpreventive measures. (Fleisher, 1945)

22. (A) Epiphyseal separations are common child-hood injuries. The growth plate or epiphysis isgenerally the weakest part of a child’s bone,weaker even than surrounding ligaments. Traumawhich would result in a tear of the ligament in anadult, often results in an epiphyseal fracture in achild. A fall onto an outstretched arm, whichmight result in a Colles fracture in an adult, islikely to cause a separation fracture of the distalradial epiphysis in a child. (Rudolph, 1436)

23. (C) Aspirin poisoning results in a mixed distur-bance of metabolic acidosis and respiratory alka-losis. In adolescents and adults, the predominantearly abnormality usually is the respiratory alka-losis. Before the age of 2 years, however, meta-bolic acidosis is the predominant process andthe net change in arterial pH generally is adecrease. (Rudolph, 375–376; Gaudreault,1982;70:566)

24. (B) Salicylate poisoning results in an increase inboth rate and depth of ventilation. The latterusually is especially striking. Generally, neithertachypnea nor deep respirations are clinicallyapparent until several hours after ingestion.Even young infants show this response. (Rudolph,375–376; Gaudreault, 1982;70:566)

25. (B) Retinal hemorrhages and cerebral hemor-rhages are characteristic of the abusive headtrauma syndrome. Often, there are no externalsigns of trauma. This type of child abuse occursmost commonly in children less than 1 year ofage and is associated with high morbidity andmortality. Of children who survive this injury,35% will be blind or visually impaired.Increasingly, this syndrome is referred to byother names such as shaken baby/impact syn-drome, nonaccidental head trauma, and abu-sive head trauma. (Fleisher, 1677; Rudolph, 2415;Caffey, 1974;54:396)

26. (C) The child described most likely has sus-tained an injury to the spleen. Splenic trauma,with or without rupture, is a common sleddinginjury. Shock can occur rapidly, or some timelater. Pain in the left shoulder is common and

Page 192: Questions - uhs.edu.kh You suspect Lyme disease, most likely contracted by which of the following? (A) ingestion of unripe fruit (B) ingestion of spoiled fruit (C) drinking of contaminated

Answers: 19–32 193

reflects irritation to the left diaphragm by sub-phrenic blood. Hospitalization with carefulmonitoring is the current approach to man-agement, with every attempt to salvage, ratherthan remove, the spleen. (Fleisher, 1365)

27. (A) Unintentional injuries remain the leadingcause of death among children aged 1–19 years.Of these fatal unintentional injuries, two-thirdsinvolve motor vehicles. (McMillan, 9; MacDorman,2002;110:1037–1052)

28. (E) All states have laws requiring any personhaving reason to suspect child abuse or neglectto report the case to the proper child protectiveauthority. Proof is not a prerequisite for report-ing. In a case of suspected abuse, physicians mustidentify and treat physical injuries and mustensure the child’s immediate safety. Detailedrecording of exact history given, including his-tory in the child’s own words if he/she is oldenough to speak, and physical findings (photo-graphs or videos) can be extremely useful to thechild protective team. In this case, further eval-uation by scanning, or blood tests are not neces-sary since the fracture is clearly seen on plainfilms. (Rudolph, 2002;415–418)

29. (E) In November 2003, the American Academyof Pediatrics (AAP) issued a policy statement,Poison Treatment in the Home, which statedsyrup of ipecac should no longer be used rou-tinely as a home treatment strategy. Further, itrecommends safe disposal of all ipecac fromhomes. The policy identifies the incidence of sig-nificant vomiting often with prolonged duration(potentially impacting the efficiency of activatedcharcoal therapy), lack of efficacy for the vastmajority of children, and potential for abuse (eg,eating disorders and Munchausen by proxy ie,Pediatric Falsification Condition). Caregivers of achild who may have ingested a toxic substanceshould consult with local poison control centers bytelephoning 800-222-1222. (AAP Committee on Injury,Violence and Poison Prevention, 2003;112:1182–1185)

30. (A) The most likely villain in this case is thebrown recluse spider (fiddler spider, Loxoselesreclusa). Envenomation is characterized by severelocal reaction, often a blister at the bite site

which often progresses to local tissue necrosis,particularly in bites that occur over fatty areasof the body. An associated rash is often noted.Severe systemic symptoms and painful musclecramps are characteristic of black widow spider(Latrodectus mactans) bite, in which case there islittle local reaction, although local pain is common.(Fleisher, 1049–1050)

31. (C) Activated charcoal administered orally orvia nasogastric tube adsorbs certain substancesonto its surface, thereby decreasing the amountavailable for absorption from the gastrointestinaltract. Multiple-dose activated charcoal (MDAC)is defined as the administration of more thantwo doses of activated charcoal in the treatmentof a given poisoning. It is believed that drugswith a prolonged elimination half-life, lowplasma protein binding, and a small apparentvolume of distribution (eg, ≤ 0.6 L/kg) may bemore quickly eliminated by use of multipledoses of activated charcoal and as a result, theclinical course of the patient improved. In addi-tion, some drugs undergo enterohepatic orenteroenteric recirculation which, in the case ofan overdose, may be enhanced by such ther-apy. Multiple-dose activated charcoal shouldbe considered for life-threatening ingestions ofcarbamazepine, dapsone, phenobarbital, quinine,and theophylline. The decision to use multiple-dose activated charcoal should be based on thephysician’s clinical judgment, lack of contraindi-cations to its use, and the potential of other alter-native therapies. (Lowry, 2008; Flomenbaum, 115–116,130–131)

32. (D) The most immediate concern in patientssuccessfully resuscitated from a near-drowningepisode is correction of hypoxemia and acido-sis. The consequences of cerebral hypoxia withacute brain swelling are the major causes ofmorbidity and mortality. Aspiration pneumo-nia is common. Life-threatening electrolyte dis-turbances are quite rare in patients who surviveto the emergency room. Most victims of near-drowning aspirate relatively late in the immer-sion episode, after they have become severelyhypoxic secondary to apnea. Aspiration oflarge quantities of fluid, therefore, is rare. (Note:A key to answering this question correctly is

Page 193: Questions - uhs.edu.kh You suspect Lyme disease, most likely contracted by which of the following? (A) ingestion of unripe fruit (B) ingestion of spoiled fruit (C) drinking of contaminated

194 8: Injuries, Poisoning, and Substance Abuse

attention to the phrase “require immediateattention.”) (Fleisher, 1009–1012)

33. (A) The clinical manifestations of iron poison-ing have been organized into four stages. Thefirst stage is characterized by gastrointestinal(vomiting, diarrhea, abdominal pain, and gas-trointestinal bleeding) and neurologic (lethargyor coma) signs. This is followed by a secondstage of deceptive quiescence, of up to 48 hours,and then a third stage characterized by shockand metabolic acidosis, with or without evidenceof hepatic injury. Leukocytosis is common. Latesequelae (stage four) include pyloric or antralstenosis and hepatic cirrhosis. (Fleisher, 979–982,Behrman, 350–351)

34. (E) The head of an infant is relatively larger(compared to total body size or weight) thanthe older child or adult. For this reason, theneck is subjected to proportionally increasedforce during a crash. Having the infant facebackward diffuses the blow (deceleration) overthe entire back. Infants under the age of 1 yearshould ride facing the rear of the car. All olderchildren should ride properly restrained, facingforward. It has been shown that the force gen-erated by a 1-year-old infant in a front-endcrash far exceeds the ability of an adult to holda child in the lap. The child will be propelledagainst the interior of the automobile or outsidethe automobile. (McMillan, 139–141)

35. (D) Drowning is defined as death from suffoca-tion within 24 hours of submersion in water.Near-drowning victims survive at least 24 hours.Therefore, near-drowning victims may surviveor die from complications of the submersion.As with many injuries and poisonings, there arepeaks in occurrence of drownings in toddlersand in teenagers. In the United States, almostone-half of those who die of drowning aredeclared dead at the scene and never present toa medical facility for care. Of children and ado-lescents who survive near-drowning, an esti-mated 35% will have significant neurologicimpairment. Prevention is of utmost importance.Installation of four-sided fences around poolsdecreases the number of pool immersion injuriesby more than 50% in children aged 1–4 years.

As with all injuries, brief lapses in appropriatesupervision may result in tragedies. (Fleisher,1011–1012; Behrman, 438–449)

36. (B) Childhood sexual abuse is defined as “theengaging of dependent, developmentallyimmature children in sexual activities that theydo not fully comprehend and to which theycannot give consent or activities that violatethe laws and taboos of a society.” The abusersfrequently are relatives or others well known tothe child. Impotence and low self-esteem areoften seen in abusers. It is estimated that about1% of children are sexually abused each year inthe United States. Girls are six times more likelythan boys to be victims of sexual abuse. (Fleisher,1780, Behrman, 178–182)

37. (A) Firearm-related deaths in children aged1–19 years are overwhelmingly intentional,with fewer than 8% unintentional. Most of thesedeaths occur prior to arrival at the hospital.Firearms remain the most common means ofsuicide in males of all ages. Though the rates ofmotor vehicle accident deaths have declinedover the past two decades, they remain the lead-ing cause of injury-related deaths in this agegroup. (McMillan, 135–144, Behrman, 373)

38. (E) It is extremely important to recognize pat-terns of abuse to prevent escalating abuse oradditional injury and also to prevent unneces-sary family anxiety with unwarranted referrals tochild protective authorities. Trends found incases of child abuse include a history not con-sistent with the injury or the developmentalstage of the child (he rolled off the bed and brokehis leg, said by the parent of the 1-month-oldbaby), the presence of a pattern of injuries knownnot to occur except with abuse (handprint bruiseon the face), a pattern of injury that reflects injurywith an instrument in a manner that would notoccur in play or natural injury (hanger or loopcord marks on buttocks), delay in seeking med-ical attention, a history that changes during thecourse of the evaluation, and a history of recur-rent injuries. On the other hand, bruises along theanterior lower leg, where children often bumpinto objects and where the bone is close to theskin, are characteristic of active preschoolers.

Page 194: Questions - uhs.edu.kh You suspect Lyme disease, most likely contracted by which of the following? (A) ingestion of unripe fruit (B) ingestion of spoiled fruit (C) drinking of contaminated

Questions: 33–43 195

Figure 8-1 demonstrates a burn injury incurredwhen a child was placed in scalding water. Thestory the parents gave that the child fell forwardinto the water in a pot on the stove is inconsistentwith the pattern of burn. (Behrman, 171–178)

39. (D) Motor vehicle accidents, homicide, and sui-cide account for approximately 70% of alldeaths in this age group. Motor vehicle injurieslead the list of injury deaths at all ages duringchildhood and adolscence, even in childrenyounger than 1 year of age. Homicide is thethird leading cause of injury death in childrenages 1–4 years and second for the 15–19-year-old group. Suicide is the third leading cause ofdeath for 15–19-year olds. (McMillan, 9; MacDorman,2002(110):1037–1052, Behrman, 366–375)

40. (A) Ingestion of an overdose of salicylates,ethanol, or an oral hypoglycemic agent mayresult in symptomatic hypoglycemia. Closemonitoring and aggressive therapy with dex-trose is appropriate. (Fleisher, 968–970, 1174;Flomenbaum, 750,764–767)

41. (C) Compared to all other substances andfirearms, tobacco kills more individuals in theUnited States each year than all other causescombined. Among adults aged >25 years, 59%of all deaths in the United States result fromcardiovascular disease (36%) and cancer (23%).The leading causes of morbidity and mortality

among youth and adults in the United Statesare related to six categories of health-riskbehaviors: violence, tobacco use; alcohol andother drug use; sexual activity that contributeto unintended pregnancy and STDs, includingHIV infection; unhealthy dietary behaviors;and physical inactivity. These behaviors fre-quently are interrelated and are establishedduring childhood and adolescence and extendinto adulthood. The average smoker in theUnited States starts at age 12 years and mostare regular smokers by age 14 years.Adolsecent smokers may become nicotinedependent after smoking fewer cigarettes thanadults. Nicotine’s effect in the brain takes lessthan 20 seconds. While cigarettes are the mostcommon tobacco products used by adolescents,cigars and smokeless tobacco use is common(11% and 6% respectively). Cotinine is themajor metabolite of nicotine and can bedetected in urine. (Behrman, 829)

42. (A) This clinical scenario is classic for a tod-dler fracture, which is seen most often in chil-dren aged 9–36 months. History of injury maynot be elicited, and physical findings often aresubtle. Care and time must be taken in thephysical examination. Approximately one-fourthof the time, these fractures are not evident radi-ographically and must be diagnosed by phys-ical examination or bone scan, with confirmatoryx-ray revealing subperiosteal new bone 2–3 weekslater. (Fleisher, 1561–1562)

43. (G), 44. (H), 45. (F), 46. (B), 47. (D), 48. (C), 49.(I), 50. (A), 51. (D), 52. (E) There are specificantidotes for a number of toxic ingestions.These antidotes should be readily available inemergency room settings, and administered asearly as possible in the appropriate dose.Antidote administration should be used in con-junction with meticulous ongoing evaluation,management, and supportive care. (McMillan,752, Behrman, 343)

Figure 8-1(Courtesy of Mary Anne Jackson, MD)

Page 195: Questions - uhs.edu.kh You suspect Lyme disease, most likely contracted by which of the following? (A) ingestion of unripe fruit (B) ingestion of spoiled fruit (C) drinking of contaminated

196 8: Injuries, Poisoning, and Substance Abuse

Garry Gardner H and the Committee on Injury, Violence,and Poison Prevention. Office-based counseling for unin-tentional injury prevention. Pediatrics. 2007;119:202–206.

Hegenbarth MA and the Committee on Drugs. Preparingfor pediatric emergencies: Drugs to consider. Pediatrics.2008; 121:433–443.

Williams JF, Storck M, and the Committee on SubstanceAbuse, and Committee on Native American Child Health.Inhalant abuse. Pediatrics. 2007;119:1009–1017.

Brenner RA and Committee on Injury, Violence, andPoison Prevention., Prevention of drowning in infants,children, and adolescents. Pediatrics. 2003;112:440–445.

AAP Committee on Injury and Poison Prevention.Firearm-related injuries affecting the pediatric popula-tion (RE9926). Pediatrics. 2000;105:888–895.

AAP Committee on Sports Medicine and Fitness.Adolescents and anabolic steroids: A subject review(RE9720). Pediatrics. 1997;99:6.

Committee on Sports Medicine and Fitness. Use ofperformance-enhancing substances. Pediatrics. 2005;115:1103–1106.

CDC. Youth risk behavior surveillance—United States,2001. MMWR. 2002;51:6–7.

Hughes JM, Merson MH. Fish and shellfish poisoning.N Engl J Med. 1976;295:1117–1120.

Koch JJ. Performance-enhancing substances and their useamong adolescent athletes. Pediatr Rev. 2002;23:310–317.

McHugh MJ. The abuse of volatile substances. PediatrClin North Am. 1987;34:333–339.

Reece RM, Grodin MA. Recognition of nonaccidentalinjury. Pediatr Clin North Am. 1985;32:41–60.

Tobias JD. Pediatric Critical Care: The Essentials. Armonk,NY: Futura Publishing Co.; 1999.

Tong T, Boyer EW. Club drugs, smart drugs, raves andcircuit parties: An overview of the club scene. PediatrEmerg Care. 2002;18:216–218.

SELECTED READINGS

Page 196: Questions - uhs.edu.kh You suspect Lyme disease, most likely contracted by which of the following? (A) ingestion of unripe fruit (B) ingestion of spoiled fruit (C) drinking of contaminated

CHAPTER 9

Pediatric TherapeuticsJason W. Custer, MD

Kristine A. Parbuoni, PharmD, BCPS

R. Blaine Easley, MD

197

Page 197: Questions - uhs.edu.kh You suspect Lyme disease, most likely contracted by which of the following? (A) ingestion of unripe fruit (B) ingestion of spoiled fruit (C) drinking of contaminated

DIRECTIONS (Questions 1 through 73): For eachof the multiple choice questions in this sectionselect the one lettered answer that is the bestresponse in each case.

1. A 5-year-old, 15 kg girl presents to the emer-gency room 10 hours after ingesting 7.5 g ofacetaminophen. Plasma acetaminophen andhepatic enzymes are obtained and oral N-acetylcysteine is started. The patient is admittedand repeat acetaminophen levels and hepaticenzymes are followed. Peak AST levels are300 U/L, but return to normal. Her hospitalcourse is uneventful. Young children are lessprone to develop hepatic injury following acet-aminophen overdoses compared to adults dueto which of the following?

(A) Children are healthier with greater renalclearance of acetaminophen.

(B) Children have an active sulfate conjuga-tion pathway.

(C) Acetaminophen undergoes glucuronida-tion only in young pediatric patients.

(D) Glutathione stores are larger in adults.

2. A 24-month-old, 10 kg boy with a history ofepilepsy, develops a tonic-clonic seizure athome. EMTs arrive and are unable to get IVaccess. He has been seizing for 60 minutes whenyou are called. The EMTs have a vial of diazepam.Which route will achieve the safest and quickesttherapeutic serum diazepam level?

(A) administering the diazepam via intra-muscular route

(B) administering the diazepam viaintranasal

(C) administering the diazepam via oral/sublingual route

(D) administering the diazepam via rectalroute

(E) administering the diazepam via transcu-taneous route

3. An 8-year-old male with cystic fibrosis is admit-ted to the hospital with acute decline in pul-monary function and fever. He is started onceftazidime and tobramycin. This is his secondhospitalization this year. The pharmacy recom-mends tobramycin be dosed at 2.5 mg/kg IVevery 8 hours based on his pharmacokineticsfrom the last admission. Tobramycin levelsobtained appropriately after the third dose are:peak of 3 mg/L and trough of 1.8 mg/L. Thetiming of the draws is double checked anddetermined to be accurate. What is the mostlikely cause of the unexpected tobramycin level?

(A) The tobramycin dose is too low; he needshis tobramycin dose increased.

(B) The patient has a much larger volume ofdistribution than expected.

(C) The IV pump/system caused a delayeddelivery of the drug.

(D) The ceftazidime is interfering with thedrug level.

4. A 2-year-old comes into your clinic with a chiefcomplaint of a rash. You see multiple circularareas with a raised boarder and a scaling inte-rior on the upper arms and back. You also notea bald patch with a similar-looking lesion on thescalp. Which of the following medications isthe most appropriate treatment for this patient?

Questions

198

Page 198: Questions - uhs.edu.kh You suspect Lyme disease, most likely contracted by which of the following? (A) ingestion of unripe fruit (B) ingestion of spoiled fruit (C) drinking of contaminated

Questions: 1–9 199

(A) griseofulvin(B) ketoconazole cream(C) amoxicillin(D) fluconazole(E) triamcinolone cream

5. A 6-year-old male with a known history ofasthma presents to the emergency departmentwith the most severe asthma attack of his life.When you listen, you hear very little air move-ment and your patient has a declining level ofconsciousness. After your initial measures, yousee very little improvement and a decision ismade to intubate the patient’s trachea. Whichcombination of drugs is most appropriate in thissituation to prevent worsening bronchospasm?

(A) phenobarbital, atropine, and succinyl-choline

(B) pancuronium and lidocaine(C) ketamine, lidocaine, and rocuronium(D) versed, fentanyl, and pancuronium(E) perform endotracheal intubation

without medication

6. A 3-year-old girl presents to your clinic with5 days of rhinorrhea, cough, and congestion.Her mother has been using an over-the-countercough and cold remedy to help control thesymptoms. What is the most appropriate coun-seling to give this mother regarding over-the-counter medication usage in children?

(A) All over-the-counter medications are safe.(B) All over-the-counter medications

labeled pediatric or children’s are safe.(C) Many over-the-counter medications

have a combination of medications thatcan have serious side effects in children,always consult a physician for dosingand safety.

(D) It is very rare that over-the-counter coldmedications have acetaminophen as an ingredient.

(E) It is safe to give over-the-counter med-ications with acetaminophen as well asregular doses of acetaminophen if achild has a fever.

7. A 4-year-old with a history of renal failure,encopresis, and asthma is brought to the emer-gency room because of lethargy and difficultybreathing. She has been taking her usualbreathing treatment and has not had any res-piratory symptoms or fevers. She had recentlybeen severely constipated and her parents havebeen administering multiple enemas to try tohelp her stool with no result. Which of the fol-lowing is the most likely cause of her lethargyand respiratory distress?

(A) hyperphosphatemia secondary to enemas(B) albuterol overdose(C) sepsis(D) asthma exacerbation(E) anaphylaxis

8. An 11-year-old boy presents to the clinic com-plaining of itchy, watering eyes; stuffy nose;and a “tickle” in the back of his throat. He hadsimilar symptoms at this time last year. Hismother is concerned that he may be developingallergies. Which of the following allergy med-ications can cause tachycardia and hypertension?

(A) intranasal pseudoephedrine spray(B) diphenhydramine(C) intranasal steroid spray(D) oral decongestants (pseudoephedrine)(E) cromolyn sodium

9. You are in the delivery room for the birth of afull-term infant who is delivered by c-sectionfor fetal distress. The baby is limp, cyanotic,and not making respiratory effort when placedunder the warmer. The heart rate is found tobe less than 60 beats per minute. The decisionis made to place an endotracheal tube andmechanically ventilate the newborn. Which ofthe following medications can be safely admin-istered via endotracheal tube?

(A) calcium chloride(B) atropine(C) amiodarone(D) dopamine(E) phenobarbital

Page 199: Questions - uhs.edu.kh You suspect Lyme disease, most likely contracted by which of the following? (A) ingestion of unripe fruit (B) ingestion of spoiled fruit (C) drinking of contaminated

200 9: Pediatric Therapeutics

10. A 1-year-old presents to the emergency depart-ment with the chief complaint of teething. Younoticed in physical examination that the patienthas peri-oral cyanosis with an oxygen satura-tion of 98% on room air. There is no significantpast medical history. What is the most likelymedication that causes this condition?

(A) acetaminophen(B) ibuprofen(C) pseudoephedrine(D) orajel (topical benzocaine)(E) aspirin

11. A 4-year-old boy presents to the primary careclinic with a complaint of cough. The patienthas a history of wheezing with colds, but hasrecently had a persistent cough and has beentaking an albuterol MDI with some relief. Thepatient’s mother says that he has symptoms atleast 1 night out of the week and almost everyday. What is the most appropriate medicationregimen for this patient?

(A) albuterol as needed(B) over-the-counter cough medication(C) albuterol twice daily(D) inhaled corticosteroid twice daily plus

albuterol as needed(E) daily systemic corticosteroids

12. A 5-year-old presents to the emergency depart-ment with 6 days of fever, conjunctivitis, a“strawberry tongue,” swelling of the hands, adiffuse rash, and a large swelling that appearsto be a lymph node in the cervical area. Whichmedication should be included in the initialtreatment regimen for this disorder to lessenthe risk of complications?

(A) acetaminophen(B) IV antibiotics(C) intravenous immunoglobin (IVIG)(D) ibuprofen(E) corticosteroids

13. A 14-year-old presents to the emergency depart-ment after being involved in a car accident.He has a GCS of 15; however he has multiple

lacerations which require suturing. In which ofthe following locations is the use of a local anes-thetic with epinephrine contraindicated?

(A) shoulder(B) cheek(C) ear(D) neck(E) forehead

14. A 10-year-old with a history of asthma presentsto the emergency department in respiratory dis-tress. You auscultate diffuse wheezing and thepatient has increased work of breathing. Youinitiate albuterol via nebulizer, oral steroids,and establish IV access. The patient is showingsigns of worsening work of breathing, whichone of the following medications can act as anacute bronchodilator?

(A) magnesium sulfate(B) calcium chloride(C) sodium phosphate(D) potassium chloride(E) sodium bicarbonate

15. A 4-year-old with a seizure disorder presents tothe emergency department with increasedseizure frequency. Her medication regimen iscurrently phenytoin twice daily. She has beencompliant with her medication and has been onthe same dose for over 2 years. What is the nextbest step in her management?

(A) add Tegretol (carbamazepine) to herregimen

(B) add ethosuximide to her regimen(C) add phenobarbital to her regimen(D) measure a serum phenytoin level(E) perform an EEG to be sure that these are

seizures

16. A 5-year-old who has just started first gradepresents to your clinic with his mother. Hismother has a note from his teacher that yourpatient frequently misbehaves in class, gets upout of his seat, and bothers other children. Hismother says that she has a hard time control-ling his behavior at home. He is unable to sit

Page 200: Questions - uhs.edu.kh You suspect Lyme disease, most likely contracted by which of the following? (A) ingestion of unripe fruit (B) ingestion of spoiled fruit (C) drinking of contaminated

Questions: 10–21 201

still during your examination and is busy play-ing with magazines and toys when you try totalk to him. You suspect a diagnosis of ADHDfor this patient. Which of the following is themost appropriate medication to offer thispatient as a trial?

(A) methylphenidate(B) imipramine(C) haloperidol(D) paroxetine(E) phenytoin

17. You are caring for a now 5-day-old infant bornafter 28 weeks of gestation. On physical exam-ination you note bounding pulses and a harsh“washing machine” murmur. The patient hasalso had an increasing oxygen requirement.You order an echocardiogram and diagnose apatent ductus arteriosus. Which of the follow-ing therapies may correct this problem?

(A) opamine infusion(B) 100% oxygen administration(C) indomethacin(D) prostaglandin infusion(E) corticosteroids

18. A previously healthy 12-year-old presents tothe emergency department with nausea andvomiting for 2 days, there are other sick con-tacts at home. He has no meningeal signs. AnIV is placed and he is given IV fluids for rehy-dration. The vomiting continues and youdecide to give him an antiemetic. After themedication is given, the patient begins toslowly turn his head to the right and hasextreme arching of his back. He is awake andalert. What is the most likely cause of this reaction?

(A) hypocalcemia(B) tetanus(C) seizure disorder(D) metoclopramide administration(E) meningitis

19. A 9-year-old boy with Duchenne muscular dys-trophy presents to the pediatric emergencyroom with severe respiratory distress related topneumonia. The decision is made to intubatethe patient for respiratory failure. Which of thefollowing paralytics are contraindicated in thispatient?

(A) pancuronium(B) vecuronium(C) atracurium(D) succinylcholine(E) rocuronium

20. A 6-month-old infant with chronic lung dis-ease (CLD) on home oxygen and high-caloriefeeds presents with slow growth. You considerstarting a thiazide diuretic to decrease airwayresistance and improve lung compliance. Whichof the following electrolyte abnormalities mayresult from the use of thiazide-type diuretics?

(A) hypoglycemia(B) hyperuricemia(C) hyperkalemia(D) hypernatremia(E) hypocalciuria

21. A 2-year-old child presents from home with atemperature of 39°C and seizure activity. Theseizure lasts less than 15 minutes. Despite hermother’s aggressive antipyretic administrationduring the child’s febrile illnesses, this is thegirl’s fourth febrile seizure this year. Which ofthe following drugs can decrease the rate ofrecurrence of febrile seizures?

(A) carbamazepine(B) clonazepam(C) phenobarbital(D) phenytoin(E) felbamate

Page 201: Questions - uhs.edu.kh You suspect Lyme disease, most likely contracted by which of the following? (A) ingestion of unripe fruit (B) ingestion of spoiled fruit (C) drinking of contaminated

202 9: Pediatric Therapeutics

22. A 6-year-old is brought to your office becauseof “poor color”. At the visit, the child is other-wise appropriate, but slightly less active thannormal, and grayish in appearance. He admitsto drinking water from a pump-well he foundwith his older brother. You suspect nitrite poi-soning. Which of the following drug shouldyou administer for treatment of acute nitratepoisoning?

(A) hydralazine(B) morphine(C) methylene blue(D) corticosteroids(E) thiosulfate

23. A 12-year-old child with a surgically correctedpulmonary shunt is to undergo an invasivedental procedure. Which of the followingantibiotics would be most appropriate for pro-phylaxis against bacterial endocarditis?

(A) amoxicillin(B) clindamycin(C) erythromycin(D) oxacillin(E) vancomycin

24. A 4-year-old child has a history of developinga persistent cough that has endured the past3 weeks. There are multiple sick contacts in thehome with similar respiratory complaints. Yoususpect atypical pneumonia. Which of the fol-lowing is most appropriate for the treatment ofMycoplasma pneumoniae infection?

(A) cefuroxime(B) chloramphenicol(C) erythromycin(D) penicillin(E) tetracycline

25. You are caring for a 6-month-old infant fol-lowing aortic reconstruction in the intensivecare unit. You are called with a panic value thatcyanohemoglobin levels were detected on theinfant. You assess the infant and find heroral-tracheally intubated and mechanicallyventilated on numerous vasoactive drips and

sedatives. Cyanide is produced in the metabo-lism of which of the following?

(A) sodium nitroprusside(B) nitroglycerin(C) labetalol(D) dobutamine(E) milrinone

26. You have followed a 5-year-old for a series ofneurologic and behavioral issues. You referher to a pediatric neurologist for evaluation.After her evaluation, she returns to yourclinic for follow-up and is on ethosuximide.Ethosuximide (Zarontin) is most useful in thetreatment of which symptom?

(A) phenobarbital overdosage(B) absence (petit mal) seizures(C) akinetic seizures(D) tonic-clonic seizures(E) complex partial seizures

27. A 14-month-old male is admitted to the PICUfor bleeding from his nose and gums. There isno family history of bleeding tendency. On fur-ther history you determine the family has aproblem with mice and rats and have rodenti-cides around the house. Which of the followingshould be included in initial treatment?

(A) vitamin C(B) vitamin K(C) copper sulfate(D) a phenothiazine(E) atropine

28. A 14-year-old female presents to the emergencyroom 20 hours following ingestion of 8.5 g ofacetaminophen. Therapy with which of the fol-lowing should be initiated?

(A) deferoxamine(B) physostigmine(C) N-acetylcysteine(D) glutathione(E) no therapy is indicated

29. A 6-day-old male infant is brought to the emer-gency center with complaints of poor feeding

Page 202: Questions - uhs.edu.kh You suspect Lyme disease, most likely contracted by which of the following? (A) ingestion of unripe fruit (B) ingestion of spoiled fruit (C) drinking of contaminated

Questions: 22–34 203

and lethargy for 2 days. On physical examina-tion, the infant is hypotonic and respondspoorly to painful stimuli. The initial Dextrostixis 30 mg/dL. You administer 2 mL/kg of D25Wintravenously in addition to 20 mL/kg of normalsaline. Initial laboratory evaluation includessodium 128 meq/L, potassium 8.4 meq/L, bicar-bonate 8 meq/L, and chloride 104 meq/L.The next intervention should be which of thefollowing?

(A) obtain plasma for 17-OH progesteronelevel and administer 10 mg/kghydrocortisone

(B) start prostaglandin E(C) administer 3 mL/kg of hypertonic saline(D) obtain a urine metabolic screen(E) check serum ammonia and liver func-

tion tests

30. An 8-year-old with complex medical historyhas transferred care to your clinic. The child’scurrent medication list includes: albuterol, car-bamazepine, ranitidine, and methylphenidate.Methylphenidate is most commonly prescribedin the management of children with which ofthe following?

(A) poor appetites(B) temper tantrums(C) seizure disorders(D) breath-holding spells(E) attention-deficit hyperactivity disorders

31. A 6-month-old infant presents from day carewith fever, vomiting, and lethargy. On physicalexamination, you confirm the lethargy andnote a full fontanel despite signs of dehydra-tion. Which of the following would be the ini-tial antibiotic(s) of choice?

(A) cefuroxime and vancomycin(B) ceftriaxone and vancomycin(C) clindamycin(D) vancomycin(E) amoxicillin

32. A 3-day-old male was born at the hospital anddischarged home within 24 hours. The motherrelates that he has been a poor breast-feeder.His parents bring him to the emergency roombecause he has become very pale and listlessover a matter of hours. On examination, younote a lethargic newborn in shock with a largeliver and a gallop. Chest x-ray reveals increasedpulmonary vasculature and cardiomegaly. Youbegin antibiotic therapy but suspect congenitalcardiac disease. Which of the following is thedrug of choice?

(A) dobutamine(B) digoxin(C) prostaglandin E1

(D) milrinone(E) nicardipine

33. A 7-year-old male adopted from Romania pres-ents with hematemesis. He is known to havesevere chronic hepatitis. An upper endoscopyreveals esophageal variceal bleeding. Whichof the following agents is most likely to bebeneficial in controlling the esophageal bloodloss?

(A) octreotide(B) prostaglandin E1

(C) nitric oxide(D) prostacyclin(E) dexamethasone

34. A 15-year-old adolescent comes to spend thesummer with her aunt. She has an appoint-ment to be seen in your clinic because of herneed for frequent medical care. Prior to seeingthe patient you review her records and noticeshe is on oral pancreatic enzyme replacementtherapy. This is most helpful in patients withwhich of the following?

(A) protein-losing enteropathy(B) celiac disease(C) ulcerative colitis(D) alpha1-antitrypsin deficiency(E) cystic fibrosis

Page 203: Questions - uhs.edu.kh You suspect Lyme disease, most likely contracted by which of the following? (A) ingestion of unripe fruit (B) ingestion of spoiled fruit (C) drinking of contaminated

204 9: Pediatric Therapeutics

35. A 2-week-old neonate presents with lethargy,hypothermia, and poor feeding. Evaluation ofthe cerebrospinal fluid reveals a white bloodcell count of 1200 cells/mm3 (95% polymor-phonucleocytes) with an elevated protein anddecreased glucose. Which of the following isthe initial antibiotic regiment of choice?

(A) ampicillin(B) ceftriaxone(C) cefotaxime(D) cefuroxime with ampicillin(E) gentamicin with ampicillin

36. A 14-year-old undergoing consolidation ther-apy for high-risk, chronic lymphocytic leukemiadevelops burning and discomfort with urina-tion. Urine analysis demonstrates large amountof hemoglobin, along with red and white bloodcells. Urine culture is negative. You diagnosethe child with hemorrhagic cystitis. Hemorrhagiccystitis is associated most closely with largedoses of which of the following?

(A) cyclophosphamide(B) methotrexate(C) actinomycin D(D) L-Asparaginase(E) prednisone

37. A 2-month-old female presents with a 4-dayhistory of coughing spells that last at least30 seconds. After the coughing spells shevomits. She is otherwise well. Laboratory eval-uation reveals a peripheral white blood cellcount of 42,000/mm3 with 86% lymphocytes.Which of the following prophylaxis should thefamily and day-care contacts of this childreceive?

(A) rifampin(B) erythromycin(C) gentamicin(D) penicillin(E) trimethoprim-sulfamethoxazole

38. An 8-year-old male presents to your office withhis parents with the chief complaint of wettingthe bed. His parents say that he has never had

a period longer than 2 weeks in his life when hehas not wet the bed in the middle of the night.He does not want to go to sleepovers nowbecause he is embarrassed. He has tried cuttingdown on nighttime fluids and this has notmade a difference. His parents are looking fora solution. What therapy has been shown tohave the best success rate?

(A) behavioral conditioning (alarm/arousalsystem)

(B) DDAVP(C) imipramine(D) motivational therapy(E) furosemide

39. A 4-year-old presents to the emergency depart-ment with a chief complaint of swelling aroundthe eyes and ankles. You note the patient tohave pitting edema up to the knees bilaterallyand peri-orbital edema with no erythema. Youastutely send off a urine analysis and find thatthe patient has 3+ protein in the urine withoutsigns of infection or red blood cells. You makethe diagnosis of nephrotic syndrome, likelyminimal change disease. What is the next stepin this patient’s management?

(A) intravenous immunoglobulin (IVIG)(B) corticosteroids(C) kidney biopsy(D) cyclophosphamide(E) antibiotic therapy

40. A well-appearing, 10-year-old presents to theemergency department with a red rash on histrunk, ankles, elbows, and red lesions on histongue. You suspect that these look likepetechiae and order some laboratory testsincluding a complete blood count. When thelaboratory values come back, you see that theplatelet count is read as 5,000/µL. The patient’sblood type is A and he is Rh positive. You diag-nose the patient with idiopathic thrombocy-topenia purpura. What is the next step in yourmanagement?

(A) intravenous immunoglobulin (IVIG)(B) amoxicillin(C) anti-D immunoglobulin

Page 204: Questions - uhs.edu.kh You suspect Lyme disease, most likely contracted by which of the following? (A) ingestion of unripe fruit (B) ingestion of spoiled fruit (C) drinking of contaminated

Questions: 35–45 205

(D) platelet transfusion(E) observation

41. A 5-year-old with a history of sickle cell diseasepresents to the emergency department with a2-day history of cough and a 1-day history offever. You order blood work and a chest x-rayand she is found to have an infiltrate that isconsistent with pneumonia. What is the nextstep in your management?

(A) amoxicillin(B) ceftriaxone(C) azithromycin(D) ceftriaxone and azithromycin(E) no antibiotics needed, likely viral

42. A 12-month-old comes to the emergency depart-ment after having 4 days of upper respiratorytract symptoms and fever. Her mother has beengiving her acetaminophen every 4 hours and anover-the-counter cold and cough remedy sixtimes daily. She says today that the child hasbeen more sleepy and not as interactive. Sheshows you the bottle of cough and cold remedyand you notice that it also contains acetamino-phen. You suspect acetaminophen toxicity andthis is confirmed with a serum acetaminophenlevel in the toxic range, but liver enzymes arenormal. How should you proceed?

(A) contact poison control and administersteroids

(B) administer furosemide and arrange foradmission

(C) administer naloxone and monitor forimprovement

(D) contact poison control and administerN-acetylcysteine

(E) no therapy needed and observe

43. A 16-year-old presents to your office com-plaining of abdominal pain and vaginal dis-charge. She has been sexually active withmultiple partners and is concerned that shemay be pregnant. You perform a pregnancytest that is negative and during your pelvicexamination she is noted to have severe cervi-cal motion tenderness. You suspect a diagnosis

of pelvic inflammatory disease. Which of thefollowing is the most appropriate antibioticregiment for this patient?

(A) clindamycin(B) ceftriaxone(C) doxycycline(D) ceftriaxone plus doxycycline(E) metronidazole

44. A 15-year-old comes to your clinic complainingof frequent and heavy menstrual cycles. Shehad her first period at the age of 12, had someirregularity; however began having regular28 day cycles at the age of 13. For the last 4 monthsshe has been having bleeding on and off every1–2 weeks. She is not and has never been sex-ually active, has not had any vaginal discharge,has no abdominal pain, or other bleeding prob-lems. She is concerned and wants to better pre-dict her menstrual cycle. You tell her that this islikely dysfunctional uterine bleeding. Whatshould you recommend?

(A) testosterone cream(B) platelet infusion(C) oral contraceptive pill(D) nonsteroidal anti-inflammatory drug

(NSAID)(E) careful observation

45. You are called to the newborn nursery to see a3-day-old that has a rash. The parents are con-cerned because they did not notice the rash untilthis morning and are sure that it was not presentthe first 2 days of life. You find a well-appearingbaby who has scattered red papules all over thebody and are in clumps on the cheeks and on thetrunk, some of them even look like vesicles. Youtake a scraping of one of the vesicles to examineunder the microscope and find that these vesiclescontain a large number of eosinophils. What isthe recommended treatment for this patient?

(A) acyclovir(B) topical steroids(C) antidandruff shampoo(D) phototherapy(E) observation

Page 205: Questions - uhs.edu.kh You suspect Lyme disease, most likely contracted by which of the following? (A) ingestion of unripe fruit (B) ingestion of spoiled fruit (C) drinking of contaminated

206 9: Pediatric Therapeutics

46. A 14-month-old child is brought into the emer-gency room unresponsive. He is pale, hasdelayed capillary refill, and minimal respira-tory effort. Pulses are barely palpable. You arerequested to obtain vascular access. Accordingto current resuscitation guidelines, what shouldyour next step be?

(A) Attempt a central line.(B) Place an arterial line.(C) Attempt a peripheral IV catheter, if unsuc-

cessful place an intraosseous needle.(D) Administer intracardiac medications.(E) Attempt a central line, if unsuccessful

call the surgeon for a vascularcut-down.

47. A 24-month-old returns to your clinic aftertreatment for Streptococcal pharyngitis. Hismother complains that he continues to have anirregular gait after his penicillin shot. Youexamine the child and find he has a foot drop.The most likely cause of this finding is which ofthe following?

(A) reaction to penicillin(B) complication of intramuscular injection

into gluteal area(C) complication of intramuscular injection

into anterolateral thigh(D) complication of the bacterial infection(E) cerebral vascular accident

48. A 10-year-old boy with Hodgkin lymphoma isundergoing chemotherapy. The oncologistwants to start a new regimen but is worriedabout cardiac toxicity. Cardiomyopathy is mostlikely to be caused by which of the following?

(A) bleomycin(B) doxorubicin(C) cytarabine(D) etoposide(E) cisplatin

49. A 16-year-old girl with acute lymphocyticleukemia nearing the end of induction therapypresents with tingling in her fingers. Peripheralneuropathy is the most common adverse effectof which chemotherapeutic agent?

(A) methotrexate(B) cyclophosphamide(C) cytarabine(D) procarbazine(E) vincristine

50. A 10-year-old girl with acute myeloid leukemia(AML), now day 75 status post a matched,unrelated bone marrow transplant presentswith voluminous diarrhea, nausea, and vomit-ing. Acute graft-versus-host disease is suspectedand high-dose corticosteroids are initiated. Side-effects of corticosteroids most commonlyinclude which of the following?

(A) peripheral obesity(B) gigantism(C) cataracts(D) hypoglycemia(E) hypotension

51. A 6-year-old girl is brought to the emergencyroom with head injuries after being hit by acar. Her heart rate is 42 beats per minute, bloodpressure is 160/100 mm Hg, respiratory rate is8 breaths per minute, and she is minimallyresponsive. The left pupil is larger than theright and minimally reactive to light. The deci-sion is made to perform a rapid sequence intu-bation since she is unable to protect her airway.Which of the following sedative agents couldcause worsening of her condition?

(A) thiopental(B) propofol(C) fentanyl(D) etomidate(E) ketamine

52. A full-term baby became cyanotic 5 hours afterbirth. The baby was found to have persistentpulmonary hypertension of the newborn(PPHN) and is now being managed in theNICU on a mechanical ventilator. The baby’scondition is worsening, which of the followinginterventions is indicated?

(A) prostacyclin(B) arachidonic acid

Page 206: Questions - uhs.edu.kh You suspect Lyme disease, most likely contracted by which of the following? (A) ingestion of unripe fruit (B) ingestion of spoiled fruit (C) drinking of contaminated

Questions: 46–57 207

(C) leukotriene C(D) nitric oxide(E) adenosine

53. A 10-year-old boy presents to the emergencydepartment with a 4-day history of fever, runnynose, vomiting, and diarrhea. His mother hasbeen giving him aspirin (325 mg) two tabletsevery 4–6 hours for the fever and Pepto Bismol(175 mg/15 mL salicylate), one tablespoonevery 4–6 hours for the upset stomach. He islethargic, diaphoretic, and febrile to 102.4°F,BP is 100/60, HR is 120, and RR 40. Arterialblood gas testing shows a pH of 7.44, PCO214 mm Hg, PO2 93 mm Hg. Giving sodiumbicarbonate at this time would have what primaryeffects?

(A) worsen his acid-base status(B) prevent cardiac arrhythmias(C) worsen his hydration status(D) prevent seizures(E) prevent absorption of salicylates

54. A 16-year-old male with sickle cell anemia pres-ents to clinic for evaluation of his pain man-agement. He is having acute pain on top of hischronic pain. Which of the following opioids isavailable as a long-acting and short-acting oralformulation as well as intravenous?

(A) fentanyl(B) morphine(C) oxycodone(D) meperidine(E) methadone

55. A 3-year-old girl is admitted to the burn unitfollowing an immersion hot water injury withresultant second- and third-degree burns cov-ering 60% of the body surface area. Fluids arestarted at maintenance plus 4 mL/kg/%BSAburned according to the Parkland formula withhalf of the fluid administered over the first8 hours and the remainder over the following16 hours. A Foley catheter is placed to monitorurine output. Six hours after admission, the

urine output has decreased to 0.5 mL/kg/h forthe past 2 hours. The most appropriate therapywould be which of the following?

(A) continue to observe the urine output(B) furosemide 1 mg/kg intravenously(C) furosemide 2 mg/kg orally(D) mannitol 0.5 g/kg intravenously(E) fluid bolus of 20 mL/kg of normal

saline

56. A 2-day-old, 1.9-kg boy was delivered to a19-year-old mother at 36 weeks gestation, withno prenatal care. Upon preparing the infant fordischarge, the patient was noted to be tachyp-neic, with weak peripheral pulses and grayishcyanosis of the lips. Oxygen saturations werenoted to be 85% and declining. Chest x-rayshows cardiomegaly and an echocardiogramis ordered which reveals the presence ofhypoplastic left heart syndrome (HLHS).Considering the patient’s present condition,which one of the following is the best initialintervention that should be taken?

(A) give indomethacin(B) consult a pediatric cardiologist for surgi-

cal options(C) start prostaglandin E1

(D) start phenylephrine(E) give oxygen at 100%

57. A 10-year-old child presents to the emergencydepartment with an erythematous rash on histhigh, fever, and myalgia 2 weeks after huntingwith his father in eastern Pennsylvania. Therash appeared about 4 days prior to presenta-tion and has been gradually increasing in size.He thinks he had a bug bite in the same spot ofthe rash. The initial treatment of choice is whichof the following?

(A) streptomycin(B) azithromycin(C) clindamycin(D) vancomycin(E) doxycycline

Page 207: Questions - uhs.edu.kh You suspect Lyme disease, most likely contracted by which of the following? (A) ingestion of unripe fruit (B) ingestion of spoiled fruit (C) drinking of contaminated

208 9: Pediatric Therapeutics

58. A 16-year-old male presents with fever, chills,nausea, vomiting, and headache. He was previ-ously healthy, but over the last 24 hours he hasfelt awful and states he has been unable to con-centrate for his exams. He also has a petechialrash on his trunk. Gram stain of CSF fluid showsgram-negative diplococcus. Which of the fol-lowing is the drug of choice for the treatment ofmeningococcal meningitis?

(A) cefotaxime(B) ceftriaxone(C) vancomycin(D) chloramphenicol(E) penicillin

59. A3-year-old female was found with her mother’sprenatal vitamins. The bottle was empty andwas estimated to contain 20–25 tablets. At4 hours postingestion, her serum iron level is550 mg/dL. Which of the following should beadministered?

(A) dimercaprol (BAL)(B) deferoxamine (desferal)(C) edetate calcium disodium (EDTA)(D) ipecac(E) activated charcoal

60. A 10-year-old male is admitted to the hospitalwith a 4-day history of nausea, vomiting, andabdominal pain. The pain is located in theepigastrium and radiates to the back. He is cur-rently completing a course of azithromycin for asinusitis diagnosed 3 days prior. Home medica-tions include acetaminophen, metoclopramide,ranitidine, valproic acid, fluticasone nasal spray,and loratadine. The drug most likely to causethese symptoms is which of the following?

(A) acetaminophen(B) ranitidine(C) azithromycin(D) valproic acid(E) loratadine

61. An adolescent male presents with a urethraldischarge. Gram stain of the exudate revealsintracellular gram-negative diplococci. The initialtreatment of choice is which of the following?

(A) azithromycin, 1 g orally once(B) doxycycline, 100 mg twice a day for 7 days(C) procaine penicillin G, 1.2 million units

IM once(D) benzathine penicillin G, 2.4 million units

IM once(E) ceftriaxone, 125 mg IM once

62. A 16-year-old girl presents to the emergencydepartment 12 hours after ingesting 30 extrastrength Tylenol (acetominophen) tablets. Sheis vomiting, diaphoretic, and lethargic. Whichof the following statements is true regardingthe toxicity of acetaminophen overdose?

(A) primarily involves the liver(B) primarily involves the central nervous

system(C) is most severe in patients less than

2 years of age(D) is best treated with sodium bicarbonate

to alkalinize the urine(E) is best treated with dialysis

63. Aneonate is receiving intravenous ampicillin andgentamicin for sepsis and possible meningitis. Ablood culture reveals group B streptococcus. Theantibiotic of choice is which of the following?

(A) clindamycin(B) penicillin(C) cefuroxime(D) vancomycin(E) tobramycin

Questions 64 and 65

64. A 14-year-old girl is admitted to the emergencydepartment with an acute change in mentalstatus. She recently had a fight with her parentsand threatened to kill herself. Her HR is 130, BPis 106/68, and T 37°C. She is placed on a cardiacmonitor and an electrocardiogram is obtainedwhich shows prolongation of the QRS interval.Which of the following agents was most likelyingested by this patient?

(A) nortriptyline(B) phenobarbital(C) acetaminophen

Page 208: Questions - uhs.edu.kh You suspect Lyme disease, most likely contracted by which of the following? (A) ingestion of unripe fruit (B) ingestion of spoiled fruit (C) drinking of contaminated

Questions: 58–71 209

(D) cocaine(E) phencyclidine

65. The patient was given activated charcoal and afluid bolus with normal saline. Minutes later,the QRS complex abruptly widens to 0.16 sec-onds, BP 86/50, and HR 140. Treatment shouldinclude the immediate administration of whichof the following?

(A) N-acetylcysteine(B) sodium bicarbonate(C) sodium nitroprusside(D) amyl nitrite(E) lidocaine

66. A 14-year-old patient with acute lymphocyticleukemia presents with fever to 39.6°C. Shewas most recently discharged 1 week ago aftercompleting a round of chemotherapy. Bloodculture reveals a nonlactose fermenting gram-negative rod, which is oxidase positive. Whichof the following is the appropriate antibiotic?

(A) levofloxacin(B) azithromycin(C) trimethoprim-sulfamethoxazole(D) piperacillin-tazobactam(E) ceftriaxone

67. A 2-year-old male presents to the emergencydepartment with fever to 39.1°C, and abnormalmental status for the past 12 hours. Laboratoryresults show a WBC of 567,000, hemoglobin3.2, and LDH 1197. Acute lymphoblasticleukemia is suspected and the oncologist asksyou to initiate allopurinol therapy. Allopurinolis used most commonly to manage what inacute leukemia of childhood?

(A) to induce remission(B) to maintain remission(C) to help alkalinize the urine(D) to prevent vomiting from chemotherapy(E) to prevent hyperuricemia associated

with tumor lysis syndrome

68. A 4-year-old child presents with profusewatery diarrhea which is positive for occultblood. She complains of abdominal pain and

cramping. She is febrile to 38.8°C and her WBCis 19,000. She is currently completing a 7-daycourse of amoxicillin-clavulanate for an earinfection. Which of the following is the mostappropriate therapy at this time would be

(A) IV vancomycin(B) PO vancomycin(C) IV metronidazole(D) PO metronidazole(E) PO clindamycin

69. A15-year-old boy with central core disease and ahistory of malignant hyperthermia crises is under-going surgery. He would benefit from whichagent during a malignant hyperthermia crisis?

(A) ethanol(B) succinylcholine(C) halothane(D) potassium(E) dantrolene

Questions 70 and 71

Gentamicin and piperacillin-tazobactam is initiated inan 8 year old with acute myeloblastic leukemia for dualcoverage of Pseudomonas aeruginosa bacteremia.Gentamicin levels will be checked to ensure that thepeaks are high enough to kill the bacteria relative to theMIC (peak concentration to MIC or peak/MIC ratio).

70. Which of these drugs is concentration-dependentlike the aminoglycosides?

(A) doxycycline(B) ciprofloxacin(C) vancomycin(D) cefdinir(E) erythromycin

71. For the above patient, the appropriate time tocheck serum gentamicin levels at steady statewould be after how many half-lives?

(A) one(B) two(C) three(D) five(E) seven

Page 209: Questions - uhs.edu.kh You suspect Lyme disease, most likely contracted by which of the following? (A) ingestion of unripe fruit (B) ingestion of spoiled fruit (C) drinking of contaminated

210 9: Pediatric Therapeutics

72. A 33-week, 1200 g boy is born to a teenagemother who had poor prenatal care. The baby’shead circumference, height, and weight arebelow average for his age, indicating poorintrauterine growth, likely due to exposure todrugs during pregnancy. Intrauterine growthretardation (IUGR) is a common adverse effectof which drug if taken by the mother duringpregnancy?

(A) warfarin(B) acetaminophen(C) lithium(D) lisinopril(E) phenytoin

73. An 8-year-old girl with uncontrolled asthmais receiving inhaled corticosteroids and beta-agonists (Advair, fluticasone/salmeterol) aswell as a leukotriene antagonist (Singulair,montelukast). The decision is made to begintheophylline; however you are worried aboutthe potential toxicities. What side effects shouldyou monitor for that would indicate high levelsof theophylline?

(A) nystagmus(B) myopathy(C) polyuria(D) rash(E) vomiting

DIRECTIONS (Questions 74 through 100): Thefollowing group of questions is preceded by a listof lettered answer options. For each question,match the one lettered option that is most closelyassociated with the question. Each lettered optionmay be selected once, multiple times, or not at all.

Questions 74 through 78

(A) dopamine(B) epinephrine(C) isoproterenol(D) norepinephrine(E) dobutamine(F) milrinone

74. Indicated for septic and spinal shock

75. Has most significant impact on metabolism;possesses β1 and β2 effects

76. May lower systemic vascular resistance andpulmonary vascular resistance

77. Increases cardiac output without increasingheart rate or systemic vascular resistance

78. Has greatest effect on increasing heart rate

Questions 79 through 84

(A) albuterol(B) cromolyn sodium(C) theophylline(D) corticosteroids(E) epinephrine (racemic)(F) ipratropium bromide(G) caffeine

79. Reduces airway edema rapidly, with risk ofpotential recurrence of symptoms

80. Mast cell stabilizer

81. Anticholinergic agent used for airway smoothmuscle relaxation

82. Reduces upper and lower airway inflammation

83. Narrow therapeutic window

84. May be given by inhalation, orally, intra-venously, and intramuscularly

Questions 85 through 88

(A) µ1 (mu1)(B) µ2 (mu2)(C) κ (kappa)(D) δ (delta)(E) σ (sigma)

Page 210: Questions - uhs.edu.kh You suspect Lyme disease, most likely contracted by which of the following? (A) ingestion of unripe fruit (B) ingestion of spoiled fruit (C) drinking of contaminated

Questions: 72–96 211

85. Sedation, miosis

86. Analgesia

87. Hypertonia, dysphoria

88. Physical dependence

Questions 89 through 91

In a trial evaluating the pharmacology of a newdrug in children, the drug is administered in threedifferent fashions and the serum concentration ofthe drug is measured at different time points. Thearea in gray represents the minimal effective con-centration of the drug in the serum. Match theresulting serum concentration curves (labeled A, B,and C) with the most likely method of administration(see Fig. 9-1).

89. Continuous intravenous infusion

90. Intermittent intravenous dosing

91. Single intravenous bolus dose

Questions 92 through 93

A 15-year-old male is found unresponsive in hisbedroom. The child is intubated but remains mini-mally responsive. He is admitted directly to the inten-sive care unit and toxicology screens demonstratealcohol and benzodiazepines. No other medicationsare found. If repeat serum tests are performed onthese two agents, the illustrated elimination curveswould result. Match the agents to their respectiveelimination kinetics (see Fig. 9-2).

(A) first order(B) zero order

92. Ethanol is eliminated following ___________kinetics.

93. Diazepam is eliminated following ___________kinetics.

Questions 94 through 96

On the following graph, (A) represents the dose-response of a drug acting alone, and (B) depicts thedose-response relationship of the same drug in thepresence of a competitive antagonist (see Fig 9-3).

94. If the amount of the competitive antagonist isincreased, which is the resulting dose-responsecurve?

95. Which curve illustrates a dose-response curveof lower efficacy than B?

96. Which curve illustrates a dose-response curveof greater potency than B?

Time

Ser

um c

once

ntra

tion

of d

rug A

B

C

Time (hours)0 2 4 6

Ser

um c

once

ntra

tion Ethanol

Diazepam

A B C

D

100

75

50

25

1 10 100Log Dose (mg)

% E

ffect

Figure 9-1

Figure 9-2

Figure 9-3

Page 211: Questions - uhs.edu.kh You suspect Lyme disease, most likely contracted by which of the following? (A) ingestion of unripe fruit (B) ingestion of spoiled fruit (C) drinking of contaminated

212 9: Pediatric Therapeutics

Questions 97 through 100

Patients A and B are receiving the same medicationat the same dosing interval, and have their serumlevels of the drug monitored at the same time inter-vals. The drug is hepatically metabolized, and thenactive metabolites are cleared renally. Based on thegraph of their serum levels following repetitive dos-ing of the medication, match the patients with themost likely clinical scenarios (see Fig 9-4).

97. This patient has renal insufficiency.

98. This patient has elevated cytochrome P-450activity.

99. This patient demonstrates cumulative drugproperties.

100. This patient would benefit in an increase inboth the dose amount and frequency of drugadministration.

12

10

8

6

4

2

00 1 2 3 4 5 6 7 8 9

Pla

sma

conc

entr

atio

n

Dose

Toxic range A

B

Therapeutic range

Subtherapeutic

Figure 9-4

Page 212: Questions - uhs.edu.kh You suspect Lyme disease, most likely contracted by which of the following? (A) ingestion of unripe fruit (B) ingestion of spoiled fruit (C) drinking of contaminated

213

1. (B) Children younger than 6 years of age areunlikely to develop significant toxicity after asingle ingestion of even relatively large doses ofacetaminophen. Although prepubescent chil-dren have relatively high hepatic, mixed func-tion oxidase (MFO) activity, they also exhibit agreater capacity than adults to detoxify aceta-minophen by phase II metabolic reactions, pri-marily sulfate conjugation. In addition, childrenpossess higher glutathione turnover whichmay contribute to protection from hepatotoxicity.Thus, the lower susceptibility of children toacetaminophen poisoning is due to their greatercapacity to eliminate the drug by nontoxic path-ways. Nevertheless, children with a significantingestion should have their plasma acetamino-phen level measured and receive treatment withN-acetylcysteine (NAC) if the level falls withinthe toxic range on the nomogram. Notably, ado-lescents have a higher incidence of toxic plasmalevels after ingestion than do younger children.Even if a serious case of hepatotoxicity devel-ops, the mortality rate is less than 0.5%. Childrenwho recover have no sequelae when observed3–12 months after the acute toxicity; however,severely affected patients may require livertransplantation. (McMillan, 760–761)

2. (D) Alternative routes for drug administrationare required in children more often than adults.Routes to consider are oral, intramuscular (IM),transdermal, and rectal administration. If anIV line cannot be established or the child issome distance from a medical center, rectaldiazepam or lorazepam can be used safely.IM midazolam is as effective as IV diazepaminitially or refractory. Diazepam diluted in3 mL 0.9% NaCl is placed into the rectum by

a syringe and a flexible tube at a dose of0.3–0.5 mg/kg. The effective dose of rectallorazepam is 0.05–0.1 mg/kg. Therapeuticserum levels occur within 5–10 minutes. IMadministration of diazepam is an option, but isfelt to be less efficacious than rectal dosing.(McMillan, 703, 2296; Brunton, 4–7)

3. (C) Prolonged administration (from an infu-sion error) resulted in the abnormally low peakof this scenario. Several steps can be taken tominimize problems with intravenous drugadministration to infants and children. Theseinclude the following: standardization and doc-umentation of the total administration time;documentation of the volume and content ofthe solution used to “flush” an intravenousdose; standardization of specific infusion tech-niques (infusion duration, volumes) for drugswith a narrow therapeutic index; standardizationof dilution and infusion volumes for drugs givenby intermittent IV injection; avoidance of attach-ing lines for drug infusion to a central hub withother solutions infused concurrently at widelydisparate rates; preferential use of large-gaugecannula; maintenance of the recommended solu-tion at a specified height above the infusion sitefor use with a gravity-based controller; and theuse of low-volume tubing and the most distal sitesfor access of the drug into an existing intravenousline. (Behrman, 333–336; Brunton, 4–7)

4. (A) The case describes a child with tinea corporisand tinea capitus. These infections are caused bydermatophytes of the genera Trichophyton andMicrosporum. These are infections of the dermisthat can invade hair follicles when found inhair-containing areas such as the scalp. The typical

Answers and Explanations

Page 213: Questions - uhs.edu.kh You suspect Lyme disease, most likely contracted by which of the following? (A) ingestion of unripe fruit (B) ingestion of spoiled fruit (C) drinking of contaminated

214 9: Pediatric Therapeutics

presentation is a circular rash with raised bor-ders and a scaling interior. This diagnosis can beconfirmed by a potassium hydroxide testing ofa scraping of the lesion. Tinea capitus needs tobe treated with a systemic antifungal—mostcommonly griseofulvin is used (A). In tinea capitus(scalp infection), the fungus invades the hairfollicle and topical creams will not effectivelytreat the organism (B, D, E). Tinea corporis (bodyinfection) can be treated with topical antifungalcreams. Amoxicillin (C) is an antibiotic effectiveagainst bacterial infections and will not treatfungal infection. (McMillan, 870–872)

5. (C) Patients with severe asthma exacerbationsoften need admission to the hospital for admin-istration of medications and supportive care. Invery rare circumstances a patient with very severeasthma and poor air exchange will becomeobtunded and will need endotracheal intubation.The medications used to facilitate intubation inthis scenario have two additional goals overroutine intubation: first, it should not worsen thebronchospasm; and second, it should allow thephysician/patient to ventilate once the endotra-cheal tube is placed as soon as possible. Ketamineand lidocaine can be used to prevent or reducebronchoconstriction. Ketamine has been shownto have direct bronchodilatory effects, and lido-caine blunts the airway responsiveness, such asthe coughing reflex. Rocuronium is one of thenondepolarizing muscle relaxants that does notcause bronchospasm and is indicated in rapidsequence intubation. The other medication com-binations listed are not ideal because they do notincorporate medications that help to reducebronchospasm and to achieve rapid return tospontaneous ventilation (A, B, D). Intubatingwithout medication (E) in this case would beinappropriate. (McMillan, 2682; Brunton, 346–375)

6. (C) More than 800 cough and cold remediesare available over the counter in the UnitedStates. The majority of these medications con-tain some form of antihistamine, decongestant,antitussive, expectorants, and analgesics.A review article of the use of over-the-countermedications from 1950–1991 showed no goodevidence for the effectiveness of over-the-countermedications for preschool-age children. A 2000

report from the American Association of PoisonControl Centers showed that 5% of poisonousexposures to children under 6 years of age wererelated to cough and cold preparation exposure.Parents should always be counseled to consulta physician for dosing and safety of any over-the-counter medication (C). All over-the-countermedications whether or not they are labeledpediatric or children’s possess some risk, thereforethey are not safe (A, B). Most over-the-countermedications do contain acetaminophen (D). It isnot safe to give over-the-counter medicationscontaining acetaminophen, along with additionalacetaminophen; there is risk for an overdose (E).(McMillan, 764; Kelly, 25(4):115–123)

7. (A) A Pediatric Fleet Enema is a hypertonicsolution with high levels of phosphate. If theseenemas are retained or are given in quick suc-cession, there is a chance of inducing hyper-phosphatemia, especially in conjunction withrenal failure, although this can happen evenwithout renal insufficiency. Hyperphosphatemiaoften manifests with the signs of an inducedhypocalcemia—which are lethargy, tetany, andoccasionally seizures. An albuterol overdosewould most likely present as tachycardia, sweat-ing, and tachypnea (B). Sepsis would likely pres-ent with fever, hypotension, and tachycardia (C).An asthma exacerbation would present withtachypnea and wheezing (D). Anaphylaxiswould present with hypotension, wheezing,hives, and difficulty breathing (E). (McMillan,672–673;Maraffa, 20(7):453–456.)

8. (D) Many allergy medications use deconges-tants which are often a form of an adrenergicagonist. Pseudoephedrine acts to release nor-epinephrine and causes a sympathetic surge.One of the useful effects of this as a deconges-tant is that it causes constriction of blood ves-sels and decreases edema by allowing less bloodflow to the lining of the nose and sinuses. Thesystemic side effects manifest as tachycardia,restlessness, and hypertension (D). Intranasalpseudoephedrine will act locally, but doesnot have the degree of systemic action of oralpseudoephedrine. Diphenhydramine or Benadrylis an antihistamine. The most common sideeffect is drowsiness (B). Intranasal steroid sprays

Page 214: Questions - uhs.edu.kh You suspect Lyme disease, most likely contracted by which of the following? (A) ingestion of unripe fruit (B) ingestion of spoiled fruit (C) drinking of contaminated

Answers: 5–13 215

have limited systemic side effects (C). Cromolynis a mast cell stabilizer that blocks the release ofhistamine. Adverse effects are typically GI upsetand throat irritation (E). (McMillan, 2428–2432)

9. (B) Atropine is one of the medications that canbe delivered via endotracheal tube. It is an anti-cholinergic used to block vagal stimulation tothe heart and raise the heart rate in bradycardicneonates. Other medications that can be admin-istered via endotracheal tube are lidocaine, epi-nephrine, atropine and naloxone—the acronymLEAN can be used to remember these medications.Recently, vasopressin has been added as anotheragent that may be given via the endotrachealtube during cardiac arrest. This route can be aneffective way to administer medication beforeIV access is obtained in any setting. Calciumchloride, amiodarone, dopamine, and phenobar-bital should not be administered via endotrachealtube (A, C, D, E). (McMillan, 208–210)

10. (D) Topical benzocaine has been associatedwith methemoglobinemia. Methemoglobin isan oxidized form of hemoglobin that is notcapable of carrying oxygen. This can manifestas cyanosis without showing a drop in thepulse oximeter. This state can be reversed byadministering methylene blue. Acetaminophenoverdose can manifest as hepatic dysfunction,not with cyanosis (A). Ibuprofen overdose canmanifest with platelet dysfunction and renalimpairment, not cyanosis (B). Pseudoephedrinehas the side effect of tachycardia and hyper-tension (C). Aspirin side effects are metabolicacidosis and ringing in the ears (tinnitus).(McMillan, 2696; Dahshan, 117:806–809)

11. (D) The case describes a patient with mildpersistent asthma. Asthma severity is classi-fied by frequency of symptoms during the dayand symptoms at night.

The treatment of mild persistent asthma isthe initiation of a controller medication plusthe use of albuterol as needed––the most effectivecontroller is a low dose inhaled corticosteroidtwice daily (D). Albuterol alone will not controlthe persistent nature of this patient’s symptoms(A). Over the counter cough medication doesnot address the root of the problems and symp-toms will persist (B). Albuterol twice dailymay help with symptoms, but albuterol shouldbe given on an as needed basis and a controllermedication given twice daily (C). Systemicsteroids daily are used for acute exacerbationsof asthma for a short course and for severepersistent asthma as a long term therapy (E).(McMillan, 2407)

12. (C) The above case describes mucocutaneouslymph node syndrome or Kawasaki disease.This is a disease of unknown etiology whichhas a set of diagnostic criteria that need to bemet to make the diagnosis. The presence ofhigh spiking fever for 5 days plus at least fouror five of the following: (1) bilateral bulbar con-junctival injection; (2) mucous membranechanges; (3) extremity changes—erythema,edema, desquamation; (4) polymorphousrash—primarily truncal; (5) single large cervi-cal lymph node. The treatment of choice afterrecognition is a combination of IVIG and high-dose aspirin. This treatment has been shown todecrease the complication rate of coronaryartery aneurysm if administered during thefirst 10 days of fever (C). Acetaminophen andibuprofen will lower the fever, but will notlessen the complication rate (A, D). IV antibi-otics have no role in the treatment of Kawasakidisease (B). Steroids are generally not consid-ered part of the initial treatment for Kawasakidisease (E). (McMillan, 1015–1020)

13. (C) The use of local anesthetic with epinephrineis contraindicated in areas that have an end-organ arterial supply. These areas include theear, tip of the nose, distal extremities, and thedigits. The epinephrine will cause vasocon-striction and will allow the local anesthetic towork more effectively and longer, howevervasoconstriction in an end-organ vascular bedcan cause ischemia. The shoulder, cheek, neck,

TABLE 9-1.

Classification Days with Nights withSymptoms Symptoms

Mild intermittent ″ 2/week ″ 2/monthMild persistent 3-6/week 3-4/monthModerate persistent Daily ≥ 5/monthSevere persistent Continuous Frequent

Page 215: Questions - uhs.edu.kh You suspect Lyme disease, most likely contracted by which of the following? (A) ingestion of unripe fruit (B) ingestion of spoiled fruit (C) drinking of contaminated

216 9: Pediatric Therapeutics

and forehead would be appropriate places touse local anesthetic mixed with epinephrine(A, B, D, E). (McMillan, 2672)

14. (A) Magnesium sulfate is used as a smoothmuscle relaxant and can be used to help controlbronchospasm in severe asthma and causevasodilatation (A). Calcium chloride, sodiumphosphate, potassium chloride, and sodiumbicarbonate do not have any bronchodilatingproperties (B, C, D, E). (McMillan, 699)

15. (D) Children with epilepsy often will outgrowtheir seizure disorder and be medication free inlater life. Some patients have a progressive neu-rologic disorder that will require escalating med-ication doses and the addition of medications. Ina child with increasing seizure frequency who ison a medication, it is important to check theserum drug level to be sure that it is in a thera-peutic range. If the medication is subtherapeutic,it may indicate noncompliance with the med-ication or that the dose needs to be increaseddue to increasing volume of distribution orchanges in metabolism (D). Adding a secondagent at this time would not be the next best stepin the management before knowing if the pheny-toin level is normal (A, B, C). Performing an EEGis unlikely to be helpful because you alreadyknow that the patient has a seizure disorder andthe child’s clinical presentation is consistent withseizures (E). (McMillan, 2288)

16. (A) The diagnosis of ADHD is made by observ-ing the symptoms of impulsivity and hyperac-tivity in more than one setting. This childclearly exhibits these behaviors in school and inthe home. Occasionally it is worth a trial of astimulant medication to see if this helps to con-trol these behaviors and allows the child to per-form better in the school setting. Stimulantsare reported to be effective in 70% of individu-als with ADHD. Methylphenidate is one stim-ulant medication that is commonly prescribed.Common side effects include decreasedappetite and decreased sleep (A). Imipramineis a tricyclic antidepressant (TCA) and paroxe-tine is a selective serotonin reuptake inhibitor(SSRI), both used to treat major depressive dis-order (B, D). Haloperidol is an antipsychotic

used to treat acute psychosis (C). Phenytoin isan antiepileptic used to treat seizure disorders (E).(McMillan, 674–680)

17. (C)Apatent ductus arteriosus is a common abnor-mality seen in premature infants secondary tohypoxia, acidosis, and increased pulmonary pres-sure from significant lung disease. The ductusarteriosus should close within the first 2–3 days oflife, when this does not occur properly the patientcan exhibit signs of pulmonary edema and heartfailure. One medical option for assisting theductus to close is the initiation of indomethacin.Indomethacin inhibits prostaglandin synthesisand allows the ductus to close. Repeat coursesmay be needed. If this is ineffective, surgical cor-rection may be needed. A dopamine infusionwould be needed if the patient were hypoten-sive, and is not appropriate in this case (A).Administration of 100% oxygen may cause a low-ering of the pulmonary vascular resistance andcause more pulmonary blood flow, worseningpulmonary edema (B). A prostaglandin infusionis used to keep a ductus arteriosus open in thecase of cyanotic heart disease (D). Corticosteroidshave no role in the closure of a patent ductusarteriosus (E). (McMillan, 226)

18. (D) This is most likely a dystonic reactioncaused by the antiemetic, metoclopramide.This is an uncommon side effect of metoclo-pramide administration. Dystonia often mani-fests with head turning (acute torticollis) andsevere back arching (opisthotonos). Acute dys-tonia should be treated with IV diphenhy-dramine or benztropine. This is unlikely to behypocalcemia or tetany, given the time rela-tionship to the medication administration (A, B).This is unlikely to be a seizure because he isawake and alert during the episode and hasno mention of a seizure history (D). This isunlikely to be meningitis; the child is awakeand alert and does not show meningeal signs (E).(McMillan, 2372–2373)

19. (D) Neuromuscular blocking agents are classi-fied as either nondepolarizing or depolarizingagents. Nondepolarizing agents act as com-petitive antagonists for acetylcholine at theacetylcholine receptor of the neuromuscular

Page 216: Questions - uhs.edu.kh You suspect Lyme disease, most likely contracted by which of the following? (A) ingestion of unripe fruit (B) ingestion of spoiled fruit (C) drinking of contaminated

Answers: 14–25 217

junction. By blocking the receptor without stim-ulating it, they stop neuromuscular transmis-sion from occurring. Pancuronium, vecuronium,atracurium, and rocuronium are examplesof nondepolarizing neuromuscular blockers(A, B, C, E). The depolarizing agents such assuccinylcholine interact with the receptor andcause the muscle end-plate to depolarize. Unlikeacetylcholine, succinylcholine is resistant todegradation by acetylcholinesterase and as suchcontinues to occupy the receptor preventing themuscle from repolarizing. The depolarizing neu-romuscular blockers, succinylcholine, are con-traindicated in patients with muscular dystrophy.(McMillan, 2682; Brunton, 196–199)

20. (B) Long-term therapy with thiazide diureticshas been shown to be beneficial in the man-agement of CLD-related pulmonary edema (alsocalled, bronchopulmonary dysplasia or BPD).A thiazide diuretic acts by blocking the reab-sorption of sodium and chloride in the thin por-tion of the ascending limb of the loop of Henle.In addition, they block the reabsorption of cal-cium leading to hypercalciuria, which can leadto the production of nephrocalcinosis and even-tual renal failure. Increased urinary excretion ofpotassium can result in hypokalemia. The thi-azides block the excretion of uric acid and canlead to hyperuricemia with symptomatic gout insusceptible patients. (McMillan, 323–324)

21. (C) While daily administration of phenobarbi-tal may reduce the recurrence rate for febrileseizures, the continuous administration of phe-nobarbital is not necessary or recommended.The therapy does not prevent the developmentof epilepsy. In addition, undesirable side effects(rashes, hyperactivity, irritability, lethargy) arefound in at least 20% of patients taking contin-uous phenobarbital. Phenytoin has not beenshown to be effective. Although valproic acidand phenobarbital are effective, the potentialtoxicities outweigh the benefit and their routineuse. Intermittent therapy with phenobarbitalis ineffective. (McMillan, 2298)

22. (C) Nitrite poisoning can result from medicationoverdose or ingestion (amyl nitrate, nitroglyc-erin, nitrate food additives, or contaminated

well water). Nitrites increase the oxidation ofhemoglobin, resulting in methemoglobinemia.Methemoglobin contains oxidized iron (ferric orFe3+) as opposed to reduced iron (ferrous or Fe2+)found in normal hemoglobin. Methemoglobincan be converted to hemoglobin by a reduc-ing agent such as methylene blue, which isadministered intravenously as a 1% solution.(Behrman, 1610)

23. (A) Although many organisms can cause endo-carditis following a dental procedure, Streptococcusviridans remains the most common. Penicillin ishighly effective, with a longer half-life, and is thedrug of choice. For those allergic to penicillins oramoxicillin against this organism, but amoxicillin,the use of cephalexin or another first-generationoral cephalosporin, clindamycin, or a macrolidesuch as azithromycin, or clarithromycin isrecommended. (McMillan, 1652–1655)

24. (C) Penicillins and cephalosporins are ineffectiveagainst Mycoplasma. Erythromycin, azithromycin,and tetracycline are all effective. However, tetra-cycline causes discoloring deposited in growingbone and teeth and is generally not recommededfor those under 8 years of age. Although depositshave not been shown to have significant patho-physiologic effects, it does cause staining ofthe teeth that is cosmetically unacceptable.Chloramphenicol is effective; however, the infre-quent but lethal side effect of aplastic anemiarestricts use of this drug to certain serious infec-tions. (McMillan, 540, 1399)

25. (A) The sodium nitroprusside molecule con-tains cyanide which is released following themetabolism of the parent compound. Cyanidenormally is converted by the rhodanese enzymeof the liver to thiocyanate, which is thenexcreted by the kidneys. The risk of cyanidetoxicity increases with the dose and the dura-tion of administration of sodium nitroprusside.Cyanide binds to cytochrome oxidase andinhibits oxidative phosphorylation leading to ametabolic acidosis. Cyanide also increases theeffects of catecholamines on the smooth muscleof the arterioles leading to an increase in bloodpressure and a need to increase the dose ofnitroprusside (tachyphylaxis). Treatment of

Page 217: Questions - uhs.edu.kh You suspect Lyme disease, most likely contracted by which of the following? (A) ingestion of unripe fruit (B) ingestion of spoiled fruit (C) drinking of contaminated

218 9: Pediatric Therapeutics

cyanide toxicity includes discontinuation ofsodium nitroprusside and administration ofthiosulfate. Thiosulfate provides a sulfur donorfor the rhodanese enzyme to convert cyanide tothiocyanate. In extreme cases, amyl nitrite orsodium nitrite is administered to induce methe-moglobinemia since cyanide will bind to themethemoglobin instead of the cytochrome oxidase.(Brunton, 88–90)

26. (B) Ethosuximide is one of the drugs of choicein the treatment of absence (petit mal) seizures.Amotrigine (Lamictal) or valproic acid (Depakene,Depakote) are other commonly prescribed anti-convulsants for absence seizure. (McMillan, 2289)

27. (B) Many rodent poisons contain warfarin, ananticoagulant that inhibits the production of pro-thrombin by the liver, leading to hemorrhage.Symptoms typically are delayed for 12–24 hoursfollowing ingestion. The degree and duration oftoxicity is variable depending on the size of thechild, amount of ingestion, and formulation ofthe rodenticide. The hepatic toxicity can be effec-tively counteracted by large doses of vitamin K.(McMillan, 1747–1748; Brunton, 1529–1531)

28. (C) Ingestion of toxic doses of acetaminophen(> 140 mg/kg) results in the depletion ofhepatic glutathione stores and the productionof the toxic intermediate metabolite, N-acetyl-benxoquinoneimine. The toxic intermediatemetabolite covalently binds to hepatocytes andleads to cellular death. N-acetylcysteine acts asa surrogate glutathione or sulfate donor andthereby prevents the binding of the toxic inter-mediary to the hepatocyte. Therapy is indicatedbased on the history and should be administeredregardless of the level. N-acetylcysteine is mosteffective if administered within 12–24 hourspostingestion. (McMillan, 759–764)

29. (A) The most likely diagnosis is congenitaladrenal hyperplasia or CAH. CAH results froman enzymatic defect in the adrenal cortical pro-duction of corticosteroids. Ninety percent ofcases and the one presenting as outlined aboveresult from deficiency of the 21-hydroxylaseenzyme. Without this enzyme, there is deficiencyof corticosteroids resulting in an Addisonian-like

state and deficiency of aldosterone resulting inhyperkalemia. The overproduction of adrenalandrogens can result in virilization of femaleinfants. Careful physical examination is neces-sary to be certain that the testes are palpableand that the infant in question is not a virilizedfemale. (McMillan, 2138–2140)

30. (E) Stimulant drugs such as methylphenidate(Ritalin), amphetamines, and atomoxetine(Strattera) have been shown to have a beneficialeffect in many children with attention-deficithyperactivity disorder. The seemingly para-doxical calming action of stimulant drugs inthese children may be related to increasedawareness of, and therefore response to, exter-nal stimuli, permitting sustained attention. Thedrugs have no role in the management ofseizure disorders, temper tantrums, or breath-holding spells. Appetite suppression often is aside effect of these drugs. (McMillan, 678–680)

31. (B) The increasing percentage of Streptococcuspneumoniae resistant to penicillin has followedwidespread overuse of antibiotics. Isolates whichare nonsusceptible to penicillin (including bothintermediate and resistant strains) are definedby minimum inhibitory concentration (MIC) test-ing and the definition of nonsusceptible variesdepending on whether or not the isolate is froma meningeal or nonmeningeal source. Meningitisor septicemia potentially caused by S pneumoniaeshould be initially treated with a third-generationcephalosporin plus either vancomycin. Therapyshould be appropriately tailored based on sus-ceptibility testing of the organism isolated.Cefuroxime is a second-generation cephalosporinand is not recommended in the treatment ofmeningitis. (McMillan, 929–930)

32. (C) Although a diagnosis of sepsis should notbe eliminated immediately, hypoplastic leftheart syndrome or other obstructive left-sidedcardiac lesions are most likely, given thisneonate’s clinical presentation. The mostcommon clinical presentation of hypoplasticleft heart syndrome is respiratory distress onthe second or third day of life. Prostaglandin E1maintains patency of the ductus arteriosus andthereby preserves blood flow to the systemic

Page 218: Questions - uhs.edu.kh You suspect Lyme disease, most likely contracted by which of the following? (A) ingestion of unripe fruit (B) ingestion of spoiled fruit (C) drinking of contaminated

Answers: 26–39 219

circulation. Although this is deoxygenatedblood due to total mixing, there is adequateoxygen delivery to maintain life. Milrinone is aphosphodiesterase inhibitor whose cardiovas-cular effects include increased inotropy andperipheral vasodilatation. These effects are sim-ilar to dobutamine which acts through beta1,2-adrenergic receptors. (McMillan, 1558–1561)

33. (A) Octreotide is a somatostatin analogue thatincreases the resistance in the splanchnic vascu-lature and reduces portal venous pressure,thereby decreasing the bleeding from esophagealvarices. It has replaced vasopressin in this clini-cal situation as octreotide has less of an effect onsystemic blood pressure. (McMillan, 2064–2066)

34. (E) Cystic fibrosis is the most common cause ofpancreatic insufficiency in childhood. Mostpatients with this disease have less steatorrheaand improved weight gain when receivingpancreatic replacement therapy. (McMillan, 1436)

35. (E) Neonatal meningitis and sepsis most com-monly are caused by group B streptococcus,gram-negative enteric bacilli, or Listeria mono-cytogenes. Initial combination therapy withampicillin plus gentamicin; or ampicillin pluscefotaxime is recommended though the latterregimen is preferred if meningitis is present.Neither the cephalosporins nor gentamicinprovides coverage against L monocytogenes.Cefuroxime is not recommended for treatmentof meningitis because of reports of delayed CSFsterilization. (McMillan, 493–495)

36. (A) In addition to treating oncologic conditions,cyclophosphamide is used in immunosup-pressive regimens for autoimmune disordersand organ transplantation. High doses ofcyclophosphamide results in bladder epithelialdysplasia, hemorrhagic cystitis, and sterility.(Brunton, 1395–1396)

37. (B) Erythromycin has historically been the drugof choice to prevent spread of pertussis thoughcurrently, azithromycin is generally preferred astherapy. On rare instances Bordetella pertussis hasbeen found to be macrolide resistant; in such acase, trimethoprim-sulfamethoxazole is an

alternative but its efficacy is less well docu-mented. Treatment of the index case generallywill eradicate the organism from the tracheo-bronchial tree and reduce spread. Althougherythromycin alters the clinical course of pertus-sis when given early in the paroxysmal stage, itdoes not seem to alter the clinical course if startedlater in the illness. A 14-day course of erythro-mycin is recommended for prevention and treat-ment of pertussis; a 5 day course is recommendedfor azithromycin. (McMillan, 1094–1097)

38. (A) This case describes an 8-year-old with pri-mary nocturnal enuresis. The male to femaleratio is 3:2 for this disorder. About 15% of5-year-olds and 5% of 10-year-olds have enure-sis. There are many therapy options; howeverbehavioral conditioning has been shown tohave the best success rate of 70%–80% (A).DDAVP (B) is used to decrease urine outputfor 1–7 hours, is very expensive therapy, andhas been shown to have a 40%–60% successrate with a high rate of relapse after discontin-uation (B). Imipramine is a tricyclic antide-pressant which is FDA approved to treatenuresis, it has been shown to be 30%–50%effective (C). Motivational therapy, such as areward system for not wetting the bed has beenshown to be only 25% effective (D). Furosemideis a diuretic and would not be useful in thetreatment of enuresis (E). (McMillan, 670–672)

39. (B) Minimal change disease is characterized bynephrotic syndrome without systemic disease.Two-thirds of the cases present between the agesof 2 to 6. The recommended treatment regimentis to start a course of high-dose corticosteroidsand to follow urine protein to see if there is aresponse. Severe cases may need hospitaliza-tion for diuretic therapy; however most casescaught early can be treated as an outpatient.IVIG (A) is not indicated as a first-line therapyfor minimal change disease, it is a first-line treat-ment for Kawasaki disease. A kidney biopsy (C)is not indicated with this presentation ofnephrotic syndrome. Minimal change disease isby far the most likely diagnosis and a kidneybiopsy will be of little help and a high mor-bidity. If the patient does not respond to thecourse of steroids, a biopsy may be appropriate.

Page 219: Questions - uhs.edu.kh You suspect Lyme disease, most likely contracted by which of the following? (A) ingestion of unripe fruit (B) ingestion of spoiled fruit (C) drinking of contaminated

220 9: Pediatric Therapeutics

Cyclophosphamide (D) is a second-line therapyfor minimal change disease. Antibiotic therapy(E) is not indicated for minimal change disease;however an early presentation may be confusedwith a peri-orbital cellulitis. (McMillan, 1867–1868)

40. (C) Idiopathic thrombocytopenia purpura (ITP)is an autoimmune-mediated disease that leadsto the destruction of platelets via the reticuloen-dothelial system. Serious complications are rare;however with a platelet count of less than 20,000there is a risk for intracranial hemorrhage. Anti-d immunoglobulin (or Rhogam) is the treatmentof choice for patients with ITP who are Rh posi-tive. The mechanism of action is not fully under-stood, however the theory is that the anti-d coatsred blood cells and these antibody-coated redblood cells compete with the platelets for destruc-tion by the reticuloendothelial system. Acompli-cation of this therapy is a mild anemia from thered cell destruction. This therapy only works inpatients who are Rh-positive blood type. IVIG(A) is a second-line therapy for patients withRh-positive blood type and first line for patientswho are Rh negative. Amoxicillin (B) and antibi-otics have no role in the treatment of ITP. Aplatelettransfusion (D) is not indicated in the acute settingof ITP, the transfused platelets will be destroyedvia the same mechanism outlined above and willoffer little benefit. Observation (E) is not a goodoption in this patient. With a platelet count ofless than 20,000 and small mucosal bleeding,there is a risk of intracranial hemorrhage.(McMillan, 1731–1733)

41. (D) The child in the case has the diagnosis ofacute chest syndrome. The diagnosis of acutechest syndrome is made in children with sicklecell disease when a new infiltrate is found onchest x-ray. Acute chest syndrome can haveserious short- and long-term complications andlead to respiratory failure. Antibiotics shouldbe started in a timely manor and both a third-generation cephalosporin and a macrolide arerecommended (D). Amoxicillin, ceftriaxone,and azithromycin alone would not be sufficienttherapy (A, B, C). Withholding antibiotic therapywould not be appropriate in this case, even ifthis is a viral process, acute chest syndromeshould be treated with antibiotics. (McMillan, 1697)

42. (D) Notification to the local poison center canprovide guidance and follow-up of all forms oftoxic exposures, including acetaminophen tox-icity. Acetaminophen toxicity can occur in achild if the dose is in the range of 150 mg/kgper 24 hour period. It should be treated witheither oral or IV N-acetylcysteine as soon asthe elevated acetaminophen level is detected,regardless of hepatic enzyme levels. This treat-ment can help prevent further liver dysfunctionand avoid liver transplantation (D). Steroidsand furosemide have no role in the treatment ofacetaminophen toxicity (A, B). Naloxone isused to reverse opioid toxicity (C). Providingno treatment to this child may allow the liverdysfunction to worsen and the child mayprogress toward fulminate liver failure andliver transplant (E). (McMillan, 751–752)

43. (D) Pelvic inflammatory disease is caused byChlamydia and gonococcal infection of theupper reproductive tract. It can have long-termcomplications if not treated properly, such asinfertility, ectopic pregnancy, and abdominaladhesions. Since PID can be polymicrobialinfections, it is important to treat all potentialcauses. Ceftriaxone and doxycycline (D) is anacceptable outpatient treatment regimen forPID. Clindamycin in conjunction with gen-tamicin is an acceptable inpatient treatmentregimen; however clindamycin alone is not suf-ficient (A). Neither ceftriaxone nor doxycyclinealone is an adequate treatment regimen.Metronidazole is used to treat trichomonalinfections and can be used in conjunction withan accepted treatment regimen, but should notbe used as a single agent (E). (McMillan, 587–590)

44. (C) Dysfunctional uterine bleeding (DUB) iscommon during adolescence and can be con-cerning to the patients, but is often self-resolvingover time. If the patient is having frequentbleeding and wants to become regular, the besttherapy is an oral contraceptive pill with estro-gen and progestin (C). A platelet disorder maymanifest as heavy menses, but often has othersigns of bleeding and low platelets, this patientdoes not have any other signs (B). Testosteronecream is not indicated for dysfunctional uterinebleeding (A). NSAIDs are the first-line therapy

Page 220: Questions - uhs.edu.kh You suspect Lyme disease, most likely contracted by which of the following? (A) ingestion of unripe fruit (B) ingestion of spoiled fruit (C) drinking of contaminated

Answers: 40–50 221

for dysmenorrhea, or painful menses, not forDUB (D). Careful observation may be a poten-tial choice for a patient with DUB as most caseswill self-resolve. This patient is looking for anintervention that will help make her mensesmore regular and oral contraceptives are theclear choice. (McMillan, 564–565)

45. (E) Careful examination, close observation, andreliable follow-up represent an importantapproach to many pediatric complaints. Thiscase describes erythema toxicum neonatorumwhich is a benign rash of the neonate that isself-resolving and requires no treatment (E).The rash typically starts out as papules of 2–3 mmin diameter and progresses to vesicles and thenresolves over 2 weeks. The vesicles are noted tocontain a large number of eosinophils whenlooked at under a microscope. Acyclovir (A) isthe treatment for cutaneous herpes simplex infec-tion, this patient’s history and microscopic find-ings are not consistent with this diagnosis.Topical steroids are often used for eczema andatopic dermatitis (B). Antidandruff shampoo isthe first-line treatment for seborrhea (cradle cap).Phototherapy is the first-line treatment for hyper-bilirubinemia (D). (McMillan, 461)

46. (C) Intraosseous needle insertion is often thequickest means to vascular access, however, thispatient’s status provides for a reasonable attemptat peripheral venous access. Intracardiac (D) andarterial access (B) are not appropriate for thispatient. Surgical cut down (E) should only bepursued when intraosseous and central venousaccess methods have failed or cannot be pursued(such as multiple trauma). (McMillan, 2675–2677)

47. (B) This case scenario describes a child withsciatic nerve injury following intramuscularinjection into the gluteal region. Althoughpostinjection injury can occur in both adultsand children, children appear to be at higherrisk. The degree of nerve injury appears to berelated to direct nerve trauma by the needle, hostresponse to the trauma (inflammation), andtoxicity of the agent injected. For instance,Penicillin G is considered to be more toxic tothe nerve. An allergic reaction to penicillinwould not cause these symptoms (A). The

anterolateral thigh (C) and deltoid (D) muscleappear to have the lowest risk for complica-tions such as bleeding, muscle contracture, andnerve injury. Though cerebral vascular acci-dent (E) is within the differential for new onsetof neurologic findings, it does not best describethis clinical scenario. (McMillan, 2685–2687)

48. (B) Cardiomyopathy most commonly is anadverse effect related to the administration ofanthracyclines such as doxorubicin. Childrenare at higher risk for cardiac toxicity and thoseyounger than 5 years are at higher risk thanolder children. Both acute and chronic cardiactoxicity can occur. The acute phase is charac-terized by arrhythmias and conduction abnor-malities. Chronic cardiomyopathies can occurearly (within a year of anthracycline therapy)and late (greater than a year after completinganthracycline therapy). The early form of car-diomyopathy is related to the cumulative doseof anthracyclines. A high-dose rate and preex-isting cardiac disease can also increase the risk.(Brunton, 1428; Behrman, 2112)

49. (E) Neurotoxicity in this form of peripheralneuropathy is a dose-limiting toxicity that canoccur with the use of vincristine. It is related tothe cumulative dose and occurs more com-monly on a weekly schedule. It often presentsas a peripheral mixed neuropathy involvingboth the sensory and motor nerves. Loss ofdeep tendon reflexes, paresthesias, and wristand foot drop are the most common manifes-tations. (Behrman, 1967, 21126, Brunton, 1418–1419)

50. (C) Although steroids are first-line therapy foracute graft-versus-host disease (GVHD), theyare not without their risks, particularly in thispatient population that may require long-termtherapy to suppress GVHD symptoms. Infection,hypertension, hyperglycemia, weakness, andHPA-axis suppression are common side effectsof corticosteroids. Growth retardation, psy-chosis, osteoporosis, central obesity, moonfacies, acne, hirsutism, pseudotumor cerebri,and glaucoma can also occur. Cataracts canoccur with long-term therapy. (Behrman, 930–931;McMillan, 1765)

Page 221: Questions - uhs.edu.kh You suspect Lyme disease, most likely contracted by which of the following? (A) ingestion of unripe fruit (B) ingestion of spoiled fruit (C) drinking of contaminated

222 9: Pediatric Therapeutics

51. (E) The patient has a head injury and is at riskfor increased intracranial pressure. Ketamineincreases ICP and is relatively contraindicatedin patients with intracranial hypertension, andshould be avoided in this patient. All of theother agents decrease the cerebral metabolicrate for oxygen and thereby decrease cerebralblood flow and intracranial pressure, whichcould be beneficial if there is a head bleedincreasing the ICP. (Behrman, 466; McMillan, 2682)

52. (D) Nitric oxide (NO) is a naturally occurringvasodilator. PPHN occurs in term and post-terminfants. Predisposing factors include birthasphyxia, meconium aspiration pneumonia,early-onset sepsis, HMD, hypoglycemia, poly-cythemia, maternal use of nonsteroidal anti-inflammatory drugs with in utero constriction ofthe ductus arteriosus, and pulmonary hypoplasiaas a result of diaphragmatic hernia, amniotic fluidleak, oligohydramnios, or pleural effusions.PPHN is often idiopathic. Some patients withPPHN have low plasma arginine and nitric oxidemetabolite concentrations and polymorphismsof the carbamoyl phosphate synthase gene, find-ings suggestive of a possible subtle defect in nitricoxide production. Exogenous surfactant therapyhas been beneficial in some patients. iNO (inhalednitric oxide), a potent and selective pulmonaryvasodilator (equivalent to endothelium-derivedrelaxation factor), when given at an initial doseof 1–20 ppm, has improved oxygenation inpatients with PPHN and reduce the need forECMO. Although brief exposure to higherdoses (40–80 ppm) appears to be safe, long-termtreatment with 80 ppm increases the adverseeffects. Responses to iNO include no improve-ment; initial improvement but not sustained, withECMO being required; initial and sustainedimprovement, usually weaned by the fifth day oftherapy; and an initial response but prolongeddependency, possibly as a result of pulmonaryhypoplasia or alveolar capillary dysplasia.Methemoglobinemia is a rare but possible com-plication of iNO. The maximal safe duration ofiNO therapy is unknown. (McMillan, 316–318)

53. (E) The treatment of acidosis is critical in themanagement of salicylate intoxication. Acidosisenhances the passage of salicylate (nonionized

form) from the extracellular space into the cells,including the blood–brain barrier, where it dis-rupts mitochondrial function. Salicylates areweak acids that in an alkaline medium dissociateto the ionized form and can be “trapped” there.Sodium bicarbonate’s alkalinization effect occurssolely in the extracellular space and increases thelevel of ionized drug in the extracellular plasma.The intracellular-to-extracellular gradient of dif-fusible, nonionized drug is increased, thusenhancing the trapping of salicylate in the extra-cellular plasma. Alkalinization of the blood withsodium bicarbonate can prevent salicylates fromentering the brain and CSF, alkalinization of theurine results in enhanced urinary salicylate excre-tion. CNS status may improve as the plasma pHincreases, because of a shift of salicylate equilib-rium from brain to blood despite a lack of urinealkalinization. Cardiac arrhythmias and seizuresare not common in salicylate poisoning. Sodiumbicarbonate alone will not worsen his hydrationstatus, but fluids should be replaced as neededand fluid status and urine output should be care-fully monitored. (McMillan, 757)

54. (B) Morphine is a commonly used opioid forthe treatment of sickle cell disease both chron-ically and for acute pain crises. It is available asan intravenous medication, and can be givenon a PCA pump for those old enough to usethem. It is also available in several long-actingand short-acting preparations for chronic painand breakthrough pain as needed. Fentanyl isthe shortest acting of the agents listed and isavailable intravenous, as a lozenge, or as atransdermal long-acting patch, but oral for-mulations are not available. Oxycodone comesas long- and short-acting oral preparations, butis not available IV. Meperidine is available IV oras a short-acting oral preparation, but shouldbe avoided due to the adverse effects of itsactive metabolite, normeperidine. Methadone isavailable as IV and oral, but has the longest half-life and is not used as a short-acting medication.(Berhman, 2027–2028, 479–480)

55. (E) This child is most likely under volumeresuscitated and requires an intravascular fluidbolus. In the setting of high insensible losses (suchas a burn injury), low urine output suggests low

Page 222: Questions - uhs.edu.kh You suspect Lyme disease, most likely contracted by which of the following? (A) ingestion of unripe fruit (B) ingestion of spoiled fruit (C) drinking of contaminated

Answers: 51–58 223

intravascular fluid volume. For most childrenwith burns, the Parkland formula is an appropri-ate starting guideline for fluid resuscitation (4 mLlactated Ringer/kg/% BSA burned). One-half ofthe fluid is given over the first 8 hours calculatedfrom the time of onset of injury. The remaininghalf is given at an even rate over the next 16 hours.The rate of infusion is adjusted according to thepatient’s response to therapy. Pulse and bloodpressure should return to normal, and an ade-quate urine output (1 mL/kg/h) should beaccomplished by varying the intravenous infu-sion rate. Inadequate urine output reflectsdepleted intravascular volume. Although it willtemporarily increase urine output, the use ofdiuretics will only further decrease the intravas-cular volume. (Behrman, 452–454; McMillan, 711–712)

56. (C) Management strategies of infants born withHLHS classically have been divided into threebasic approaches: staged palliative reconstruc-tion, cardiac transplantation, and supportive careonly. Immediate medical management is centeredon maintaining ductal patency and optimizingthe balance between systemic and pulmonaryvascular resistance. Intravenous prostaglandinE1 is effective in maintaining patency of theductus arteriosus and enables the neonate to haveoptimized cardiac hemodynamics and a normal-ized acid-base balance before surgical interven-tion. Intravenous indomethacin and ibuprofenwill close the ductus arteriosus. Preoperativemanagement should avoid excessive pulmonaryblood flow; oxygen administration, hyperventi-lation, and high-dose inotropic agents should beavoided if possible so as not to alter the tentativebalance between the pulmonary and systemicvascular beds. (McMillan, 1560; Behrman, 1926–1928)

57. (E) Lyme disease is the most common vector-borne disease in the United States. Most casesoccur in southern New England, the easternparts of the Middle Atlantic states, and theupper Midwest, with a smaller endemic focus ofLyme disease along the Pacific coast. Lyme dis-ease is a zoonosis caused by the transmission ofthe spirochete Borrelia burgdorferi to humansthrough the bite of an infected tick of theIxodes species. The first clinical manifestationof Lyme disease is the typical annular rash,

named erythema migrans. The initial lesionoccurs at the site of the bite. The rash may beuniformly erythematous, or it may appear as atarget lesion with central clearing; rarely, theremay be central vesicular or necrotic areas.Erythema migrans may be associated with sys-temic features including fever, myalgia, headache,or malaise. Without treatment, the rash graduallyexpands (hence the name migrans) to an averagediameter of 15 cm and remains present for atleast 1–2 weeks. No clinical trials of treat-ment for Lyme disease have been conducted inchildren. Recommendations for the treatment ofchildren are extrapolated from studies of adults.Doxycycline (100 mg twice daily × 14-21 days)for older children or amoxicillin (50 mg/kg/daydivided three times daily × 14–21 days) are first-line therapy. Patients who are treated withdoxycycline should be alerted to the risk ofdeveloping dermatitis in sun-exposed areaswhile taking the medication. Cefuroxime is alsoapproved for the treatment of Lyme disease andis an alternative for persons who cannot takedoxycycline and who are allergic to penicillin.Preliminary results with azithromycin have beendisappointing. There is little need to use neweragents because the results of treatment witheither amoxicillin or doxycycline have beengood. (McMillan, 1046–1049; Behrman, 1274–1278)

58. (E) The most common organisms causingmeningitis in children are Streptococcus pneu-moniae and Neisseria meningitidis. After theintroduction of Haemophilus influenzae type bconjugate vaccine in 1988, there was a greaterthan 99% decline in invasive disease caused byH influenzae type b, including meningitis.Penicillin remains the drug of choice for thetreatment of meningococcal meningitis. It ishighly effective, inexpensive, and generally freeof significant side effects, except for allergicreactions. Ceftriaxone, cefotaxime, or ampicillinare acceptable alternatives. Several of the third-generation cephalosporins are useful for thetreatment of bacterial meningitis (includingH influenzae and S. pneumoniae meningitis) priorto identification of the infecting organism.However, once N meningitides has been identi-fied, penicillin becomes the antibiotic of choice.Penicillin G should be administered IV at a

Page 223: Questions - uhs.edu.kh You suspect Lyme disease, most likely contracted by which of the following? (A) ingestion of unripe fruit (B) ingestion of spoiled fruit (C) drinking of contaminated

224 9: Pediatric Therapeutics

dose of 250,000–400,000 units/kg/day dividedevery 4–6 hours. (Behrman, 2514–2519, 1164–1168)

59. (B) Deferoxamine is the chelating agent ofchoice for iron poisoning. Deferoxamine bindswith iron to form a water-soluble compound, fer-rioxamine, which can be renally excreted or dia-lyzed. Deferoxamine may also limit the entranceof iron into the cell and help chelate intracellulariron. Because of its short half-life, it is administeredas a continuous infusion at a dose of 15 mg/kg/hfor up to 24 hours. Rapid administration ofdeferoxamine can lead to hypotension, whichshould be treated by reducing the infusion rate.Deferoxamine is indicated for severe intoxication(ingestion of more than 25 mg/kg of elementaliron or severe illness) or a serum iron concentra-tion greater than 350 mg/dL. The use of activatedcharcoal, ipecac, and gastric lavage for iron poi-soning are no longer recommended for acute ironingestion. Dimercaprol (BAL) actually enhancesthe toxicity of iron and is contraindicated in casesof iron ingestion. EDTAis used for lead poisoning,not iron poisoning. (McMillan, 751–752)

60. (D) The most likely diagnosis is acute pancreati-tis, given this clinical scenario. Though the dif-ferential diagnosis includes gastritis and pepticulcer, the epigastric pain with radiation into theback is more characteristic of pancreatitis. Themost likely etiologies of pancreatitis in the pedi-atric age group include trauma, viral infections,gallbladder disease, drugs, and toxins. The lattergroups include corticosteroids, alcohol, valproicacid, tetracycline, and furosemide. Diagnosticevaluation should include a serum amylase andlipase. (Behrman, 1652–1654)

61. (E) Treatment of gonococcal urethritis dependson achieving a high antimicrobial level in theserum for a short time; prolonged treatment isnot required. Benzathine penicillin should neverbe used as it will not achieve sufficiently highblood levels. Procaine penicillin 4.8 million unitsIM with 1 g of probenecid is effective againstpenicillin-sensitive strains. However, penicillinresistance is so common that a single dose of cef-triaxone, 125 mg IM, is generally the treatment ofchoice and provides high bactericidal levels inthe blood. Extensive clinical experience indicates

that ceftriaxone is safe and effective for the treat-ment of uncomplicated gonorrhea at all anatomicsites, curing 98.9% of uncomplicated urogenitaland anorectal infections in published clinicaltrials. Patients infected with Neisseria gonorrhoeaefrequently are coinfected with Chlamydia tra-chomatis; this finding has led to the recommen-dation that patients treated for gonococcalinfection also be treated routinely with a regimenthat is effective against uncomplicated genitalC trachomatis infection. To maximize compliancewith recommended therapies, medications forgonococcal infections should be dispensed onsite. Azithromycin 1 g as a single oral dose ordoxycycline 100 mg PO bid for 7 days are effec-tive for the treatment of chlamydial infections,not gonococcal infections. Either regimen shouldbe added to the ceftriaxone to treat the possiblecoinfection with chlamydia. (McMillan, 584–590)

62. (A) Liver injury is the major toxic effect of acet-aminophen poisoning and generally is seen onlywith large overdoses or poisonings. In adults,greater than 150mg/kg is considered toxic,whereas in children, greater than 200 mg/kg isconsidered toxic. Generally, children less than2 years old tend to have the least toxicity. Thehepatic damage is not caused by the acetamin-ophen itself, but rather by the toxic metabolicproduct. The metabolic pathway producing thehepatotoxic metabolite is less well developed ininfants and young children, and therefore, therisks of liver injury are actually somewhat less inyoung children. Treatment involves adminis-tration of N-acetylcysteine, either oral or IV,which minimizes metabolism of the acetamino-phen to the toxic metabolite. Treatment is mosteffective if started within 24 hours of ingestion.Alkalinization of the urine and dialysis are inef-fective treatment modalities. (McMillan, 759–764)

63. (B) Ampicillin plus an aminoglycoside is theinitial treatment of choice for a newborn infantwith presumptive group B streptococcus infec-tion. However, once group B streptococcus hasbeen identified and demonstrated to be suscep-tible, then switching to penicillin G is the drugof choice to treat group B streptococcal infec-tions, including meningitis. Treatment shouldbe continued for 14–21 days. (McMillan, 505)

Page 224: Questions - uhs.edu.kh You suspect Lyme disease, most likely contracted by which of the following? (A) ingestion of unripe fruit (B) ingestion of spoiled fruit (C) drinking of contaminated

Answers: 59–67 225

64. (A) The most important toxic effect of the tri-cyclic antidepressants is sodium channel block-ade. The cardiac sodium channel is responsiblefor cardiac cell depolarization (action potentialphase 0); its inhibition leads to slowed depolar-ization of individual cardiac cells. This, in turn,leads to slowing of the wave of depolarizationacross the myocardium. The ECG manifestationof slowed depolarization is prolongation of theQRS complex, the hallmark of TCA overdose.QRS prolongation serves as a marker for TCAingestion and the risk of adverse cardiac or cen-tral nervous system events; such as ventriculartachycardia or seizures. This is true not justbecause QRS prolongation identifies patientswho have ingested a substantial overdose, butalso because slowed cardiac conduction is theproximate cause of much of the cardiovasculartoxicity of these drugs. Additional toxic effects ofTCAs include alpha-adrenergic blockade (caus-ing systemic vasodilation/hypotension), andcholinergic muscarinic receptor blockade maycause an “anticholinergic syndrome,” includingsinus tachycardia, delirium, coma, mydriasis,impaired gut motility, urinary retention, impairedsweating, and dry mucosa. (McMillan, 753–754)

65. (B) The most effective intervention for the car-diovascular toxicity of TCA overdose is sodiumbicarbonate (NaHCO3 = 1 molar, or 1 meq/mL).NaHCO3 is useful in stabilizing cardiac mem-branes and consequently reduces arrhythmias.NaHCO3 is effective in reducing TCA-inducedQRS prolongation, reversing hypotension, andtreating ventricular dysrhythmias. NaHCO3 isgenerally administered as intravenous boluses of1 meq/kg up to adult dose of 50 meq as neededto correct acidosis if pH less than 7.1, QRS greaterthan 0.16 seconds, arrhythmias are present orhypotension is not responding to fluids. In gen-eral, antiarrhythmics should be avoided in TCAingestion, and supportive care provided includingadmission to a monitored setting, intravenousfluids, and inotropic support. (McMillan, 754)

66. (D) A gram-negative rod, non-lactose fermenter,oxidase positive is most concerning for P aerug-inosa. Children with leukemia or other debilitatingmalignancies, particularly those who are receiv-ing immunosuppressive therapy and who are

neutropenic, are extremely susceptible to sep-ticemia by P aeruginosa usually from endogenousseeding from the patient’s respiratory or gas-trointestinal tract. Empiric therapy should includean antipseudomonal agent tailored to the antibioticresistance patterns of P aeruginosa for the hospital.Many strains are susceptible to the anti-pseudomonal penicillins (ticarcillin or piperacillin),fourth-generation cephalosporins (cefepime),carbapenems (imipenem or meropenem), aztre-onam, ciprofloxacin, and aminoglycosides.Ceftazidime is a third-generation cephalosporinthat has good activity against P aeruginosa,other third-generation cephalosporins, includ-ing ceftriaxone, have limited activity againstP aeruginosa. Fluoroquinolones other thanciprofloxacin have variable activity againstP aeruginosa. Aminoglycosides should not beused as monotherapy. Due to the rapid emer-gence of resistance, double therapy (usually asemisynthetic penicillin and aminoglycoside)is often recommended, but the evidence tosupport this is not robust. (Behrman, 1208–1210)

67. (E) Tumor lysis syndrome results when theintracellular contents released from dyingleukemic blasts exceeds the ability of the body toadequately metabolize and excrete them. This ismost frequently associated with leukemiapresenting with a very high WBC counts.Hyperuricemia, a result of the metabolism ofexcess amounts of released nucleic acids, canresult in renal failure that then can furtherworsen the patient’s ability to excrete othermetabolites. Hyperphosphatemia and second-ary hypocalcemia may lead to further renal fail-ure and the danger of seizures. Hyperkalemiamay lead to alterations in the electrocardiogramwith increased amplitudes of T waves, arrhyth-mia, and cardiac arrest. Patients presenting withtumor lysis syndrome or those at high risk withinduction of chemotherapy require immediateintervention with intravenous hydration tomaintain urine flow and alkalinization withintravenous sodium bicarbonate and allopurinolor other agents to increase the solubility andrenal excretion of insoluble urate. Allopurinolinhibits the enzyme that accelerates the conver-sion of xanthine and hypoxanthine to uric acidand is used to prevent hyperuricemia in children

Page 225: Questions - uhs.edu.kh You suspect Lyme disease, most likely contracted by which of the following? (A) ingestion of unripe fruit (B) ingestion of spoiled fruit (C) drinking of contaminated

226 9: Pediatric Therapeutics

at risk for the tumor lysis syndrome whenreceiving chemotherapy. Frequent monitoringof serum electrolytes, in addition to potassium,calcium, phosphorus, creatinine, and urine output,is critical. (McMillan, 1756–1757)

68. (D) Clostridium difficile–associated diarrhea,also known as pseudomembranous colitis orantibiotic-associated diarrhea, usually occurs inpatients receiving antimicrobial therapy. Virtuallyall known antibiotics have been implicated; peni-cillins, broad-spectrum cephalosporins, and clin-damycin are the most frequent offenders. Theclassic picture of pseudomembranous colitis isdiarrhea with blood and mucus accompanied byfever, cramps, abdominal pain, nausea, and vom-iting. Disease occurs during and as long as weeksafter antibiotic therapy. The first and essential stepin treatment is the discontinuation of the currentantibiotics, if possible. In most instances, thiscourse combined with appropriate fluid and elec-trolyte replacement is sufficient. Metronidazole ispreferred as primary therapy, (20–40 mg/kg/24 hdivided q 6–8 hours PO for 7–10 days) becauseoral therapy with this agent is similar in efficacyto vancomycin given orally for mild and moder-ate cases of disease, has lower cost (about $20versus $200), and avoids the emergence of ente-rococcal resistance to vancomycin. If parenteraltherapy is the only option, metronidazole is pre-ferred because achievable colonic concentrationsare superior, and because clinical failure has beenreported with the use of parenteral vancomycin.Oral vancomycin (25–40 mg/kg/24h dividedq 6 hours PO) is preferred for severe or complicateddisease and for direct instillation in treatment ofintestinal obstruction or adynamic ileus. (Behrman,1230–1231; McMillan, 1038–1040)

69. (E) Malignant hyperthermia (MH) has occurredin association with several forms of myopathy,especially central core disease. MH is charac-terized by symptoms of rapidly rising bodytemperature, tachycardia, tachypnea, cyanosis,and respiratory and metabolic acidosis. Manypatients with malignant hyperthermia have noobvious clinical symptoms of muscle disease.Usually, affected patients experience rigidityand myoglobinuria. Unexplained fevers, muscle

cramps, and increased serum creatine kinasevalues may represent clinical clues to the pres-ence of malignant hyperthermia. The clinicalsyndrome can be aborted by the intravenousadministration of dantrolene. Dantrolene canprevent or reduce the increase in myoplasmiccalcium ion concentration that activates theacute catabolic processes associated with malig-nant hyperthermia. (McMillan, 2326)

70. (B) Ciprofloxacin and the other fluoro-quinolones exert their antibacterial activitybased on their peak/MIC ratio, termed con-centration-dependent activity. Higher concen-trations above the MIC exert greater killingactivity. Fluoroquinolones do not have thenephrotoxic and ototoxic effects that amino-glycosides do; therefore therapeutic drug mon-itoring by levels is not necessary to ensure highpeaks for efficacy and low troughs for toxicity.Recommended doses of fluoroquinolonesensure adequate peak levels. Other antibioticsexert their efficacy based on the time the antibi-otic concentration exceeds the MIC (termedtime dependent), and these drugs are typicallydosed more frequently to ensure efficacy.Examples of time-dependent drugs includebeta-lactams, macrolides, and clindamycin.(Behrman, 334–336; Brunton, 1182, 1225–1226)

71. (D) The concept of a steady-state concentra-tion after repeated doses of a drug is importantfor the appropriate timing of serum samplingin monitoring drug therapy. Blood levels meas-ured before a steady state is reached do notrepresent or predict drug concentrations atsteady state. Dosage adjustments based on suchpremature data can result in serious dosageerrors. Steady-state blood levels are achievedafter approximately five drug half-lives haveelapsed since the first dose, if no loading doseis given. A loading dose of drug may be neces-sary or desirable to rapidly achieve adequatetherapeutic drug concentrations. (McMillan, 49)

72. (E) Phenytoin has been associated with IUGRas well as other congenital anomalies. IUGRis associated with medical conditions thatinterfere with the circulation and efficiency of

Page 226: Questions - uhs.edu.kh You suspect Lyme disease, most likely contracted by which of the following? (A) ingestion of unripe fruit (B) ingestion of spoiled fruit (C) drinking of contaminated

Answers: 68–74 227

the placenta, with the development or growthof the fetus, or with the general health andnutrition of the mother. IUGR is often classifiedas reduced growth that is symmetric (head cir-cumference, length, and weight equallyaffected) or asymmetric (with relative sparingof head growth). Symmetric IUGR often hasan earlier onset and is associated with diseasesthat seriously affect fetal cell number, such asconditions with chromosomal, genetic, mal-formation, teratogenic, infectious, or severematernal hypertensive etiologies. AsymmetricIUGR is often of late onset and is associatedwith poor maternal nutrition or with late onsetor exacerbation of maternal vascular disease(preeclampsia, chronic hypertension). Otherdrugs such as narcotics, alcohol, cigarettes,cocaine, and antimetabolites are commonly acause of IUGR. (Behrman, 692)

73. (E) Theophylline is rarely used in pedi-atric asthma because of its potential toxicity.Theophylline, when used chronically, can

reduce asthma symptoms and need for supple-mental beta-agonist use. Because theophyllinemay have some glucocorticoid-sparing effects inindividuals with oral glucocorticoid-dependentasthma, it is still sometimes used in this groupof asthmatic children. Theophylline has a narrowtherapeutic window; therefore, serum theo-phylline levels need to be routinely monitored,especially if the patient has a viral illness asso-ciated with a fever or is placed on a medicationknown to delay theophylline clearance, such asmacrolide antibiotics, cimetidine, oral antifun-gals, oral contraceptives, and ciprofloxacin. Toxiceffects include both mild symptoms (nausea,vomiting, abdominal pain, headache, irri-tability) and severe and life-threatening reac-tions (intractable convulsions, tachyarrhythmia).Both therapeutic and toxic effects are dosedependent, and it is difficult to maintain thera-peutic effects without some toxicity. (Behrman,961–965; McMillan, 2406–2409)

74. (D) 75. (B) 76. (F) 77. (E) 78. (C)

TABLE 9-2.

Drug Indications Pharmacology Dose range

Dopamine (1) Hypotension Dopaminergic effects 2–20 µg/kg/min(2) Low cardiac output (vasodilatation at 1–5 µg/kg/min)

β-adrenergic effects(inotropy at 5–10 µg/kg/min)

α-adrenergic effects(vasoconstriction at >15 µg/kg/min)

Epinephrine (1) Cardiogenic and septic shock β1 > β2 > α adrenergic effects 0.1–1 µg/kg/min(2) Low cardiac output with low but all are present.

systemic vascular resistance(3) Bradycardia

Dobutamine (1) Myocardial dysfunction β1-adrenergic effects 2–20 µg/kg/min(2) Low cardiac output in setting

of elevated pulmonary resistanceIsoproterenol (1) Slow heart rate β1-adrenergic effects with some β2 0.05–0.5 µg/kg/min

effects at higher doses.(2) Myocardial dysfunction with normal

systemic vascular resistanceNorepinephrine (1) Hypotension α1-adrenergic effects (vasoconstriction) 0.1–2 µg/kg/min

(2) Spinal shock β1-adrenergic effects (increased inotropy)(3) Inadequate cardiac output(4) α-adrenergic blockade(5) Low cardiac output with low

systemic vascular resistanceMilrinone (1) Cardiogenic shock Phosphodiesterase type III inhibitor Load:

(2) Low cardiac output with elevated 50 µg/kgpulmonary and systemic vascular Infusion:resistance 0.75–1 µg/kg/min

Page 227: Questions - uhs.edu.kh You suspect Lyme disease, most likely contracted by which of the following? (A) ingestion of unripe fruit (B) ingestion of spoiled fruit (C) drinking of contaminated

228 9: Pediatric Therapeutics

These drugs are usually administered in anintensive care setting, where the dose can becarefully titrated to hemodynamic response.Continuous determinations of arterial bloodpressure and heart rate are performed; measur-ing cardiac output at the bedside with a pul-monary thermodilution (Swan-Ganz) cathetermay also be helpful in assessing drug efficacy.

Dopamine is a predominantly beta-adrener-gic receptor agonist, but it has alpha-adrenergiceffects at higher doses. Dopamine has lesschronotropic and arrhythmogenic effect thanthe pure beta-agonist isoproterenol. In addition,it results in selective vasodilatation at lowerdosages (< 5 µg/kg/min) via its interactionwith dopamine receptors. At an intermediatedose of 5–10 µg/kg/min, dopamine results inincreased contractility via its beta-adrenergiceffects with little peripheral vasoconstrictiveeffect. However, if the dose is increased beyond15 µg/kg/min, its peripheral alpha-adrenergiceffects may result in vasoconstriction.

Dobutamine, a derivative of dopamine, isuseful in treating low cardiac output condi-tions. Infusions cause direct inotropic effectswith a moderate reduction in peripheral vas-cular resistance. Dobutamine can be used as anadjunct to dopamine therapy to avoid thevasoconstrictive effects of high-dose dopamine.Dobutamine is also less likely to cause cardiacrhythm disturbances than isoproterenol. Theusual dose is 2–20 µg/kg/min.

Isoproterenol is a pure beta-adrenergic ago-nist that has a marked chronotropic effect; it ismost effective in patients with slow heart ratesand should be used with caution in those whoalready have significant tachycardia. Childrenreceiving isoproterenol must be carefully mon-itored for arrhythmias.

Epinephrine is a mixed alpha- and beta-adrenergic receptor agonist that is usuallyreserved for patients with cardiogenic shockand low arterial blood pressure. Although epi-nephrine can raise blood pressure effectively, italso increases systemic vascular resistance andtherefore increases the afterload against whichthe heart has to work.

Milrinone is useful in treating patientswith low cardiac output who are refractory tostandard therapy. It works by inhibition of

phosphodiesterase, which prevents the degrada-tion of intracellular cyclic adenosine monophos-phate. Milrinone has both positive inotropiceffects on the heart and significant peripheralvasodilatory effects and has generally beenused as an adjunct to dopamine or dobutaminetherapy in the intensive care unit. It is givenby intravenous infusion at 0.5–1 µg/kg/min,sometimes with an initial loading dose of50 µg/kg. A major side effect is hypotensionsecondary to peripheral vasodilatation, especiallywhen a loading dose is used. The hypotensioncan generally be managed by the administra-tion of intravenous fluids to restore adequateintravascular volume. (McMillan, 2582–2587)

79. (E) 80. (B) 81. (F) 82. (D) 83. (C) 84. (D) Althoughseveral recent studies have failed to demonstratethe effectiveness of intravenous theophylline inhospitalized children receiving frequent inhaledbeta-agonist and systemic glucocorticoid therapy,theophylline may still have a role in the treatmentof children with severe, life-threatening asthmaexacerbations. If a child responds poorly to inten-sive therapy with nebulized albuterol, iprat-ropium, and parenteral glucocorticoids, thenadding intravenous theophylline may be a con-sideration. It is important to remember that thetherapeutic window for theophylline is narrowand patients treated with this therapy must haveserum levels routinely checked.

The indications for the administration ofnebulized epinephrine include moderate-to-severe stridor at rest, respiratory distress,hypoxia, and when stridor does not respondto cool mist. The duration of activity ofracemic epinephrine is less than 2 hours. Themechanism of action is believed to be con-striction of the precapillary arterioles throughthe β-adrenergic receptors causing fluid reup-take from the interstitial space and a decreasein the laryngeal mucosal edema. The symptomsof croup may reappear, but racemic epineph-rine does not “cause” rebound worsening ofthe obstruction. (McMillan, 2406–2407, 1505–1506;Brunton, 961–968)

85. (C) 86. (A) 87. (E) 88. (D) Opioids act by mim-icking actions of endogenous opioid peptidesin binding to receptors located in the brain,

Page 228: Questions - uhs.edu.kh You suspect Lyme disease, most likely contracted by which of the following? (A) ingestion of unripe fruit (B) ingestion of spoiled fruit (C) drinking of contaminated

Answers: 79–97 229

brainstem, spinal cord, and peripheral nerv-ous system. Opioids have dose-dependent res-piratory depressant effects and directly bluntventilatory responses to hypoxia and hyper-carbia. The respiratory depressant effects ofopioids can be increased with coadministra-tion of other sedating drugs, such as benzo-diazepines or barbiturates. Other commonside effects can include constipation, nausea,vomiting, urinary retention, and pruritus.Optimal use of opioids requires proactive andanticipatory management of these side effects.Constipation is the most common side-effectof opioid usage in all age groups. (Behrman,466; Brunton, 569–575)

89. (C) 90. (B) 91. (A) Serum concentration of drugscan reflect the route of administration. Continuousinfusions (C) to obtain steady serum concen-trations and effect can minimize the adverseeffects seen with bolus administration (A) andintermittent administration (B) of drugs. (McMillan,45–46; Brunton, 4–7)

92. (B) 93. (A) Alcohol (ethyl alcohol or ethanol) israpidly absorbed in the stomach and is trans-ported to the liver and metabolized by two path-ways. The primary pathway involves removal oftwo hydrogen atoms to form acetaldehyde, areaction catalyzed by alcohol dehydrogenasethrough reduction of a cofactor nicotinamide-adenine dinucleotide. Some drugs, such asphenytoin, salicylate, and alcohol, saturate theirelimination pathways. When these pathwaysbecome “saturated,” the resultant drug concen-tration in the blood changes disproportionatelyto the dose administered. These drugs followthe principles of zero-order, or Michaelis-Menten, kinetics. The classic principles of elim-ination half-life (t1/2) and clearance (Cl) do notapply to drugs that exhibit zero-order kinetics.

In contrast, the behaviors of most clinicaldrugs follow the principles of linear or first-order pharmacokinetics. This means that theserum concentration or amount of drug in thebody is directly proportional to the doseadministered. For example, if dosing of a drugwhich follows linear pharmacokinetics is dou-bled, the resulting drug concentration in the

blood (at steady state) also doubles. Such pre-dictable behaviors of a drug become the basisfor serum drug level monitoring and doseadjustment. (McMillan, 47; Brunton, 1886-87)

94. (C) 95. (D) 96 (A) Pharmacokinetic interactionsbetween drugs can occur through a variety ofmechanisms. Such interactions may affect theabsorption, distribution, metabolism, and/orexcretion of any given drug. Dose-responsecurves are a useful way to observe the impactof one drug on the pharmacodynamic profile oreffect of a drug or group of drugs. Potencyrelates to the relative concentration of a drug toachieve an effect. In the graph, A is greater thanB is greater than C. Efficacy is the maximaleffect of the drug. In the graph, efficacybetween drugs is A = B = C; and all are moreefficacious than D. (Brunton, 24–29)

97. (A) 98. (B) 99. (A) 100. (B) The clinical responseto recommended dose of a drug can vary con-siderably, even when the dose is administeredrelative to a patient’s body weight, surface area,and age. This variation is a result of inter-individual differences in drug pharmacokineticsand pharmacodynamics and a number of bio-logic variables, including genetic differencesin drug metabolism or drug receptors, and con-current pathophysiology (such as renal orhepatic insufficiency). Individual variabilitywith respect to drug efficacy and possible tox-icity frequently necessitates the adjustment ofdosage regimens for specific patients, espe-cially when prescribing drugs with a narrowtherapeutic window. Problems with clearanceand elimination of the drug can result in asteady rise in serum drug concentration. Often,such medications must be adjusted for thisproblem in clearance, such as in renal insuffi-ciency. “Cumulative” properties refer to theadministration of a “routine” dose at an inter-val that precludes adequate clearance of thedrug. This can be addressed by adjustingthe dose amount and/or dosing interval. TheCYP-450 enzyme system represents more than13 primary enzymes and a number of isozymesof specific gene families. Activity of this systemis affected by genetic polymorphisms, normal

Page 229: Questions - uhs.edu.kh You suspect Lyme disease, most likely contracted by which of the following? (A) ingestion of unripe fruit (B) ingestion of spoiled fruit (C) drinking of contaminated

230 9: Pediatric Therapeutics

development, and presence of other drugs.An understanding of the substrates for spe-cific isozymes and the effects certain drugsmay have on isozyme activity (eg, induction,inhibition) allows the clinician to predict thepossibility of clinically important metabolic-based drug-drug interactions. (McMillan, 47)

Page 230: Questions - uhs.edu.kh You suspect Lyme disease, most likely contracted by which of the following? (A) ingestion of unripe fruit (B) ingestion of spoiled fruit (C) drinking of contaminated

CHAPTER 10

Case Diagnosis and Management

Joseph Y. Allen, MD

This chapter contains clinical scenarios followed by a set of questions. These “real life” scenarios andquestions are designed to assess the examinee’s clinical judgment and practical thinking ability as well as heror his fund of knowledge. Case-based questions are common on national examinations, so it is important tobecome comfortable with them. To maximize your learning experience, answer all of the questions about thecase before looking at the answers. And, refrain from looking ahead at the next question about the case inhopes of obtaining additional information for the question at hand. Not only would this negate the learning-testing experience, it might actually lead to an incorrect answer! For example, diagnostic or therapeutic stepsthat might be correct later in the case might by considered incorrect earlier when certain information was notavailable.

231

Page 231: Questions - uhs.edu.kh You suspect Lyme disease, most likely contracted by which of the following? (A) ingestion of unripe fruit (B) ingestion of spoiled fruit (C) drinking of contaminated

232

DIRECTIONS: This part of the test consists of aseries of cases. Each case is followed by a group ofrelated questions. For each of the multiple choicequestions in this section select the one letteredanswer that is the best response in each case.

Questions 1 through 3

An 18-month-old male toddler presents with pallor.He drinks 64 oz of cow milk per day. The examina-tion is significant only for an obese and playful malewith pallor. Stool is negative for blood.

1. Which laboratory test would most likely revealthe diagnosis?

(A) chest x-ray(B) examination of stool for ova and parasites(C) complete blood count(D) serum haptoglobin(E) bone marrow aspirate

2. Testing reveals a mean corpuscular volume of60 fL (nL 72–86 fL) and an elevated red cell dis-tribution width. The most likely diagnosis is

(A) vitamin B12 deficiency(B) B-cell leukemia(C) hemosiderosis(D) iron deficiency anemia(E) sickle cell disease

3. Appropriate therapy is started for this patient.When should reticulocytosis peak?

(A) 12–24 hours(B) 1–3 days

(C) 5–10 days(D) 2–4 weeks(E) 1–2 months

Questions 4 through 6

A 10-month-old female presents in winter with 2 daysof rhinorrhea, tachypnea, and wheezing. She hasrespirations of 60 breaths per minute, HR of 160 beatsper minute, and an oxygen saturation of 90% onroom air. Examination reveals an alert infant in mildrespiratory distress with mild intercostals retractionsand coarse bilateral expiratory wheezing.

4. Appropriate therapy for this infant shouldbegin with which of the following?

(A) intravenous access and a 20 cc/kg bolusof normal saline

(B) oxygen by nasal canula(C) bag-valve mask followed by rapid-

sequence intubation(D) posterior-anterior and lateral chest

radiographs(E) intravenous corticosteroids

5. Chest radiography reveals bilateral air trap-ping, chest hyperexpansion, and peribronchialthickening. Which of the following is the mostlikely diagnosis?

(A) congestive heart failure(B) acute respiratory distress syndrome

(ARDS)(C) pneumococcal pneumonia(D) Mycoplasma pneumoniae pneumonia(E) viral bronchiolitis

Questions

Page 232: Questions - uhs.edu.kh You suspect Lyme disease, most likely contracted by which of the following? (A) ingestion of unripe fruit (B) ingestion of spoiled fruit (C) drinking of contaminated

Questions: 1–12 233

6. Which of the following is the best additionaltherapeutic option for this patient?

(A) inhaled beta agonist therapy(B) ribavirin(C) extracorporeal membrane oxygenation(D) palivizumab (Synagis)(E) Heliox

Questions 7 through 9

A 6-year-old previously well male presents with 1 week of worsening elbow swelling and fever. Hedenies trauma. Examination reveals a male in milddistress with a temperature to 102°F. His left elbowis warm, erythematous, edematous, and tenderaround the joint. He is holding it in mid-flexion andstrongly resists passive movement.

7. Which is the most likely offending organism forhis condition?

(A) Neiserria gonorrheae(B) Group B streptococcus(C) Pseudomonas aeruginosa(D) Mycobacterium tuberculosis(E) Staphylococcus aureus

8. Which of the following should be performedfirst?

(A) administration of oral antibiotics(B) arthrocentesis of left elbow(C) administration of nonsteroidal anti-

inflammatory agents(D) fasciotomy of the affected limb(E) laboratory evaluation for immunologic

deficiencies

9. Which of the following is a poor prognosticfactor for this condition?

(A) age greater than 6 months(B) gram-positive infection(C) absence of physeal involvement(D) positive Gram stain of joint fluid(E) hip or shoulder involvement

Questions 10 and 11

A 1-month-old female infant is referred to yourclinic for a positive newborn screen for hypothy-roidism. On history, the mother reports she is “agood baby who sleeps all the time but is a sloweater.” She was jaundiced for the first 2 weeks andstools twice a week. Examination reveals an alertinfant with a large tongue, cool skin, a large umbil-ical hernia, edematous extremities, and hypotonia.

10. The most likely cause of this infant’s condi-tion is which of the following?

(A) maternal ingestion of propylthiouracil(B) thyroid dysgenesis(C) iodide transport defects(D) thyrotropin deficiency(E) thyrotropin receptor–blocking antibodies

11. Which of the following management optionsshould be initiated next?

(A) levothyroxine therapy at a dose of 10–15 mcg/kg/day

(B) thyroid scintiscan at the next availabledate

(C) neurodevelopmental consultation(D) radiographs of the legs(E) confirmation of the state screen findings

with a TSH level

Questions 12 through 14

A 4-year-old previously healthy boy presents with 1 day of scrotal swelling. His mother noted his scro-tum to be markedly swollen and thinks his eyes arepuffy. Examination reveals an afebrile child with aBP of 90/50 mm Hg. He is alert with significantbilateral periorbital edema. His abdomen hasascites with no organomegaly. His scrotum andlower extremities have tense pitting edema.

12. The initial laboratory test most likely to pointto the etiology of his illness is as which of thefollowing?

(A) chest radiograph(B) liver biopsy(C) urine analysis(D) hepatitis panel(E) stool guaiac and pH

Page 233: Questions - uhs.edu.kh You suspect Lyme disease, most likely contracted by which of the following? (A) ingestion of unripe fruit (B) ingestion of spoiled fruit (C) drinking of contaminated

234 10: Case Diagnosis and Management

13. Subsequent testing reveals a serum albumin of1 g/dL, a cholesterol level of 560 mg/dL (nL109–189 mg/dL), and normal complement andliver enzyme levels. The most likely diagnosisfor this patient is which of the following?

(A) membranous glomerulonephritis(B) focal segmental glomerulosclerosis(C) poststreptococcal glomerulonephritis(D) membranoproliferative glomeru-

lonephritis(E) minimal-change disease

14. Which of the following statements regardingthis patient’s most likely condition is true?

(A) It almost never responds to steroidtherapy.

(B) Spontaneous bacterial peritonitis is not aconcern.

(C) These patients have an increased ten-dency to hemorrhage.

(D) A low-salt diet is essential during flaresof the illness.

(E) Nonresponse to therapy for at least 1 yearshould prompt a nephrology referral forbiopsy.

Questions 15 through 17

A 7-year-old well female presents to your emer-gency department with episodic headache andhypertension. During the episodes she is sleepy,complains of headaches, vomits, and becomessweaty. Her current vital signs are: T 101°F, HR 150 beats per minute, BP 220/130 mm Hg. She isdiaphoretic, sleepy but arousable, and clutches herhead. Pupils are reactive and papilledema is present.There is no organomegaly and femoral pulses arenormal.

15. After determining that her airway is intact andbreathing sufficient, the first course of actionshould be which of the following?

(A) immediate CT scan of the head to evalu-ate for a mass lesion

(B) lumbar puncture to rule out meningitis(C) administration of an antihypertensive

medication

(D) administration of a benzodiazepine torelieve the patient’s anxiety

(E) placement of a large bore centralcatheter for dialysis

16. Her vital signs are stabilized on the appropri-ate therapy. Now happy and interactive, she istransferred to the ICU. Which option is mostlikely to lead to a diagnosis?

(A) magnetic resonance imaging of the brain(B) urine for catecholamines(C) ECG and echocardiography(D) inquiry into a family history of essential

hypertension(E) surgical consult for laparotomy

17. The urinary vanillyl mandelic acid level returnsat 400 mg/g creatinine (nL < 8 mg/g creatinine).Which of the following is the best treatmentfor this condition?

(A) surgical removal of all tissue(B) radiologic injection of cyclophos-

phamide directly(C) watchful waiting as most regress

spontaneously(D) oral mineralcorticoid therapy(E) oral propylthiouracil therapy

Questions 18 through 20

You are seeing a 7-day-old male infant for a well-child check. The baby is breastfeeding well. He hashad no fever or emesis. He passed his first stool at 3 days of age, and has not passed another stool.Examination reveals an afebrile, well appearing,and vigorous baby. The abdomen is firm andslightly distended with bowel sounds present. Theperianal area is slightly erythematous and rectalexamination reveals increased tone with no stoolpresent in the rectal vault.

18. Of the following, which is the best initial diag-nostic test?

(A) stool assay for Clostridium difficile toxin(B) abdominal ultrasound(C) serum TSH(D) barium enema(E) diagnostic trial of mineral oil

Page 234: Questions - uhs.edu.kh You suspect Lyme disease, most likely contracted by which of the following? (A) ingestion of unripe fruit (B) ingestion of spoiled fruit (C) drinking of contaminated

Questions: 13–24 235

19. The initial diagnostic approach is unsuccessful.Which test to should you perform next to diag-nose this patient?

(A) serum TSH(B) stool for esosinophils(C) anal manometry(D) upper gastrointestinal series with small

bowel follow through(E) rectal biopsy

20. Testing reveals an absence of the Meissner andAuerbach plexus. With proper treatment, whatis the most likely prognosis for this patient?

(A) he will likely be continent(B) he will need a colostomy for his entire

childhood(C) he will need a total colectomy and likely

have serious incontinence problems(D) he will need a small bowel transplant

and require lifelong parenteral nutrition(E) he will be at great risk for frequent life-

threatening intra-abdominal infections

Questions 21 and 22

A previously well 2-year-old girl is brought to you forevaluation of a “broken elbow.” The father reportsswinging her round and round by her left arm and leg.She now is crying and not moving her left arm as it isheld in a flexed and slightly pronated position at herside. No effusions or point tenderness are discernible.

21. Which of the following is the most appropriateinitial management?

(A) intravenous line placement for sedationand reduction

(B) hyperpronation or supination-flexion ofthe arm

(C) complete blood count and erythrocytesedimentation rate

(D) social work evaluation for abuse(E) radiographic evaluation of the left elbow

22. When counseling the father regarding his child’scondition, you should tell him which of thefollowing?

(A) recurrence is unlikely

(B) splinting at bedtime is desirable(C) gymnastics should be avoided(D) suspicion of abuse requires that you

report this injury(E) children outgrow predisposition for this

condition

Questions 23 through 25

A 16-year-old female living in a shelter presents with3 days of worsening lower abdominal pain. Shereports multiple unprotected sexual encounters. Sheis febrile to 39°C and is ill appearing. She has bilaterallower quadrant tenderness without rebound. Herpelvic examination reveals purulent vaginal dis-charge with adnexal and cervical motion tenderness.Pregnancy test is negative.

23. What is the most appropriate managementstrategy at this time?

(A) inpatient admission with intravenousantibiotics for pelvic inflammatorydisease

(B) social service referral for in loco parentisdesignation prior to initiation of therapy

(C) initiation of antibiotic therapy in theemergency department and dischargewith close outpatient follow-up

(D) urine analysis and empiric intramuscu-lar antibiotic therapy for cystitis

(E) nonsteroidal therapy and oral contra-ceptive pills for pregnancy prophylaxis

24. The patient remains febrile and has worseningabdominal pain despite 72 hours of antibiotictherapy. At this time what is the best next step?

(A) continue therapy at the scheduled doses(B) add diphenhydramine for a potential

drug reaction(C) consult gynecology for imaging and

drainage of a possible tubo-ovarianabscess

(D) repeat blood cultures and obtain a C-reactive protein to help monitorresponse to therapy

(E) consult infectious disease for evaluationof the possibility of methicillin-resistantS aureus infection

Page 235: Questions - uhs.edu.kh You suspect Lyme disease, most likely contracted by which of the following? (A) ingestion of unripe fruit (B) ingestion of spoiled fruit (C) drinking of contaminated

236 10: Case Diagnosis and Management

25. The patient improves and is ready for discharge.Which of the following is true about thispatient’s condition?

(A) recurrence is rare(B) recurrence is associated with an

increased risk of infertility(C) douching and oral contraceptives may

decrease the risk of recurrence(D) male partners with N gonorrheae are

always symptomatic(E) ectopic pregnancies are not a concern

after PID

Questions 26 and 27

An 11-year-old boy presents after cutting his ankle ona rusty piece of metal at the junkyard. His fatherreports he is positive his son’s immunizations are cur-rent because the patient received booster doses justprior to kindergarten at 5 years of age. Examinationreveals a healthy boy in no distress with a 4 cm bleed-ing laceration on the posterior aspect of the lower leg.

26. What should be the first step in management ofthis wound?

(A) radiography to evaluate for metallicparticles

(B) irrigation of the wound with normalsaline

(C) intravenous antibiotics directed againstgram-positive organisms

(D) surgery assistance for this complicatedinjury

(E) direct pressure on the wound forhemostasis

27. The wound is treated. For tetanus prophylaxis,which of the following should this patient receive?

(A) no prophylaxis(B) Td only(C) Td and tetanus immune globulin (TIG)(D) DTaP(E) TIG only

Questions 28 and 29

A 6-month-old boy is brought in by his mother forcrying. Your examination reveals a thin boy with

tender swelling around the midshaft of his left femur.He cries when the leg is manipulated, but is comfort-able when left alone. An x-ray reveals a transversemidshaft femur fracture. The father tells you thepatient’s 16-month-old brother lifted him off the bedand dropped him on the floor earlier that evening.

28. What should be the next step in the manage-ment of this patient?

(A) review with parents the history of theinjury

(B) MRI of left lower extremity(C) skeletal survey(D) internal fixation of the fracture(E) fibroblast assay

29. During the evaluation the father asks that theybe discharged immediately as he has to be atwork early. What is the most appropriatecourse of action?

(A) let them go home and follow-up withtheir pediatrician the next day

(B) splint the child with follow-up by anorthopedist

(C) call security to forcibly incarcerate thefather

(D) directly confront the parents of theircomplicity in allowing this to happen

(E) inform the family that the safety of thechild may be at risk as the history givendoes not match with his pattern ofinjuries

Questions 30 and 31

A 3-month-old female presents with 2 days of cryingand decreased oral intake. She is afebrile, HR is 280beats per minute, and BP is 85/40 mm Hg. Her satu-rations are 99% on room air. She is alert and easilyconsolable, her lungs are clear, and she is tachycardicwith no murmurs audible. There is no organomegalyand the peripheral pulses are normal in strength.

30. Which test should you order first to confirm thediagnosis?

(A) complete blood count and differential(B) echocardiogram(C) blood glucose

Page 236: Questions - uhs.edu.kh You suspect Lyme disease, most likely contracted by which of the following? (A) ingestion of unripe fruit (B) ingestion of spoiled fruit (C) drinking of contaminated

Questions: 25–37 237

(D) thyroxine and TSH levels(E) electrocardiogram

31. Her heart rate and blood pressure remainunchanged after testing. The initial treatmentfor the most likely cause of her condition iswhich of the following?

(A) adenosine(B) defibrillation(C) verapamil(D) digoxin(E) ibuprofen

Questions 32 and 33

A 15-year-old previously well female presents with1 week of hair loss. She denies fever, weight loss, ormedications. On examination, she is pleasant butnervous about her hair loss. Her scalp revealspatches of complete hair loss with small brokenhair that easily pull out at the edges. The scalp issmooth and no inflammation is seen. Microscopicexamination of the shafts reveals the stubs toresemble exclamation points.

32. What illness does this patient most likely have?

(A) tinea capitis(B) alopecia areata(C) trichotillomania(D) hair traction alopecia(E) chemical exposure

33. You tell the patient the likely course of thisillness is which of the following?

(A) spontaneous resolution(B) resolution after chemotherapy(C) difficult to predict(D) progression to total hair loss and then

resolution(E) improvement with dietary change

Questions 34 through 36

A 6-month-old female develops a persistent coughwith progressively worsening paroxysms andcyanosis. There is occasional posttussive emesis.The child is afebrile. Between coughing spells, thephysical examination is normal.

34. At this time, what would be most importantquestion to ask the family regarding patienthistory?

(A) birth weight(B) immunizations(C) consanguinity(D) early infant deaths in relatives(E) family history of reactive airways disease

35. The white blood cell count on the patient is32,000/mm3, with 80% lymphocytes and 2%mononuclear cells. What is the most appropri-ate next step at this time?

(A) order a bone marrow examination(B) prescribe intravenous gammaglobulin(C) prescribe oral azithromycin(D) perform a lumbar puncture(E) repeat the blood count in 24 hours

36. The most appropriate method to identifythe responsible organism is via which of thefollowing?

(A) throat swab(B) nasopharyngeal swab(C) blood culture(D) sputum culture(E) bronchoscopy

Questions 37 through 39

A 17-year-old female presents to your clinic withcomplaints of recurrent headaches for 6 months.They are described as circumferential; onset is notassociated with time of day. There has been no eme-sis and the headaches have not interfered with activ-ities. Her weight is 140 kg and her BP is 140/90 mmHg. Her examination reveals bilateral papilledema,and an otherwise normal neurologic examination.

37. What is the best next step in management ofthis patient?

(A) oral administration of nifedipine(B) determination of renin levels(C) intravenous nitroprusside drip(D) computerized tomography of the head(E) intravenous mannitol and furosemide

Page 237: Questions - uhs.edu.kh You suspect Lyme disease, most likely contracted by which of the following? (A) ingestion of unripe fruit (B) ingestion of spoiled fruit (C) drinking of contaminated

238 10: Case Diagnosis and Management

38. The imaging study is normal, reveals no massand normal-sized ventricles. At this point youshould do which of the following?

(A) perform lumbar puncture(B) treat with dexamethasone(C) treat with acetazolamide(D) refer her for opthalmalogic evaluation(E) refer her for psychiatric evaluation

39. You proceed and she has relief from her symp-toms. Despite appropriate care she begins todevelop visual loss and optic nerve atrophy issuspected. Your best course of action is to dowhich of the following?

(A) increasing the dose of dexamethasone(B) increasing the frequency of lumbar

punctures(C) increasing the dose of acetozolamide(D) refer her to a surgeon for gastric banding

to speed up weight loss(E) refer to an ophthalmologist for evalua-

tion for optic nerve fenestration

Questions 40 through 42

A 10-month-old female presents to the emergencydepartment with a 2-day history of runny nose andfever to 102°F. On examination, her temperature is103°F, HR 140 beats per minute, and a RR 30 timesper minute. She is alert and playful with copiousrhinorrhea. After the examination is complete, shebecomes stiff and displays tonic-clonic movementsof all four extremities.

40. Which of the following should be your firsttask?

(A) obtain whole blood glucose(B) administer intravenous lorazepam

(Ativan)(C) perform lumbar puncture(D) establish airway patency(E) administer intramuscular fosphenytoin

(Cerebyx)

41. After 5 minutes, the seizure ceases and the res-piratory rate is 30 breaths per minute. Thepatient is sleepy but arousable. Fundoscopicexamination is normal as is the remainder of the

physical examination. Rectal acetaminophen isgiven. Which of the following tests should beperformed at this time?

(A) electroencephalogram(B) skull radiographs(C) arterial blood gas(D) computerized tomography scan of the

head(E) lumbar puncture

42. The patient quickly becomes alert, happy, andplayful. She is afebrile and has a normal exam-ination. All ordered tests are normal. Which ofthe following management plans is mostappropriate at this time?

(A) discharge home after instructionsregarding home management of feverand seizures

(B) discharge home on phenytoin(C) discharge home on phenobarbital(D) discharge home with alternating doses

of ibuprofen and acetaminophen every3 hours for the next 3 days

(E) admission for observation and MRI

Questions 43 through 45

A 6-year-old female presents with short stature. Herfamily reports she has been well and has had noother medical problems. Her diet and review of sys-tems is unremarkable.

43. Which of the following would be most impor-tant to know at this time?

(A) maternal age at menarche(B) parental growth rate and height(C) paternal age at conception(D) maternal age at conception(E) sibling growth rate and height

44. The physical examination reveals a pleasantgirl at less than the 5th percentile for heightand 10th percentile for weight. In addition toher short stature, she has a broad chest, cubi-tum valgum, and 2/6 systolic ejection murmurat the right upper sternal border. Which test ismost likely to confirm the etiology of her shortstature at this time?

Page 238: Questions - uhs.edu.kh You suspect Lyme disease, most likely contracted by which of the following? (A) ingestion of unripe fruit (B) ingestion of spoiled fruit (C) drinking of contaminated

Questions: 38–50 239

(A) serum LH/FSH levels(B) abdominal ultrasound(C) growth hormone levels(D) karyotype(E) bone age analysis of the nondominant

hand

45. Of the following management options, which isindicated for this patient?

(A) transthoracic echocardiogram(B) halo bracing for cervical laxity(C) counseling regarding the likelihood of

severe mental deficiency(D) hysterectomy to prevent the develop-

ment of endometrial carcinoma(E) initiation of thyroid hormone replace-

ment therapy

Questions 46 through 48

A 13-month-old boy presents with 5 days of fever to103°F. His temperature is 102.8°F, HR 160 beats perminute, and RR 36 times per minute. On examination,he is found to be irritable with markedly injected con-junctiva, a strawberry tongue, and red, cracked lips. A 2-cm lymph node is present in the left anterior cer-vical chain. There is no meningismus. His lungs areclear and he is tachycardic. A diffuse erythematousblanching rash is present on his chest and extremities.No desquamation of the fingertips is noted.

46. Among the following, which diagnostic test ismost important in establishing the diseasemost likely to be causing this constellation ofsymptoms?

(A) complete blood count(B) erythrocyte sedimentation rate(C) viral culture(D) clinical judgment(E) presence of antinuclear antibody

47. After the diagnosis is confirmed, therapy is ini-tiated with improvement in symptoms within24 hours. Which of the following complicationsis most likely to occur in this illness?

(A) thrombocytopenia(B) sterility

(C) hydrops of the gallbladder(D) ulcerative colitis(E) transverse myelitis

48. Prior to discharge, the parents state that theirchild needs his immunizations updated. Youshould counsel the parents that their sonshould follow which course of treatment?

(A) stay on schedule for his vaccinations(B) have measles and varicella immuniza-

tions deferred for 11 months after IVIG(C) not be given diphtheria-tetanus-acellu-

lar pertussis vaccine because it maypotentiate a relapse of his illness

(D) not receive influenza vaccine because hewill be on chronic aspirin therapy

(E) have his primary series of vaccinesrepeated

Questions 49 and 50

A 2-year-old is brought to your emergency departmentfor refusal to walk after he tripped and fell while run-ning. Examination reveals a well-appearing afebrilechild in no distress. His left leg has full range of motionand some point tenderness in the distal tibia.

49. The most appropriate first step in your evalu-ation is to obtain which of the following?

(A) a C-reactive protein(B) a blood culture(C) radiographs of the tibia(D) a social work consultation(E) a serum alkaline phosphatase

50. Your initial evaluation confirms your suspi-cion. At this point you should perform whichtask next?

(A) make a referral to child protective serv-ices for abuse

(B) place a long-term intravenous catheterfor intravenous antibiotics

(C) arrange an immediate orthopedic referral(D) perform a needle aspirate of the affected

area(E) immobilize the affected limb in a splint

with outpatient follow-up

Page 239: Questions - uhs.edu.kh You suspect Lyme disease, most likely contracted by which of the following? (A) ingestion of unripe fruit (B) ingestion of spoiled fruit (C) drinking of contaminated

240 10: Case Diagnosis and Management

Questions 51 through 53

You are called to the nursery to evaluate a 3-day-old, full-term male infant with lethargy. The nursereports the infant was feeding well on standard for-mula until 4 hours previously. He has had no emesis.There is no maternal history of fever or rash. Theinfant currently is afebrile with a HR 110 beats perminute, RR of 50 times per minute, and a BP of80/45 mm Hg. He is lethargic but the examination isotherwise unremarkable. There are no dysmorphicfeatures. Whole blood glucose is normal. Serum cal-cium and electrolyte results are pending.

51. What should be your next step in the evalua-tion of the patient?

(A) obtain emergent abdominal ultrasound(B) obtain complete blood count, urine

analysis and lumbar puncture as well ascultures of blood, urine, and CSF

(C) urine drug screen(D) obtain emergent upper gastrointestinal

series with small bowel follow-through(E) obtain emergent magnetic resonance

imaging of the head to evaluate for anintracranial hemorrhage

52. The neonatal evaluation for sepsis is completeand the neonate is placed on antibiotics. The nextday there is no evidence of infection and theinfant has not improved. Additional studies areobtained, including a serum ammonia which is1150 mmol/L (nL 64–107) and a blood pHof 7.36. Medical therapy is initiated. The urinaryorotic acid level returns and is markedly ele-vated as well. Which of the following is the mostlikely diagnosis?

(A) ornithine transcarbamylase deficiency(B) carbamoyl synthetase deficiency(C) methylmalonic academia(D) carnitine palmitoyl transferase deficiency(E) glycogen synthetase deficiency

53. Six hours after institution of treatment withappropriate intravenous doses of arginine, theserum ammonia is 1200 mmol/L. The best courseof action at this time is which of the following?

(A) increase the rate of IV arginine to clearammonia faster

(B) begin a double volume exchangetransfusion

(C) transfer the patient to a center wherehemodialysis can be performed

(D) discuss DNR status with the family(E) allow another 6 hours of therapy as

ammonia in tissue needs to be metabo-lized first and therapy can be toxic aswell

Questions 54 through 56

A 3-year-old girl is seen in the emergency depart-ment for bruising. Her family denies fever or weightloss but states she had a “cold” 3 weeks ago. She isafebrile and the remaining vital signs are normal.She is happy and playful and has generalized ecchy-moses and petechiae.

54. What should be the first test you obtain?

(A) bone marrow aspirate(B) Neiserria meningitidis latex assay of the

cerebrospinal fluid(C) Rickettsia rickettsiae serology(D) skeletal survey looking for healing

fractures(E) complete blood count and differential

55. The laboratory results return and the platelets are10,000/mm3. A bone marrow aspirate demon-strates increased megakaryocytes but is other-wise normal. Which of the following is anindication for WinRho (anti-RhoD antibodies)?

(A) platelet count less than 100,000/mm3

(B) fever greater than 39°C(C) splenomegaly(D) epistaxis(E) bone marrow with megakaryocyte

hypoplasia

56. Which of the following would be an indicationfor splenectomy in this patient?

(A) platelet count below 10,000/mm3

(B) gingival bleeding(C) persistence of thrombocytopenia for

more than 1 month

Page 240: Questions - uhs.edu.kh You suspect Lyme disease, most likely contracted by which of the following? (A) ingestion of unripe fruit (B) ingestion of spoiled fruit (C) drinking of contaminated

Questions: 51–61 241

(D) persistence of thrombocytopenia formore than 1–2 years

(E) presence of splenomegaly and anemia

Questions 57 and 58

A 4-year-old boy presents to the emergency depart-ment with a chief complaint of pallor. He was welluntil 1 week previously when he developed bloodydiarrhea that resolved with oral antibiotics. He isafebrile with a BP of 150/100 mm Hg and a HR of130 beats per minute. He is alert and fundoscopicexamination is normal. His examination is signifi-cant for pallor.

57. Which test is most likely to reveal the diagnosis?

(A) complete blood count and smear(B) stool culture(C) computerized tomography of the head(D) renal ultrasound(E) urine myoglobin levels

58. The laboratory results reveal a hemoglobin of8 g/dL, a BUN and creatinine of 40 and 1.8,respectively. The urine output is normal. Whatshould you tell the family regarding the treat-ment of this disease?

(A) aggressive medical management anddialysis result in the majority of thepatients doing well

(B) high-dose steroids are essential in thetreatment of this illness

(C) most patients eventually require renaltransplantation

(D) antibiotics need to be continued for 21 days

(E) recurrence of this illness is frequent

Questions 59 and 60

An 18-month-old female presents to the emer-gency department with fever up to 102°F, a barkycough, and stridor. The examination shows an RRof 60 breaths per minute with an oxygen saturationof 95% while breathing room air, marked stridor,moderate substernal retractions, and equal aerationwithout wheezes or rhonchi.

59. As the patient is being evaluated, initial ther-apy for this patient should begin with which ofthe following?

(A) acetaminophen per rectum(B) nebulized dexamethasone(C) nebulized racemic epinephrine(D) humidified air only in a position of

comfort(E) stat portable AP and lateral neck films

60. Despite therapy the patient becomes more toxicappearing and is drooling. The parents subse-quently report the child has received no immu-nizations. Her respiratory rate is now 70 breathsper minute and she appears sleepy. At this pointthe best intervention is which of the following?

(A) blind intubation with a laryngeal maskairway

(B) humidified oxygen and observationwith the patient in a position of comfort

(C) rapid sequence intubation in the emer-gency department

(D) anesthesia assistance for intubation inthe operating room with surgical availability

(E) prompt administration of intravenouscefotaxime

Questions 61 and 62

A 4-year-old male presents to the office with wheez-ing and increased work of breathing for the fifth timein the past year. On examination, his RR is 30 breathsper minute and oxygen saturation is 98% whilebreathing room air. He is talking in complete sen-tences and there is expiratory wheezing with a pro-longed expiratory phase.

61. What is the first treatment he should receive?

(A) nebulized beta agonist therapy(B) intravenous beta agonist therapy(C) subcutaneous epinephrine(D) oxygen via 15 L nonrebreather(E) intramuscular steroids

Page 241: Questions - uhs.edu.kh You suspect Lyme disease, most likely contracted by which of the following? (A) ingestion of unripe fruit (B) ingestion of spoiled fruit (C) drinking of contaminated

242 10: Case Diagnosis and Management

62. The patient improves after receiving the treat-ment selected. Further history reveals that thepatient has been using inhaled albuterol threetimes per week to control wheezing. He awakenseach night with coughing, but seems better in themorning. In addition to albuterol and a shortcourse of oral steroids what is the best additionalmedicine he should receive at this time?

(A) epinephrine auto injector (Epi-Pen) to beused as needed

(B) salmeterol to be used in divided dosesdaily

(C) fluticasone to be used in divided dosesdaily

(D) theophylline to be used in divided dosesdaily

(E) cetirizine to be used in divided dosesdaily

Questions 63 through 65

A previously well 9-year-old Hispanic male presentswith 2 days of yellow eyes and abdominal pain. Hevisited Mexico 1 month previously. On examinationhe is febrile to 101°F, has scleral icterus, and moderateright upper quadrant tenderness. The liver is moder-ately enlarged.

63. What would be the most helpful additionalhistory?

(A) sickle cell disease in the family(B) excessive carrot intake(C) history of malar rash(D) acetaminophen usage(E) vaccination status prior to travel

64. What test would most likely confirm his diag-nosis?

(A) hepatitis A serology(B) hepatitis B serology(C) 24-hour copper excretion(D) abdominal ultrasound(E) acetaminophen level

65. Diagnostic testing confirms the diagnosis andthe family has questions about the disease. Youshould tell them which of the following?

(A) an effective vaccine is not routinelyavailable

(B) the recurrence risk is approximately 25%(C) if he were to contract hepatitis D simul-

taneously, he would likely need a livertransplant

(D) fulminant hepatitis is uncommon inchildren with hepatitis A infection

(E) intimate sexual contact is the most com-mon route of transmission

Questions 66 through 68

You are called to see a 2-hour-old male with cyanosisand tachypnea. Oxygen saturation is 80% whilebreathing room air and the RR is 60 breaths perminute with BP of 80/50 mm Hg. The baby is cyan-otic and there are no murmurs. The rest of the exam-ination is unremarkable. The baby is placed in on anFiO2 of 1.0 by head hood and arterial blood gasesreveal the PaO2 to be 30 mm Hg.

66. What is the most likely etiology of the hypox-emia?

(A) methemoglobinemia(B) cyanotic congenital heart disease(C) sepsis(D) pneumonia(E) arteriovenous fistula

67. A chest x-ray shows normal lung fields, slightlygenerous cardiothymic silhouette, and a narrowupper mediastinum. An electrocardiogram isnormal.What would the most likely diagnosisbased on this information?

(A) transposition of the great arteries(B) tetralogy of Fallot(C) ventricular septal defect(D) endocardial fibroelastosis(E) total anomalous pulmonary venous

return

68. The airway is stabilized and prostaglandin E1therapy is initiated. Echocardiography con-firms the diagnosis. At this point what is thebest intervention?

Page 242: Questions - uhs.edu.kh You suspect Lyme disease, most likely contracted by which of the following? (A) ingestion of unripe fruit (B) ingestion of spoiled fruit (C) drinking of contaminated

Questions: 62–73 243

(A) emergent cardiac catheterization andatrial septostomy

(B) nothing until the patient’s hypoxemiaimproves

(C) empiric antibiotics for sepsis(D) emergent cardiovascular surgery

consultation for immediate arterialswitch

(E) dobutamine and milrinone to enhancecardiac function

Questions 69 through 71

A 6-year-old previously well African Americanchild presents with new-onset jaundice, dark urine,and pallor. There is a history of a recent mild upperrespiratory tract infection. Vital signs are normal.Physical examination is remarkable for icterus andpallor. Laboratory examination reveals hemoglobinof 7 g/dL with a normal platelet and white bloodcell count. The total bilirubin is 4 mg/dL.

69. Which of the following historical findings ismost likely to indicate the diagnosis?

(A) His father has sickle cell trait.(B) Baby aspirin given to him last year was

associated with dark urine.(C) His grandmother was diagnosed with

leukemia last year.(D) He was diagnosed as being iron defi-

cient 4 years ago.(E) His healthy cousin returned from a trip

to Mexico 1 month ago.

70. The bilirubin is predominantly unconjugatedand the reticulocyte count is 12%. Further his-tory reveals that for the past 2 days he has beengiven an antibiotic that his grandfather wastaking for a urinary tract infection. What is themost likely diagnosis?

(A) sickle cell disease(B) preleukemia(C) infectious hepatitis(D) spherocytosis(E) glucose-6-phophate-dehydrogenase

deficiency

71. What is the most appropriate management atthis time?

(A) immediate transfusion to bring thehemoglobin to 14 g/dL

(B) immediate transfusion to bring thehemoglobin to 10 g/dL

(C) withhold transfusion and follow vitalsigns and hemoglobin level

(D) initiate prednisone at 2 mg/kg/day(E) transfusion to bring the hemoglobin to

10 g/dL and prednisone at a dose of 2 mg/kg/day

Questions 72 and 73

A 16-year-old female presents with leg weaknessafter recovering from an upper respiratory illness.On examination her vital signs are normal. She isunable to stand alone. Motor strength is 5/5 in thearms and 2/5 in the legs. Deep tendon reflexes areabsent in the legs.

72. What finding is classically associated with thisillness?

(A) hydrocephalus(B) elevated serum C-reactive protein(C) myoglobulinuria(D) elevated cerebrospinal fluid protein(E) presence of Clostridium species on stool

culture

73. Testing demonstrates marked slowing of nerveconduction velocity. What would the mostappropriate intervention be at this time?

(A) administration of intravenous immuneglobulin

(B) discharge and reassurance about the overall benign nature of this disease

(C) administration of intravenous freshfrozen plasma

(D) supplemental oxygen via nasal cannula

(E) administration of intravenous interferon-beta

Page 243: Questions - uhs.edu.kh You suspect Lyme disease, most likely contracted by which of the following? (A) ingestion of unripe fruit (B) ingestion of spoiled fruit (C) drinking of contaminated

244 10: Case Diagnosis and Management

Questions 74 and 75

A 5-year-old male presents with a 5-lb weight lossover the previous 2 weeks and nighttime enuresis.On examination, he is alert and talkative with a BPof 90/60 mm Hg and a HR of 130 beats per minute.Mucous membranes are sticky and respirations arerapid and deep. The bedside whole blood glucose is750 mg/dL. Intravenous access is obtained.

74. What should the first step in management be?

(A) regular insulin 0.1 U/kg IV push(B) bicarbonate 1 meq/kg IV over 1 hour(C) endotracheal intubation(D) 20 cc/kg isotonic crystalloid solution

over 1 hour(E) a diet soda

75. After your intervention, the whole blood glu-cose is 485 mg/dL. Intravenous fluids andinsulin are given, and the patient is admittedfor further care. Which of the following meta-bolic abnormalities is most likely to occurduring insulin therapy?

(A) hypokalemia(B) hyperkalemia(C) hyperphosphatemia(D) hypercalcemia(E) hypermagnesemia

Questions 76 through 78

A 13-month-old female in day care presents to theoutpatient clinic with 3 days of rhinorrhea, cough,and fever to 101°F. Her examination is significantfor clear nasal congestion and red tympanic mem-branes that are mobile with insufflation. The motherasks for antibiotics.

76. What is the most appropriate therapy at thistime?

(A) amoxicillin 80–90 mg/kg/day for 10 days(B) amoxicillin 40–50 mg/kg/day for 5 days(C) ceftriaxone 50 mg/kg IM in one dose(D) otorhinolaryngology consult for

tympanocentesis(E) acetaminophen and fluids

77. The mother brings her child in 6 days later forpersistent fever (up to 103°F) and pulling atthe left ear. On examination, the left tympanicmembrane is bulging and immobile. The bonylandmarks are not visible. The mother statesthe day care won’t let her back in withoutantibiotics. The most appropriate therapy atthis time is which of the following?

(A) amoxicillin 80–90 mg/kg/day for 10 days

(B) erythromycin 40–50 mg/kg/day for 10 days

(C) ceftriaxone 50 mg/kg IM in one dose(D) otorhinolaryngology consult for

drainage(E) acetaminophen and fluids

78. The patient returns again in 2 days with contin-ued fevers to 104°F. The child is still fussy. Thereis redness, swelling, and tenderness posterior tothe left ear and the pinna is displaced forward.The most appropriate therapy at this time iswhich of the following?

(A) amoxicillin 80–90 mg/kg/day for 10 days

(B) amoxicillin 40–50 mg/kg/day for 5 days(C) ceftriaxone 50 mg/kg IM in one dose(D) otorhinolaryngology consult for drainage(E) acetaminophen and fluids

Questions 79 through 81

A mother brings her 17-year-old daughter to yourclinic for evaluation of 6 weeks of fatigue, 10-lbweight loss, and listlessness. The vital signs includea HR of 70 beats per minute and BP of 90/50 mmHg. Examination reveals a thin girl with a flat affect,but no other abnormalities.

79. The best next step in this patient’s manage-ment would be which of the following?

(A) obtain a serum beta-human chorionicgonadotrophin (b-HCG)

(B) a referral to an oncologist for a bonemarrow aspirate

(C) obtain stat urine drug screen(D) reassure the mother that this is normal

behavior(E) interview the patient alone

Page 244: Questions - uhs.edu.kh You suspect Lyme disease, most likely contracted by which of the following? (A) ingestion of unripe fruit (B) ingestion of spoiled fruit (C) drinking of contaminated

Questions: 74–85 245

80. Further discussion with the patient reveals signsof depression. However, she is interactive anddenies any drug use or suicidal ideation. Shehas friends and makes As and Bs in school. Theresults of screening laboratory work are normal.Appropriate initial management should bewhich of the following?

(A) immediate psychiatry consultation due tothe medicolegal risk posed by the patient

(B) immediate inpatient admission with 24-hour suicide watch

(C) urine collection for illicit substancesagainst the patient’s will

(D) initiation of a selective serotonin reup-take inhibitor (SSRI) and scheduling ofoutpatient counseling with a therapist

(E) reassurance to the patient that it is a signof puberty and the feelings will pass

81. You are called 3 weeks later as the patient isbrought to an outside emergency departmentbecause of increasing hyperactivity, loqua-ciousness, and 3 days of insomnia. The patient’sparents state that she purchased a large amountof goods. Urine toxicology is negative. Thispatient most likely has which of the following?

(A) a drug reaction(B) bipolar disorder(C) schizophrenia(D) reaction to sexual abuse(E) normal adolescent behavior

Questions 82 through 84

A 7-year-old girl presents with a 3-week history offatigue, 5-lb weight loss, and listlessness. Examinationis significant for a thin girl who appears tired.Petechiae and ecchymoses are present over her trunkand extremities. The complete blood count reveals awhite blood cell count of 85,000/mm3, hemoglobin of7 g/dL, and platelets of 15,000/mm3. The differentialreveals 80% blasts and 20% lymphocytes. She is febrileto 103°F and has a BP of 90/50 mm Hg.

82. Appropriate empiric antibiotic therapy wouldbegin with which of the following?

(A) intramuscular ceftriaxone(B) intravenous clindamycin and cefuroxime

(C) intravenous ceftazidime(D) intravenous amphotericin and

voriconazole(E) oral ciprofloxacin

83. Bone marrow examination confirms the diag-nosis of acute lymphocytic leukemia. Which ofthe following would indicate a poor prognosis?

(A) patient’s age (7 years)(B) patient’s initial white blood count

(85,000/mm3)(C) hyperdiploidy(D) absence of the Philadelphia chromo-

some (9:22)(E) absence of blasts in the cerebrospinal fluid

84. Induction chemotherapy is begun. Which ofthe following patterns of metabolic abnormal-ities might be expected to occur?

(A) hypokalemia, hypouricemia, hypophos-phatemia

(B) hypokalemia, hypouricemia, hyper-phosphatemia

(C) hyperkalemia, hyperuricemia, hyper-phosphatemia

(D) hyperkalemia, hypouricemia,hypophosphatemia

(E) hypokalemia, hyperuricemia, hypophos-phatemia

Questions 85 through 87

A first-time mother brings her 6-week-old full-termmale infant in because of excessive crying. Shestates he is feeding well and has had no fever butseems to cry “all the time” for the last 3 weeks.Examination reveals an alert and vigorous infantwho has gained weight very well since the 2-weekcheck-up and is currently cooing with no abnormal-ities noted on examination.

85. What should the initial approach to manage-ment be?

(A) a full sepsis evaluation(B) a skeletal survey(C) a radioisotope milk scan(D) a urine analysis and urine culture(E) counseling, reassurance, and close

follow-up

Page 245: Questions - uhs.edu.kh You suspect Lyme disease, most likely contracted by which of the following? (A) ingestion of unripe fruit (B) ingestion of spoiled fruit (C) drinking of contaminated

246 10: Case Diagnosis and Management

86. What would be the most appropriate manage-ment option at this time?

(A) suggesting that a single individual, prefer-ably the mother, act as the sole caregiver

(B) swaddling of the infant(C) warm water feedings three times a day

to decrease gastric upset(D) vigorous shaking of the baby back and

forth to simulate uterine movements(E) oral paregoric to calm the infant’s

immature nervous system

87. If this infant had presented with acute onset ofirritability and crying and an episode of bil-ious emesis, differential diagnosis should focuson which of the following diagnoses?

(A) hair tourniquet around a digit(B) milk protein intolerance(C) occult fracture(D) gastroesophageal reflux(E) malrotation with volvulus

Questions 88 and 89

A 10-month-old female is brought to the emergencydepartment for evaluation of streaks of blood on thesurface of the stools. The parents deny any historyof travel, diarrhea, or fever.

88. What additional history would be most likelyto point to a specific diagnosis in this patient?

(A) family history of hemophilia(B) dietary iron intake(C) history of constipation(D) presence of rotavirus in day care

contacts(E) family history of Crohn disease

89. Examination reveals a small fissure without erythema in the posterior area of the anus. Whatis the most appropriate treatment at this time?

(A) prescribing milk of magnesia(B) encouraging the use of a rectal dilator to

stretch the internal anal sphincter(C) prescribing a stool softener and titrating

the dose to desired stool consistency

(D) prescribing an oral antibiotic to preventsecondary infection of the fissure

(E) referral to a naturopathic practitionerfor colonic irrigation

Questions 90 and 91

An 8-year-old girl presents with a 3-month historyof intermittent joint swelling and stiffness and issubsequently diagnosed with juvenile idiopathicarthritis (JIA).

90. Which of the following types of JIA is mostcommonly associated with development ofsevere arthritis?

(A) systemic onset JIA(B) pauciarticular JIA without ocular

involvement(C) pauciarticular JIA with associated ocular

involvement(D) polyarticular rheumatoid factor positive

JIA(E) polyarticular rheumatoid factor negative

JIA

91. Vision loss associated with JIA is most com-monly due to which of the following?

(A) optic neuritis(B) retinal artery thrombosis(C) retinal detachment(D) beta carotene malabsorption(E) chronic iridocyclitis/uveitis

Questions 92 through 94

A 4-month-old male infant presents with 3 days ofprofuse watery nonbloody diarrhea and nonbiliousemesis. He was being fed milk, orange juice, and ricewater. Vital signs are T 99°F, HR 170 beats per minute,and BP 66/40 mm Hg. His eyes and fontanelle aresunken and he has poor skin turgor with a capillaryrefill time of 3–4 seconds; pulses are normal.

92. This description is most consistent with whichof the following?

(A) hypovolemic shock, compensated(B) hypovolemic shock, uncompensated(C) cardiogenic shock, compensated

Page 246: Questions - uhs.edu.kh You suspect Lyme disease, most likely contracted by which of the following? (A) ingestion of unripe fruit (B) ingestion of spoiled fruit (C) drinking of contaminated

Questions: 86–99 247

(D) septic shock, uncompensated(E) normal cardiovascular state

93. His weight just prior to intravenous fluid ther-apy is 6.3 kg. His fluid deficit is approximatelywhat measurement?

(A) 50 cc(B) 250 cc(C) 650 cc(D) 850 cc(E) 1100 cc

94. Assuming normal kidney function, the initialhydrating fluid bolus in this patient should bewhich of the following?

(A) sodium 140 meq/L(B) sodium 100 meq/L(C) sodium 80 meq/L(D) sodium 40 meq/L(E) glucose 10 mg%

Questions 95 through 97

An 8-day-old full-term neonate presents with 1 day of vesicular lesions of the skin and mouth. Sheis afebrile and alert. There are multiple 3–5 mm vesi-cles on an erythematous base present on her trunkand mouth. The pregnancy was uncomplicated andthe parents have no history of genital lesions.

95. Which of the following is the best course ofaction at this time?

(A) discharge home and follow up in 72 hours(B) culture the lesions and begin treatment

with oral clindamycin (Cleocin)(C) check a complete blood count and

urine analysis and, if normal, dischargepatient after intramuscular ceftriaxone(Rocephin)

(D) perform bacterial cultures of blood,urine, cerebrospinal fluid, bacterial andviral cultures of the lesions, and initiatebroad-spectrum antibacterial therapy

(E) perform bacterial cultures of blood,urine, cerebrospinal fluid, bacterial andviral cultures of the lesions, and initiatebroad-spectrum antibacterial therapyand acyclovir

96. Viral shedding due to maternal HSV reactiva-tion is associated with transmission to the fetusin approximately what proportion of cases?

(A) 2%–5%(B) 10%–15%(C) 35%–40%(D) 70%–75%(E) 90%

97. Risk factors for vertical transmission of HSVtype 2 include which of the following?

(A) maternal coinfection with HSV type 1(B) physician rupture of membranes just

prior to delivery(C) scalp electrode usage(D) coinfection with the human immunode-

ficiency virus(E) delayed administration of acyclovir to

the mother in labor

Questions 98 and 99

A 15-day-old, full-term, large-for-gestational ageinfant is brought to your clinic for evaluation of achest mass. Examination reveals an afebrile vigor-ous baby with a firm, nontender mass palpable overthe middle third of the right clavicle.

98. What is the most appropriate course of actionfor this patient?

(A) reassurance to the family about the condition

(B) weight-bearing and comparison viewsof the clavicles

(C) skeletal survey to rule out abuse(D) fibroblast assay for osteopetrosis(E) report to child protective services for

failure to seek care in a timely fashion

99. Risk factors for this condition include which ofthe following?

(A) congenital syphilis(B) breech presentation(C) maternal drug use(D) female gender(E) thanatophoric dysplasia

Page 247: Questions - uhs.edu.kh You suspect Lyme disease, most likely contracted by which of the following? (A) ingestion of unripe fruit (B) ingestion of spoiled fruit (C) drinking of contaminated

248 10: Case Diagnosis and Management

Questions 100 through 102

You are called to evaluate the cause of hypotonia in a 1-day-old full-term female infant born to a 28-year-old mother. The baby is alert and moves allextremities well but is hypotonic. She has upwardslanting palpebral fissures, speckled irides, a largetongue, short fifth digits, and bilateral transversepalmar creases. Karyotype is 46,XX.

100. What is the most likely reason for this result?

(A) the number of chromosomes was miscounted

(B) she has a partial translocation involvingchromosome 21

(C) there is no chromosomal abnormality(D) interference of the test by maternal-fetal

blood transfusion(E) she has Turner syndrome

101. What is the frequency of Down syndrome?

(A) 1/12 live births(B) 1/600 live births(C) 1/4000 live births(D) 1/8000 live births(E) 1/20,000 live births

102. Patients with Down syndrome are at increasedrisk for which of the following?

(A) hyperthyroidism(B) arthritis of the cervical spine(C) streak gonads(D) cardiac malformations(E) rhabdomyosarcoma

103. You are seeing a previously well 22-month-oldmale with fever to 102°F, rhinorrhea, and coughfor 3 days. His parents report that he remainsplayful with no alteration in his activity. Yourexamination reveals rhinorrhea and is otherwiseunremarkable. The complete blood count revealsa white blood count (WBC) of 6000/mm3,normal hemoglobin and platelet count; WBCdifferential is 8% neutrophils, 40% lympho-cytes, 50% monocytes, 2% basophils. The bestcourse of action at this time is which of thefollowing?

(A) hospitalization and treatment withintravenous ceftazidime and tobramycin

(B) intramuscular dosages of ceftriaxone fora 7-day period

(C) bone marrow aspirate to rule outleukemia

(D) initiation of treatment with granulocytecolony-stimulating factor

(E) close outpatient follow-up and repeti-tion of the complete blood count in 1–2 days

Questions 104 through 106

A 3-year-old boy with sickle cell disease (hemoglo-bin SS) presents with a 1-hour history of right-sidedweakness. Examination reveals right hemiparesis. A CT scan of the brain is normal.

104. Acute treatment for this patient’s condition iswhich of the following?

(A) observation(B) anticoagulation with heparin(C) exchange transfusion(D) intravenous tissue plasminogen activator(E) hydroxyurea

105. The patient recovers completely, but prior todischarge develops a fever to 102°F. He appearsotherwise well. Infection with which of the fol-lowing organisms would be of most concern inthis patient?

(A) Mycobacterium tuberculosis(B) Clostridium botulinum(C) group A beta-hemolytic streptococcus(D) Streptococcus pneumoniae(E) influenza A

106. The patient’s father inquires about the possi-bility that his son’s children will have sicklecell disease. You should you inform him whichof the following about the risk?

(A) is negligible(B) is approximately 25%(C) is approximately 50%(D) approaches 100%(E) cannot be determined at this time

Page 248: Questions - uhs.edu.kh You suspect Lyme disease, most likely contracted by which of the following? (A) ingestion of unripe fruit (B) ingestion of spoiled fruit (C) drinking of contaminated

Questions: 100–112 249

Questions 107 through 109

During a well-child examination, the mother of apreviously healthy 5-year boy inquires whether youthink her child is unusually clumsy. The motherthinks her son is going to be strong as he has “bigcalves,” but her family tells her he seems to trip alot. Your examination reveals a pleasant male with aslightly waddling gait and mildly enlarged calves.

107. Which of the following physical examinationfindings would suggest the diagnosis ofDuchenne muscular dystrophy?

(A) Rovsing sign(B) opisthotonus(C) Gower sign(D) Chovstek sign(E) Grey-Turner sign

108. Complications of Duchenne muscular dys-trophy most commonly include which of thefollowing?

(A) renal failure from myoglobinuria(B) ophthalmoplegia(C) seizures(D) immunosuppression(E) aspiration pneumonia

109. The mother is devastated to learn the diagno-sis and that she carries the affected gene. A truestatement about Duchenne muscular dystro-phy inheritance is which of the following?

(A) 50% of her daughters will be affectedwith the disease

(B) about 25% of the cases are from newmutations

(C) women are never affected with thedisease

(D) the risk of recurrence is minimal(E) an affected male will pass the illness on

to all his sons

Questions 110 and 111

An 8-year-old girl is seen in the emergency depart-ment with a complaint of 1-week worsening short-ness of breath. She denies fever or cough. Vitalsigns include HR 140 beats per minute and BP78/40 mm Hg. She is alert with mild tachypnea and

is most comfortable leaning forward. Cardiac exam-ination reveals tachycardia and muffled heart tones.

110. What finding is the most suggestive of peri-cardial tamponade?

(A) pulsus paradoxus of greater than 10–20 mm Hg

(B) electrocardiogram with low voltages inall leads

(C) chest radiograph demonstrating anenlarged cardiothymic silhouette

(D) egophany over the left anterior chest(E) bounding carotid pulses

111. Echocardiography confirms a large pericardialeffusion compromising left ventricle function.After ensuring that the airway is stable, whatshould the next intervention be?

(A) intravenous solumedrol 30 mg/kg rapidinfusion

(B) continuous intravenous milrinone infusion(C) thoracotomy and placement of a peri-

cardial window(D) pericardiocentesis under ultrasound

guidance(E) intravenous albumin and furosemide

Questions 112 through 114

A previously healthy 7-day-old, full-term infantpresents to your clinic with jaundice which beganon the first day of age. He is afebrile with normalvital signs and growth indices. He is breastfeedingwell and has five gray stools and six wet diapers perday. Examination reveals a vigorous infant withmarked scleral icterus and jaundice to the level ofthe umbilicus. The liver is palpable 3 cm below theright costal margin. Your nurse was only able toobtain a limited sample of blood.

112. What are the most important tests to directfurther evaluation at this time?

(A) complete blood count and culture(B) total and direct bilirubin(C) blood type and Coomb test(D) aspartate aminotransferase and alanine

aminotransferase(E) alpha1-antrypsin levels

Page 249: Questions - uhs.edu.kh You suspect Lyme disease, most likely contracted by which of the following? (A) ingestion of unripe fruit (B) ingestion of spoiled fruit (C) drinking of contaminated

250 10: Case Diagnosis and Management

113. The child is sent to the emergency department,and testing reveals the total bilirubin to be 16 mg/dL with a conjugated fraction of 14 mg/dL. Which of the following tests wouldbe most likely to reveal the cause of this infant’sconjugated hyperbilirubinemia?

(A) urine analysis and culture(B) urine succinylacetone(C) sweat chloride test(D) abdominal ultrasound(E) thoracolumbar films

114. The above test is ordered and is abnormal. A subsequent liver biopsy to confirm thediagnosis reveals a paucity of biliary chan-nels and plugging with the channel diameterof greater than 150 mm. What is the besttreatment for this condition?

(A) phenobarbital to promote bile flow(B) immediate liver transplant using a

parental split liver graft(C) hepatocyte infusion directly into the

residual portal vein(D) Kasai hepatoportoenterostomy after

6 months of life to make it technicallyeasier

(E) Kasai hepatoportoenterostomy as soonas possible after diagnosis

Questions 115 and 116

A 4-year-old boy is brought in for evaluation becauseof perianal irritation and itching and parental con-cern that this might be a manifestation of sexualabuse. Examination reveals a playful boy; inspectionof the perianal region demonstrates several superfi-cial excoriations and several thread-like worms.

115. What is the best next step in management?

(A) antiparasitic treatment and reassuranceabout the benign nature of the condition

(B) stool for ova and parasite examination(C) social work evaluation and child protec-

tive services referral(D) instructions to the family regarding

improved hygiene(E) antiparasitic treatment of household pets,

the most likely reservoir for this parasite

116. Which of the following should you treat thiscondition with?

(A) mebendazole(B) metronidazole(C) diethylcarbamazine(D) praziquantel(E) thiabendazole

Questions 117 through 119

A 2-year-old 13-kg female is brought to the emer-gency department by her parents after they foundher with an empty bottle of cherry-flavored aceta-minophen drops about 2 hours prior. You notethat the bottle contained 1 oz at a concentration of80 mg/0.8 cc. The parents report they just pur-chased it earlier in the day for her 2-month-oldbrother. No treatment was given prior to arrival atthe emergency department. The patient’s vitalsigns are normal, and her physical examination isunremarkable.

117. What should the next step in this patient’smanagement be?

(A) intubate to secure the airway(B) do nothing until 4 hours have elapsed

since the purported time of ingestion(C) give activated charcoal with sorbitol at a

dose of 1 g/kg orally or via nasogastrictube

(D) induce emesis with syrup of ipecac(E) proceed with gastric lavage using a

12-French orogastric tube

118. You proceed with management. The 4-hour acet-aminophen level is 200 mg/kg. What is the bestnext step?

(A) continue observation in the emergencydepartment for 6 hours and discharge ifthe patient remains asymptomatic

(B) start the antidote, N-acetylcysteine(C) continue activated charcoal every

4 hours in addition to the antidote, N-acetylcysteine

(D) begin preparations for liver transplant(E) place a hemodialysis catheter for initia-

tion of hemodialysis

Page 250: Questions - uhs.edu.kh You suspect Lyme disease, most likely contracted by which of the following? (A) ingestion of unripe fruit (B) ingestion of spoiled fruit (C) drinking of contaminated

Questions: 113–121 251

119. The toxicity of acetaminophen comes fromwhich of the following?

(A) its renal metabolite(B) acetaminophen directly(C) its glucoronidate metabolite(D) its sulfated metabolite(E) its cytochrome P-450

metabolite

Questions 120 and 121

A previously healthy 3-year-old male is brought in byhis parents with a chief complaint of increased sleepi-ness, which began during a visit to his grandmother’shouse earlier that day. The patient’s vital signs areHR of 50 beats per minute, temperature of 97°F, RRof 8 times per minute, and a BP of 98/65 mm Hg.Examination reveals an obtunded male toddler whois unresponsive to sternal rub and lacks a gag reflex.He also has miosis.

120. The first step in managing this patient is

(A) bedside whole blood glucose(B) urinary catherization to obtain urine

drug screen(C) intravenous flumazenil(D) securing an airway and insuring ade-

quate breathing(E) intravenous epinephrine for bradycardia

121. The grandmother reports that she has multiplemedications in her house and is concerned thatthe patient’s problems may be due to an inges-tion. Which one of her medicines would mostlikely cause the symptoms reported?

(A) clonidine(B) nifedipine(C) meperidine(D) labetalol(E) diazepam

Page 251: Questions - uhs.edu.kh You suspect Lyme disease, most likely contracted by which of the following? (A) ingestion of unripe fruit (B) ingestion of spoiled fruit (C) drinking of contaminated

252

Answers and Explanations

1. (C) Given the history of excessive intake ofmilk, a food known to be low in iron, and thepresentation of pallor, initial testing shouldfocus on the hemoglobin and iron status. Thereticulocyte count and the mean corpuscularvolume (MCV) generally are low in iron defi-ciency. Chest radiography and stool smears forova and parasites are not indicated. Serum hap-toglobin is low in state of hemolysis, but that isnot the most likely cause of anemia in this case.A bone marrow biopsy is not indicated at thistime. (Behrman, 2014–2016)

2. (D) In iron deficiency anemia, the red bloodcells are smaller than normal and are variablein size; therefore a decreased MCV and ele-vated red cell distribution width (RDW) areclassic with this history. Vitamin B12 defi-ciency from a poor diet or intrinsic factor defi-ciency presents with an elevated MCV. Noblasts indicating leukemia or stippling sug-gesting hemosiderosis were noted on thesmear. Additionally, there is no evidence ofsickled cells. (Behrman, 1469–1471)

3. (C) When iron deficiency anemia is suspected,elemental iron at a dose of 6 mg/kg/daydivided into two or three doses should be initiated. Appetite will return to normal in12–24 hours. Bone marrow response will beginin 36–48 hours. Reticulocytosis will peak around5–7 days. Repletion of stores will occur in 1–3 months depending on the severity of theanemia. Contrary to common belief, iron doesnot constipate patients. Increasing vitamin Cintake via diet or supplementation can increaseiron absorption. Parents should be counseledregarding the importance of a diverse diet.

Milk intake should be limited to 16–24 oz/day.(Behrman, 1469–1471)

4. (B) The initial step in the management ofpatients presenting in respiratory distress isassessment of the “ABCs” (airway, breathing,and circulation). The presence of hypoxemiais an indication for administration of supple-mental oxygen. There is no indication of acompromised airway (eg, stridor) in thispatient. While there is mild distress, there isno indication at this time of respiratory failurenecessitating emergent intubation. Chest radi-ographs and administration of corticosteroidsmay be necessary, but are not the first prior-ity in this patient’s management. (Behrman,1388–1390, 1773–1777)

5. (E) The age, physical findings, and season allpoint to bronchiolitis, most likely due to res-piratory syncytial virus (RSV), as the mostlikely cause of this patient’s problem. RSV isthe etiologic agent in about 70% of patientswith bronchiolitis and results in approximately70,000 inpatient admissions yearly in theUnited States. Congestive heart failure likelywould be associated with other physical find-ings such as hepatomegaly, or have demon-strable cardiomegaly on the chest radiograph.Respiratory distress syndrome typically has aground-glass appearance on chest radiogra-phy and presents with more severe clinicalsymptoms. Pneumococcal pneumonia classi-cally presents with a more focal infiltrate withconsolidation, is unilateral, and is not associ-ated with wheezing. Mycoplasmal infectionsare not common at this age. (Behrman, 991–993,1285–1287)

Page 252: Questions - uhs.edu.kh You suspect Lyme disease, most likely contracted by which of the following? (A) ingestion of unripe fruit (B) ingestion of spoiled fruit (C) drinking of contaminated

Answers: 1–12 253

6. (A) A trial of beta agonist therapy, such asalbuterol or racemic epinephrine should beoffered to all patients with bronchiolitis withsymptoms of distress; however, not all patientswill respond. Ribavirin aerosal treatment is avail-able but not recommended routinely. Guidelinesas delineated by the American Academy ofPediatrics suggest that the high cost, technicalissues associated with aerosal administration,potential toxic effects among exposed health careworkers, and lack of convincing efficacy datalimit its usefulness and at this time. It may be bestutilized in the immunocompromised host (eg,bone marrow transplant recipient with RSVinfection). Ahumanized mouse monoclonal anti-body product (palivizumab) is licensed for pre-vention of infection with RSV, but has nowell-defined role in treatment of confirmed infec-tions. Heliox (70/30 mixture of helium/oxygen)has been used for upper airway obstructions andextracorporeal membrane oxygenation may beutilized in severe bronchiolitics but is not indi-cated here. (Behrman, 991–993, 1285–1287)

7. (E) Gram-positive organisms, in particular S aureus, are a significant source of soft tissueinfections in the immunocompetent child.Spread to deep tissues can occur directly orhematogenously. Arthritis due to infection withNeisseria typically presents as a monoarticularseptic joint in sexually active individuals. Group Bstreptococcal arthritis is most commonly seen inneonates. P aeruginosa or other gram negativesshould be considered in drug users or immun-odeficient individuals. Arthritis due to M tuber-culosis is uncommon and would have a moreindolent course. (Behrman, 2845–2847)

8. (B) The examination is very concerning for aseptic arthritis of the left elbow. Erosion andpermanent damage of the joint can occurquickly. Obtaining fluid for cultures by arthro-centesis prior to antibiotic therapy is preferred,particularly in light of the increasing frequencyof antibiotic-resistant organisms. Oral antibi-otics are inappropriate for initial treatment ofseptic arthritis. Nonsteroidal anti-inflammatoryagents provide symptomatic relief only and arenot a primary treatment modality in septicarthritis. (Behrman, 777–780)

9. (E) Young infants, patients with hip and shoulderinvolvement, and those whose antibiotic therapyis delayed are at increased risk of a poor outcomefollowing bacterial arthritis. Additionally, septicarthritis due to gram-negative and fungal organ-isms is more difficult to eradicate. The Gram stainis only positive in approximately 50% of cases,and the presence or absence of organisms onGram stain is of no prognostic value in septicarthritis. (Behrman, 777–780)

10. (B) Thyroid dysgenesis accounts for 85% of thecases of congenital hypothyroidism. Maternalingestion of propylthiouracil causes a transi-tory hypothyroidism, but history should revealmaternal use of this drug. The other causes ofcongenital hypothyroidism are uncommonlyseen. (Behrman, 2319–2325)

11. (A) When a patient presents with a state screen that is positive and has obvious clinical symptoms of hypothyroidism, initiating thyroxine prior to confirmation is essential. Therapy initiated prior to2–4 weeks of life can ensure near-normal intelli-gence. Dosing for infants starts at 10–15 mcg/kg/day. Older children require about 4 mcg/kg/dayand adults require only 2 mcg/kg/day. A scintis-can to look for ectopic thyroid tissue is helpful.Close neurodevelopmental follow-up is necessary.Radiography of the distal femur of patients withcongenital hypothyroidism frequently revealsabsent distal epiphysis. This finding is occasionallyused as a quick indirect screen for hypothyroidism.(Behrman, 1698–1703)

12. (C) Edema results from two pathophysiologicconditions: increased venous hydrostatic pres-sure or decreased oncotic pressure. Increasedhydrostatic pressure, resulting in extravasation offluid into the interstitial space, results from con-ditions such as venous thrombosis and conges-tive heart failure. Reduced oncotic pressure mostcommonly results from hypoalbuminemia due todecreased production of albumin (eg, liver fail-ure) or increased loss of albumin (eg, via the gas-trointestinal tract or kidneys); reduction in oncoticpressure in turn results in fluid extravasation intothe interstitial tissues. Of the choices given, aurine analysis for protein is the quickest, cheap-est, least invasive, and most helpful initial testto direct further evaluation. (Behrman, 2190–2194)

Page 253: Questions - uhs.edu.kh You suspect Lyme disease, most likely contracted by which of the following? (A) ingestion of unripe fruit (B) ingestion of spoiled fruit (C) drinking of contaminated

254 10: Case Diagnosis and Management

13. (E) In conjunction with proteinuria, elevatedserum cholesterol, hypoalbuminemia, andedema constitute the nephrotic syndrome.Membranous glomerulonephritis, focal seg-mental glomerulosclerosis, membranoprolifer-ative disease, and acute poststreptococcalglomerulonephritis all may be associated withthe nephrotic syndrome. However, minimal-change disease is much more common in chil-dren, accounting for 85% of protein-losingnephropathies. (Behrman, 1592–1595)

14. (D) Fortunately, 90% of patients with minimal-change disease respond rapidly to steroids. Lossof properdin factor B that opsonizes bacteriacan occur, leading to an increased risk of spon-taneous bacterial peritonitis. Antithrombin III isalso lost in the urine, so children are at increasedrisk of thrombosis, not hemorrhage. Duringflares low-sodium diets, combined with diuret-ics and albumin infusions may be necessary todecrease edema. A referral and renal biopsyshould be performed after 1 month (not 1 year)of nonresponse to steroid therapy to determinethe etiology of the illness. (Behrman, 1592–1595)

15. (C) Hypertension presenting with symptoms(headache, altered mental status, papilledema)requires immediate antihypertensive therapy.Causes of severe hypertension (> 95th percentile)include renal artery stenosis, malignant hyper-thyroidism, aldosteronism, pheochromocytoma,and a coarctation of the aorta. An intracranialmass with Cushing triad causing hypertensionshould present with bradycardia but this can bea late presenting sign. An LP should be deferredin a patient with papilledema until imagingdemonstrates the absence of an intracranial mass.(Behrman, 1990, 2372–2373)

16. (B) Determination of urine level of vanillylmandelic acid (VMA, a metabolite of epineph-rine), thyroid function analysis, and CT imag-ing of the abdomen for a mass as well as renalartery caliber delineation are all essential inlooking for the etiology of her hypertension.Laparotomy may be needed for eventual ther-apy but is not a part of the initial evaluation.(Behrman, 1184, 1741–1742)

17. (A) An elevated VMA level with this patient’ssymptoms point to a pheochromocytoma as thecause of her hypertension. Pheochromocytomasarise from the chromaffin cells of the sympatheticnervous system; they can arise in the adrenalmedulla or anywhere along the sympathetic chain.They can be inherited sporadically or as a compo-nent of types IIA and IIB multiple endocrine neo-plasia syndromes from a mutation in the RETproto-oncogene. The definitive treatment is surgi-cal with exquisite detail to peri- and postoperativehypertension. (Behrman, 1184, 1741–1742)

18. (D) Ninety-nine percent of healthy neonates passtheir first stool (meconium) within 48 hours afterbirth. Delayed passage of the meconium raisesconcern for Hirschsprung disease. Barium enemamay demonstrate a “transition zone” at the junc-tion of the dilated normal bowel proximally andthe narrowed aganglionic segment distally. Asfrequent rectal stimulation and digital examina-tion may alter the radiographic appearance, thebarium enema should be delayed until 2–3 daysafter these procedures, if feasible. Botulism andhypothyroidism may present with constipation,but this neonate’s vigorous appearance and his-tory of feeding make these unlikely. Mineral oilshould not be given to infants because of the riskof aspiration and subsequent lipoid pneumonia(see Figure 10-1). (Behrman, 1565–1567)

Figure 10-1(Courtesy of Doug Rivard, DO)

Page 254: Questions - uhs.edu.kh You suspect Lyme disease, most likely contracted by which of the following? (A) ingestion of unripe fruit (B) ingestion of spoiled fruit (C) drinking of contaminated

Answers: 13–23 255

19. (E) The diagnostic ability of the barium enemadepends on demonstration of a transition zoneof narrow distal bowel expanding into morenormal caliber proximal bowel. However, false-negative studies occur, especially if performedafter previous rectal stimulation (eg, digitalexamination). Biopsy above the dentate lineremains the gold standard for diagnosis; inHirschsprung disease, it reveals a lack of theneuronal plexus (Meissner and Auerbach),hypertrophied nerve bundles, and increasedacetylcholinesterase levels. Stool examinationfor eosinophils may be useful in cases of milkprotein allergy, but is not indicated in thisbreastfed baby. Anal manometry to measurepressure changes of the rectum is technicallychallenging in young infants but can be per-formed in older patients. Upper GI series withsmall bowel follow through is useful in thediagnosis of intestinal malrotation, but is notindicated in the patient described. (Behrman,1139–1141)

20. (A) The prognosis for surgically treated HD gen-erally is satisfactory with a majority of patientsachieving fecal continence. In 75% of patients,the aganglionic segment is limited to the distalrectosigmoid area. Colostomy placement is rareand is only temporary until the proximal seg-ment recovers from its dilated state. A totalcolectomy is necessary in fewer than 10% ofpatients. A small bowel transplant is neededonly if a large proportion of the small bowel isaganglionic (total intestinal Hirschsprung dis-ease). Enterocolitis can occur in untreatedHirschsprung disease, but is uncommon fol-lowing surgical correction. (Behrman, 1139–1141)

21. (B) The history of longitudinal traction in achild less than 4 years old, the lack of tender-ness or effusion on examination, and the char-acteristic posture (arm flexed at elbow andslightly pronated) point to nursemaid elbow(radial head subluxation) as the most likelydiagnosis. This occurs when the radial head issubject to longitudinal traction causing thedeveloping annular ligament to remain par-tially entrapped in the radiohumeral joint. Inpatients with a typical history and physical

examination, maneuvers to reduce the sublux-ation are both diagnostic and therapeutic.Hyperpronation or supination/flexion of theforearm are two maneuvers that can reduce thesubluxation. Although causing brief pain,reduction does not require sedation. On suc-cessful reduction a “clunk” can be felt over thelateral portion of the elbow, and the patientwill generally begin using the arm normallywithin a short period. Radiographs are notindicated as they will be normal. While nonac-cidental trauma may result in a nursemaidelbow, the history given is compatible with theinjury and, in the absence of other suspiciousfindings, is not an indication for social serviceevaluation. (Behrman, 2827)

22. (E) Recurrence of nursemaid’s elbow has been reported in as many as 30% of children.Therefore, counseling the parents on themechanism of this injury is important. Nosplints are necessary and physical activity as tolerated should be permitted. A radialhead subluxation by itself typically is notindicative of abuse. The annular ligament andradial head strengthen with age and radialhead subluxation is rare after 4 years of age.(Behrman, 2092)

23. (A) This adolescent has several risk factorsthat make her a candidate for admission. Thefever and toxic appearance suggest that shehas an illness beyond a cystitis or simple cervicitis caused by N gonorrhea or C trachomatis.Residence in a shelter also suggests increaseddifficulty with outpatient follow-up. Treatmentfor an emergent condition as well as for sex-ually transmitted diseases does not requirelegal proceedings. Appropriate pain controlwill be necessary, but admission and intra-venous antibiotic therapy are the first criticalstep in this patient with pelvic inflammatorydisease (PID). Prior to discharge from the hos-pital and hopefully after this patient hasdeveloped a trusting professional relation-ship with her physician, counseling regardingcontraception and further protection as wellas lifestyle choices are indicated. (AAP:RedBook,493–498)

Page 255: Questions - uhs.edu.kh You suspect Lyme disease, most likely contracted by which of the following? (A) ingestion of unripe fruit (B) ingestion of spoiled fruit (C) drinking of contaminated

256 10: Case Diagnosis and Management

24. (C) Complications of PID including chronicpelvic pain, perihepatitis, ectopic pregnancies,infertility, and tubo-ovarian abscess should beworsening signs and symptoms despite appro-priate antimicrobial therapy. These are sugges-tive of tubo-ovarian abscess. Consultation witha gynecologist and diagnostic imaging (usu-ally ultrasound) is appropriate at this time. A C-reactive protein to monitor therapy is notneeded when the clinical picture clearly sug-gests lack of response to therapy. Methicillin-resistant S aureus is an uncommon cause ofPID. (AAP:Red Book,493–498)

25. (B) PID causes scarring of the reproductivetract and can lead to many complications laterin life. There is a 10% risk of tubal infertilityafter a single episode of PID, with a 50% riskafter three or more episodes. Patients whohave had PID are at increased risk for repeatinfections, and douching or oral contracep-tives do not prevent them. Male carriers arenot always symptomatic. PID is a risk factorfor future ectopic pregnancies. (AAP:Red Book,493–498)

26. (E) The first step in management of any lacer-ation is hemostasis. Once this is achieved, irri-gation, evaluation of the wound for foreignbodies, and antimicrobial therapy can begin.If the wound injury is found to involve ten-dons, ligaments, vessels, or may be associatedwith severe cosmetic imperfection, surgicalassistance for repair is appropriate. (AAP:RedBook,649)

27. (B) Two questions to ask when confrontedwith tetanus prophylaxis are (1) has thepatient received three or more doses of teta-nus toxoid and (2) is the wound clean. Teta-nus immune globulin is necessary in thosewho have a dirty wound and fewer than three doses of tetanus-containing vaccine or those with a history of HIV infection. Inthose with greater than or equal to threetetanus-containing vaccines, TIG is not nec-essary but the age-appropriate tetanus vac-cine should be given if immunization isincomplete (see Table 10-1). (AAP:Red Book,650–651)

28. (C) The injury this child suffered is not consis-tent with the story that is presented. Other“red flags” suggesting nonaccidental traumainclude stories that are developmentally impos-sible or change over time, reticence on the partof one caregiver to speak, and delay in seekingmedical care. This child clearly is at risk andneeds an evaluation with a skeletal survey.Findings suggestive of abuse are multiple heal-ing injuries, rib fractures, bucket handle frac-tures, or healing long bone fractures. A hipspica is the treatment for a femur fracture ininfants; internal fixation is not indicated.Although osteogenesis imperfecta may pres-ent with long bone fractures, it is rare and eval-uation for more common conditions such asabuse takes precedence. Eighty percent ofinflicted skeletal trauma occurs in childrenunder 18 months of age. Among fracturesresulting from abuse, skull fractures are mostcommon followed by fractures of the extremities.(Rudolph, 463–469, 561)

TABLE 10-1 Guide to Tetanus Prophylaxis in RoutineWound Management (Centers for Disease Control andPrevention. Manual for the Surveillance of Vaccine-Preventable Diseases 4th Edition, 2008. Published 2008.Updated April 8, 2009. http://www.cdc.gov/vaccines/pubs/surv-manual/chpt16-tetanus.htm)

History of adsorbedtetanustoxoid(doses) Clean minor wounds All other woundsa

Tdap or TIGc Tdap or TIGc

Tdb Tdb

<3 or Yes No Yes Yesunknown≥3 dosesd Noe No Nof No

aSuch as (but not limited to) wounds contaminated with dirt, feces,soil, and saliva; puncture wounds; avulsions; and wounds resultingfrom missiles, crushing, burns, and frostbite.bFor children younger than 7 years of age, DTaP is recommended; if pertussis vaccine is contraindicated, DT is given. For persons 7–9 years of age or 65 years or older, Td is recommended. Forpersons 10–64 years, Tdap is preferred to Td if the patient hasnever received Tdap and has no contraindication to pertussisvaccine. For persons 7 years of age or older, if Tdap is notavailable or not indicated because of age, Td is preferred to TT.cTIG is human tetanus immune globulin. Equine tetanus antitoxinshould be used when TIG is not available.dIf only three doses of fluid toxoid have been received, a fourthdose of toxoid, preferably an adsorbed toxoid, should be given.Although licensed, fluid tetanus toxoid is rarely used.eYes, if it has been 10 years or longer since the last dose.fYes, if it has been 5 years or longer since the last dose. Morefrequent boosters are not needed and can accentuate side effects.

Page 256: Questions - uhs.edu.kh You suspect Lyme disease, most likely contracted by which of the following? (A) ingestion of unripe fruit (B) ingestion of spoiled fruit (C) drinking of contaminated

Answers: 24–34 257

29. (E) When confronted with possible nonacci-dental trauma, the safety of the affected child aswell as other members of the household is ofprimary importance. Evaluation for factors thatmay contribute to the potential for abuse shouldbe undertaken. The assistance of a social workeris often valuable in this regard. All states man-date that physicians report any case of sus-pected abuse. Mandated reporters are immunefrom prosecution, even if abuse is not proven, aslong as the report is made in good faith. Directconfrontation and accusation are not productiveand may hinder further efforts in the inves-tigative process. Incarcerating the father is notappropriate as he has made no threats, nor isthere sufficient evidence to implicate him as theperpetrator of the injury. However, this childshould not be discharged until there is suffi-cient investigation of the social situation topermit safe placement. (Rudolph CD 463–469, 561)

30. (E) A patient with this degree of tachycardianeeds immediate assessment of her cardiores-piratory status. As her airway, breathing, andcirculation are intact with no evidence ofimpending failure, a brief evaluation can beperformed to evaluate her tachycardia.Obtaining an electrocardiogram during theepisode likely will be valuable for analysis ofthe rhythm and determining the best treatmentmodality. The other tests are helpful in evalu-ating sinus tachycardia, which usually peaksaround 220–230. A tachycardia over 230 is moresuggestive of a supraventricular tachycardia(SVT), which is the most common dysrhyth-mia requiring medical intervention in children.(Rudolph CD 1854–1856)

31. (A) Once the diagnosis of supraventriculartachycardia is made, adenosine 0.1 mg/kg(maximum 6 mg) rapid intravenous push isthe treatment of choice. If the initial attempt isunsuccessful, the dose can be doubled andrepeated (to a maximum of 12 mg). Side effectsinclude transient headache, flushing, and chestpain. Serious side effects of adenosine includeatrial fibrillation, apnea, bronchospasm, briefasystole, accelerated ventricular rhythms, andwide complex tachycardia. Cardioversion (notdefibrillation) is indicated if the patient is

unstable. Digoxin is also used for recalcitrantSVT. Calcium channel blockers are contraindi-cated for children under 1 year of age as theymay precipitate cardiovascular collapse.Ibuprofen would be useful for sinus tachycar-dia due to fever, but this patient had no feverand the heart rate is above that seen with sinustachycardia. (Rudolph CD, 1854–1856)

32. (B) All of the listed answers may cause hairloss. Tinea capitis generally has more patchyhair loss and may have “black dots,” repre-senting broken hair. Trichotillomania will havebizarre patterns of hair loss in linear bands thathas stubs of varying length. Hair traction alope-cia occurs in areas of stress where the hair ispulled into braids or ponytails. It may becomepermanent if scarring occurs. Chemical expo-sure should be elicited from history. Alopeciaareata is the most likely cause in this patient,given the complete hair loss and the charac-teristic exclamation point appearance of thehair. (Rudolph CD, 1210–1212)

33. (C) Alopecia areata occurs when hair that is ina growing phase (anagen) abruptly stops grow-ing, causing the hair shafts to taper and loseadhesion to the follicle. Round patches of hairanywhere on the body can be affected, but aremost noticeable when on the scalp. Clues todiagnosis include lack of inflammation, easilyremoved hair at the borders, and the exclama-tion point appearance of the hair under themicroscope. Unfortunately the course is difficultto predict, a fact that should be stressed to thepatient. Treatments with steroids, minoxidil,and immunomodulators such as cyclosporinehave shown some success, but their use shouldbe undertaken in consultation with a derma-tologist. (Rudolph CD, 1210–1212)

34. (B) This history is highly suggestive of pertus-sis. The clinical course is divided into threestages: (1) the catarrhal stage, 2–10 days in dura-tion, characterized by rhinorrhea, lacrimation,and sometimes low-grade fever; (2) the parox-ysmal stage, lasting 1–6 weeks, during whichthere are intermittent episodes of coughing thatmay terminate with a forced inspiration againsta partially closed glottis resulting in a “whoop”

Page 257: Questions - uhs.edu.kh You suspect Lyme disease, most likely contracted by which of the following? (A) ingestion of unripe fruit (B) ingestion of spoiled fruit (C) drinking of contaminated

258 10: Case Diagnosis and Management

or may terminate with vomiting; (3) the conva-lescent stage, lasting up to 6 months, duringwhich the coughing episodes gradually resolve.Infants with pertussis often do not whoopbecause of their inability to generate sufficientinspiratory forces. Between episodes of cough,the examination is often normal. Of the choicesgiven, the immunization status is of mostimportance. (McMillan, 1094–1097)

35. (C) A high white blood count with a markedlymphocytosis is characteristic of pertussis.Therefore, prescribing a macrolide, specificallyazithromycin, is the most appropriate choice. A repeat CBC is not needed to follow the treat-ment. In fact, antibiotics do not hasten the res-olution of the illness, but will decrease spreadto other household members. Even so, allhousehold contacts should also be treated pro-phylactically. (McMillan, 1094–1097)

36. (B) Bordetella pertussis is most likely to be recov-ered from the nasopharynx and more likely tobe positive early in the illness. Throat swabs areless likely to be positive and sputum is almostnever obtainable from infants. Bronchoscopy isnot necessary to make the diagnosis and bac-teremia does not occur. Special media areneeded for cultivation of the organism.(McMillan, 1094–1097)

37. (D) This history and the finding of papilledemais strongly suggestive of pseudotumor cerebri,also known as benign intracranial hyperten-sion, a condition marked by overproduction of cerebrospinal fluid leading to increasedintracranial pressure. However, imaging to ruleout a mass or a sinus thrombosis should be per-formed. A history of ingestion of vitamin A,tetracycline, or steroids, or a history of preg-nancy would further support the diagnosis ofpseudotumor cerebri. Her blood pressure ismildly elevated, but does not warrant treatmentat this time with oral or intravenous antihyper-tensive agents. Mannitol and furosemide wouldbe used in the emergent management of sus-pected herniation, but the patient currently hasno signs to suggest that problem. (Rudolph CD,2398; Behrman, 2525–2526)

38. (A) The next step would be lumbar puncturewith determination of opening and closingpressures. Removal of sufficient fluid to reducethe CSF pressure is therapeutic. At times a non-steroidal drug or even a narcotic may be nec-essary to temporarily relieve the headachesassociated with pseudotumor cerebri. Seriallumbar punctures may be needed for contin-ued symptomatic relief. Medications to reduceCSF production include acetazolamide andsteroids. (Rudolph CD,2398; Behrman, 1862)

39. (E) Indications for surgical interventionsinclude vision loss despite adequate therapy orinability to tolerate treatment. Surgical optionsinclude optic nerve fenestrations or placementof a lumboperitoneal shunt; repeat or additional(MRI) imaging should also be performed in casea small tumor was missed initially. (Rudolph CD,2398; Behrman, 1862)

40. (D) The first priority in managing this patientis ensuring that the airway is patent and ven-tilation/oxygenation is adequate. Suction ofvomitus may be needed as well. (McMillan,702–704, 2297–2299; Pediatrics, 97:769–772, 1996)

41. (E) Given the patient’s age, fever, and paucity ofabnormal physical findings, the most likely diag-nosis is febrile seizure. The most pressing need isto rule out meningitis. The American Academyof Pediatrics strongly recommends considera-tion of a lumbar puncture in all patients less than12 months of age who present with a simplefebrile seizure. In the absence of signs ofincreased intracranial pressure, a head CT is notindicated prior to lumbar puncture. Skull x-rays,arterial blood gas, and electroencephalogramshave very low yield in this clinical situation andare not routinely warranted. (McMillan, 702–704,2297–2299; Pediatrics, 97:769–772, 1996)

42. (A) It is reassuring that this patient’s physicalexamination has returned to normal, compati-ble with the diagnosis of a simple febrileseizure. The parents should be counseled thatabout one-third of children experiencing afebrile seizure will have at least one recurrence.The risk is higher in children experiencing thefirst seizure before the age of 1 year. Chronic

Page 258: Questions - uhs.edu.kh You suspect Lyme disease, most likely contracted by which of the following? (A) ingestion of unripe fruit (B) ingestion of spoiled fruit (C) drinking of contaminated

Answers: 35–46 259

anticonvulsant prophylaxis is not indicated forsimple febrile seizures. While attempts to control fever are certainly warranted in patientswith a history of febrile seizures, the use ofantipyretics alone has not proven effective inthe prevention of recurrence. In particular,there is no evidence that the use of alternatingdoses of acetaminophen and ibuprofen is nec-essary or effective in seizure prevention.Hospital admission and neuroimaging are notnecessary in patients whose examination returnsto normal prior to discharge from the emergencydepartment. (McMillan, 702–704, 2297–2299;Pediatrics, 97:769–772, 1996)

43. (B) Before a lengthy and expensive diagnosticevaluation is undertaken, the growth patterns ofthe biological parents should be evaluated, asthey are predictive of the child’s growth pattern.Sibling height may be helpful, but is not predic-tive. Parental age does not affect offspring heightper se, but may be indirectly associated in casesof age-related disorders such as Down syndrome.Social factors may influence growth rate as psy-chosocial dwarfism is seen in institutionalizedand maltreated children. (Behrman, 2386–2389)

44. (D) The physical findings described suggestthe diagnosis of Turner syndrome. Girls withTurner syndrome often present with shortstature without typical stigmata includingwebbed neck, low hairline, and a broad chestwith widely spaced nipples. Other findingsinclude dysgenic ovaries, and estrogen levelsare low resulting in a lack of negative feedbackon the pituitary gland. Consequently, luteinizing-and follicle-stimulating hormone levels arehigh. An abdominal ultrasound will reveal thestreak ovaries. Growth hormone levels are lowand the bone age is delayed. Although all of thetests listed will likely reveal abnormalities, it isthe karyotype that will ultimately confirm thediagnosis. (Behrman, 1753–1754)

45. (A) Echocardiography is indicated in patientswith Turner syndrome because of the associa-tion with various cardiac anomalies, includingbicuspid aortic valve, aortic stenosis, and anom-alous pulmonary venous return. While charac-teristic of Down syndrome, cervical laxity is not

a feature of Turner syndrome. Growth hormonemay increase adult height, and estrogenreplacement therapy can improve bone density,growth, and sexual maturation. However, thy-roid hormone replacement is not routinely indi-cated. The risk of endometrial carcinoma is notincreased in these patients. Although Turnersyndrome is associated with increased risk forsome learning problems, mental retardation isnot a feature of the disorder. Psychosocial sup-port is important as well; the Turner SyndromeSociety has chapters in the United States thatcan provide a support network for patients andtheir families. (Behrman, 1753–1754)

46. (D) The constellation of these symptoms is strongly suggestive of Kawasaki disease (Figure 10-2). The disease is most commonlyseen in children under 5 years of age. Diagnosticcriteria include fever of at least 5 days’ durationtogether with at least four of the following fea-tures: (1) conjunctival injection; (2) oral changes,including erythema of the oral mucosa andpharynx, strawberry tongue, and red, crackedlips; (3) an erythematous, generalized rash; (4) changes in the peripheral extremities con-sisting of swelling of the hands or feet, erythe-matous palms or soles, or desquamation of thefingertips; and (5) a unilateral lymph node atleast 1.5 cm in diameter. In addition, otherpotential causes of the illness (eg, measles, scar-let fever, and Stevens-Johnson syndrome) mustbe excluded. (AAP:Red Book, 412–415)

Figure 10-2

Page 259: Questions - uhs.edu.kh You suspect Lyme disease, most likely contracted by which of the following? (A) ingestion of unripe fruit (B) ingestion of spoiled fruit (C) drinking of contaminated

260 10: Case Diagnosis and Management

47. (C) Kawasaki manifestations and complica-tions include arthralgias, irritability, vomiting,sterile pyuria, mild hepatic dysfunction, asep-tic meningitis, pericardial effusion, myocardi-tis, and coronary artery ectasia and aneurysms.In addition to these problems, hydrops of thegallbladder is a well-described complication.Although thrombocytopenia has rarely beendescribed in association with this disease,thrombocytosis is the more characteristic find-ing. Sterility, ulcerative colitis, and transversemyelitis are not associated with this disease.(AAP:Red Book,412–415)

48. (B) Receipt of IVIG may interfere with theimmune response to some live viral vaccina-tions. Therefore, measles and varicella immu-nization should be deferred for 11 monthsfollowing IVIG administration. Other rou-tine vaccines should be given as scheduled.Influenza vaccine should be encouraged, asthere is a potential for Reye syndrome whenaspirin is given. No repetition of previouslyadministered vaccines is necessary. (AAP:RedBook,415)

49. (C) The history of trauma with the presence oflocalized tenderness on examination and thelack of systemic symptoms such as fever makeinfection unlikely. Therefore, C-reactive pro-tein and blood culture are not indicated. At thispoint there is no indication of underlying meta-bolic bone disease and alkaline phosphatase isnot warranted. Although a high index of sus-picion for nonaccidental trauma is appropriate,the history given is consistent with the physicalfindings described, and social service consulta-tion is not warranted at this time. Given thepossibility of fracture, performance of radi-ographs would be the most appropriate choice.(Behrman, 2839)

50. (E) The history and physical findings describedare suggestive of a “toddler’s fracture,” alsotermed childhood accidental spiral tibial(CAST) fracture. These fractures occur mostcommonly in the distal tibia, often followingrelatively minor trauma. They are not sugges-tive of abuse. The diagnosis is confirmed byradiographs; occasionally, the fracture may be

radiographically occult or demonstrable onlyon oblique films. Immobilization of the limbin a posterior short leg splint will provide sup-port and reduce the pain that occurs withmovement of the limb. Casting can occur laterwhen swelling has subsided. Healing typicallyoccurs in 4–6 weeks. (Behrman, 2097)

51. (B) The differential diagnosis of the lethargicneonate is extensive and includes infection,effect of maternal medications, electrolyte dis-turbances, metabolic abnormalities, congenitalabnormalities, and trauma. The delay in onsetof symptoms until day 3 of life make maternalmedications an unlikely cause of lethargy inthis neonate. There is no history of abdominaldistention or bilious emesis to suggest anintraabdominal emergency such as midgutvolvulus and diagnostic imaging is thereforenot indicated. If there were reason to suspecthead trauma or intracranial bleeding, CTwould be the imaging modality of choice. Ofthe options presented, performance of an eval-uation for infection is the most appropriate firststep. (Rudolph CD,106–107)

52. (A) Inborn errors of metabolism are likely inthis patient. While rare individually, these dis-orders in aggregation approximate the inci-dence of sepsis. Therefore, consideration ofone of these disorders, particularly when theevaluation for infection does not yield any def-inite evidence of infection, should not bedelayed. In a male with an elevated ammonialevel, especially without acidosis, a urea cycledefect is the most likely possibility. Elevationof urine orotic acid, a by-product produced inincreased amounts due to a lack of ornithinetranscarbamylase, is consistent with ornithinetranscarbamylase deficiency. Inheritance is X-linked dominant, and there are over 20 vari-ants. Females with the defective gene typicallyhave milder forms of the illness. (Behrman,558–559)

53. (C) Therapy for suspected urea cycle defectsinvolves parenteral nutrition with glucose,lipids, and essential amino acids to promoteanabolism. Arginine hydrochloride intra-venously is essential to supply the urea cycle

Page 260: Questions - uhs.edu.kh You suspect Lyme disease, most likely contracted by which of the following? (A) ingestion of unripe fruit (B) ingestion of spoiled fruit (C) drinking of contaminated

Answers: 47–58 261

with ornithine, which now becomes an essen-tial amino acid. Additional emergent med-ications to lower the ammonia includesodium benzoate, that combines with glycineto form hippuric acid, and sodium phenylac-etate, that combines with glutamine to formphenylacetylglutamine, both of which aremore easily excreted by the kidney thanammonia. Citrulline is necessary as 1 mol ofcitrulline will remove 1 mol of ammonia.Frequent monitoring of ammonia levels isnecessary. If no appreciable change in ammo-nia is noted within a few hours, the only ther-apeutic option left is dialysis, which may betechnically difficult or impossible if accesscannot be obtained. Increasing arginine infu-sion rates or prolonging an ineffective ther-apy will provide no benefit to this child.Exchange transfusion has little effect on low-ering total body ammonia. While many chil-dren do suffer permanent debilitating effectsof the initial hyperammonemia, considera-tion of DNR status at this time is inappropriate.(Behrman, 367–370)

54. (E) In a well-appearing child with a recent viralillness and without fever, toxicity, or systemicsigns of illness who presents with unexplainedpetechiae and ecchymoses, idiopathic throm-bocytopenic purpura (ITP) is the most likelydiagnosis. A complete blood count is indicatedto establish the presence of thrombocytopeniaand assess the other cell lines. When other celllines are affected, a bone marrow may beneeded to rule out leukemia and other hema-tologic disorders. Trauma presenting only withgeneralized petechiae would be rare. rickettsialor neisserial infections can cause petechiae butmost affected patients have other symptoms.(Rudolph CD,1556–1557)

55. (D) The presence of isolated thrombocytopeniaand increased megakaryocytes on bone marrowexamination establishes the diagnosis of idio-pathic thrombocytopenic purpura. If examina-tion of the bone marrow reveals a lack ofmegakaryocytes, the diagnosis is not ITP andother causes of thrombocytopenia should bepursued. Several treatment options for ITP exist,including simple observation. Corticosteroids,

intravenous immunoglobulin, and anti-Rhoantibodies have all been used with success.Indications for treatment include evidence ofbleeding, thrombocytopenia less than 50,000, orprolonged thrombocytopenia. The presence offever is not relevant to the treatment decision.The mechanism of action of anti-Rho antibodiesis uncertain. However, it is thought that reticu-loendothelial Fc receptor blockade is involved.This type of therapy will only be effective inpatients that are Rh-positive. (Rudolph CD,1556–1557)

56. (D) ITP resolves completely within 6 months in80%–90% of cases. Splenectomy is seldom nec-essary, and usually is reserved for chronic caseslasting greater than 6–9 months. Splenectomymay also be required for control of severe recal-citrant bleeding. Low platelet counts, bleeding,splenomegaly, and anemia are not in them-selves indications for splenectomy. (Rudolph CD,1556–1557)

57. (A) In a patient presenting with hypertensionand pallor following a history of hemorrhagicenteritis, the presence of hemolytic-uremic syn-drome (HUS) should be considered. The mostfrequent cause is infection with E coli O157:H7.The primary event in this condition is endothe-lial injury from verotoxin release, resulting in amicroangiopathic hemolytic anemia. Renal vas-cular involvement results in hypertension. TheCBC and smear will show anemia, thrombo-cytopenia, and schistocytes. The other testsmay be abnormal but will not establish thediagnosis. (Behrman, 2181–2182)

58. (A) Additional evidence for HUS is mani-fested by the elevated BUN and creatinine.Care for HUS is primarily careful medicalmanagement in conjunction with early andfrequent dialysis to control fluid overload,manage electrolyte abnormalities, and removeplasminogen activator inhibitor-1 so thatendogenous fibrinolytic mechanisms can dis-solve vascular thrombi. Plasmapheresis andfresh frozen plasma have been used with somesuccess as well. Steroids and antibiotics haveno ameliorative effect, and the illness rarelyrecurs. Ninety percent of patients survive the

Page 261: Questions - uhs.edu.kh You suspect Lyme disease, most likely contracted by which of the following? (A) ingestion of unripe fruit (B) ingestion of spoiled fruit (C) drinking of contaminated

262 10: Case Diagnosis and Management

acute phase and the majority of patients regainnormal renal function. (Behrman, 367–370)

59. (C) This patient has typical signs and symp-toms of croup which is caused primarily byparainfluenza viruses. Infection results in tra-cheal airway inflammation and edema. In mildcases, positive intrathoracic airway pressuregenerated by coughing generates a high-pitched, seal-like barky cough. As narrowingcontinues, the negative thoracic pressure gen-erated by the diaphragm will cause the airwayto collapse, requiring more accessory muscleeffort to generate airflow. Nebulized epineph-rine provides immediate effect in relief ofedema, primarily through its alpha agonisteffect. Steroids are effective in relieving theedema of croup, but typically take severalhours for effects to be seen. The hypoxia anddegree of respiratory distress this child is man-ifesting requires more than just humidified air.While the other options presented may all beuseful in the management of croup, racemicepinephrine will provide the fastest relief.(Behrman, 1762–1765)

60. (D) If a patient is not responsive to initial thera-pies, other etiologies of upper airway obstruc-tion must be considered. Of those (a foreignbody, retropharyngeal abscess, epiglottitis, peri-tonsillar abscess, or Ludwig angina), epiglottitisis the diagnosis suggested by the clinical pres-entation in this patient. Compared to patientswith viral croup, patients with bacterial epiglot-titis typically are more toxic appearing. Theypresent with lethargy, drooling, and a rapidonset of respiratory failure. When the diagnosisof bacterial epiglottitis is highly suspected, inter-vention (intubation) should not be delayed toawait radiographic confirmation (thumb printsign), antibiotic administration, or observation.Because of the risk of the development of com-plete airway obstruction, establishment of anartificial airway is the first priority. Controlledendotracheal intubation, performed in the oper-ating suite if possible, should be undertakenwith immediate surgical backup. Rapid-sequence intubation in the emergency depart-ment may become necessary, but is not preferred

to the more controlled environment of the oper-ation suite. (Behrman, 836, 1275–1278)

61. (A) The patient described is not in severe dis-tress as indicated by the ability to talk in com-plete sentences and the lack of hypoxia. Therecurrent episodes of wheezing suggest asthmaas the most likely etiology. An initial trial of aninhaled beta agonist is the most appropriatechoice. These agents’ primary mode of action isrelaxation of airway smooth muscle. The othertherapies listed are sometimes used, but not asthe initial management for acute exacerbationsof mild-to-moderate asthma. (Rudolph CD,1950–1963; Behrman, 958–969)

62. (C) An expert panel report provides guidelinesfor management of asthma based on the sever-ity of the disease. For infants and young chil-dren who have had four or more episodes ofwheezing in the past year that lasted more than1 day and impaired sleep plus additional riskfactors for developing persistent asthma,including parental history of asthma, physiciandiagnosis of atopic dermatitis, or evidence ofsensitization to aeroallergens, two of the fol-lowing including evidence of sensitization tofoods, greater than or equal to 4% peripheralblood eosinophilia, or wheezing apart fromcolds, long-term control therapy is recom-mended. The patient described has persistentasthma. Recommended management includesthe daily use of an anti-inflammatory agent.Therefore, inhaled fluticasone is the mostappropriate choice given. Autoinjectable epi-nephrine should be used for anaphylaxis and is not indicated in the routine management of asthma. Salmeterol, a long-acting beta2-adrenergic agonist, is indicated for patients withmore severe asthma. Because of its narrow ther-apeutic index and need for frequent monitoring,theophylline’s use has declined and currently isconsidered only a second line-agent for patientswith asthma refractory to standard therapy.Antihistamines do not decrease bronchocon-striction and are not routinely part of chronicasthma management (see Table 10-2). (Rudolph CD,1950–1963; Behrman, 674–679; http://www.nhlbi.nih.gov/guidelines/asthma/asthgdln.htm)

Page 262: Questions - uhs.edu.kh You suspect Lyme disease, most likely contracted by which of the following? (A) ingestion of unripe fruit (B) ingestion of spoiled fruit (C) drinking of contaminated

Components ofSeverity

Classifying Asthma Severity andInitiating Therapy in Children

Symptoms

Nighttimeawakenings

Short-actingbeta2-agonist use forsymptom controlInterference withnormal activity

Lung Function

• FEV1 (predicted) or peak flow (personal best)

Exacerbationsrequiring oral systemic

corticosteroids(consider severity and

interval since lastexacerbation)

0-1/year (see notes)

Intermittent

Ages0-4

Ages5-11

Ages0-4

Ages5-11

Ages5-11

Persistent

ModerateMild Severe

≤2 days/week

≤2 days/week

None Minor limitation Some limitation Extremely limited

N/AN/AN/AN/A

Normal FEV1between

exacerbations

>80% >80%

≥ 2x/year(see

notes)

≥2 exacerbationsin 6 months

requiring oralsystemic

corticosteroids, or≥4 wheezing

episodes/1 yearlasting

>1 day and riskfactors for

persistent asthma

Relativeannual

risk maybe relatedto FEV1

<60%

<75%

60-80%

74-80%>80%>85%

0≤2x/

month1-2x/month

3-4x/month

3-4x/month

>2 days/weekbut not daily

>2 days/weekbut not daily

Daily Throughout the day

Several times per dayDaily

>1x/week but not nightly

>1x/week

Often7x/week

Ages 5-11Ages0-4

Ages0-4

Step 1(for both age groups)

Step 2(for both age groups)

Step 3 andconsider

shortcourse of

oralsystemiccortico-steroids

Step 3:medium-dose

ICS optionand considershort course

of oralsystemiccortico-steroids

Step 3:medium-dose

ICS optionor step 4

and considershort course

of oralsystemiccortico-steroids

Step 3and

considershort

courseof oral

systemiccortico-steroids

• FEV1/FVC

Risk

Impairment

Recommended Step for Initiating Therapy

(See “Stepwise Approach for ManagingAsthma” for treatment steps.)

The stepwise approach is meant to assist, not replace,the clinical decisionmaking required to meet individual

patient needs.

Key: FEV1, forced expiratory volumein 1 second; FVC, forced vital capacity;ICS, inhaled corticosteroids; ICU,intensive care unit; N/A, not applicable

Notes:• Level of severity is determined by both impairment and risk. Assess impairment domain by caregiver’s recall of previous 2-4 weeks. Assign severity to the most severe category in which any feature occurs.• Frequency and severity of exacerba- tions may fluctuate over time for patients in any severity category. At present, there are inadequate data to correspond frequencies of exacerbations with different levels of asthma severity. In general, more frequent and severe exacerba- tions (eg, requiring urgent, unscheduled care, hospitalization, or ICU admission) indicate greater underlying disease severity. For treatment purposes, patients with ≥2 exacerbations described above may be considered the same as patients who have persistent asthma, even in the absence of impairment levels consistent with persistent asthma.

In 2-6 weeks, depending on severity, evaluate level of asthma control that is achieved,• Children 0-4 years old: If no clear benefit is observed in 4-6 weeks, stop treatment and consider alternative diagnoses or adjusting therapy.• Children 5-11 years old: Adjust therapy accordingly.

TABLE 10-2. National Heart Lung and Blood Institute. National Asthma Education and Prevention Program Expert Panel Report 3. Guidelines for the Diagnosis and Management ofAsthma (epr-3). http://www.nhlbi.nih.gov/guidelines/asthma/asthsumm.pdf. Published July 2007. Updated 2002.

Page 263: Questions - uhs.edu.kh You suspect Lyme disease, most likely contracted by which of the following? (A) ingestion of unripe fruit (B) ingestion of spoiled fruit (C) drinking of contaminated

264 10: Case Diagnosis and Management

63. (E) The history of recent travel to a countrywith high rates of contagious illness makes an infectious etiology more likely in this previously healthy child. His icterus andhepatomegaly point to an infectious hepatitis.Whether he had received the hepatitis A vac-cine would be most useful to know. Sickle cellmay be associated with jaundice, but is mostcommon in individuals of African descent.Consumption of large amounts of carotene-containing vegetables such as carrots mayresult in carotenemia. This may cause cuta-neous discoloration resembling jaundice, butwould not cause scleral icterus or systemicsymptoms. Malar rash might suggest the pos-sibility of systemic lupus erythematosis whichmight be associated with hepatitis, but the con-stellation of findings presented is more sug-gestive of an infectious illness. Acetaminophencan result in hepatic dysfunction but generallyin the context of overdose. (Behrman, 1680–1682)

64. (A) Given the history of travel to Mexico,hepatits A is the most likely etiology of this ill-ness, and may be diagnosed by serology. Theaspartate aminotransferase will be elevated butdoes not establish the etiology of the hepatitis.Twenty-four hour urinary copper excretion isuseful in the diagnosis of Wilson disease whichcan present with hepatic dysfunction, but isnot indicated in the clinical context presentedhere. Unless there is a suspicion of acetamino-phen overdose, determination of serum level isnot warranted. (Behrman, 769–771)

65. (D) Hepatitis A is a picornavirus spread viathe oral-fecal route that has a mean incubationperiod of approximately 4 weeks. Most infectedchildren are asymptomatic. In certain devel-oping countries, prevalence of infection withthis agent approaches 100%. After infection,protective antibodies persist for life. An effec-tive vaccine currently is available and is inwidespread use. Fortunately, most childrencompletely recover from hepatitis A infection.Rarely, it will progress to fulminant hepaticfailure. In patients with hepatitis B infection,coinfection with hepatitis D can lead to fulmi-nant hepatic failure. Hepatitis B, C, and D arespread via exposure to blood products or via

intimate sexual contact. Hepatitis A could the-oretically be spread this way but that is not themost common manner. (Behrman, 769–771)

66. (B) Response to supplemental oxygen is help-ful in the differential diagnosis of cyanosis inthe neonate. The lack of response to increasedoxygen suggests a right-to-left shunt, which ina newborn would most likely be due to cyan-otic congenital heart disease. Hypoxemia asso-ciated with sepsis and pneumonia shouldrespond to an increase in inspired oxygen.Because the PaO2 is a measure of oxygen dis-solved in plasma, it will be normal in patientswith methemoglobinemia and will increasewith administration of oxygen. However,cyanosis will persist as oxygen saturation ofhemoglobin will remain impaired. While anarteriovenous fistula might produce right-to-leftshunting and would not respond to increasedoxygen, this is a much less likely entity thancyanotic congenital heart disease. (Rudolph CD,1824–1836)

67. (A) Transposition of the great arteries (TGA) isthe most likely cardiac malformation to presentin the early newborn period with cyanosis andabsence of heart murmur. TGA occurs in about1/5000 live births and is more common inmales and infants of diabetic mothers. Themediastinum often appears narrowed on chestradiographs. Tetralogy of Fallot usually pres-ents slightly later in infancy with cyanosis anda systolic murmur from the associated ventric-ular septal defect. Total anomalous pulmonaryvenous return presents in a similar fashion but usually is associated with increased pul-monary vascular markings on chest radiography.(Rudolph CD,1824–1836)

68. (A) This infant is critically ill and immediateintervention is needed. The hypoxemia requiresimmediate cardiac catheterization for atrial septostomy to create a shunt and improve oxy-genation. Antibiotics or pressors such as dobu-tamine may slightly improve the clinical state,but do nothing to correct the underlying prob-lem. An arterial switch generally is deferreduntil the patient’s electrolytes and medical statushave stabilized. (Rudolph CD,1824–1836)

Page 264: Questions - uhs.edu.kh You suspect Lyme disease, most likely contracted by which of the following? (A) ingestion of unripe fruit (B) ingestion of spoiled fruit (C) drinking of contaminated

Answers: 63–74 265

69. (B) The presence of anemia, jaundice, and darkurine suggests hemolysis as the most likely causeof the anemia. Evaluation should inquire aboutprevious illnesses, episodes of pallor, and a list ofall medicines taken, such as aspirin, antibiotics,or antimalarials. A family history of leukemia orsickle cell trait alone in a previously healthy childis unlikely to be relevant. Hepatitis A may causejaundice but not pallor; also the cousin isdescribed as healthy. Initial laboratory results toobtain include a smear to define red cell mor-phology, a reticulocyte count to determinemarrow response, and the type of bilirubinmade. A bone marrow may need to be per-formed at a later time. (Behrman, 739, 2040–2042)

70. (E) The unconjugated bilirubinemia is compat-ible with increased bilibrubin productionresulting from hemolysis. Initial presentation ofsickle cell disease at 6 years of life would beunusual. Absence of spherocytes on the periph-eral smear weighs against the diagnosis ofspherocytosis, although they may be absent in the face of acute hemolysis. Glucose-6-phosphate-dehydrogenase deficiency is themost likely etiology of those presented.Hemolysis associated with this disorder maybe precipitated by sulfa-based antibiotics, anti-malarials, aspirin, as well as organic solvents.Diagnosis depends on measurement of G6PDlevels in older red cells. (Behrman, 1474, 1489–1491)

71. (C) The patient described has no evidence ofcardiovascular instability; therefore, transfu-sion is not necessary or indicated. Close obser-vation and serial hemoglobin determinations tomonitor for continued hemolysis are sufficient.Removal of the offending agent usually resultsin cessation of hemolysis. Prednisone and othersteroids have not been shown to change thecourse of the illness. (Behrman, 1474, 1489–1491)

72. (D) This is a classic presentation of Guillain-Barrésyndrome (GBS), a postinfectious polyneuropa-thy. Illness is frequently preceded by a nonspe-cific viral illness. The patient experiencesascending paralysis with the bulbar musclesaffected last, if at all. Deep tendon reflexes arelost, usually early in the course of the illness.Sensory function usually is spared. Nerve

conduction velocity is decreased in Guillain-Barré syndrome. While there is no specific diag-nostic test, the clinical picture combined with acerebrospinal fluid showing an elevated proteinand normal cell count is highly suggestive. Occa-sional cases of GBS can be associated with infec-tion due to M pneumonia or Campylobacter species.The other laboratory abnormalities listed are notassociated with Guillain-Barré syndrome.(Behrman, 1200–1201, 2565–2566)

73. (A) Patients in the early phase of the illnessshould be admitted for monitoring of their res-piratory status as the muscles of respirationmay become affected. Administration of IVIGhas been shown to decrease the length of illnessand lessen the associated long-term disabil-ity. Most patients will recover fully within 2–3 weeks and supportive care is all that is rou-tinely needed. Other therapies that have beenused for rapidly progressive courses includeplasmapheresis, high-dose steroids, and immu-nosuppressive agents. Intravenous interferon-beta is a therapeutic agent that has shownpromise in the treatment of multiple sclerosisbut not GBS. (Behrman, 1335–1336, 1892–1893)

74. (D) The most likely cause of this patient’s ill-ness is diabetes mellitus with diabetic ketoaci-dosis (DKA). After assessing mental status andensuring airway and breathing are intact,restoration of circulating volume using isotonicfluids is the first priority in the treatment ofDKA. Diabetic ketoacidosis usually has aninsidious onset as the body tries to compen-sate for the hyperglycemia and acidosis. Thedeficiency of insulin increases production offatty acids and ketone bodies resulting in meta-bolic acidosis. Breathing becomes deep andrapid (Kussmaul respirations) as the respira-tory centers increase ventilation to compensatefor the metabolic acidosis. Additional lactic aci-dosis, when the osmotic diuresis is associatedwith hyperglycemia, results in increased uri-nary output (polyuria) and volume depletion.Dehydration and the caloric loss from gluco-suria lead to weight loss. As thirst increases,patients will increase volume intake (polydipsia),but the acidosis and other metabolic derange-ments may lead to nausea, vomiting, and

Page 265: Questions - uhs.edu.kh You suspect Lyme disease, most likely contracted by which of the following? (A) ingestion of unripe fruit (B) ingestion of spoiled fruit (C) drinking of contaminated

266 10: Case Diagnosis and Management

abdominal pain. Once these compensatorymechanisms are overwhelmed, DKA willbecome manifest as in this patient. Bicarbonateboluses have been associated with the devel-opment of cerebral edema. Although oralintake is not necessarily contraindicated inDKA, it is not the initial step in management.(Berhman, 2415–2417)

75. (A) A variety of metabolic derangements mayresult from the treatment of DKA. One of themore common is hypokalemia. Althoughserum potassium may be elevated in theuntreated state, total body potassium is actu-ally low. With insulin replacement and resolu-tion of acidosis, hypokalemia may becomemanifest. Hypophosphatemia may also occur,as phosphate is transported intracellularly torestore adenosine triphosphate stores. Likewise,depletion of total body stores of magnesiumand calcium occurs during DKA. (Berhman,1772–1777)

76. (E) Parental requests for antibiotic therapy arefrequent. A red eardrum alone is not diagnos-tic of otitis media. Diagnosis should be madeonly in the presence of a bulging tympanicmembrane, opaque middle ear fluid, andabnormal membrane mobility on pneumaticotoscopy. For this child’s likely viral illness,antipyretic therapy and fluids are all that isindicated at this time. The physician shouldtake the time to discuss the development of abacterial ear infection so the parents under-stand why antibiotics are not necessary at thistime, but may become necessary over thecourse of the illness. (Rudolph CD,1249–1255)

77. (A) The child now has developed acute otitismedia with effusion. Viral upper respiratoryinfections are the most frequent antecedent ofotitis media in young children. They result ineustachian tube dysfunction that predisposesto middle ear bacterial superinfection, mostcommonly by S pneumoniae, nontypeableHaemophilus influenzae, or Moraxella catarrhalis.Amoxicillin is considered by many authoritiesto be first-line therapy for otitis media. Becauseof increasing pneumococcal penicillin resist-ance rates, high doses (ie, 80–90 mg/kg/day)

are appropriate. This is especially true in thecontext of day care attendance because of theincreased risk for resistant pneumococci amongchildren in this setting. Ceftriaxone has beendemonstrated to be effective in the manage-ment of otitis media, but should generally bereserved for cases not responsive to initialantibiotic management. Erythromycin is notappropriate in the treatment of this infectionbecause many of the offending organisms areresistant. An otorhinolaryngology consult isnot routinely necessary in the management ofacute otorrhea. (Rudolph CD,1249–1255)

78. (D) The physical findings described indicatethat mastoiditis has developed as a complica-tion of the otitis media. The organisms causingmastoiditis are the same as for otitis media.Computerized tomography of the mastoidspermits delineation of the area of involvement.Intravenous antibiotic therapy including athird-generation cephalosporin and possiblyadditional gram-positive coverage are alsoindicated. Consultation with an otolaryngolo-gist for drainage is appropriate. The other choicesare inappropriate at this stage. (Rudolph CD, 1249 –1255)

79. (E) Diagnostic options for evaluating the etiol-ogy of this patient’s problems are numerous.However, the first step is to interview thepatient alone. Her symptoms and flat affectwarrant a frank, private discussion with thepatient regarding issues such as depression,suicidal ideation, abuse (physical, mental, andsexual), eating disorders, and drug abuse. Itshould be stressed that theses discussions willbe kept in confidence. Although both organicand psychiatric conditions may cause the con-stellation of findings seen in this patient, theinitial step in the evaluation of either is a care-ful history. The behavior described is not typi-cal for a healthy adolescent and should not beignored. (Behrman, 122–123)

80. (D) Psychiatric illness needs to be aggressivelyaddressed. The health care professional needsto assess whether the patient poses a threat toherself or others; the rate of suicide in the15–19-year-old age group is around 9/100,000.

Page 266: Questions - uhs.edu.kh You suspect Lyme disease, most likely contracted by which of the following? (A) ingestion of unripe fruit (B) ingestion of spoiled fruit (C) drinking of contaminated

Answers: 75–85 267

If there is any indication of suicidal ideation,the patient should be evaluated emergently bya psychiatrist. Inpatient admission with closemonitoring may be warranted in such cases.Collection of urine for drug screening againstthis patient’s will increases barriers to estab-lishing a therapeutic alliance. Although comor-bidity with drug use is often seen in patientswith psychiatric disorders, screening for druguse should only proceed with the patient’s con-sent. Variation in mood is characteristic of ado-lescents, but the severity indicated here isoutside the norm. Selective serotonin reuptakeinhibitors are safe and possess minimal sideeffects. Initiation of this class of therapeuticagent along with counseling and close follow-upis appropriate in the case described. (Behrman,80, 92)

81. (B) Bipolar disorder usually presents in thethird decade of life but can present in late ado-lescence. Patients with this disorder vacillatefrom depression to mania or may present withmania alone. During manic spells the patient’sbehavior is characterized by high levels ofactivity, loquaciousness, insomnia, feelings ofgrandiosity, a tendency toward overspending,and an expansive mood that may persist forweeks. SSRIs can heighten energy but not tothis extent. The lack of delusions and the abil-ity to function well enough to purchase itemsmake schizophrenia less likely. Sexual abusecan present in a variety of fashions but givenher history, a bipolar disorder is more likely.Lithium carbonate, valproic acid, and/or car-bamazepine along with intense psychiatrictherapy may be necessary to control the symp-toms of this illness. (Behrman, 80, 92)

82. (C) The findings indicate that this patient hasleukemia and is presently neutropenic. Mostinfections in neutropenic patients arise fromendogenous flora, primarily the gastrointesti-nal tract. Bacteria intermittently seed the bloodstream by translocating across the mucosal sur-face due to impaired immune surveillance.Empiric antibiotics should be directed againstgram-negative organisms, including P aeruginosa.Of the options presented, ceftazidime is the mostappropriate, offering coverage of gram-negative

organisms including P aeruginosa. Despitebroad activity against gram-negative bacteria,including P aeruginosa, ciprofloxacin givenorally is not appropriate in the situationdescribed. (Behrman, 1103–1104, 2116–2120)

83. (B) Indicators of a more favorable prognosis in acute lymphocytic leukemia include age 1–10 years, hyperdiploidy over 50 chromo-somes, absence of blasts in the CSF, initialwhite blood cell count of less than 50,000,absence of the 9:22 translocation (Philadelphiachromosome), and a TEL/AML-1 transloca-tion. Poor prognostic factors include 11q23 or4:11 translocations or lack of the previouslydescribed favorable conditions. (Behrman,785–787, 1543–1546)

84. (C) Induction chemotherapy may produce thetumor lysis syndrome, a constellation of meta-bolic abnormalities resulting from destructionof large numbers of tumor cells with release ofintracellular constituents. In particular, potas-sium, calcium, and phosphorus levels can riseprecipitously. Hyperkalemia can lead to dan-gerous arrhythmias. Simultaneous develop-ment of hyperphosphatemia and hypercalcemiacan lead to heterotopic calcifications in variousorgan systems; kidney involvement may resultin renal failure. The release of uric acid can leadto nephrolithiasis. Preventative measures forthese complications include aggressive hydra-tion, alkalinization of the urine, and allopurinoladministration. Dialysis may be needed if thesepreventative efforts fail. (Behrman, 785–787,1543–1546)

85. (E) Colic typically begins prior to 3 months ofage, and is characterized by 3 or more hours ofunexplained crying at least three times a week.The cry is spontaneous and continuous, withthe face appearing flushed, the legs drawn up,and the hands clenched. Bouts of crying asso-ciated with colic may resolve when the patientbecomes exhausted or with the passage offlatus or belching. Without fever, ill appear-ance, vomiting, weight loss, or other abnormalsigns or symptoms, reassurance and follow-upare the most appropriate initial steps. (Rudolph CD,414–417)

Page 267: Questions - uhs.edu.kh You suspect Lyme disease, most likely contracted by which of the following? (A) ingestion of unripe fruit (B) ingestion of spoiled fruit (C) drinking of contaminated

268 10: Case Diagnosis and Management

86. (B) A wide variety of colic treatment modalitieshave been suggested. The most effective oneremains time, as colic rarely persists over 3 months of age. Swaddling has been shown tocalm some infants. Stress is a precipitant of colic,and subjecting a single caregiver to the unremit-ting crying associated with colic is likely toworsen the stress (of both caretaker and baby).Water has not been demonstrated to be effectivein this condition and excessive amounts givento young infants may precipitate hyponatremia.Likewise, vigorous shaking obviously is veryharmful to infants. Paregoric (morphine, alco-hol, and anise oil) is ineffective and potentiallyharmful. (Rudolph CD,414–417)

87. (E) A preverbal child cries to express him- orherself, and this needs to be stressed to par-ents. A search for causes of discomfort needs tobe performed by the physician. Potential causesinclude hair tourniquets, occult fractures orinfection, corneal abrasions, and severe diaperrashes. Irritability can also be due to sepsis,intussusception, hernia, or feeding intolerance.Given this patient’s appearance, and the asso-ciated bilious emesis, a prompt evaluation toexclude the diagnosis of volvulus is necessary.(Rudolph CD,1410)

88. (C) A history of constipation and the potentialfor an associated anal fissure would be useful,given the history of streaks of blood on the sur-face of the stools. Hemophilia rarely presentswith gastrointestinal bleeding. Iron intake isirrelevant as iron (except in overdose) does notcause gastrointestinal bleeding. Rotavirus isunlikely in the absence of diarrhea and typi-cally does not result in the presence of grossblood in the stools. Crohn disease would berare in a patient of this age. (Behrman, 1639)

89. (C) Anal fissures are the most common cause ofbloody stools in the pediatric population.Continued passage of hard stool may preventhealing, a fact that should be stressed to theparents. Initial treatment should be aimed atsoftening the stool. Increasing fiber intake isvery useful but hard to do as it may requiredrastically altering the diet of the child (and theparents as well). A stool softener (such as

lactulose or mineral oil) titrated to the desiredconsistency is usually needed as well. Infectionsof anal fissures are rare; parents should betaught what to look for, but routine antibioticsare not warranted. The use of rectal dilators,frequent rectal temperatures, or colonic manip-ulation should not be performed. (Behrman, 1181)

90. (D) Juvenile rheumatoid arthritis (now knownas juvenile idiopathic arthritis or JIA) affectsabout 250,000 children in the United States witha yearly incidence of approximately 14/100,000children under 16 years of age. Joint symptomsare typically more severe in the morning andimprove through the day. Large joints are usu-ally affected. Progression of disease may resultin persistent joint effusion, arthritis, and fusionof wrist joints with loss of movement. JIA isclassically divided into three types: polyartic-ular, pauciarticular (with two subtypes), andsystemic onset. Of these, polyarticular, rheuma-toid factor positive JIA tends to have the worstprognosis with respect to joint involvement.(Behrman, 1001–1010)

91. (E) Eye involvement can be seen in either pol-yarticular or pauciarticular type of JIA, but ismore commonly seen in the pauciarticulartype. Affected children complain of redness,pain, photophobia, and decreased visual acuity.Early detection by serial ophthamologic exam-inations is essential for prevention of blindnessin these children. (Behrman, 704–709)

92. (B) The patient described is in shock which ischaracterized by inadequate end-organ perfu-sion. Evidence of inadequate perfusion in thispatient includes tachycardia and delayed cap-illary refill time. Shock is considered to be com-pensated in the presence of a normal bloodpressure. Hypotension is defined by a bloodpressure less than the 5th percentile for age. Forchildren aged 1–10 years, the lower limits ofnormal systolic blood pressure (in mm Hg) canbe estimated using the following formula: 70 +(age in years X 2). Other causes of shock includecardiogenic (eg, due to myocarditis) and sepsis.However, the presence of sunken eyes andfontanelle, along with poor skin turgor and thehistory of fluid loss from vomiting and diarrhea

Page 268: Questions - uhs.edu.kh You suspect Lyme disease, most likely contracted by which of the following? (A) ingestion of unripe fruit (B) ingestion of spoiled fruit (C) drinking of contaminated

Answers: 86–99 269

implicate hypovolemia as the most likely etiol-ogy of this patient’s shock. (McMillan, 63–64,2582–2587)

93. (C) Mild dehydration, that is, 3%–6% loss ofbody weight, is manifested by increased thirstand tachycardia but normal skin turgor and cap-illary refill time. Moderate dehydration in younginfants such as this patient occurs when there isa fluid loss approximating 10% body weight,whereas older children have moderate dehy-dration with fluid losses of about 6%–9% bodyweight. Moderate dehydration is characterizedby the signs and symptoms of mild dehydrationtogether with weak peripheral pulses. Patientswith severe dehydration (fluid loss > 10% bodyweight) frequently have associated hypotension,not present in the patient under discussion.Therefore, this child has an estimated fluid lossof 10% body weight (approximately 600–700 cc).(McMillan, 63–64, 2582–2587)

94. (A) The initial hydrating solution should haveclose to 130–155 meq/L of sodium so that itcan expand the intravascular volume quickly. Ifthe patient is hypernatremic, it will notdecrease the serum sodium too quickly, andwill also correct any hyponatremia slowly aswell. Appropriate resuscitation fluid could benormal saline (154 meq/L) or lactated Ringer(130 meq/L). (McMillan, 63–64, 2582–2587)

95. (E) Onset of a vesicular exanthem and enan-them in the early neonatal period should sug-gest the possibility of herpes simplex infection.Most women giving birth to infected babieshave no known history of herpes infection.Evaluation of babies with suspected herpesinfection should include cultures of the lesionsfor virus. Examination of the CSF is also impor-tant to evaluate for concomitant central nerv-ous system infection. Prompt initiation ofacyclovir is important in reducing morbidityand mortality. Institution of antiviral therapy inpatients with compatible signs and symptomsshould not await culture confirmation. (AAP:RedBook, 365–366)

96. (A) The overall incidence of HSV infection isfrom 1/3000 to 1/20,000 of all live births. The

risk of vertical transmission in cases of primarymaternal infection is 33%–50%. The risk oftransmission is substantially lower (> 5%) inthe setting of reactivated disease. (AAP:RedBook,362–363)

97. (C) Risk factors for vertical transmission includeprolonged rupture of membranes greater than4–6 hours, primary maternal infection, presenceof active lesions on the vaginal mucosal sur-face, and the use of scalp electrodes. Maternalcoinfection with HSV type 1 or human immun-odeficiency virus is not known to increase riskof transmission of HSV type 2. Peripartumadministration of acyclovir has not beendemonstrated effective in reducing the risk oftransmission. (A AAP:Red Book,368–369)

98. (A) This child has a healing clavicle fracturethat likely occurred during childbirth. Largebabies have an increased risk of this problem.Associated symptoms may be minimal orabsent. Callus formation does not becomeapparent until about 2 weeks of age, so it is notsurprising that this is when the parents broughtthe infant in for evaluation. Some infants willnot display irritability despite this obvious frac-ture. Comparison views are not necessary inthe evaluation. A fibroblast assay is not war-ranted as this fracture is most commonly seen inotherwise normal neonates. (Behrman, 727)

99. (B) Factors that increase the difficulty of deliv-ery may increase the risk of clavicular fracture.These include maternal diabetes (because of the association with fetal macrosomia),cephalopelvic disproportion, shoulder dysto-cia, or extension of the arms in breech position.Congenital syphilis may be associated withskeletal abnormalities, but there is no increasein risk for clavicle fractures. Likewise, maternaldrug use has not been implicated as a cause ofthis problem. Thanatophoric dysplasia, lethalform of chondrodysplasia with associateddefects in bone growth, is a rare,. These infantshave short limbs, narrow chests, and largeheads, but do not have an increased risk ofclavicle fractures. (Behrman, 2877–2878)

Page 269: Questions - uhs.edu.kh You suspect Lyme disease, most likely contracted by which of the following? (A) ingestion of unripe fruit (B) ingestion of spoiled fruit (C) drinking of contaminated

270 10: Case Diagnosis and Management

100. (B) The patient described has features typical ofDown syndrome. Ninety-five percent of patientswith Down syndrome have trisomy of chro-mosome 21, usually due to mitotic nondis-junction of maternal gonadal cells. About 4% of affected patients have a centromerictranslocation of chromosome 21 onto another chromosome, usually 13, 14, 15, or 21. Trans-locations account for 9% of Down syndromepatients born to mothers under 30. (Behrman,507–509)

101. (B) Down syndrome is the most common tri-somy, occurring in about 1/600–1/800 births.For comparison, sickle trait occurs in approxi-mately 1/12 of African Americans, trisomy 18(Edwards syndrome) occurs in about 1/8000births, and trisomy 13 (Patau syndrome) occursin about 1/20,000 births. (Behrman, 324–328)

102. (D) Individuals with Down syndrome are atincreased risk for a variety of disorders. Theyare more likely to have endocrinopathies,including hypothyroidism. Congenital heartdisease (most commonly endocardial cushiondefects) occurs in almost half of these children.They also have an increased likelihood ofleukemia, but not solid tumors. Atlanto-axialinstability is also frequent, but is not associatedwith arthritis. Streak gonads are a finding seenin Turner syndrome. (Behrman, 507–509, 2122)

103. (E) Neutropenia generally is defined as anabsolute neutrophil count less than 1500/mm3.Neutropenia is one of the major risk factors forinfection in patients undergoing chemother-apy for malignancy. Conversely, the previouslyhealthy child with an acute undifferentiatedfebrile illness and a nontoxic appearance has avery low risk of serious infection. The fact thatthe other blood cell lines are normal is reas-suring. The most likely cause of the neutrope-nia is transient bone marrow suppression froma viral infection. Antibiotic therapy is not rou-tinely necessary. Observation and follow-up ofthe WBC count to document resolution of theneutropenia would be the most appropriatemanagement. (Behrman, 910–912)

104. (C) The onset of hemiparesis most likely is dueto a thrombotic stroke, a known complication ofsickle cell disease. Rapid intervention is essentialin order to minimize morbidity. Computerizedtomography may not show evidence of ischemiafor several hours after onset of symptoms.Exchange transfusion, which acts by decreasingthe proportion of hemoglobin S, is the thera-peutic intervention of choice. Unlike the situa-tion in adults where strokes are primarily due toatherosclerosis and activation of the clotting cas-cade, heparinization or infusion of plasmino-gen activator provides no benefit in strokeassociated with sickle cell. Hydroxyurea maybe helpful in long-term treatment to increasehemoglobin F but is not useful in the acute man-agement of stroke. (Behrman, 2026–2031, 2509)

105. (D) Patients with sickle cell disease have func-tional asplenia by 1–2 years of age and areparticularly susceptible to infection by poly-saccharide-encapsulated bacteria, including S pneumoniae, Haemophilus influenzae, and N meningitidis. Of these, the pneumococcus isthe most frequent cause of bacteremia.(Behrman, 1479–1483)

106. (E) Sickle cell disease is an autosomal reces-sive disorder. Approximately 12% of AfricanAmericans are heterozygous for the abnormalgene. However, this risk varies in other ethnic/racial groups and the risk of this patient’s chil-dren cannot be determined without knowingthe risk of his future mate carrying the abnor-mal gene. However, the risk for future childrenof couples with an affected child is approxi-mately 25%. (Behrman, 1479–1483)

107. (C) This boy has stigmata of Duchenne muscu-lar dystrophy (DMD) including pseudohyper-trophy of the calves and a waddling gait.Because of muscle weakness, patients withDMD may use their hands to “walk” up theirlegs when arising from a sitting position (Gowersign). Rovsing sign occurs in appendicitis whenpalpation of the left lower quadrant elicits painin the right lower quadrant. Opisthotonus(hyperextension of the head, neck, and trunk)occurs in tetanus and kernicterus. Chvostek signis present when tapping over the course of the

Page 270: Questions - uhs.edu.kh You suspect Lyme disease, most likely contracted by which of the following? (A) ingestion of unripe fruit (B) ingestion of spoiled fruit (C) drinking of contaminated

Answers: 100–114 271

facial nerve causes contraction of the muscles ofthe eye, mouth, or nose and occurs with hypocal-cemia. Grey-Turner sign (bruising of the flanks)is associated with hemorrhagic pancreatitis.(Behrman, 2540–2544)

108. (E) As weakness progresses and difficulty han-dling oral secretions ensues, aspiration pneu-monia becomes a problem for patients withDuchenne muscular dystrophy. Other compli-cations seen include osteopenia and scoliosis dueto the weakness and immobility. Scoliosis maybecome severe enough to contribute to impairedventilation. Cardiomyopathy progressing to con-gestive heart failure may also occur. The myo-globinuria associated with this condition doesnot cause renal failure and the bulbar muscles arerarely affected. (Behrman, 1966, 2540–2544)

109. (B) DMD is primarily an X-linked recessive dis-order, occurring in about 1/3600 male infants. Aswith other X-linked recessive diseases, approxi-mately 50% of male offspring will be affectedwhile 50% of female offspring will be carriers.Affected males will not pass the disease to theirsons, but all of their daughters will be carriers.Although female carriers are generally notseverely affected, Barr body inactivation canresult in the expression of some stigmata of dis-ease. Approximately 30% of cases result fromnew mutations. (Behrman, 1432, 1797, 1873–1877)

110. (A) Of the findings listed, pulsus parodoxus(decline in systolic BP > 10 mm Hg duringinspiration) is the most suggestive of cardiactamponade. This results from impeded left ven-tricular outflow and restricted left atrial fillingduring inspiration caused in part by the descentof the diaphragm exerting traction on the tautpericardium. Enlargement of the cardiac sil-houette is neither sensitive nor specific for thediagnosis of tamponade. Neither boundingpulses nor egophany result from cardiac tam-ponade. Although the electrocardiogram maydemonstrate low voltage, this finding can alsobe seen with myocarditis. (Rudolph CD,1865–1866)

111. (D) In the presence of symptoms, particularlyhypotension, prompt intervention to removethe fluid can be lifesaving. Steroids, inotropes,

diuretics, and/or pericardial window may beneeded as part of further management, butimmediate pericardiocentesis is needed topermit patient stabilization. (McMillan, 2683;Rudolph CD,1865–1866)

112. (B) Fractionation of the bilirubin will guide fur-ther workup of the jaundice. The history ofacholic stools suggests the presence of anobstructive jaundice. The other tests listed mayall be indicated, but it is the bilirubin determi-nation that will provide the most immediatelyuseful data. (Behrman, 1671–1673)

113. (D) The presence of an elevated conjugatedbilirubin in a thriving infant makes an obstruc-tive lesion such as bile duct paucity or a chole-dochal cyst within the liver most likely. Theinitial diagnostic study in the evaluation of thiscondition is ultrasound. Tyrosinemia, which ischaracterized by elevated urinary succinylace-tone levels might present with conjugatedhyperbilirubinemia but not acholic stools.Cystic fibrosis is less likely to present with jaun-dice alone, especially at this age. Finally, plainfilm radiography will not add specific infor-mation. (Behrman, 1205–1206)

114. (E) Biliary atresia affects about 1/10,000–1/15,000infants and varies in severity. An exploratorylaparotomy and cholangiography should bedone to determine if a surgically correctablelesion is present. For those in whom none is seen,the Kasai hepatoportoenterostomy should beperformed as soon as possible to prevent wors-ening of cholestasis and hepatic damage. Therationale is that minute channels may be presentin the porta hepatis and permit reestablishmentof some bile flow, particularly if the channels aregreater than 150 mm in diameter. The rate ofsuccess with this procedure is greatest if per-formed prior to 8 weeks of age. Some patientscan survive indefinitely with this procedure; forothers it is palliative, permitting increased timefor growth and location of a suitable liver fortransplantation. Hepatocyte infusion has beenperformed on an experimental basis, but is notcurrently part of standard care for this condition.(Behrman, 1205–1206)

Page 271: Questions - uhs.edu.kh You suspect Lyme disease, most likely contracted by which of the following? (A) ingestion of unripe fruit (B) ingestion of spoiled fruit (C) drinking of contaminated

272 10: Case Diagnosis and Management

115. (A) The signs and symptoms presented are typ-ical of pinworm infestation. Treatment includesantiparasitic agents and reassurance regardingthe benign nature of the condition. Visualizationof the worms establishes the diagnosis andobviates the need to perform stool ova and par-asite examination. Pinworms are easily trans-mitted and lack of personal hygiene is notthought to be of primary importance in spread.Household pets do not harbor pinworms andare therefore not involved in perpetuation oftransmission. Finally, the pruritis and perianalexcoriations noted are common with pinworminfestation and are not suggestive of sexualabuse in this setting. (Rudolph CD,1105–1106)

116. (A) Appropriate choices for treatment of pin-worms include mebendazole, albendazole, andpyrantel pamoate. An antipruritic agent such ashydroxyzine or diphenhydramine may be givenfor symptomatic relief. No special local hygieneis necessary. The other drugs listed are useful ina variety of other parasitic infections but not forpinworm infections. (Rudolph CD,1105–1106)

117. (C) The initial history in cases of accidental inges-tion should include specific substance ingested,amount and concentration of ingested substance,potential for coingestants, and interventions (eg, induction of emesis) prior to emergencydepartment evaluation. For this patient, it wouldbe prudent to assume she drank the entire bottleof acetaminophen. This would be about 3000 mg(100 mg/cc ¥ 30 cc = 3000 mg). If so, the ingesteddose would be approximately 230 mg/kg, apotentially toxic dose. Administration of acti-vated charcoal with sorbitol to bind acetamino-phen still within the gastrointestinal tract is thebest option of those presented. There is no evi-dence of airway compromise, and acetamino-phen is not associated with rapid loss of airwayreflexes; therefore, intubation is not indicated.Gastric emptying by lavage or induced emesis isnot recommended more than 1–2 hours follow-ing ingestion, unless the ingested substance isknown to delay gastric emptying. Although anacetaminophen level should be obtained at 4 hour ingestion, administration of charcoalshould not delay until the results are available.(Rudolph CD,356–362)

118. (B) The Rumack-Matthew nomogram is used toassess the risk of acetaminophen toxicity after anacute overdose. The initial serum level should bedetermined 4 hours after ingestion. At that time,levels over 150 µg/mLfall into the possible hepatictoxicity range, while levels over 200 µg/mL fallinto the probable hepatic toxicity range.

For patients with levels in the pos-sible or probable toxic range, institution ofN-acetylcysteine is indicated. Prompt initiationmay be lifesaving, and long-term follow-up indi-cates that most patients receiving prompt treat-ment recover completely. (Rudolph CD, 356–362)

119. (E) Acetaminophen is metabolized primarily(94%) to the sulfate or glucoronide form within theliver. About 4% is metabolized via the cytochromeP-450 system and glutathione reductase to themercapturic acid conjugate; the rest is secretedunchanged by the liver and, to a small amount,the kidney. When the load of acetaminophendepletes the glutathione to less than 70% ofnormal, the highly reactive P-450 metabolite gen-erates significant amounts of free radicals, leadingto hepatocyte destruction. NAC serves as aprecursor for glutathione synthesis and preventsthe free radical formation. (Rudolph CD,356–362)

120. (D) The first priority in this patient’s manage-ment is to secure the airway. The deep depres-sion of consciousness, reflected by lack ofresponse to painful stimuli and absence of gagreflex, place the patient at risk for aspiration. Inaddition, ventilation is likely inadequate. Drugscreen and determination of blood glucose willlikely be indicated, but should not delay estab-lishment of a secure airway. Flumazenil may beuseful in benzodiazepine overdose, but is notthe next step in this patient’s management.(Gunn, 19–20, 39–40, 748, 752, 894)

121. (C) The toxidrome of bradycardia, hypopnea,marked lethargy, and miosis is most suggestiveof a narcotic overdose. Clonidine ingestion canhave bradycardia, hypotension, and respiratorydepression but usually not miosis. Labetalolwould cause bradycardia and hypotension butusually not the mental status changes. Diazepamcan cause sleepiness but not usually miosis.(Gunn, 19–20, 39–40, 748, 752, 894)

Page 272: Questions - uhs.edu.kh You suspect Lyme disease, most likely contracted by which of the following? (A) ingestion of unripe fruit (B) ingestion of spoiled fruit (C) drinking of contaminated

Questions: 115–121 273

American Academy of Pediatrics Policy Statement.Practice parameter: the neurodiagnostic evaluation ofa child with a first simple febrile seizure. Pediatrics1996;97(5):769–772.

Lang DM. An overview of EPR3 asthma guidelines:what’s different? Allergy Asthma Proc. 2007 Nov-Dec;28(6):620–627.

National Asthma Education and Prevention Program,National Heart, Lung, and Blood Institute. Expert panelreport 2, guidelines for the diagnosis and managementof asthma. NIH Publication no. 97-4051; July 1997.

Ruddy S, Harris ED, Sledge CB, et al. Kelley’s Textbook ofRheumatology. 6th ed. Philadelphia, PA: WB Saunders;2001.

SELECTED READINGS

Page 273: Questions - uhs.edu.kh You suspect Lyme disease, most likely contracted by which of the following? (A) ingestion of unripe fruit (B) ingestion of spoiled fruit (C) drinking of contaminated

This page intentionally left blank

Page 274: Questions - uhs.edu.kh You suspect Lyme disease, most likely contracted by which of the following? (A) ingestion of unripe fruit (B) ingestion of spoiled fruit (C) drinking of contaminated

CHAPTER 11

Practice TestMary Anne Jackson, MD

Sara S. Viessman, MD

This chapter is a 162-question sample examination that provides a final opportunity to review, evaluate,and improve your clinical knowledge in general pediatrics. You should be able to complete the sampleexamination within 4 hours. Good luck!

275

Page 275: Questions - uhs.edu.kh You suspect Lyme disease, most likely contracted by which of the following? (A) ingestion of unripe fruit (B) ingestion of spoiled fruit (C) drinking of contaminated

276

DIRECTIONS (Questions 1 through 106): For eachof the multiple choice questions in this section selectthe one lettered answer that is the best response ineach case.

1. Among children, short stature and infertilityare most commonly associated with which ofthe following?

(A) Klinefelter syndrome(B) Beckwith-Wiedemann syndrome(C) Turner syndrome(D) Marfan syndrome(E) Pierre Robin sequence

2. Unilateral multicystic dysplastic kidney in aninfant usually presents with which of thefollowing?

(A) an abdominal mass(B) hematuria(C) hypertension(D) nephrotic syndrome(E) oliguria

3. The Fanconi syndrome is characterized bywhich of the following?

(A) azotemia, edema, and hypertension(B) glycosuria, aminoaciduria, and

phosphaturia(C) hematuria, glycosuria, and proteinuria(D) hypoglycemia, glycosuria, hypoglycine-

mia, and glycinuria(E) uremia, phosphaturia, and albuminuria

4. Which of the following sets of values is mostsuggestive of proximal renal tubular acidosis?

Serum SerumUrine bicarbonate chloride (pH) (meq/L) (meq/L)

(A) 8 22 98(B) 8 15 115(C) 7 22 115(D) 6 22 98(E) 6 15 115

5. The presence of bilateral renal masses and a mid-line suprapubic mass in a newborn male infantis most suggestive of which of the following?

(A) bilateral Wilms tumor(B) congenital neuroblastoma(C) congenital rubella infection(D) rhabdomyosarcoma of the bladder(E) congenital urethral or bladder neck

obstruction

6. Wilms tumor (nephroblastoma) is the mostfrequent malignant tumor of the genitourinarytract in childhood. The most common pre-senting sign of this neoplasm is which of thefollowing?

(A) abdominal mass(B) abdominal pain(C) edema(D) hematuria(E) hypertension

7. A newborn with abdominal distension is foundto have a meconium ileus. Which of the fol-lowing conditions is most likely present in thisneonate?

(A) trisomy 13(B) trisomy 21

Questions

Page 276: Questions - uhs.edu.kh You suspect Lyme disease, most likely contracted by which of the following? (A) ingestion of unripe fruit (B) ingestion of spoiled fruit (C) drinking of contaminated

Questions: 1–15 277

(C) Tay-Sachs disease(D) cystic fibrosis(E) maple syrup urine disease

8. A 5-year-old male presents with a 48-hour his-tory of headache, and meningismus. Evaluationof the CSF reveals clear fluid with normal proteinand glucose content. The CSF cell count reveals300 WBC/hpf, 90% lymphocytes. Which of thefollowing is the most likely etiologic agent?

(A) Streptococcus pneumoniae(B) Haemophilus influenzae(C) Neisseria meningitidis(D) adenovirus(E) enterovirus

9. Compared to other methods of dialysis, peri-toneal dialysis offers many advantages for chil-dren and their families including increasedautonomy and flexibility, with resultantenhanced school attendance and peer interac-tions. Peritoneal dialysis is less costly thanhemodialysis and can be used to treat evensmall infants. The major complication of peri-toneal dialysis, either continuous ambulatory(CAPD) or continuous cycling (CCPD), is

(A) a high incidence of disequilibrium syndrome

(B) a high incidence of peritonitis(C) a need for severe dietary restriction(D) frequent electrolyte problems(E) poor growth

10. Which of the following represents the occurrenceof significant bilateral hearing loss in infants inwell newborn nurseries in the United States?

(A) 1/100(B) 1/1000(C) 1/5000(D) 1/10,000(E) 1/100,000

11. The peak age of onset of childhood nephroticsyndrome associated with minimal-changemorphology is which of the following?

(A) under 6 months of age(B) between 12 and 18 months of age(C) between 2 and 5 years of age(D) between 5 and 10 years of age(E) between 10 and 15 years of age

12. The finding of a low serum concentration of C3component of complement in a child with neph-rotic syndrome indicates which of the following?

(A) a high likelihood of spontaneousremission

(B) a high likelihood of good response tosteroid therapy

(C) the presence of focal segmental sclerosis(D) the presence of membranous glomeru-

lonephritis(E) the presence of membranoproliferative

glomerulonephritis

13. A 4-year-old presents for a well-child visit. Theparents state their family has practiced a strictvegan diet since the child was 2 years of age. Ofthe following, which is most likely to be defi-cient now or in the near future?

(A) vitamin C(B) folic acid(C) thiamine(D) vitamin B6

(E) vitamin B12

14. The most common roentgenographic abnormalityin a child with asthma is which of the following?

(A) bronchiectasis(B) generalized hyperinflation(C) lower lobe infiltrates(D) pneumomediastinum(E) right middle lobe atelectasis

15. The pattern of inheritance of achondroplasia iswhich of the following?

(A) autosomal recessive(B) autosomal dominant(C) X-linked recessive(D) X-linked dominant(E) polygenetic

Page 277: Questions - uhs.edu.kh You suspect Lyme disease, most likely contracted by which of the following? (A) ingestion of unripe fruit (B) ingestion of spoiled fruit (C) drinking of contaminated

278 11: Practice Test

16. The bacterial pathogens most commonlyencountered in the lungs of patients with cysticfibrosis are which of the following?

(A) Escherichia coli and alpha streptococcus(B) Escherichia coli and Pseudomonas

aeruginosa(C) Staphylococcus aureus and Proteus species(D) Staphylococcus aureus and P aeruginosa(E) Haemophilus influenzae and Streptococcus

pneumoniae

17. A 16-year-old male presents for a sports physi-cal. Examination reveals hypermobile joints,pes planus, and a high-arched palate. Which ofthe following would be most appropriate atthis time?

(A) chromosomal analysis(B) cardiac ultrasound(C) urine for organic acids(D) cerebrospinal fluid for amino acids(E) urine for calcium to creatinine ratio

18. A 2-month-old infant presents with irritabilityand congestive heart failure. An ECG is inter-preted as characteristic of myocardial infarc-tion. Which of the following is the most likelyexplanation for these findings?

(A) viral myocarditis(B) ventricular septal defect(C) endocardial fibroelastosis(D) anomalous origin of the left coronary

artery(E) atherosclerotic heart disease secondary

to a congenital lipid disorder

19. A 6-month-old infant is found to be lethargicand cyanotic. Blood obtained by venipunctureappears chocolate brown and does not becomebright red when shaken in the presence of air.It is especially important to seek a history ofexposure to which of the following?

(A) automobile exhaust(B) fava beans(C) paint fumes(D) undercooked meat(E) well water

20. Peak height velocity for girls most often occursaround which sexual maturity rating (SMR)

(A) zero(B) one(C) three(D) four(E) five

21. Which of the following is characteristic of anight terror?

(A) most often occur during the first one-third of the night

(B) onset usually is during the elementaryschool years

(C) rarely persists until adolescence(D) occurs during REM sleep(E) the event is vividly recalled by the child

22. The primary lesion in acne is which of thefollowing?

(A) hyperplasia of the sweat gland(B) sterile inflammation of the sweat gland(C) plugging of the sebaceous gland(D) infection of the sebaceous gland(E) increased cornification of the

epidermis

23. A 7-year-old child is brought to the officebecause of chronic nasal obstruction. Examinationreveals bilateral, clear, serous discharge fromthe nose, but is otherwise unremarkable.Which of the following is the most likelydiagnosis?

(A) a defect in the cribriform plate(B) allergic rhinitis(C) choanal atresia(D) cystic fibrosis(E) immunodeficiency

24. Which of the following best describes atopicdermatitis in children?

(A) it tends to spare the face and arms(B) it is frequently associated with uveitis(C) it rarely begins during the first 2 years

of life

Page 278: Questions - uhs.edu.kh You suspect Lyme disease, most likely contracted by which of the following? (A) ingestion of unripe fruit (B) ingestion of spoiled fruit (C) drinking of contaminated

Questions: 16–33 279

(D) it is characterized by pruritus and lichenification

(E) it is associated with elevated serum levelsof IgA and IgM and decreased levels of IgE

25. A 15-year-old boy is bitten on the hand by asnake, which he then kills and brings to theemergency room. The snake is identified as acopperhead measuring 14 in. in length. Themost likely complication to be expected in thischild would be which of the following?

(A) fever(B) local tissue necrosis(C) paralysis(D) renal failure(E) shock

26. The major concern regarding chronic otitismedia with effusion is the development ofwhich of the following?

(A) meningitis(B) mastoiditis(C) permanent nerve deafness(D) perforation of the tympanic membrane(E) impaired speech and language

development

27. Which of the following is the most common indi-cation for surgical repair of pectus excavatum?

(A) thoracic scoliosis(B) cardiac dysfunction(C) pulmonary compromise(D) cosmetic appearance(E) cervical pain

28. Most nasal polyps in children are due to whichof the following?

(A) allergy or immunodeficiency(B) allergy or infection(C) allergy or cancer(D) cystic fibrosis or cancer(E) cystic fibrosis or allergy

29. Which of the following is the most commonmanifestation of alpha1-antitrypsin deficiencyin infancy?

(A) pulmonary cysts(B) myocarditis(C) hepatic cirrhosis(D) pancreatic insufficiency(E) obstructive lung disease

30. The earliest indicators of Cushing syndrome(glucocorticoid excess) in children are whichof the following?

(A) weight gain and growth arrest(B) growth arrest and acne(C) acne and hypertension(D) hypertension and striae(E) acne and striae

31. A12-year-old girl presents with headache, visualchanges, and papilledema. A CT scan reveals amass lesion in the region of the anterior pituitarygland which of the following is the most likelydiagnosis?

(A) chromophobe adenoma(B) craniopharyngioma(C) ganglioneuroma(D) medulloblastoma(E) neuroblastoma

32. An 8-year-old male with sickle cell diseasepresents with 2–3 days of runny nose and mild-to-moderate anterior chest pain. On chest x-raya new infiltrate is noted in the right middlelobe. Which of the following diagnoses, is mostlikely?

(A) acute chest syndrome(B) right middle lobe syndrome(C) aspiration pneumonia(D) foreign body(E) RSV bronchiolitis

33. Which of the following is frequently seen insickle cell patients with splenic sequestration?

(A) hyposplenism(B) pneumonia(C) polycythemia(D) eosinophilia(E) thrombocytopenia

Page 279: Questions - uhs.edu.kh You suspect Lyme disease, most likely contracted by which of the following? (A) ingestion of unripe fruit (B) ingestion of spoiled fruit (C) drinking of contaminated

280 11: Practice Test

34. Which of the following is the most commoncause of deaths in infants under 12 months ofage each year in the United States?

(A) sudden infant death syndrome (SIDS)(B) RSV bronchiolitis(C) child abuse(D) infantile leukemia(E) congenital heart disease

35. Of the following, which is a significant riskfactor for SIDS?

(A) firm mattress(B) breastfeeding(C) early introduction of solid foods(D) pacifier use(E) maternal smoking during pregnancy

36. Which of the following in an 8-year-old child ismost likely to indicate an underlying psycho-logic or behavioral problem?

(A) enuresis(B) encopresis(C) motion illness(D) migraine headache(E) recurrent pharyngitis

37. A 4-year-old girl presents with a 1-year his-tory of monthly episodes of fever and apht-hous stomatitis. A CBC 6 months prior wasreportedly normal, but a CBC now reveals atotal neutrophil count of less than 200/mm3.The remainder of the CBC is normal. Physicalexamination reveals an inflamed pharynxand oral mucosa with several aphthouslesions on the gingival and buccal mucosa.There are bilateral, tender anterior cervicallymph nodes, the largest of which is about 4 cm in diameter. The remainder of the phys-ical examination is normal. The most likelydiagnosis is which of the following?

(A) Chediak-Higashi syndrome(B) cyclic neutropenia(C) HIV infection(D) leukemia(E) Schwachman-Diamond syndrome

38. A 3-year-old male presents with random eyemovements, ataxia, and developmental delay.Of the following, which is most likely thediagnosis?

(A) acute cerebellar ataxia(B) migraine variant(C) neuroblastoma(D) retinoblastoma(E) rhabdomyosarcoma

39. An 11-week-old male presents with a 1-weekhistory of poor feeding and constipation. Onexamination, you note poor head control anda weak cry. Though the infant’s face isexpressionless, he displays a paradoxicalalertness and has normal features. The mostlikely condition of this infant is which of thefollowing?

(A) bacterial meningitis(B) infantile botulism(C) hypothyroidism(D) Hirschsprung disease(E) iron poisoning

40. Chronic upper airway obstruction from enlargedtonsils and adenoids in a child may cause whichof the following?

(A) convulsions(B) cor pulmonale(C) a pneumothorax(D) thymic hyperplasia(E) reactive airway disease

41. Febrile seizures occur most frequently at whatage?

(A) in the first month of life(B) in the first 6 months of life(C) between 6 months and 5 years of age(D) between 5 and 10 years of age(E) around the time of puberty

42. A mother calls to inform you that her previ-ously well 4-year-old child has been com-plaining of headaches for about a month. Forthe past 2 weeks he has been keeping his headin a tilted position, and for the past few days he

Page 280: Questions - uhs.edu.kh You suspect Lyme disease, most likely contracted by which of the following? (A) ingestion of unripe fruit (B) ingestion of spoiled fruit (C) drinking of contaminated

Questions: 34–50 281

has been vomiting in the morning. The mostlikely diagnosis is which of the following?

(A) acute torticollis(B) brain abscess(C) brain tumor(D) degenerative brain disease(E) meningitis

43. Episodes of cerebellar ataxia may be seen inwhich of the following?

(A) cystinuria(B) Gaucher disease(C) Hartnup disease(D) oxalosis(E) tyrosinosis

44. An 8-year-old child develops an intensely pru-ritic rash on the legs only. There are patches oferythematous papules and vesicles and sev-eral streaks of erythematous vesiculation. Thechild is afebrile and otherwise well. The mostlikely diagnosis is

(A) eczema(B) Henoch-Schönlein purpura(C) poison ivy dermatitis(D) scabies(E) varicella

45. A 2-day-old male presents with upper abdom-inal distension and bilious vomiting. He passeda normal meconium stool shortly after birth.Abdominal x-rays suggest duodenal obstruc-tion. An upper gastrointestinal contrast studyreveals a “bird’s beak” appearance of the distalportion of the duodenum. Which of the fol-lowing is most likely?

(A) malrotation with midgut volvulus(B) infantile hypertrophic pyloric

stenosis(C) duodenal atresia(D) intussusception(E) Hirschsprung disease

46. The usual presentation of an annular pancreasin childhood is which of the following?

(A) hypoglycemia(B) hyperglycemic acidosis(C) jaundice(D) vomiting(E) steatorrhea

47. The classic radiologic finding in duodenalatresia is which of the following?

(A) a totally gasless abdomen(B) free air below the diaphragm(C) the double bubble sign(D) the anchor sign(E) the string sign

48. During the first year of life, birth lengthincreases by what percent?

(A) 25%(B) 50%(C) 75%(D) 100%(E) 125%

49. Infants typically double their birth weight bywhat age?

(A) 2 weeks(B) 2 months(C) 4 months(D) 8 months(E) 12 months

50. A female-appearing infant is operated on forbilateral inguinal hernias only to have the sur-geon discover that the masses are undescendedtestes. Chromosomal analysis reveals XY consti-tution. The patient’s adolescent sister has pri-mary amenorrhea. Chromosomal analysis of thesister also reveals XY constitution. The most likelycause of this syndrome in the patient and sister iswhich of the following?

(A) 20, 22-desmolase deficiency complex(B) end-organ insensitivity to androgens(C) congenital adrenal hyperplasia(D) true hermaphrodism(E) Turner syndrome

Page 281: Questions - uhs.edu.kh You suspect Lyme disease, most likely contracted by which of the following? (A) ingestion of unripe fruit (B) ingestion of spoiled fruit (C) drinking of contaminated

282 11: Practice Test

51. Which of the following is the normal, or aver-age, hemoglobin concentration in a 12-month-old infant?

(A) 8 g/dL(B) 10 g/dL(C) 12 g/dL(D) 15 g/dL(E) 17 g/dL

52. Breath-holding spells are best described bywhich of the following?

(A) are most common between 4 and 6 years of age

(B) are a common cause of sudden infant death(C) are a manifestation of infantile colic(D) represent a type of epilepsy(E) may terminate in cyanosis and loss of

consciousness

53. Which of the following is the most commonpresentation of Enterobius vermicularis infection?

(A) appendicitis(B) diarrhea(C) intussusception(D) perianal pruritus(E) vaginitis

54. A 10-day-old infant is evaluated for recurrentblisters and sores, mostly on the extremitiesat areas of friction or trauma. The child hasbeen afebrile and well except for irritability.Examination is unremarkable except for blis-ters, varying in stage from fresh to rupturedand crusted, mostly on the extremities, espe-cially the dorsal surfaces of the hands and feet.Which of the following would be highest onyour differential diagnosis?

(A) bullous impetigo(B) congenital syphilis(C) drug-induced toxic epidermal

necrolysis(D) epidermolysis bullosa(E) staphylococcal scalded skin syndrome

55. Which of the following best describes juvenilegastrointestinal polyps?

(A) they occur most commonly in the ileum(B) they rarely present in the first 5 years of

life(C) they usually present as blood-streaked

stools(D) they often already have malignant ele-

ments when first discovered(E) they have a significant risk of malignant

transformation after puberty

56. A 2-week-old male presents with irritability,decreased feeding, and fever. Cerebrospinalfluid examination reveals cloudy fluid with5500 WBC/mm3 and a low glucose. Gram stainrevealed no bacteria. Which of the following isthe most likely causative agent?

(A) Treponema pallidum(B) cytomegalovirus(C) Mycobacterium tuberculosis(D) group B streptococcus(E) Staphylococcus epidermidis

57. In comparison to cow milk, human milk con-tains more of what substance?

(A) protein(B) sodium(C) casein(D) calcium(E) calories

58. On routine examination, a 2-month-old African-American male infant is noted to have a mod-erate size umbilical hernia. The contents of thehernia are easily reduced, and it is noted thatthe abdominal wall defect easily admits oneexamining finger but not quite two. The remain-der of the examination is normal and the infantis asymptomatic. What is the most appropriatenext step in management?

(A) order thyroid function tests(B) refer the infant to a surgeon(C) instruct the parent in how to tape the

defect(D) obtain an abdominal roentgenogram or

ultrasound(E) advise the parent that the defect will

probably close spontaneously

Page 282: Questions - uhs.edu.kh You suspect Lyme disease, most likely contracted by which of the following? (A) ingestion of unripe fruit (B) ingestion of spoiled fruit (C) drinking of contaminated

Questions: 51–66 283

59. Which of the following best describes chylousascites in infancy?

(A) Congenital nephrosis is the most com-mon cause.

(B) Hypoalbuminemia and lymphopeniaare common.

(C) The condition usually is benign andtransient.

(D) Hypergammaglobulinemia is common.(E) Hepatic involvement is common.

60. A 12-year-old child presents with severeabdominal pain, nausea and vomiting, abdom-inal distension, and epigastric tenderness.Chest roentgenogram reveals a small pleuraleffusion. The child is afebrile but blood countreveals a marked leukocytosis. Which of thefollowing is the most likely cause for this con-dition?

(A) blunt abdominal trauma(B) systemic lupus erythematosis(C) hemolytic uremic syndrome(D) alcohol ingestion(E) Kawasaki disease

61. Which of the following best describes idiopathicscoliosis?

(A) it is more common in boys than in girls(B) it is diagnosed by x-ray(C) it is generally associated with mental

retardation(D) it is most commonly noted during

preadolescence or adolescence(E) it is usually associated with consider-

able pain on motion of the back

62. A 6-month-old male presents with failure tothrive, eczema, and a history of recurrent bac-terial infections. On evaluation, a thrombocytecount of 20,000/mm3 is noted. The peripheralsmear reveals microthrombocytes. Which ofthe following is the most likely condition causingthese signs and symptoms?

(A) Wiskott-Aldrich syndrome(B) 22q11 deletion syndrome(C) celiac disease

(D) idiopathic thrombocytopenic purpura(E) leukemia

63. A 10-year-old male presents with an acuteepisode of nausea, vomiting, and severe testic-ular pain. On examination, scrotal edema anderythema with lack of cremasteric relex arenoted. Color-flow Doppler ultrasonographyconfirms the suspected diagnosis. What is theinitial treatment of choice?

(A) observation for 24 hours(B) immediate surgical exploration(C) chemotherapy(D) intravenous antibiotics(E) intramuscular ceftriaxone

64. You examine a school-age child because ofitchy scalp and note minute white-gray struc-tures firmly attached to the hair shafts. You recommend which of the following?

(A) a selenium-containing shampoo(B) a 1% permethrin cream rinse(C) oral tetracycline(D) oral and topical tetracycline(E) oral griseofulvin

65. A 2-year-old child drinks kerosene that hadbeen left in a glass. After the first swallow, shecries and drops the glass. She is most likely todevelop which of the following?

(A) aplastic anemia(B) chemical pneumonitis(C) coma and/or convulsions(D) hepatitis(E) peripheral neuritis

66. Congenital hypothyroidism should be includedin the differential diagnosis of a newbornwith which of the following?

(A) coma(B) prolonged jaundice(C) pulmonary edema(D) renal failure(E) severe anemia

Page 283: Questions - uhs.edu.kh You suspect Lyme disease, most likely contracted by which of the following? (A) ingestion of unripe fruit (B) ingestion of spoiled fruit (C) drinking of contaminated

284 11: Practice Test

67. A 2-month-old infant has had inspiratory stridorsince the first month of life, but has been otherwisewell. Physical examination is unremarkableexcept for moderate inspiratory stridor and retrac-tions, which are worse when the infant is supineor agitated and better when he is prone and quiet.What is the most likely cause of these findings?

(A) reactive airway disease(B) laryngomalacia(C) viral croup(D) an aspirated foreign body(E) a tracheoesophageal fistula

68. A significant number of children with recur-rent gross or microscopic hematuria, normalrenal ultrasound, and normal tests of renalfunction have which of the following?

(A) hyperkaliuria(B) hypercalciuria(C) hypercalcemia(D) hyperphosphatemia(E) hemorrhagic urethritis

69. A 4-year-old child, previously well, presentswith the rather sudden onset of wheezingwhich does not respond to treatment withaerosolized albuterol. An x-ray examinationof the chest (see Figure 11–1) is obtained.What is the most likely diagnosis?

(A) asthma(B) foreign body(C) infantile lobar emphysema(D) pulmonary hypoplasia(E) right middle lobe syndrome

70. Physical examination of a 6-month-old childwho does not walk reveals the findings shownbelow (Figures 11–2 and 11–3). What is the mostlikely diagnosis?

Figure 11-3(Courtesy of Mark R. Sinclair, MD )

Figure 11-2(Courtesy of Mark R. Sinclair, MD )

Figure 11-1(Courtesy of Doug Rivard, DO )

Page 284: Questions - uhs.edu.kh You suspect Lyme disease, most likely contracted by which of the following? (A) ingestion of unripe fruit (B) ingestion of spoiled fruit (C) drinking of contaminated

Questions: 67–76 285

(A) arthrogryposis congenita(B) congenital hip dislocation(C) equinovarus deformity(D) rickets(E) tibial torsion

71. A 12-year-old girl complains of decreasingvisual acuity and a slight feeling of discomfortin both eyes. Examination by an ophthalmolo-gist reveals anterior uveitis. You suspect thatthe child may have which of the following?

(A) leukemia(B) toxoplasmosis(C) toxocara infection(D) hypoparathyroidism(E) juvenile idiopathic arthritis

72. A 5-year-old female presents with intoeing. Hermother states the child is very limber and hasno complaints of pain. On examination, withthe child in prone position and knees flexed 90 degrees the child can easily place her lateralmalleoli on the table. What is the most likelycondition described here?

(A) Perthes disease(B) acute slipped femoral epiphysis(C) metatarsus adductus(D) tibial torsion(E) femoral anteversion

73. For which of the following would laboratorytesting for group A streptococcus (GAS) be rec-ommended in the United States?

(A) an 18-month-old with pharyngitis(B) a 5-year-old with pharyngitis and anterior

stomatitis with discrete ulcerative lesions(C) a 10-year-old with pharyngitis, conjunc-

tivitis, and cough(D) all household contacts of a child with

documented GAS pharyngitis(E) all household contacts of a child with

documented poststreptococcal glomeru-lonephritis

74. The most consistent finding in lymphocyticthyroiditis (Hashimoto thyroiditis) is which ofthe following?

(A) enlargement of the thyroid gland(B) hyperthyroidism(C) hypothyroidism(D) eosinophilia(E) associated disturbances of the

parathyroids

75. A 6-month-old male presents with diaper rashthat is resistant to therapy. He has intermittentdiarrhea. He is breastfeeding, and rarely seemsinterested in solid foods. Mother is a vegetarian.On examination, he is thin and listless. He hasdry, plaque-like, sharply demarcated lesionsaround his mouth and eyes. His hair is coarseand scanty. A deficiency of which of the fol-lowing could explain this condition?

(A) iron(B) zinc(C) vitamin A(D) vitamin B6

(E) vitamin D

76. A 2-year-old child has had recurrent episodesof fever, cough, and pulmonary infiltrates onchest roentgenograms. Diagnostic evaluationreveals a normal sweat test, barium swallow,and serum immunoglobulins, but a severemicrocytic, hypochromic anemia. The child’sdiet is normal for age. Which of the followingwould be highest on your list of differentialdiagnoses?

(A) cystic fibrosis(B) diffuse pulmonary hemangiomatosis(C) extramedullary pulmonary erythropoiesis(D) gastroesophageal reflux and hemor-

rhagic esophagitis(E) primary pulmonary hemosiderosis

Page 285: Questions - uhs.edu.kh You suspect Lyme disease, most likely contracted by which of the following? (A) ingestion of unripe fruit (B) ingestion of spoiled fruit (C) drinking of contaminated

286 11: Practice Test

77. A 13-year-old female athlete presents withanterior knee pain of 2–3 weeks’ duration. Shehas no history of trauma. On examination, shehas good flexibility of lower extremities. Thereare no effussions. There is no tenderness overthe tibial tubercle, or at the inferior pole of thepatella. She has a positive patella glide test. Ofthe following causes of anterior knee pain,which is most likely in this patient?

(A) Osgood-Schlatter disease(B) chondromalacia patellae(C) Sinding-Larsen-Johansson disease(D) osteochondritis dissecans(E) slipped capital femoral epiphysis

78. On a 6-month well-child visit, you note a healthywell-grown male with a normal physical exam-ination except for a closed anterior fontanelle.Which of the following should you consider?

(A) hypothyroidism(B) trisomy 18(C) hydrocephalus(D) craniosynostosis(E) hypophosphatasia

79. A previously well 9-year-old boy develops anintensely pruritic rash consisting of raised hive-like lesions with pink edges and pale centers.Lesions are well circumscribed and range insize from 2 to 6 cm in diameter. The history isunremarkable and physical examination isunrevealing except for the rash. Careful diag-nostic workup probably will disclose which ofthe following?

(A) a food allergy(B) an occult malignancy(C) a collagen vascular disease(D) C1-esterase inhibitor deficiency(E) no specific cause for the rash

80. Most patients with XYY constitution are whichof the following?

(A) short(B) obese(C) behaviorally normal

(D) impulsive and antisocial(E) severely mentally retarded

81. A petechial rash is noted in a 10-hour-oldnewborn who is otherwise appearing normaland vigorous. There is no hepatosplenomegalyand radii are present. There are no risk factorsfor sepsis. A complete blood count reveals anormal WBC count with differential hemoglobin17 g/dL; and platelet count 11,000/mm3 withlarge platelets present. Which of the followingwould be the most appropriate next step?

(A) maternal platelet count(B) abdominal ultrasound(C) bone marrow aspiration(D) chest x-ray(E) PT and PTT

82. A 2-month-old male presents with a cough thathas persisted for over 2 weeks. Mother hasnoted he coughs in spells and at the end ofthese spells, he vomits. She feels he has lostweight. On examination, you note a thin maleinfant who otherwise appears normal. Completeblood count reveals 30,000 white blood cellswith 95% lymphocytes. Which of the followingis the most likely diagnosis?

(A) respiratory syncitial virus bronchiolitis(B) pneumococcal pneumonia(C) pertussis(D) laryngomalacia(E) vascular ring

83. Pseudohypoparathyroidism is associatedwith which of the following?

(A) short stature(B) renal failure(C) long, spindly fingers(D) generalized increased mineralization of

bone(E) decreased serum levels of parathyroid

hormone

84. The most consistent abnormality in vonWillebrand disease is which of the following?

Page 286: Questions - uhs.edu.kh You suspect Lyme disease, most likely contracted by which of the following? (A) ingestion of unripe fruit (B) ingestion of spoiled fruit (C) drinking of contaminated

Questions: 77–92 287

(A) decreased platelet count(B) prolonged bleeding time(C) prolonged prothrombin time(D) prolonged partial thromboplastin

time(E) decreased plasma level of factor XII

85. The most common cause of congenitalhypothyroidism is which of the following?

(A) dysgenesis of the thyroid gland(B) a defect in thyroid synthesis(C) Hashimoto’s thyroiditis(D) maternal ingestion of iodides(E) maternal iodine deficiency

86. A 6-year-old Hispanic male presents with afirst-time generalized seizure lasting less than5 minutes. There is no history of trauma or ill-ness. The family recently immigrated to theUnited States from Mexico. The child is afebrile,and his examination is completely normal. Ofthe following, which would be the most likelycause for the seizure?

(A) herpes encephalitis(B) cysticercosis(C) lead poisoning(D) vitamin B12 deficiency(E) hypocalcemia

87. A 17-year-old boy with cystic fibrosis complainsof a 5–10 lb weight loss over the past month,unaccompanied by changes in his pulmonaryor gastrointestinal symptoms. You suspect thathe has developed which of the following?

(A) a bronchogenic carcinoma(B) an eating disorder(C) biliary cirrhosis(D) cor pulmonale(E) diabetes mellitus

88. The diagnosis of cystic fibrosis is usually con-firmed by the finding which of the following?

(A) elevated sweat chloride(B) decreased sweat chloride(C) elevated serum chloride

(D) decreased serum chloride(E) elevated sweat and serum chloride

89. A 2-year-old male presents with upper respi-ratory symptoms for 3 days and 1 day of fever,irritability, and right ear pain. Examinationreveals a temperature of 39°C, clear rhinorrhea,a bulging immobile right tympanic membranewith absent light reflex. What is the mostappropriate initial therapy?

(A) antihistamines and ibuprofen(B) antihistamines and acetaminophen(C) penicillin(D) amoxicillin(E) erythromycin

90. Which of the following best describes aninguinal hernia in a 2-month-old girl?

(A) it may contain an ovary(B) it is usually a direct hernia(C) it does not require surgical repair(D) it is a sign of pseudohermaphroditism(E) it is generally associated with an imper-

forate anus

91. Which of the following would be expected in a6-month-old child with a large ventricularseptal defect?

(A) cyanosis(B) an enlarged heart(C) a continuous cardiac murmur(D) decreased pulmonary vasculature on

roentgenogram(E) evidence of predominantly right ventric-

ular hypertrophy on ECG

92. Which of the following is most commonpresentation of hypoparathyroidism beyondthe neonatal period?

(A) syncope secondary to prolonged QTintervals

(B) tingling of extremities(C) seizure(D) bronchospasm(E) laryngospasm

Page 287: Questions - uhs.edu.kh You suspect Lyme disease, most likely contracted by which of the following? (A) ingestion of unripe fruit (B) ingestion of spoiled fruit (C) drinking of contaminated

288 11: Practice Test

93. What is the most common cause of schoolabsenteeism among adolescent females?

(A) depression(B) asthma(C) headache(D) dysmenorrhea(E) drug overdose

94. An irritable 12-month-old male has a 1-weekhistory of high fevers and macular truncal rash.Examination reveals bulbar conjuctivitis, brightred cracked lips, and cervical adenopathy.What is the most appropriate next step?

(A) initiate airborne barrier precautions(B) intravenous antibiotics(C) intravenous antibiotics for the infant

and oral antibiotics for all householdmembers

(D) intravenous gammaglobulin(E) intravenous corticosteroids

95. An 8-month-old female presents with failure tothrive, constipation, fevers, and polydipsia. Onevaluation, you find hypokalemia and hyper-chloremic metabolic acidosis and suspectFanconi syndrome. Which of the followingwould be the most likely inherited cause?

(A) cystinosis(B) cystic fibrosis(C) glycogen storage disease(D) Tay-Sachs disease(E) tyrosinemia

96. A 13-year-old male presents with a 3 cm annu-lar scaling lesion on his right posterior trunkunresponsive to 2 weeks of a topical antifungal.On follow-up visit he presents with dozens ofelliptical scaling lesions on his trunk. He has nopalmoplantar or mucous membrane lesions andno adenopathy. What is the most likely diagnosis?

(A) pityriasis rosea(B) secondary syphilis(C) lichen planus(D) tinea versicolor(E) Munchausen syndrome by proxy

97. An 18-month-old female presents for a well-child visit. On examination, there is an asym-metric red reflex, with the left appearing normaland the right appearing cream colored. Of thefollowing, which is the most likely diagnosis?

(A) rhabdomyosarcoma(B) retinoblastoma(C) neuroblastoma(D) leukemia(E) lymphoma

98. Which of the following represents the ethnicgroup with the highest neonatal and infantmortality rates in the United States?

(A) American Indian(B) Asian and Pacific Island(C) African-American(D) Hispanic(E) Caucasian

99. Which of the following best describes patientswith constitutional growth delay?

(A) they will begin puberty at the same timeas their peers

(B) they have a bone age equal to theirchronological age

(C) they are usually obese(D) they will achieve normal adult height(E) they demonstrate height that is inappro-

priate for bone age

100. A 4-year-old male presents with a 1-day his-tory of abdominal pain and vomiting. He isafebrile and has no diarrhea. He complains ofknee pain bilaterally, and there is some ten-derness of the knee joints but no effusions.Within 24 hours he develops a rash on his legsand buttocks which is petechial and purpuric,and his platelet count is normal. What is themost likely diagnosis?

(A) hemolytic uremic syndrome(B) Henoch-Schönlein purpura(C) acute glomerulonephritis(D) Kawasaki disease(E) systemic lupus erythematosis

Page 288: Questions - uhs.edu.kh You suspect Lyme disease, most likely contracted by which of the following? (A) ingestion of unripe fruit (B) ingestion of spoiled fruit (C) drinking of contaminated

Questions: 93–107 289

101. A 4-year-old female presents with a limp. Theweek prior she had an upper respiratory infec-tion. There is no history of trauma. She has no his-tory of fever. Examination reveals limitation ofmotion of the right hip joint, especially with inter-nal rotation. X-ray reveals some swelling in softtissues surrounding the hip joint. A completeblood count and sedimentation rate are normal.What would be the next most appropriate step?

(A) aspirate the hip joint(B) begin nonsteroidal anti-inflammatory

agents(C) CT scan of the hip(D) MRI of the hip(E) begin intravenous antibiotics

102. A newborn is noted to have a large head andshort limbs. On further examination, shortbroad fingers, a small face, and low-normallength are noted. The trunk appears long andnarrow. What should be your first step to con-firm the diagnosis?

(A) order an ophthalmologic examination(B) obtain skeletal radiographs(C) order chromosome analysis(D) examine the parents(E) perform the routine newborn screen

103. Infants fed exclusively goat milk are susceptible towhich of the following?

(A) vitamin A deficiency(B) vitamin B6 deficiency(C) vitamin E deficiency(D) folate deficiency(E) thiamine deficiency

104. A 5-month-old female presents in the winterwith a 2-day history of vomiting, diarrhea, andfever. The diarrhea is without blood or mucus.On examination, she is moderately dehydratedand has a serum sodium of 152 meq/L. Whatis the most likely etiologic agent?

(A) Giardia lamblia(B) Salmonella(C) influenza

(D) enterovirus(E) rotavirus

105. Among adolescents of 15–19 years of age, motorvehicle accidents cause the greatest number ofdeaths each year. Which of the following is thesecond leading cause of death in this age group?

(A) homicide and legal intervention(B) suicide(C) heart disease(D) cancer(E) cystic fibrosis

106. The most common congenital infection isinfection with cytomegalovirus, affecting3/1000–4/1000 livebirths. Of those babiesinfected, approximately what percent arenormal at birth and develop normally?

(A) 5%(B) 15%(C) 25%(D) 75%(E) 90%

DIRECTIONS (Questions 107 through 128): Thefollowing group of questions is preceded by a listof lettered answer options. For each question,match the one lettered option that is most closelyassociated with the question. Each lettered optionmay be selected once, multiple times, or not at all.

Questions 107 through 110

On the second day of treatment for pneumococcalmeningitis, a 10-month-old child who had beenalert is noted to be lethargic. Serum electrolytesreveal: Na 120 meq/L; Cl 82 meq/L; K 3.1 meq/L;BUN 2 mg/dL.

107. What is the most likely cause of the lethargyand electrolyte disturbance in this patient?

(A) acute hepatic failure(B) acute renal failure(C) congestive heart failure(D) inappropriate secretion of ADH(E) subdural effusions

Page 289: Questions - uhs.edu.kh You suspect Lyme disease, most likely contracted by which of the following? (A) ingestion of unripe fruit (B) ingestion of spoiled fruit (C) drinking of contaminated

290 11: Practice Test

108. What is the most appropriate test to confirmthis diagnosis?

(A) chest roentgenogram(B) measurement of serum and urine

osmolality(C) determination of serum creatinine(D) measurement of hepatic enzymes(E) CT scan of the head

109. One would also expect this patient to exhibitwhich of the following?

(A) polyuria(B) hypotension(C) metabolic alkalosis(D) high urine specific gravity(E) high serum alkaline phosphatase

110. The most appropriate management of thisproblem would be which of the following?

(A) administration of hypertonic (3%) saline

(B) restriction of fluid intake to 70% ofmaintenance requirements

(C) restriction of sodium intake to about10% of normal

(D) administration of a diuretic such asfurosemide

(E) discontinue intravenous fluids andbegin oral fluids as tolerated

Questions 111 through 113

A 12-year-old child is admitted because of the suddenonset of coma. The child had been well until about6 hours prior to admission, when he began to com-plain of a headache. The headache became moresevere and the child lapsed into coma. Physical exam-ination reveals a temperature of 38.2°C. The child isflaccid and comatose. The remainder of the physicalexamination is unremarkable. A lumbar puncturereveals grossly bloody spinal fluid. After centrifuga-tion, the fluid appears xanthochromic. There are 3000RBCs and 7 WBCs/mm3. The protein concentration is400 mg/dL and the glucose is 62 mg/dL.

111. Which of the following is the most likely etiol-ogy of the coma?

(A) intraventricular hemorrhage(B) subarachnoid hemorrhage(C) aqueductal stenosis(D) viral encephalitis(E) subdural effusion

112. Which of the following underlying structuralabnormalities would most likely have led tothe above event or condition?

(A) absence of the corpus callosum(B) porencephalic cyst(C) cerebral arteriovenous malformation(D) cerebral aneurysm(E) Arnold-Chiari malformation

113. What would the most appropriate next step atthis time would be to

(A) obtain a CT of the head(B) repeat the lumbar puncture(C) administer fresh frozen plasma(D) perform an exchange transfusion(E) initiate a transfusion of packed red

blood cells

Questions 114 through 116

A 21-day-old male infant is admitted because ofvomiting for 12 days. Birth weight was 2925 g.The child had been normal at birth and did wellfor the first 9 days of life. He was initially begunon breastfeeding only, but on the eighth day oflife, supplemental feeding with a commercially pre-pared cow milk formula was added. Vomiting beganon the 10th day of life and persisted despite discon-tinuation of the prepared formula. On the 21st dayof life, the child was hospitalized. Diarrhea hadnever been present. Several days prior to admission,the stools had become hard and infrequent. Onadmission, the anterior fontanel is sunken, themucous membranes are dry, and skin turgor is poor.The diaper is dry and the mother cannot recall whenthe child last urinated. The child appears poorlynourished. Wt 2850 g, HR 152 beats per minute,RR 12 times per minute, T 37.5°C. Pulses and color aregood. No abnormalities are palpated on examination.

114. Which of the following is the best diagnostictest for this condition?

Page 290: Questions - uhs.edu.kh You suspect Lyme disease, most likely contracted by which of the following? (A) ingestion of unripe fruit (B) ingestion of spoiled fruit (C) drinking of contaminated

Questions: 108–121 291

(A) an unprepped barium enema(B) a sweat test(C) an abdominal ultrasound(D) chromosomal analysis(E) stool for Giardia antigen

115. One would expect the initial laboratory data toreveal which of the following?

(A) mixed metabolic and respiratory acidosis(B) mixed metabolic and respiratory alkalosis(C) normal acid-base status(D) primary metabolic acidosis(E) primary metabolic alkalosis

116. Which of the following intravenous solutionswould be most appropriate as the initialhydrating fluid?

(A) 5% dextrose in water(B) normal saline(C) 140 meq/L of Na, 120 meq/L of Cl,

20 meq/L of bicarbonate(D) 5% glucose, 40 meq/L of Na, 40 meq/L

of K, 80 meq/L of Cl(E) 3% dextrose plus 140 meq/L of Na,

115 meq/L of Cl, 20 meq/L of bicarbon-ate, and 5 meq/L of K

Questions 117 through 119

A 15-year-old male presents with a 2-day history offever, chills, and cough. He complains of achingmuscles. Today he noticed his urine was red.Examination revealed a tired-appearing adolescentwith fever, pharyngitis, nasal congestion, and ten-der calf muscles.

117. Urine analysis reveals a positive test for hemo-globin with no red blood cells seen on micro-scopic examination. Which of the following ismost likely to reveal the source of the redurine?

(A) detailed dietary history(B) renal ultrasound(C) intravenous pyelogram(D) immunoglobulin levels(E) serum creatine kinase

118. You obtain the above study and it is markedlyabnormal. What is the most important nextstep?

(A) perform a lumbar puncture(B) initiate intravenous antibiotics(C) obtain a chest x-ray(D) order serum electrolytes, BUN, and

creatinine(E) order liver function tests

119. Of the following infectious agents, which ismost likely to cause this condition?

(A) influenza(B) group A streptococcus(C) group B streptococcus(D) hepatitis A(E) Escherichia coli

Questions 120 through 122

A 2-year-old child is hospitalized because of fever,abdominal pain, and hepatomegaly. The child livesin a poor, crowded home on a small farm. No oneelse at home is ill. Physical examination reveals athin child with marked hepatomegaly. The spleen isnot palpable and all lymph nodes are within normallimits. The remainder of the examination is withinnormal limits. Chest x-ray reveals bilateral scattereddensities. White blood cell count is 14,000/mm3

with 36% eosinophils.

120. Which of the following would be most likely toestablish the diagnosis in this patient?

(A) bone marrow aspiration(B) stool examination(C) liver biopsy(D) serum enzyme immunoassay(E) duodenal aspiration

121. How did this child most likely acquire thedisease?

(A) eating poorly cooked pork(B) eating dirt(C) kissing a dog(D) contact with a sick bird(E) contact with a sick rodent

Page 291: Questions - uhs.edu.kh You suspect Lyme disease, most likely contracted by which of the following? (A) ingestion of unripe fruit (B) ingestion of spoiled fruit (C) drinking of contaminated

292 11: Practice Test

122. This condition is which of the following?

(A) more prevalent in the United States thanin developing countries

(B) more common in rural areas(C) nearly always symptomatic(D) most frequently present in school-aged

children(E) transmitted only by mature dogs

Questions 123 through 125

A 6-month-old girl is admitted because of failure tothrive and persistent pneumonia. The child was well atbirth but severe, recurrent diarrhea began at about6 weeks of age. At 8 weeks the child developed pneu-monia which responded only poorly to antibiotic ther-apy and since hospitalization, she continues to requireoxygen by nasal cannula. Physical examination revealsa small, poorly nourished child with moderate respira-tory distress. There are bilateral rales, oral thrush, anda monilial-appearing rash in the diaper area. Theremainder of the examination is within normal limits.The total white blood cell count is 5200/mm3 with 87%polymorphonuclear cells, 12% lymphocytes, and 1%eosinophils; hemoglobin and platelets are normal.Serum immunoglobulin levels are: IgA not detectable;IgG 280 mg/dL; IgM less than 5 mg/dL.

123. Which of the following diagnoses best explainsthe clinical picture?

(A) chronic granulomatous disease(B) Wiskott-Aldrich syndrome(C) hereditary agammaglobulinemia(D) severe combined immunodeficiency(E) transient hypogammaglobulinemia of

infancy

124. On further evaluation one would also expect tofind which of the following?

(A) increased numbers of plasma cells onbone marrow examination

(B) enlarged superficial lymph nodes(C) absent thymic shadow on chest

roentgenogram(D) positive intradermal reaction to Candida

antigen(E) abnormal oxidative burst testing

125. Chest roentgenogram reveals bilateral patchy con-solidations and diffuse granular densities with analmost ground-glass appearance to the lungs.Which of the following is the most likely cause ofthese x-ray findings?

(A) pneumococcal pneumonia(B) Pneumocystis jiroveci pneumonia(C) pulmonary hemorrhage(D) pulmonary lymphangiectasia(E) miliary tuberculosis

Questions 126 through 128

A 4-month-old boy had been well until 4 weeksprior to admission, when vomiting and poorappetite were noted. Psychomotor developmenthad been normal. The child was being fed wholecow milk and strained foods. Stools were normal.On the morning of admission, the child had a gen-eralized convulsion and was brought to the emer-gency room where the seizure was controlled withintravenous medication. A second seizure occurredabout 1 hour later and again responded to intra-venous medication. Physical examination revealeda pale, listless infant, poorly nourished, but in noacute distress. The height was at the 25th percentile,the weight at the 3rd percentile, and the head cir-cumference over the 97th percentile on a standardgrowth curve; T 38°C, RR 16 breaths per minute, HR110 beats per minute, BP 76/50 mm Hg. The ante-rior fontanel is full, but not bulging. There are nofocal neurologic signs. The remainder of the exami-nation is within normal limits.

126. Which of the following diagnoses is most likely?

(A) tuberculous meningitis(B) mastoiditis(C) subdural hematoma(D) congenital toxoplasmosis(E) pseudotumor cerebri

127. A funduscopic examination performed afterone pupil is dilated with atropine reveals dif-fuse retinal hemorrhages. The most likely diag-nosis now is which of the following?

(A) tuberculous meningitis(B) mastoiditis

Page 292: Questions - uhs.edu.kh You suspect Lyme disease, most likely contracted by which of the following? (A) ingestion of unripe fruit (B) ingestion of spoiled fruit (C) drinking of contaminated

Questions: 122–140 293

(C) subdural hematoma(D) congenital toxoplasmosis(E) pseudotumor cerebri

128. Of the following, what is the next most impor-tant step in diagnosis and management?

(A) lumbar puncture(B) electroencephalography(C) CT scan of the head(D) bone marrow examination(E) conjunctival biopsy

DIRECTIONS (Questions 129 through 162): Thefollowing group of questions is preceded by a listof lettered answer options. For each question,match the one lettered option that is most closelyassociated with the question. Each lettered optionmay be selected once, multiple times, or not at all.

Questions 129 through 137

(A) acetylcysteine(B) atropine(C) ammonium chloride(D) calcium chloride(E) deferoxamine(F) digoxin(G) EDTA(H) ethanol(I) fresh frozen plasma(J) furosemide

(K) glucagon(L) methionine

(M) methyldopa(N) methylene blue(O) naloxone(P) nitroprusside(Q) oxygen(R) phenobarbital(S) phenylephrine(T) physostigmine(U) potassium chloride(V) potassium citrate

(W) propranolol(X) sodium bicarbonate

(Y) sodium chloride(Z) sodium nitrite

For each drug or poison, select the most specific andmost important antidote.

129. Lead

130. Iron

131. Acetaminophen

132. Organophosphate

133. Jimsonweed

134. Ethylene glycol

135. Morphine

136. Cyclic antidepressants

137. Carbon monoxide

Questions 138 through 144

(A) 2 months(B) 4 months(C) 6 months(D) 9 months(E) 12 months(F) 18 months(G) 2 years(H) 3–4 years(I) 5 years

For each description of developmental accomplish-ments, select the earliest, most appropriate age.

138. Walks if led and is able to take a few stepsunsupported, demonstrates a good pincer grip,drinks from a cup

139. Copies a circle, hops on one foot, recognizescolors

140. Smiles responsively, follows a moving object orface with turning of the head

Page 293: Questions - uhs.edu.kh You suspect Lyme disease, most likely contracted by which of the following? (A) ingestion of unripe fruit (B) ingestion of spoiled fruit (C) drinking of contaminated

294 11: Practice Test

141. Kicks a ball, builds a tower of six 1-in cubes,combines two different words

142. Sits without support, pulls to a stand, babblesloudly and happily, grabs a block with theentire hand

143. Startles at sudden noise

144. Draws a triangle, uses past and future tense inlanguage

Questions 145 through 149

(A) atrioventricular septal defect(B) mitral valve prolapse(C) supravalvular aortic stenosis(D) coarctation of the aorta(E) pulmonary stenosis

Choose the cardiovascular anomaly most closelyassociated with the condition below

145. Down syndrome

146. Williams syndrome

147. Marfan syndrome

148. Turner syndrome

149. Noonan syndrome

Questions 150 through 154

(A) tongue fasciculations(B) Gower sign(C) heliotrope sign(D) nonthrombocytopenic purpura(E) thrombocytopenia

For each of the conditions below, choose the mostclosely associated finding from the list above.

150. Dermatomyositis

151. Werdnig-Hoffmann disease

152. Hemolytic uremic syndrome

153. Muscular dystrophy

154. Henoch-Schönlein purpura

Questions 155 through 160

(A) adenovirus(B) Borrelia burgdorferi(C) Ehrlichia chaffeensis(D) Eikenella corrodens(E) Escherichia coli(F) group A coxsackievirus(G) human herpesvirus 6(H) Pasteurella multocida(I) parainfluenza virus(J) Shigella

(K) Spirillum minus

For each of the following clinical scenarios, choosethe most likely etiologic agent from the list above.

155. A 3-year-old presents 2 days after a cat bite toher face. She is febrile but not toxic and hasthree tubular shaped areas of indurationapproximately 1 ¥ 3 cm each with surroundingwarmth and erythema. The infected areas arevery painful to touch.

156. A 4-year-old child presents in Missouri in latesummer with a 3-day history of progressive vom-iting and abdominal pain, fever, and petechialrash. He is obtunded and has multiple insectbites. He has leukopenia, elevated liver enzymes,and evidence of DIC on laboratory evaluation.

157. A6-year-old presents with “pink eye,” sore throat,and fever in the summer. His mother tells youthat three or four of his friends have similar symp-toms. Examination reveals pharyngitis with largetonsils and bilaterally erythematous conjunctiva.

158. A 6-month-old female presents with a 3-dayhistory of fevers 39.5°C–40.0°C. On the fourthday, she developed an erythematous macu-lopapular rash and the fever resolved.

159. An 8-month-old with severe inspiratory stri-dor, fever, and a barking cough is admitted to

Page 294: Questions - uhs.edu.kh You suspect Lyme disease, most likely contracted by which of the following? (A) ingestion of unripe fruit (B) ingestion of spoiled fruit (C) drinking of contaminated

Questions: 141–162 295

the hospital for treatment with aerosolizedracemic epinephrine and corticosteroids.

160. A 4-week-old presents with fever. She has ahistory of being well until 1 week prior whenshe was found screaming in her bed. Hermother noted she had a bite wound on herneck and suspected a rat to be the perpetrator.She watched the infant, who seemed to bedoing well, until the past 24 hour when shedeveloped fever. On examination, the infant isirritable and has mild erythema surroundingthe bite wound.

Questions 161 through 162

(A) tuberous sclerosis(B) neurofibromatosis type 1(C) Sturge-Weber disease(D) von Hippel-Lindau disease(E) Albright syndrome

161. Facial port wine stain and intracranialangiomatosis

162. Axillary freckling, long arm of chromosome 17,autosomal dominant with 50% of cases associ-ated with new mutation

Page 295: Questions - uhs.edu.kh You suspect Lyme disease, most likely contracted by which of the following? (A) ingestion of unripe fruit (B) ingestion of spoiled fruit (C) drinking of contaminated

296

1. (C) Both Turner syndrome and Pierre Robinsequence are associated with short stature; how-ever, Turner syndrome is also associated withgonadal dysgenesis and infertility. Children withKlinefelter syndrome and Marfan syndrometypically have long extremities, and childrenwith Beckwith-Wiedemann syndrome typicallyhave gigantism. (Rudolph CD,745, 762, 2091)

2. (A) Multicystic dysplastic kidney is one type ofrenal dysplasia, a group of disorders charac-terized by developmental abnormalities ofkidney structure and differentiation. The entityis not related to polycystic kidney disease.Bilateral multicystic kidneys are rapidly fatal inthe newborn period. The unilateral multicystickidney usually presents in infancy as anabdominal mass. Hypertension has been noted,but is rare. In unilateral cases, renal function ispreserved by the opposite normal kidney.(Rudolph CD, 1704)

3. (B) The term Fanconi syndrome currently is usedto indicate a complex dysfunction of the proxi-mal tubules, characterized by glycosuria, aminoaciduria, and phosphaturia. Blood glucose aswell as urea and creatinine levels usually arenormal. It may be seen as an isolated finding(primary Fanconi syndrome) or secondary todisorders such as cystinosis or tyrosinemia.[Note: Fanconi syndrome is not related toFanconi anemia, an autosomal recessive disordercharacterized by pancytopenia and hypoplasticthumb and radius.] (Rudolph CD,1709–1710)

4. (E) The defect in proximal renal tubular acidosisis renal loss of bicarbonate due to an abnormally

low threshold. Serum chloride is elevated andserum bicarbonate is low. Therefore, choices(A), (C), and (D) can be eliminated. A distinc-tive feature of these patients is their unim-paired ability to excrete adequate amounts oftitratable acid and thereby lower urinary pH.The urine pH usually is acidic in these patientswhen the serum bicarbonate level is low.However, if they are given an infusion of bicar-bonate to raise the serum level, urine pH will bealkaline during the infusion. (Rudolph CD,1710)

5. (E) The presence of bilateral renal masses anda midline suprapubic mass (the distended blad-der) in a newborn infant is congenital urethralor bladder neck obstruction until proven oth-erwise. Posterior urethral valves are an espe-cially important cause of such obstruction inthe male infant. Early diagnosis is critical, as therenal parenchyma eventually will be destroyedby the resultant hydronephrosis. (Rudolph CD,1641)

6. (A) An abdominal or flank mass is the mostcommon presenting sign in children withWilms tumor. Often, the mass is discovered onroutine examination or during examination fora minor illness. This underscores the impor-tance of a thorough examination at everyphysician visit. Occasionally, the parents willbring the child to the physician because theynote an enlarging abdomen or feel the massthemselves. (McMillan, 1775–1777)

7. (D) Meconium ileus typically presents withinthe first 24–48 hours of life, and almost alwaysoccurs in neonates with cystic fibrosis. Of all

Answers and Explanations

Page 296: Questions - uhs.edu.kh You suspect Lyme disease, most likely contracted by which of the following? (A) ingestion of unripe fruit (B) ingestion of spoiled fruit (C) drinking of contaminated

Answers: 1–14 297

children with cystic fibrosis, 10%–20% have ahistory of meconium ileus as a neonate. Theusual location of the obstruction is the distalileum, resulting in a colon of small caliberreferred to as a microcolon. Older patients withcystic fibrosis often develop obstruction of thedistal ileum (typically in the ileocecal region)secondary to inspissated stool. This obstruc-tion, termed meconium ileus equivalent ordistal intestinal pseudo-obstruction syndrome,can be seen in cystic fibrosis patients of any agebeyond the newborn period. (Rudolph CD,1407)

8. (E) Clearly this child’s presentation suggestsmeningitis, a diagnosis which is confirmed bythe CSF results. The relatively low number oftotal white blood cells, the predominance oflymphocytes, and the normal glucose and pro-tein values are all suggestive of a viral illness.Therefore, the most likely etiologic agent ismeningitis caused by enterovirus. (Rudolph CD,1021; Hay, 1113)

9. (B) In most regards, chronic ambulatory peri-toneal dialysis and chronic cycling peritonealdialysis compare favorably to other methods ofchronic dialysis. However, there is an increasedrisk of peritonitis. Usually diagnosed by a care-taker who notices cloudy effluent dialysate, theperitonitis most often is due to coagulase-negative Staphylococcus and is treated withintraperitoneal antibiotics, often at home.Frequent episodes of peritonitis may cause thenephrologist or patient to seek another formof dialysis. (McMillan, 1846–1847)

10. (B) In the past, evaluation of hearing was rec-ommended for infants who met high-risk cri-teria such as birth weight less than 1500 g,family history of sensorineural hearing loss,and presence of congenital infections or cran-iofacial abnormalities. However, 50% of chil-dren with sensorineural deafness (1/1,000infants in well newborn nurseries) did not meetthese criteria for screening at birth. Many stateshave passed legislation which mandates uni-versal newborn hearing screening programs.The American Academy of Pediatrics and theJoint Commissions on Infant Hearing recom-mend that the diagnosis of hearing impairment

be made before 3 months of age and theinfant be receiving intervention services before6 months of age. (Hay, 474–475; Rudolph CD,487–488)

11. (C) Most cases of minimal-change nephroticsyndrome are diagnosed in children 2–5 yearsof age with a peak age of onset between 2 and3 years of age. Older children presenting withnephrotic syndrome are more likely to have anunderlying chronic nephritis (membranopro-liferative glomerulonephritis, lupus nephritis,etc.) rather than minimal-change nephrotic syn-drome. (Rudolph CD,1678, 1691)

12. (E) A low serum concentration of the C3 com-ponent of complement in a child withnephrotic syndrome almost always indicatesthe presence of membranoproliferative dis-ease (assuming that the child does not havesystemic lupus, endocarditis, or a shuntinfection). Membranoproliferative glomeru-lonephritis is very unlikely either to respondto corticosteroids or to remit spontaneously.(Rudolph CD,1681, 1694–1698)

13. (E) Strict vegans eat exclusively foods of plantorigin. Through education, strict vegans canhave a diet balanced for most requirements.However, vitamin B12 is found only in productsof animal origin. Therefore, strict vegans areat high risk for vitamin B12 deficiencies. Breastfedinfants of strict vegan mothers are at markedrisk of vitamin B12 deficiency. In children withvitamin B12 deficiency, irreversible neurologicdamage can occur before the presence ofanemia. (Rudolph CD,1338, 1530)

14. (B) Generalized or diffuse hyperinflation of thelungs, manifested by an increased anteropos-terior diameter of the thorax and flatteneddiaphragms, is the earliest and most frequentroentgenographic abnormality in children withasthma. Infiltrates, atelectasis, and pneumo-mediastinum are seen occasionally during anacute attack. Bronchiectasis is a rare complica-tion and would suggest the possibility of anunderlying disorder such as cystic fibrosis,immunodeficiency, foreign body, or recurrentaspiration. (Gershel, 309:336, 1983)

Page 297: Questions - uhs.edu.kh You suspect Lyme disease, most likely contracted by which of the following? (A) ingestion of unripe fruit (B) ingestion of spoiled fruit (C) drinking of contaminated

298 11: Practice Test

15. (B) Achondroplasia, the most common form ofshort-limbed dwarfism, is an autosomal dom-inant disorder. Most cases, however, are theresult of a sporadic mutation of FGFR3. Mostpatients have an identical missense mutation ofcodon 380 of FGFR3 causing an arginineresidue to be replaced by glycine. The incidenceof achondroplasia is approximately 1 in 26,000live births. (Hay, 791; Rudolph CD,759)

16. (D) Chronic airway colonization and infectionwith S aureus and P aeruginosa is characteristicof patients with cystic fibrosis. Antibiotic ther-apy occasionally eliminates S aureus from thebronchial tree, but almost never eradicates P aeruginosa despite in vitro susceptibility to avariety of antibiotics. (Rudolph CD,1975)

17. (B) Marfan syndrome is a disorder of connec-tive tissue affecting many organs. Affecting1/5000 individuals without ethnic or genderpredilection, this syndrome carries an increasedrisk of sudden death due to cardiovascularcomplications, primarily related to progressivedilation of the aortic root and ascending aorta.Ultrasound is a useful tool in diagnosing andfollowing this potentially fatal manifestationof Marfan syndrome. (Rudolph CD,762–763, 1897)

18. (D) In the congenital abnormality of anom-alous origin of the left coronary artery (LCA),the aberrant LCA arises from the pulmonaryartery rather than from the aorta. This results inperfusion of the left ventricle by poorly oxy-genated blood under low pressure. Myocardialischemia can be severe. The presenting signsare irritability and congestive heart failure. Theage at presentation is related to the degree ofcollateral circulation between the LCA andRCA. (McMillan, 1612–1613)

19. (E) The findings of cyanosis and blood which ischocolate brown and does not become red uponexposure to air are indicative of methemoglo-binemia. One cause of methemoglobinemia isexposure to well water which has been con-taminated with nitrites. In contrast, carbonmonoxide poisoning (automobile exhaust)results in carboxyhemoglobin, which bindsoxygen so tightly that it cannot be released to

the tissues. In this situation the blood wouldbe bright red, rather than dark brown. Favabeans are a cause of hemolysis in patients withglucose-6-phosphate dehydrogenase (G-6-PD)deficiency, but are not a cause of methemoglo-binemia. (Rudolph CD,371–372, 1544–1545)

20. (C) Peak height velocity occurs typically duringSMR 2–3 for girls, and during SMR 4 for boys.(Rudolph CD,223–224)

21. (A) Night terrors are partial arousals that occurduring stage 4 non-REM sleep to near arousaltransitions, which typically first present in thepreschool-aged child. The child appears awake,but does not respond to surroundings, includ-ing the family members attempting to calm thechild. Amnesia of the event is another one ofthe events distinguishing these from night-mares. Night terrors persist into adolescencein about one-third of the cases. (Rudolph CD,418)

22. (C) The primary lesion in acne is plugging ofthe duct of the sebaceous gland by dried orexcessive sebum and keratin. The obstructedgland (comedo) eventually becomes inflamed.The relative roles of fatty acids, bacteria such asPropionibacterium acnes and S epidermidis,and the immunologic response in determiningthis inflammatory reaction are uncertain.(McMillan, 875)

23. (B) Although all the diseases listed can causenasal obstruction and discharge, allergic rhinitisis by far the most common and the most likelyin this child. Bilateral choanal atresia causes res-piratory distress in the neonatal period; unilat-eral atresia causes persistent unilateral, purulentdischarge. Immunodeficiency is most likely tocause chronic sinusitis and purulent discharge.Cystic fibrosis is much less common than allergy,and a defect in the cribriform plate is extremelyrare. (McMillan, 2428–2429)

24. (D) Atopic dermatitis (eczema) is characterizedby exudation, pruritus, and lichenification.Involvement in infancy is common, althoughthe distribution and other features may besomewhat different than in older children. Theclassical adult distribution with predilection

Page 298: Questions - uhs.edu.kh You suspect Lyme disease, most likely contracted by which of the following? (A) ingestion of unripe fruit (B) ingestion of spoiled fruit (C) drinking of contaminated

Answers: 15–33 299

for popliteal and antecubital areas usually isnot seen in infancy. Involvement of the face iscommon. Most patients have elevated serumIgE levels and normal levels of the otherimmunoglobulins. (McMillan, 2424–2425)

25. (B) The copperhead is a member of the groupof poisonous snakes known as pit vipers orCrotalids. This group also includes the rat-tlesnake and the cottonmouth (water moc-casin). All members of the group share similartoxins, the effects of which include local tissueinjury and necrosis, neurologic manifestations,generalized bleeding, and shock. The severityof the reaction is related to the amount ofvenom injected (size of snake) and size of thevictim. Local reactions are most common andare seen in almost all significant envenoma-tions. Systemic reactions are less frequent andgenerally are seen only with severe enveno-mations. (Rudolph CD,396–398)

26. (E) The major concern for children with chronicotitis media with effusion (chronic OME) is thatthe associated conductive hearing loss willinterfere with speech and language develop-ment during the critical early years of life.(Rudolph CD,1253)

27. (D) The most common indication for repair ofpectus excavatum is to improve the appear-ance of the chest wall. (Rudolph CD,2001)

28. (E) Nasal polyps are uncommon in childhood.The two leading causes are allergy (allergicrhinitis) and cystic fibrosis. Although allergy ismore common, any child with nasal polypsprobably should have a sweat test, since aller-gic manifestations are not uncommon in chil-dren with cystic fibrosis. Benign (hemangioma,dermoid cyst) and malignant (rhabdomyosar-coma, nasal glioma) lesions are rare and can bemistaken for polyps. (Rudolph CD,1262,1265;McMillan, 2428)

29. (C) Unlike the situation in older children andadults, in infants the most frequent targetorgan of alpha1-antitrypsin deficiency is theliver. This may take the form of prolongedneonatal cholestasis and/or cirrhosis and

portal hypertension. Obstructive lung diseasecan be seen in older children, but is rare ininfancy. (Rudolph CD,1490–1491)

30. (A) Cushing syndrome (glucocorticoid excess)should be considered in any child with thecombination of weight gain and growth arrest.In children younger than 7 years of age, gluco-corticoid excess (noniatrogenic) is most oftensecondary to an adrenal tumor. In older chil-dren, the most common noniatrogenic causeof Cushing syndrome is true Cushing disease,adrenal hyperplasia caused by hypersecretionof pituitary ACTH. Over 80% of these olderchildren have surgically identifiable pituitarymicroadenoma. (Rudolph CD,2045–2046)

31. (B) The most common tumor to cause destruc-tion of the anterior pituitary gland in child-hood is the craniopharyngioma. The diabetesinsipidus that occurs as a result of the pituitarydestruction can be very challenging to manage.The chromophobe adenoma is the most commondestructive lesion in adults, but is rare in child-hood. (McMillan, 1771; Rudolph CD,2237)

32. (A) Acute chest syndrome is a well describedand potentially life-threatening complicationof sickle cell disease. Early recognition andtreatment are essential. Patients with acute chestsyndrome can deteriorate rapidly and there-fore require keen observation. (AAP Section onHematology/Oncology, 109:526–535, 2002; Rudolph CD,1533)

33. (E) Splenic sequestration is characterized byan acutely enlarging spleen and a decreasing(or decreased) hemoglobin, and frequently isassociated with thrombocytopenia. Promptrecognition and treatment is necessary becausesome of the severe cases will rapidly progressto shock and death. Most of these acute splenicsequestration crises occur during infancy andearly childhood and are uncommon after thefourth or fifth birthday, by which time auto-splenectomy (due to recurrent infarctions) hasoccurred. In patients with sickle cell disease orsickle thalassemia, however, the spleen mayremain large and sequestration crises can occurinto the teens and adulthood. (AAP Section on

Page 299: Questions - uhs.edu.kh You suspect Lyme disease, most likely contracted by which of the following? (A) ingestion of unripe fruit (B) ingestion of spoiled fruit (C) drinking of contaminated

300 11: Practice Test

Hematology/Oncology, 109:526–535, 2002; RudolphCD,1532)

34. (A) Since 1992 the American Academy ofPediatrics has recommended infants be placedon their backs to sleep in order to decrease therisk of SIDS. It has been called “The Back toSleep” campaign, and has been very successfulin helping to decrease SIDS by greater than40%. Yet SIDS remains the leading cause ofdeath in infants beyond the neonatal period.(AAP Task Force on Sleep Position and Sudden InfantDeath Syndrome, 105: 650–656, 2000)

35. (E) Modifiable risk factors for SIDS that havebeen documented thus far include prone sleep-ing, soft sleep surfaces and loose bedding, over-heating, maternal smoking during pregnancy,bed sharing (especially if the adults sharingthe bed are smokers), and preterm birth andlow birth weight. The factor most stronglyassociated with protection against SIDS issupine sleeping. Interestingly, studies recentlyhave shown a substantially lower incidence ofSIDS among pacifier-using infants compared toinfants not using pacifiers. However, there is noproof that pacifier use prevents SIDS, and theuse of pacifiers has other negative conse-quences. Therefore, a specific recommendationregarding pacifier use in SIDS prevention hasnot been made by the AAP. (AAP Task Force onSleep Position and Sudden Infant Death Syndrome,105:650–656, 2000)

36. (B) Although some cases of encopresis may besecondary to organic causes (spina bifida,Hirschsprung disease), the majority are psy-chologic or behavioral in nature. In contrast, themajority of cases of enuresis are most often dueto a disturbance of bladder physiology or sleepmechanism, and generally are not consideredto represent an emotional or behavioral disor-der. If a child has been dry, then becomesenuretic (secondary enuresis), emotional fac-tors may be involved in the process. Secondaryenuresis may indicate sexual abuse. (McMillan,670–673,1920–1923)

37. (B) This child probably has cyclic neu-tropenia, a defect in the production of

granulocyte-macrophage colony-stimulatingfactor (GM-CSF). In these patients GM-CSF isproduced in a cyclic rather than continuousmanner. Episodes of neutropenia, often accom-panied by fever, aphthous stomatitis, and cer-vical lymphadenitis recur at regular intervals,usually every 21–42 days. Few other conditionshave such a remarkable periodicity. Schwachman-Diamond syndrome is the association of neu-tropenia with exocrine pancreatic insufficiency.Chediak-Higashi syndrome is an autosomalrecessive disorder characterized by morpho-logically abnormal neutrophils, recurrent infec-tions, partial albinism, mental retardation, andeventual pancytopenia. (McMillan, 1713)

38. (C) Random eye movements, ataxia, develop-mental delay, and abnormal behavior are foundin the opsoclonus-myoclonus syndrome (oth-erwise called “dancing eyes, dancing feet” syn-drome). This syndrome is associated withneuroblastoma. Though the associated tumorstypically are small and resected, neurologicsymptoms may persist. There may be an immunemechanism for these neurologic symptoms.(McMillan, 1778; Rudolph CD,1618)

39. (B) Infantile botulism should be considered inany infant with a history of progressive weak-ness, poor feeding, and constipation. Exceptfor the insidious development of symptoms,the infants can be confused with those withsepsis or hypoglycemia. Their alertness withweakness resembles that seen in Werdnig-Hoffmann disease. However, infants withWerdnig-Hoffmann disease typically presentbefore 6 months of age, lack lower limb deeptendon reflexes, and have very expressivefaces. (Rudolph CD,917–918, 2002, 2280)

40. (B) Snoring is the hallmark of chronic upperairway obstruction from enlarged adenoidsand tonsils in children. An infrequent butimportant complication of this is pulmonaryhypertension with cor pulmonale. These chil-dren appear to hypoventilate, especially whenasleep. The hypoxemia which results from thehypoventilation causes pulmonary arteriolarvasoconstriction, and this in turn results in corpulmonale. (Rudolph CD,1270)

Page 300: Questions - uhs.edu.kh You suspect Lyme disease, most likely contracted by which of the following? (A) ingestion of unripe fruit (B) ingestion of spoiled fruit (C) drinking of contaminated

Answers: 34–50 301

41. (C) Essentially all febrile seizures occur in chil-dren between 6 months and 5 years of age.Indeed, most authors incorporate this agerange into the definition of febrile convulsionswhile some broaden the definition to between3 months and 6 years. The peak age clearly isbetween 6 months and 3 years. In one study, themean age was 23 months. (Rudolph CD,2270–2271)

42. (C) The clinical picture described is extremelysuggestive of a brain tumor. Head tilt in thesecases can be due to several mechanisms,including cranial nerve involvement with acutestrabismus and secondary compensation bytilting the head. Head tilt is particularlycommon with posterior fossa tumors. Centralnervous system tumors are now the mostcommon neoplasms of childhood, accountingfor 20%. The incidence of brain tumors isevenly distributed throughout childhood andadolescence. (Rudolph CD,2207–2211)

43. (C) Hartnup disease is an autosomal recessivedisorder with a single defect in the transport ofmonoamino-monocarboxylic amino acids bythe renal tubules and intestinal mucosa. Thereis increased loss of the involved amino acids(neutral amino acids). Most children remainasymptomatic, and the major clinical manifes-tation is cutaneous photosensitivity. Some chil-dren with this disorder will have attacks ofcerebellar ataxia, which resolve spontaneouslybut also can be reversed by the administrationof nicotinic acid. None of the other conditionslisted are associated with ataxia. (Behrman, 353)

44. (C) The rash described is that of a contact der-matitis, most likely due to poison ivy. Thelesions which are usually papules and vesiclesare intensely pruritic. Restriction of the rash tothe legs also is characteristic of poison ivy.Although the rash of Henoch-Schönlein pur-pura is usually restricted to below the waist, itis purpuric rather than pruritic. (Rudolph CD,1180)

45. (A) Bilious vomiting in the neonate can representany gastrointestinal obstruction distal to theentry of the common duct into the duodenumand requires immediate evaluation and treat-ment. In this case, the “bird beak” appearance of

the distal duodenum is diagnostic of midgutvolvulus probably secondary to malrotation. Theresultant strangulation of the superior mesen-teric artery can lead to total destruction of thejejunoileum. (Klaus, 185; Rudolph CD,1376)

46. (D) Annular pancreas is a congenital malfor-mation in which the pancreas encircles the duo-denum. The condition may be asymptomaticor may cause partial or complete duodenalobstruction. The most frequent presenting com-plaint is vomiting. (Rudolph CD,1464)

47. (C) The classical radiologic finding in duodenalatresia is dilatation of the stomach and of theproximal duodenum—the so-called “doublebubble” sign. The double bubble sign can alsobe seen with annular pancreas, and occasionallymalrotation. Remember, 40% of all neonateswith duodenal atresia have trisomy 21. (Klaus,185; Rudolph CD,1403)

48. (B) During the first year of life, the averageinfant grows about 25 cm (10 in). That is, theygrow from about 51 cm (20 in) at birth to about75 cm (30 in) at the first birthday. Another wayto phrase this is that by the end of the first year,birth length increases by 50%. (Rudolph CD,5)

49. (C) Infants typically double their birth weightby approximately 4 months of age, and tripletheir birth weight by approximately 12 monthsof age. Contrast this with height. A child dou-bles their birth length by 4 years of age andtriples their birth length by 13 years of age.(Rudolph CD,5)

50. (B) The picture described is most suggestive ofthe X-linked recessive disorder of end-organinsensitivity to androgenic hormones (testicularfeminization). Although the 20, 22-desmolasedeficiency does cause male pseudohermaphro-dism, the associated adrenal insufficiency isquickly fatal in infancy, unless diagnosed andtreated. All other forms of congenital adrenalhyperplasia are virilizing rather than feminiz-ing. True hermaphrodism is very rare andrequires that both testicular and ovarian tissuebe present. No evidence for this is presented inthis case. (Rudolph CD,2011, 2085)

Page 301: Questions - uhs.edu.kh You suspect Lyme disease, most likely contracted by which of the following? (A) ingestion of unripe fruit (B) ingestion of spoiled fruit (C) drinking of contaminated

302 11: Practice Test

51. (C) At birth, the average hemoglobin is 17 g/dLin term babies. It then falls rapidly over thenext 2–3 months to a low of about 11 g/dL.Hemoglobin values then gradually rise andeventually equal adult normal values in theearly teen years. (Rudolph CD,1519–1521)

52. (E) Breath-holding spells always terminatespontaneously and are never fatal; they haveno known relationship to sudden infant deathsyndrome. Breath-holding spells are commonin early childhood, have a peak incidencebetween 12 and 18 months and usually disap-pear by about 5 years of age. The exact mecha-nism or etiology is not known but familypedigrees suggest they are associated with anautosomal dominant trait with reduced pene-trance. Although many youngsters will holdtheir breath to the point of cyanosis and syn-cope or seizure, there is nothing to suggest thatbreath-holding is a type of epilepsy. Rather, thesyncope and seizure are secondary to cerebralhypoxia. (Rudolph CD,445)

53. (D) The major manifestation of pinworm (E vermicularis) infection is rectal or perirectalitching. The female worms leave the rectum atnight to lay their eggs in the perineal area anddie shortly thereafter. Eggs remain viable andinfective for 2–3 weeks in indoor environments.(AAP:Red Book, 520–521)

54. (D) Although all the conditions listed as possi-ble answers are associated with bullous lesions,epidermolysis bullosa (EB) is the best fit andshould be highest on the differential diagnosis.There are at least 15 known forms of EB, vary-ing in severity and age of presentation. Manypresent in the newborn period, with lesionsmostly at areas of friction or trauma asdescribed in this patient. Both drug-inducedtoxic epidermal necrolysis (rare in neonates)and staphylococcal scalded skin syndrome(common in neonates) are associated with moregeneralized erythema and lesions than describedin this patient. Neither the lesions of bullousimpetigo nor the lesions of congenital syphilisare related to areas of trauma; the latter have aspecial predilection for the palms and soles.(Rudolph CD,1173, 1187)

55. (C) Juvenile polyps are most commonly foundin the rectum. These are hamartomas withinflammatory infiltration. They are not adeno-matous polyps and are neither malignant norpremalignant. They generally present in thefirst 5 years of life with the passage of red bloodstreaking on the stool. (Rudolph CD,1453–1455)

56. (D) The elevated white blood cell count andlow glucose in the cerebrospinal fluid are mostsuggestive of a bacterial pathogen, of whichgroup B streptococcus would be the most likely.Though a Gram stain of CSF provides pre-sumptive evidence in the evaluation of menin-gitis, it frequently is misleading. The goldstandard for diagnosis of a bacterial pathogenis the culture of the spinal fluid. (AAP:Red Book,620-624; Hay,1089)

57. (E) Cow milk and human milk have many dif-ferences in composition. Cow milk has a higherprotein content (32 vs. 10.6 g/L) and humanmilk has a higher fat (45 vs. 38 g/L) and carbo-hydrate (71 vs. 47 g/L) content. Cow milk alsohas higher sodium and calcium content. Exceptfor colostrum—milk produced during the firstfew days following delivery of the baby—human milk has more calories (747 kcal/L) thandoes cow milk (701 kcal/L). (Behrman, 156)

58. (E) Umbilical hernias are quite common, espe-cially in premature and in African-Americaninfants. Although there is an increased inci-dence of umbilical hernias in infants with a vari-ety of syndromes (eg, Beckwith-Wiedemann)and diseases (eg, hypothyroidism), the greatmajority occur in otherwise normal infants.Most resolve spontaneously and no therapy isrequired except for the rare case that incarcer-ates. An abdominal ultrasound examinationshould be done only if there is clinical evidenceof an abdominal mass or ascites. Strapping ortaping are ineffective and irritating and shouldnot be advised. (Behrman, 528)

59. (B) Chylous ascites is characterized by a highconcentration of lipids in the ascitic fluid. Mostcases in infancy are due to congenital malfor-mation of lymph channels; occasional cases aredue to obstruction of the thoracic duct, tumor,

Page 302: Questions - uhs.edu.kh You suspect Lyme disease, most likely contracted by which of the following? (A) ingestion of unripe fruit (B) ingestion of spoiled fruit (C) drinking of contaminated

Answers: 51–68 303

inflammation, or intestinal obstruction. Loss ofalbumin, gammaglobulin, and lymphocytes in the ascitic fluid leads to depletion of theseelements from the blood. The prognosis isguarded, even with treatment. Spontaneousresolution has been reported, but is the excep-tion rather than the rule. (Rudolph CD,1497, 1515)

60. (A) The clinical picture described is typical ofacute pancreatitis, and can be the result ofeach condition listed. However, of these,blunt abdominal trauma (including childabuse) would be the most common cause.(McMillan, 2010)

61. (D) Although idiopathic scoliosis can present atany stage of growth, it is most commonly man-ifest during, or shortly before, adolescence. It isconsiderably more frequent in girls than inboys. These children are mentally and intellec-tually normal. The condition is never painfuland is usually detected on routine physicalexamination. The diagnosis is made on clinicalgrounds using physical examination and aninclinometer. When the scoliosis exceeds 6 or 7 degrees with the use of the inclinometer, x-rays will be used for a more precise assess-ment. (McMillan,2487–2491)

62. (A) The Wiskott-Aldrich syndrome, an X-linked disorder, typically presents during the first 6 months of life with recurrent inf-ections or bleeding episodes. The unusualaspect of the thrombocytopenia is that thethrombocytes are very small and thus calledmicrothrombocytes. These small platelets arenot common in any other thrombocytopenicdisease. The treatment of choice is with HLA-matched bone marrow transplantation. (McMillan,2467)

63. (B) Testicular torsion actually is torsion of thespermatic cord and is a true surgical emergencybecause irreversible change can occur in testeswithin 4–6 hours, and after 24 hours, infarc-tion is the rule. This condition occurs in 1 in4000 males 25 years of age and younger, withpeak occurrence in the preadolescent agegroup. (Rudolph CD,1740)

64. (B) The minute structures attached to the hairare most likely nits, the eggs of the scalp louse(pediculosis capitis). The safest and most effectivetreatment of scalp lice is the topical applicationof a 1% permethrin cream rinse. Griseofulvin isuseful for certain fungal infections such as tineacapitis, but is not effective against pediculosis.(AAP:Red Book,488–492)

65. (B) Chemical pneumonitis is the most commonclinical manifestation of poisoning with hydro-carbons such as kerosene and may result fromthe aspiration of even minute amounts. Centralnervous system changes are less common butcan be seen. (Rudolph CD,366–367, 1988)

66. (B) Congenital hypothyroidism may be associ-ated with prolonged neonatal hyperbilirubine-mia, either indirect or mixed. The indirecthyperbilirubinemia is due to impaired hepaticglucuronidation of bilirubin and to enhancedenterohepatic circulation of bilirubin secondaryto decreased intestinal motility. The mechanismof the mixed hyperbilirubinemia is uncertain.Hypothyroidism also can be associated withanemia and impaired renal function, but theseare mild. (Rudolph CD,2065)

67. (B) Laryngomalacia is a condition of excessivecompliance, or softness, of the cartilage of thelarynx. It is presumed to be a problem of mat-uration rather than a congenital abnormalityand is the most common cause of persistentinspiratory stridor in the young infant. Symp-toms usually begin in the first month of life; areworse with agitation and in the supine posi-tion; and eventually resolve, usually by a yearor so. Foreign body aspiration is an importantcause of persistent stridor in the older infantand toddler but would be very unusual at thisage. (Rudolph CD,1271)

68. (B) Hypercalciuria has been found in a verysignificant number of children with otherwiseunexplained gross or microscopic hematuria.In most cases, the hematuria is not associatedwith renal stones or precedes the formation ofstones by many years. The serum level of cal-cium is normal in these patients. (Rudolph CD,1663)

Page 303: Questions - uhs.edu.kh You suspect Lyme disease, most likely contracted by which of the following? (A) ingestion of unripe fruit (B) ingestion of spoiled fruit (C) drinking of contaminated

304 11: Practice Test

69. (B) The chest roentgenogram reveals air trap-ping of the left lung with a shift of the heart andmediastinum to the right. This is most sugges-tive of a foreign body in the left mainstembronchus. In the case of infantile lobar emphy-sema, the findings would be present in the firstfew months of life and would not cause thesudden onset of wheezing at 4 years of age.Also, in this condition, one lobe rather than anentire lung would be overdistended. Asthma isassociated with bilateral, rather than unilateral,air trapping. (McMillan, 1464–1466)

70. (B) The unequal leg length and asymmetry ofthe inguinal (anterior) and thigh (posterior)skin creases are indicative of severe develop-mental dysplasia of the hip. Neither tibial tor-sion nor equinovarus deformity affect leglength or inguinal skin creases. Arthrogryposisis associated with severe joint contractures.(Rudolph CD,2434–2436)

71. (E) Any child with uveitis involving primarilythe anterior structures of the eye should be sus-pected of having juvenile idiopathic arthritis(JIA). This association is strongest in girls withthe pauciarticular form of JIA, especially thosewho are antinuclear antibody positive. Toxo-plasmosis and toxocara are more likely to beassociated with posterior uveitis. Neither leuke-mia nor hypoparathyroidism are associatedwith uveitis. (Rudolph CD,837, 2382)

72. (E) Intoeing is a very common pediatric issueand in most cases represents a developmentalprocess which is self-limited. Femoral antever-sion is seen in preschoolers and typically resolvesaround the age of 8 years. Tibial torsion typi-cally is present in toddlers and is diagnosed withthe toddler in a sitting position with legs dan-gling from the table at 90 degrees. Metatarsusadductus is seen more commonly in infants, andinvolves the foot. (Rudolph CD,2422–2425)

73. (E) There are many factors to be considered inthe decision to obtain throat swab specimens fortesting of GAS. Testing is not recommended inchildren with signs and symptoms such ascoryza, conjunctivitis, hoarseness, anteriorstomatitis, diarrhea, and discrete ulcerative

lesions which are highly suggestive of viral syn-dromes. Testing generally is not recommendedin children younger than 3 years of age becausethe risk of rheumatic fever is so remote. Testingis recommended only for symptomatic house-hold contacts of patients with documented GASpharyngitis, but is recommended for all house-hold contacts of patients with documentedrheumatic fever or poststreptococcal glomeru-lonephritis. (AAP:Red Book,613–614)

74. (A) Lymphocytic thyroiditis (Hashimoto thyroiditis) is an autoimmune disorder andcirculating antithyroid antibodies can be demon-strated in most patients. However, other auto-immune disorders, including diabetes mellitus,adrenal insufficiency, hypoparathyroidism, andpernicious anemia, occur in only a minority ofpatients. The thyroid gland is invariably enlarged,often irregularly so. Most patients are euthy-roid, some are hypothyroid, and a few arehyperthyroid. (Rudolph CD,2070)

75. (B) Before the 1970s, zinc deficiency was notknown to be the cause of this devastating con-dition called acrodermatitis enteropathica.Breastfed infants are at risk of zinc deficiencywhen the maternal levels are low and vegetar-ians are at risk of zinc deficiency because thebioavailability of zinc in plant sources is low.Therefore, this infant was at risk. Patientsreceiving TPN also are prone to this condition,and should receive supplementation and mon-itoring. (Rudolph CD,1338; McMillan, 175)

76. (E) Primary pulmonary hemosiderosis is anuncommon disease of unknown origin charac-terized by repeated pulmonary hemorrhages.The clinical picture of recurrent episodes of fever,cough, and radiographic infiltrates is frequentlymisinterpreted as recurrent pneumonia, espe-cially in infants and young children who do notexpectorate and in whom hemoptysis may goundetected. The first clue often is a severe irondeficiency anemia. Some of these infants or chil-dren have improvement in pulmonary manifes-tations when cow milk is eliminated from theirdiet. Diffuse pulmonary hemangiomatosis andextramedullary pulmonary erythropoiesis arenonentities. (Rudolph CD,1997–1998)

Page 304: Questions - uhs.edu.kh You suspect Lyme disease, most likely contracted by which of the following? (A) ingestion of unripe fruit (B) ingestion of spoiled fruit (C) drinking of contaminated

Answers: 69–84 305

77. (B) Chondromalacia patellae (also calledpatellofemoral pain and patellar compressionsyndrome) is a repetitive stress injury result-ing from rubbing between the femoral grooveand patella associated with high forces gen-erated through sports and jumping. As withother conditions suffered from overuse or rep-etition, the pain remits with rest. Night pain orunremitting pain in this situation shouldalways prompt further evaluation. (Rudolph CD,2432–2433)

78. (D) The anterior fontanel normally closesbetween 9 and 18 months of age. Each of thechoices, except for craniosynostosis, is associ-ated with delayed closure of the anteriorfontanelle. Craniosynostosis is the prematurefusion of skull sutures which results in abnor-malities of calvarial shape and occurs in about1 in 2000 live births. The most common form ofsingle suture fusion is sagital synostosis.Although most cases of craniosynostosis arediagnosed in the newborn period, early clo-sure of the fontanelles should raise suspicionfor this diagnosis, and with careful examinationyou should be able to palpate the ridging of theprematurely fused suture. (Rudolph CD,755)

79. (E) The rash described in this question soundslike urticaria, for which there are many causes.Although allergy to foods, drugs, or chemicalsis high on the list, in most cases, no specificcause can be found. A few cases are associatedwith infections caused by hepatitis viruses orgroup A Streptococcus. A very few cases aredue to collage vascular disease or malignancy.Hereditary deficiency of C1-esterase inhibitor(extremely rare) is usually associated withangioedema rather than urticaria. (Rudolph, 1195)

80. (C) The XYY karyotype is relatively common,occurring in about 1 in 1000 males. Patientswith this karyotype generally are tall. It isdebatable whether or not the XYY karyotype isassociated with an increased risk of impulsiv-ity, antisocial behavior, and psychopathic per-sonality. But whether or not this is true, mostXYY individuals are behaviorally normal,although IQ and language development tendto be below normal. (Jones, 70–71, 707)

81. (A) Thrombocytopenia with giant platelets inan otherwise well-appearing newborn mostlikely represents platelet destruction second-ary to transplacental transmission of maternalantiplatelet IgG autoantibodies. Maternalthrombocytopenia would be expected if thiswere the case. If the maternal thrombocyteswere normal and she had no history of autoim-mune disease, alloimmune thrombocytopeniais the likely diagnosis. This would result fromtransplacental passage of a maternal antibodythat specifically recognized an antigen onbaby’s platelets that baby inherited from thefather. (Klaus, 475)

82. (C) Pertussis is most deadly for young infants.Complications include pneumonia, seizures,encephalopathy, and apnea. The case fatalityrate in the United States is 1% for those infantsless than 2 months of age. In adolescents, the ill-ness more likely presents as a persistent cough.For those in between these age groups, themore classic picture is seen: paroxysms of 10–30uninterrupted staccato coughs followed by along loud gasping inspiration known as a“whoop,” accompanied by an absolute lym-phocytosis. Following implementation of per-tussis immunization in the 1950s, the numberof annual US cases fell dramatically. Pertussisincidence has increased since 1980 in the USand increase in incidence has been mostnotable for adolescents and adults. In 2005, per-tussis containing vaccine (Tdap) for adolscentsand adults has been available with hopes thatimplementation will reduce the large group ofsusceptibles and decrease the risk of disease ininfants too young to immunize. (AAP:RedBook,498–520)

83. (A) The abnormality in pseudohypoparathy-roidism is a defect in end-organ (kidney andbone) response to parathyroid hormone. Serumlevels of parathyroid hormone are normal or elevated. Short stature and skeletal abnormali-ties, including demineralization, are common.Fingers are short and broad (brachymetacarpals).(Rudolph CD,2011)

84. (B) Also called pseudohemophilia, vonWillebrand disease is characterized by a

Page 305: Questions - uhs.edu.kh You suspect Lyme disease, most likely contracted by which of the following? (A) ingestion of unripe fruit (B) ingestion of spoiled fruit (C) drinking of contaminated

306 11: Practice Test

prolonged bleeding time. There is a variabledeficiency or molecular abnormality of factorVIII, giving rise to a variably abnormal PTT.The platelet count, factor XII level, and pro-thrombin time are all normal. The location forthe encoding of the von Willebrand factor ison chromosome 12. Most commonly, childrenwith von Willebrand disease will present witha history of nosebleeds and easy bruising.(Rudolph CD,200)

85. (A) Dysgenesis (aplasia, dysplasia, hypopla-sia) of the thyroid gland is the most commoncause of congenital hypothyroidism. Inbornmetabolic errors of thyroid synthesis are muchless frequent. Maternal ingestion of iodides (asin expectorants) is a recognized cause of neona-tal hypothyroidism but is rare today. (Rudolph CD,2066, 2068–2069)

86. (B) Cysticercosis is a condition of generalizedtissue invasion by the larva of T solium andresults from the ingestion of T solium eggswhich have contaminated food or cookingutensils. Eating undercooked infected porkresults in ingestion of an encysted worm whichcauses an intestinal tapeworm infestation, not cysticercosis. Cysticercosis is endemic inMexico and is one of the most common causesof seizures in children in that population. Inthe United States it is seen primarily in areaswith large Latin American immigrant popula-tions. The central nervous system is the mostcommon target organ and seizures are the mostfrequent presenting complaint. (AAP:Red Book,644–646)

87. (E) Approximately 40% of adults with cysticfibrosis have diabetes mellitus. It is thought toresult from chronic obstruction of ductal flowfrom inspissated secretions which results inpancreatic fibrosis and beta-cell destruction. Itbehaves like maturity onset diabetes, with littletendency to ketoacidosis. Diabetes shouldalways be considered when an older child withcystic fibrosis loses weight. (Rudolph CD,2114)

88. (A) Elevated sweat chloride concentration isan almost universal finding in patients withcystic fibrosis (CF). Even in this era of molecular

diagnostics, the diagnosis of cystic fibrosis mostoften involves a sweat test. Normal childrenhave sweat chloride concentrations less than40 meq/L while CF patients have concentra-tions greater than 60 meq/L. It is strongly rec-ommended that sweat tests be performed onlyin established CF centers because tests per-formed outside these centers have unaccept-ably high rates of false positive and falsenegative results. (Rudolph CD,1972)

89. (D) Acute otitis media (AOM) resolves sponta-neously in about 80% of cases and 60% of chil-dren are pain free in 24 hours, with or withoutantibiotic treatment. The bacteria most com-monly involved in acute otitis media in child-hood in the United States is Streptococcuspneumoniae which accounts for 30–40% of AOM;40% of which are resistant to penicillin and mostdo not spontaneously resolve. H influenzae(causes 25% of AOM, 25% are beta-lactamaseproducers), and Moraxella catarrhalis (causes12% of AOM, 100% are beta-lactamase produc-ers) are more likely to resolve without therapy.Most experts agree that amoxicillin remains theinitial drug of choice, and that an increased dose(80 mg/kg/day) is indicated given the pneu-mococcal resistance rates. (Rudolph CD,1249–1252)

90. (A) Inguinal hernias are not at all rare in femaleinfants, and the great majority of these infantsare genetically female and otherwise normal.Such hernias may contain intestine or an ovary,either of which can incarcerate. Prompt surgi-cal correction is indicated. Most inguinal her-nias in infants, males or females, are indirect.(Rudolph CD,1742; McMillan, 1640–1642)

91. (B) Large ventricular septal defects (VSD) usu-ally are associated with left deviation of theelectrical axis and evidence of left ventricularhypertrophy on electrocardiogram, althoughsigns of combined ventricular hypertrophy arenot uncommon. An enlarged heart is common,but the pulmonary vasculature is increased, notdecreased. The murmur of a VSD is systolic,rather than continuous. With large left-to-rightshunts and a high pulmonary flow, there alsomay be a mid-diastolic rumble, but still not acontinuous murmur. (Rudolph CD,1354–1356)

Page 306: Questions - uhs.edu.kh You suspect Lyme disease, most likely contracted by which of the following? (A) ingestion of unripe fruit (B) ingestion of spoiled fruit (C) drinking of contaminated

Answers: 85–99 307

92. (C) All of the choices listed are manifestationsof hypoparathyroidism, the hallmark of whichis hypocalcemia and hyperphosphatemia withresultant neuromuscular instability. The mostcommon presentation is seizure. (Rudolph CD,1768)

93. (D) Nearly 60% of adolescent women willexperience dysmenorrhea. It is the leadingcause of school absenteeism among theseyoung women. Most cases are primary dys-menorrhea (painful menses without pelvicpathology), and are associated with high levelsof prostaglandins which correlate with theseverity of the symptoms. For these youngwomen, nonsteroidal anti-inflammatory agentsare generally very effective. Oral contracep-tives often are a good therapeutic optionbecause they successfully treat the dysmenor-rhea in 80%–90%. (McMillan,565–566)

94. (D) This child meets diagnostic criteria forKawasaki disease. To establish the diagnosisthe patient must have fever for at least 5 days,then have at least four of the following five cri-teria: (1) bilateral bulbar conjunctival injectionwithout exudates; (2) changes in oral mucosaincluding erythematous mouth and pharynx,strawberry tongue, and red, cracked lips; (3) rash that is polymorphous, generalized,morbilliform, maculopapular or scarlatiniform,or erythema multiforme-like; (4) changes inperipheral extremities consisting of indurationof hands and feet, palmoplantar erythema, orperiungual desquamation; (5) acute, nonsup-purative cervical adenopathy with at least onenode measuring 1.5 cm. The complication ofcoronary artery aneurysms occurs in about 25%of untreated patients. This declines to about3%–6% with intravenous gammaglobulin treat-ment. Treatment should be initiated as soon asthe diagnosis is made. (AAP:Red Book,392–393;Rudolph CD,412–415)

95. (A) Fanconi syndrome is a proximal tubuletransport disorder which leads to hypophos-phatemia, hypokalemia, and a hyperchloremicmetabolic acidosis. The most common inher-ited cause in children is cystinosis, a disorder inwhich cystine accumulates in the lysosomes

eventually leading to cell dysfunction. Childrenwith this disorder typically are healthy for thefirst few months of life, then develop the symp-toms described. Two other choices, tyrosinemiaand glycogen storage disease also are associ-ated with Fanconi syndrome. (Rudolph CD,1708)

96. (A) Pityriasis rosea (PR) is a common dermato-sis. It is characterized by a progressive eruptionof dozens of 2–10-mm, salmon-colored flatpatches with a scaly perimeter. Lesions may beintensely pruritic. Characteristically the lesionsfollow skin lines and on the back create the so-called “Christmas tree” pattern. This general-ized rash may be preceded by several weeks bya herald patch, a larger well-circumscribed lesionon the extremities or trunk. In any patient whoalso has palmoplantar or mucous membranelesions or who is sexually active, serologic testingfor syphilis is indicated. Epidemiologic evidencehas long supported the idea this has an infectiousetiology. Recently, PR has been described inpatients with active human herpesvirus 6 andhuman herpesvirus 7 infection. (Rudolph CD,1181;Watanabe, 119:779–780, 2002)

97. (B) The presenting sign in over 60% of patientswith retinoblastoma is leukocoria, or whitepupil. Often, this is noted during a well-childexamination. This patient represents the aver-age age at diagnosis, 18 months. In 30%–35% ofpatients, tumors occur bilaterally. There is a 90%survival rate for patients with small, unilateral,promptly-treated tumors. (Rudolph CD,2396)

98. (C) A grave disparity regarding neonatal andinfant mortality rates exists between AfricanAmerican babies and babies of other races.Though preterm births are more commonamong African American mothers, this doesnot completely explain the nearly double ratesfor infant and neonatal mortality as comparedto Caucasians. (Rudolph CD,57)

99. (D) Children with constitutional growth delayhave no endocrine disease but lag behind theirpeers in both growth and physiologic matura-tion. Such children are small for their age. Boneage also is delayed and, therefore, is commen-surate with height. Puberty is delayed, and,

Page 307: Questions - uhs.edu.kh You suspect Lyme disease, most likely contracted by which of the following? (A) ingestion of unripe fruit (B) ingestion of spoiled fruit (C) drinking of contaminated

308 11: Practice Test

consequently, growth continues for a longerthan normal period of time, resulting in ulti-mate adult height which is normal. (Rudolph CD,2103)

100. (B) The most common form of systemic vas-culitis in childhood is Henoch-Schönlein pur-pura. The most consistent sign is the presenceof the purpuric rash on the lower extremities.Unfortunately, this rash does not necessarilydevelop at the beginning of the illness.Frequently a child is evaluated for arthritis orabdominal pain for 1–2 days before the onset ofthe rash makes the diagnosis apparent. Therenal disease associated with HSP typically isvariable, with most children having completerecovery. However, 3–4% of children will deve-lop end-stage renal disease. (Rudolph CD, 842,1689)

101. (B) This child most likely has transient synovi-tis of the hip, an acute inflammatory processthat is generally self-limited. It is most impor-tant for this to be distinguished from a septichip, which typically presents in youngerpatients and is accompanied by fever and lab-oratory evidence such as increased white bloodcell count, increased platelets, or increased ery-throcyte sedimentation rate. And, follow-up isyour friend. This child should be closelyobserved and examined again within 24 hours;sooner if there are any parental concerns orworsening symptoms. (Hay, 822)

102. (B) The selective disturbance of skeletal growthin achondroplasia results in disproportionatelyshort extremities. In the extremities the proxi-mal bones are further disproportionately short-ened in relation to the distal bones. The fingersare short and stubby. Hydrocephalus is seenoccasionally, possibly related to a small fora-men magnum. Overall height, of course, ismarkedly decreased. These patients character-istically have a relatively large head, bulgingforehead, and depressed nasal bridge. Thediagnosis is confirmed by skeletal radiographs.Most cases arise from a new mutation at FGFR3codon 380 to normal parents. However, achon-droplasia, when inherited, behaves as an auto-somal dominant trait. (Lissauer, 288)

103. (D) Goat milk, compared to human milk andcow milk, is low in folate and iron. Babiesexclusively fed goat milk are susceptible tomegaloblastic anemia from folate and B12 defi-ciency. Additionally, if the goat milk is freshand not boiled, the infants are at risk of bru-cellosis. (Behrman, 156–157)

104. (E) Rotavirus is the most common cause ofwinter diarrhea in the United States and is aleading cause of death among children indeveloping countries. Hypernatremic dehy-dration (serum sodium > 150 meq/L withdehydration) is frequently seen in hospitalizedinfants with rotavirus infection. A significantdecrease in hospitalizations for rotavirus enteri-tis has followed the implementation in 2006 oflive attenuated rotavirus vaccine. There are cur-rently 2 preparations on the market whichdiffer in number of required doses but bothappear to be equally efficacious at this time.(AAP:Red Book,572–574)

105. (A) Motor vehicle accidents lead to approxi-mately 5500 adolescent deaths each year in theUnited States. Homicides result in the death ofnearly 3000 adolescents per year, making thisthe second most common cause of death in thisage group. Of these 3000 deaths, 2500 are asso-ciated with firearms. The importance of fire-arms in these deaths is evident. An additional1200 suicidal deaths are associated with fire-arms each year. (McMillan, 140)

106. (E) Despite the devastating effect congenitalinfection with cytomegalovirus can have onthe fetus and newborn, most congenital infec-tions are asymptomatic. If however, symptomsof infection are present at birth, most will havesevere neurodevelopmental disabilities such ascerebral palsy, epilepsy, sensorineural hearingloss, and learning difficulties. Most severeeffects are seen in infants whose fetal infectionoccurred during the first half of pregnancy.(Lissauer, 73–74; AAP:Red Book,273–277)

107. (D) The syndrome of inappropriate secretion ofantidiuretic hormone (SIADH) is a commoncomplication of a variety of central nervoussystem problems, including bacterial meningitis.

Page 308: Questions - uhs.edu.kh You suspect Lyme disease, most likely contracted by which of the following? (A) ingestion of unripe fruit (B) ingestion of spoiled fruit (C) drinking of contaminated

Answers: 100–116 309

Usually the SIADH resolves as the meningitisis treated, even if a subdural effusion is present.(McMillan, 2603–2605)

108. (B) Theoretically, the syndrome of inappropri-ate ADH secretion would be most accuratelydiagnosed by measurement of serum ADHlevels. However, the assay for antidiuretic hor-mone is not readily available. Therefore, thediagnosis usually is confirmed by demonstrat-ing a low urine output and inappropriate highurine osmolality in the presence of low serumosmolality. (McMillan, 2605)

109. (D) Patients with inappropriate secretion ofADH have a decreased urine output and arehypervolemic. They are neither hypotensivenor alkalotic. High urine specific gravity is oneof the criteria for the diagnosis of SIADH.(McMillan, 2605)

110. (B) Therapy of inappropriate ADH secretionconsists primarily of fluid restriction and, ofcourse, treatment of the underlying condition,in this case, bacterial meningitis. Attempts tocorrect the hyponatremia by administration ofsodium usually result in more retention ofwater and further hypervolemia and edema.Intravenous administration of hypertonicsaline would be indicated only if the hypona-tremia were life-threatening. Maintenanceamounts of sodium should be administered.Diuretics are not very effective in this condition.(McMillan, 2605)

111. (B) The cerebrospinal fluid findings, especiallythe xanthochromic appearance after centrifu-gation, indicate an intracranial hemorrhage.The bleeding is primarily subarachnoid or, atleast, there has been extension of the bleedinginto the subarachnoid space. Intraventricularhemorrhage is uncommon except in the small,premature infant. Fever is seen in patients withintracranial hemorrhage. (Rudolph CD,2196, 2238)

112. (C) Of the abnormalities listed, only cerebralarteriovenous malformation (AVM) andaneurysm predispose to hemorrhagic disasters.In children, cerebral AVMs are the most commoncause of spontaneous intracranial hemorrhage.

Unlike the situation in adults, intracranial hem-orrhage in children is rarely due to a rupturedaneurysm. (Rudolph CD,2196, 2238)

113. (A) The most appropriate management at thistime would be to perform a CT scan, followed,if necessary, by further imaging. It would nothave been unreasonable to have performed theCT prior to the lumbar puncture. It is a questionof judgment as to the likelihood of meningitis inthis child with only a slight fever. Intracranialhemorrhage rarely is associated with volumedepletion or the need for transfusion. In theabsence of evidence of a bleeding disorder, thereis no indication for administration of freshfrozen plasma. (Rudolph CD,2196, 2238)

114. (C) The clinical picture described is almostpathognomonic for infantile hypertrophicpyloric stenosis, which would best be con-firmed with abdominal ultrasound. (McMillan,371–373)

115. (E) Most infants with hypertrophic pyloricstenosis develop a hypochloremic metabolicalkalosis secondary to the loss of hydrogen andchloride ions in the vomitus, which in this con-dition is essentially pure gastric secretion.Hypochloremic metabolic alkalosis occurs soregularly in this disorder, that it often is used asa diagnostic feature. (McMillan, 371–373)

116. (B) The physical findings suggest that the childis moderately dehydrated. Glucose and wateralone is never the appropriate hydrating ormaintenance solution for an infant or a child.Bicarbonate should be withheld from this childwho is not described as appearing clinically aci-dotic and who is very likely to have a metabolicalkalosis secondary to the prolonged vomiting.Indeed, the respiratory rate of 12 is rather slowfor an infant of this age and could be explainedon the basis of respiratory compensation for ametabolic alkalosis. Although the child might behypokalemic secondary to prolonged alkalosis,there also is the possibility of hyperkalemia sec-ondary to dehydration and prerenal azotemia.Normal saline is the most appropriate initialrehydrating fluid until the serum electrolytesare documented. (Behrman, 1131)

Page 309: Questions - uhs.edu.kh You suspect Lyme disease, most likely contracted by which of the following? (A) ingestion of unripe fruit (B) ingestion of spoiled fruit (C) drinking of contaminated

310 11: Practice Test

117. (E) In this clinical scenario, urine positive forhemoglobin on dipstick but negative for redblood cells on microscopic examination proba-bly represents myoglobinuria. This would bepresumptively confirmed with an elevatedserum creatine kinase. (McMillan, 1214–1215)

118. (D) Acute myoglobinuria can lead to tubularinjury and acute renal failure. Serum elec-trolytes and renal function should be evaluated.(Rudolph CD,1660)

119. (A) Acute myoglobinuria is caused by influenzaviruses A and B, some enteroviruses, and lesscommonly, adenovirus, herpes simplex virus,and Epstein-Barr virus. This patient most likelyhas influenza virus infection. (AAP:Red Book,401)

120. (D) This child most likely has visceral larvamigrans, which in this country usually iscaused by Toxocara canis, or Toxocara cati, thedog and cat ascarids, respectively. Diagnosiscan be established by biopsy of the liver butnegative biopsy does not exclude the diag-nosis. Since the human is an unnatural host,the parasite does not complete its life cycleand ova or worms do not appear in the stoolor in duodenal fluid. The diagnosis cannotbe established by examination of the bonemarrow or a blood smear. An enzymeimmunoassay for Toxocara antibodies inserum can confirm the diagnosis. Mostpatients, if not blood type AB, have markedlyelevated anti-A and anti-B isohemagglutininantibody titers. Often the diagnosis is madeon the basis of the clinical, hematologic, andserologic data, without liver biopsy. (Rudolph CD,1110–1111; AAP:Red Book,665)

121. (B) Toxocara canis infection usually is acquiredby eating soil contaminated by dog stool. TheT canis eggs which are passed in the dog’s stoolare not directly infective, but in the soil theeggs develop into infective ova containingsecond-stage larva. Infection, therefore, is prob-ably not acquired by direct contact with theanimal, but only by contact with infected soil;usually a history of pica is obtained. (Rudolph CD,1110–1111)

122. (A) If you keep a mental picture of preschool-ers playing in the dirt around puppies all overthe United States, you can begin to rememberthe epidemiology of this infection. This para-sitic infection is more common in the UnitedStates than in developing countries and isequally prevalent in urban and rural popula-tions. In humans, this infection is most fre-quently reported in children of 1–4 years ofage. The vast majority of newborn puppies areinfected. (Rudolph CD,1110–1111)

123. (D) The most likely diagnosis is severe com-bined immunodeficiency syndrome (SCIDS).Both Wiskott-Aldrich syndrome and heredi-tary agammaglobulinemia are X-linked reces-sive conditions and, therefore, are unlikely inthis female child. Most 3-month-old infantshave low, but detectable, levels of IgA and IgM(normal range for 3-month-old: 3–90 and115–120 mg/dL, respectively). The IgG ismaternally derived. HIV infection is a consid-eration but is not included in the differentialdiagnosis listed. Patients with transient hypo-gammaglobulinemia of infancy usually are notas sick as this patient and infection generallydoes not start as early as 6 weeks. Patients withchronic granulomatous disease are generallyboys and primary pathogens include S aureus,Serratia marcescens, and Aspergillus fumigatus.Of the diagnoses listed, only SCIDS explains allthe findings including lymphopenia. (RudolphCD,793–796, 799)

124. (C) Severe combined immunodeficiency is asso-ciated with an absence or paucity of plasmacells in the bone marrow, absence or paucity ofsuperficial lymphoid tissue, and an absentthymic shadow on chest roentgenogram. Cuta-neous reactivity to various common antigensis absent. In an immunologically normal childwith Candida infection, one would expect to finda positive reaction to intradermal testing withCandida antigen. In a child with severe com-bined immunodeficiency, there is global anergyand all skin tests are nonreactive. (Rudolph CD,793–796, 799)

125. (B) The roentgenographic picture describedis typical of Pneumocystis jiroveci infection

Page 310: Questions - uhs.edu.kh You suspect Lyme disease, most likely contracted by which of the following? (A) ingestion of unripe fruit (B) ingestion of spoiled fruit (C) drinking of contaminated

Answers: 117–133 311

(previously known as Pneumocystis carinii), towhich these infants are unduly susceptible.Viruses such as cytomegalovirus could pro-duce a similar picture, but it is not listed as achoice. Lymphangiectasia is a congenital lesionof the lung, unrelated to immunologic disor-ders. Neither pneumococcal pneumonia norpulmonary hemorrhage is likely to be so pro-tracted or indolent. Miliary tuberculosis is apossibility, but usually does not produce suchextensive bilateral disease or a ground-glassappearance radiographically. It also is uncom-mon in patients this young. (Rudolph CD,793–796;AAP:Red Book,537–542)

126. (C) Of the conditions listed, subduralhematoma is the most likely and would bestexplain all the clinical findings. Tuberculousmeningitis can cause obstructive hydrocephalusand a large head, but fever and cranial nerveabnormalities usually are evident on physicalexamination. Mastoiditis is generally associatedwith fever, and, by itself, would not explain theneurologic findings. Pseudotumor cerebri is rel-atively benign and is not associated with con-vulsions. All three conditions—tuberculousmeningitis, mastoiditis, and pseudotumor areuncommon in the first few months of life. Withcongenital toxoplasmosis, other findings anddelayed psychomotor development wouldlikely be present. (Rudolph CD,465, 953, 2398)

127. (C) Retinal hemorrhages may be found in up to40% of cases of subdural hematoma and usu-ally are indicative of the shaking injury as aform of child abuse now termed abusive headtrauma baby syndrome (rather than the oldname, abusive head trauma syndrome). Theforceful shaking of the infant creates shearingforces in the brain and retina, resulting inbleeding in both organs. Congenital toxoplas-mosis is associated with chorioretinitis ratherthan hemorrhage. (Behrman, 1937; Rudolph CD,465)

128. (C) The presentation of this infant necessitatesCT scan of the head to begin to define theextent of the intracranial injuries. Head injuriesremain the most common cause of death tochildren who have been abused. (Rudolph CD,469)

129. (G) The most important intervention in anychild with an elevated lead level is to stopfurther exposure of the child to the source ofthe lead. Children with blood lead levels of45–69 mg/dL should receive chelation ther-apy. Chelation with oral meso-1,3-dimercapto-succinic acid (DMSA) should be consideredfor children who have no signs of sympto-matic lead poisoning. EDTA is the chelationagent of choice for children with symptomaticlead poisoning or severe lead poisoning involv-ing the central nervous system or any childwith a blood lead level of greater than or equalto 70 mg/dL. (Rudolph CD,370)

130. (E) Children with either acute iron poisoning orchronic iron overload from repeated transfu-sions should be treated with the iron chelatingagent deferoxamine. For acute ingestion, thedrug usually is administered intravenously.(Rudolph CD,368)

131. (A) If treatment is begun early (ideally within 24 hours of ingestion), acetylcysteine may pre-vent hepatic injury from acetaminophen inges-tion. Acetaminophen itself is not hepatotoxic,but some of its metabolites are. These hepato-toxic metabolites ordinarily are inactivated byglutathione as soon as they are formed. Largeoverdoses of acetaminophen deplete the liver’sstores of glutathione, permitting the toxic meta-bolite to accumulate. Acetylcysteine replenisheshepatic glutathione, protecting the liver fromthe toxic by-products of acetaminophen metab-olism. (Rudolph CD,360; Hay, 341)

132. (B) Atropine is an important part of the treat-ment of organophosphate poisoning. Althoughit has little effect on the central nervous systemeffects of organophosphate, atropine will controlrespiratory secretions and other cholinergic eff-ects. Pralidoxime, a cholinesterase-regeneratingoxime may be needed to reverse muscle weak-ness. (Rudolph CD,373–374)

133. (T) Jimsonweed is ingested, typically by ado-lescents, for its hallucinogenic properties. Theanticholinergic symptoms this chemical causesinclude the classic “red as a beet, mad as ahatter, hot as a hare, blind as a bat, and dry as

Page 311: Questions - uhs.edu.kh You suspect Lyme disease, most likely contracted by which of the following? (A) ingestion of unripe fruit (B) ingestion of spoiled fruit (C) drinking of contaminated

312 11: Practice Test

a bone.” Physostigmine, which binds compet-itively to acetylcholinesterase in the synapse,can reverse the anticholinergic symptoms.(Tobias, 424; Rudolph CD,359)

134. (H) Ethylene glycol is the primary componentof antifreeze. Organic acids formed by metabo-lism of ethylene glycol are more toxic than theparent compound and are responsible for mostof the major toxicity. Ethanol, an effective anti-dote, works by competing for the enzyme alco-hol dehydrogenase which catalyzes the firststep in the metabolism of ethylene glycol. Thesame enzyme initiates the metabolic pathwayfor methanol and isopropyl alcohol, and there-fore, ethanol is also useful in treating poisoningwith these substances. Some prefer fomepizoleas an alternative. (Rudolph CD,359; Tobias, 423)

135. (O) Naloxone is a competitive antagonist tothe opiates, including morphine. The drug isbest administered intravenously. The usualdose is 0.03 mg/kg, but a second, larger dose(0.1 mg/kg) may be given if there is no response.Naloxone is not itself a depressant, and is a verysafe drug. (Rudolph CD,359, 372)

136. (X) Cyclic antidepressants remain a leadingcause of death from ingestions of pharmaceu-tical agents. These deaths occur principally intwo age groups: toddlers who are unaware andadolescents who are attempting self-harm. Thetreatment of the cardiovascular toxicity seenin this overdose is the administration ofsodium bicarbonate. The therapeutic effect ofalkalinization most likely is multifactorial.(Nichols,1375–1377; Rudolph CD,359)

137. (Q) The most common cause of poisoning deathis carbon monoxide inhalation. The sources mostoften are home fires and exposure to incompletecombustion of carbon fuels. When the source issmoke inhalation, most victims of fatal CO poi-soning will die before arrival to the hospital.However, CO poisoning is treated with the anti-dote 100% oxygen. Hyperbaric oxygen is used inmany centers. (Rudolph CD,359, 363)

138. (E) Most 1-year-old children are able to per-form the tasks described. The pincer grip is

developed on average at 10.5 months of age.(Lissauer, 15)

139. (H) Children typically are able to copy a circleat 3 years of age, and copy a cross at 4 years ofage. Hopping on one foot generally is accom-plished at 4 years of age. (Lissauer, 17)

140. (A) These are expected findings in the evalua-tion of the supine infant (6–8 weeks of age).An 8-week-old infant who does not demon-strate responsive smiling should be furtherevaluated. (Lissauer, 14)

141. (G) The 2-year-old is at the end of the mobiletoddler phase and is expected to walk will withnormal gait, and to be able to kick a ball.Though most 20-month-old toddlers are able tocombine two different words, toddlers aremore likely closer to 24 months of age beforethey are able to build a tower of six 1-in cubes.(Lissauer, :16)

142. (D) The sitting child, 6–9 months of age, typi-cally develops the ability to pull to a stand byage of 9 months, and babbles nonspecifically,but quite happily, by 9–10 months. The palmargrasp is predominant at 6 months of age andthe pincer grasp develops between 10 and 11 months of age. (Lissauer, 15)

143. (A) Newborns should startle to sudden loudnoise, so the best choice is the 2-month-old.(Lissauer, 14)

144. (I) Most 5-year olds are able to copy a triangle.(Remember, most children are able to copy a circle by 3 years of age, a cross by 4 yearsof age, and a triangle by 5 years of age. Theability to copy a square is demonstratedbetween 4 and 5 years of age. Past tense typ-ically is used in language by 4 years of age,and future tense by 5 years of age. (Lissauer, 17;Behrman, 43)

145. (A) Among children with Down syndrome andcongenital heart disease, 40% will have atri-oventricular defect, 30% ventricular septal defect,and 20% atrial septal defect. Approximately 5%will have tetralogy of Fallot. (Lissauer, 174)

Page 312: Questions - uhs.edu.kh You suspect Lyme disease, most likely contracted by which of the following? (A) ingestion of unripe fruit (B) ingestion of spoiled fruit (C) drinking of contaminated

Answers: 134–156 313

146. (C) Williams syndrome, also known as idio-pathic infantile hypercalcemia, features growthdelay, mental retardation, characteristic facialfeatures, stellate iris, and supravalvular aorticstenosis. A very interesting characteristic ofchildren with Williams syndrome is theirunique chatter ability, referred to as “cocktailchatter.” (Rudolph CD,740, 1805)

147. (B) Mitral valve prolapse can be demonstratedby echocardiography in about 80% of patientswith Marfan syndrome. The most importantcardiovascular manifestation, progressive dila-tion of the aortic root and ascending aorta, wasnot listed as a choice. (Rudolph CD,1797)

148. (D) Turner syndrome (45X) occurs in about1/2500 live newborns. Clinical features includeshort stature, neck webbing, ovarian dysgenesiswith infertility, cardiac defects, and normal intel-lectual development. The most common cardiacdefect in this syndrome is coarctation of the aorta.One-third of girls with Turner syndrome havebicuspid aortic valves. (Lissauer, 56; Rudolph CD,1781)

149. (E) Children with Noonan syndrome have ahigh incidence of valvular pulmonary stenosis.(Jones, 122; Rudolph CD,1810)

150. (C) The heliotrope sign is a purplish discol-oration of the periorbital skin which is frequentlyfound in children (typically school-aged chil-dren) with dermatomyositis. Dermatomyositisis a chronic inflammatory disease affecting pri-marily the skin and skeletal muscles. (Rudolph CD,1109)

151. (A) Profound muscle weakness, hypotonia,tongue fasciculations, and swallowing dys-function are characteristic of Werdnig-Hoffmanndisease, or spinal muscular atrophy type 1. Thisdisease typically presents before 6 months of ageand is fatal by 2 years of age. The infantsinvolved are of normal intelligence and normalsocial ability, making their slow demise all themore difficult for the loved ones involved.(Rudolph CD,2002, 2279)

152. (E) Hemolytic uremic syndrome is diagnosedby the presence of microangiopathic hemolytic

anemia, thrombocytopenia, and renal insuffi-ciency. Most affected children will have a pre-ceding gastrointestinal infection with E coli thatproduces a Shigella-like toxin. (Rudolph CD,1696;AAP:Red Book,291–292)

153. (B) Gower sign, or Gower maneuver, reflectship-girdle weakness and most commonly isassociated with Duchenne muscular dystrophy,although it can also be seen in spinal muscularatrophy and in some cases of dermatomyositis.The Gower maneuver enables children withproximal muscle weakness to move from aprone to standing position and involves firstmoving the feet close to the hands, then walk-ing hands up the legs to push the trunk upright.(Rudolph CD,2278, 2289)

154. (D) The most common form of systemic vas-culitis in children is HSP. Nearly 100% of chil-dren with this disorder will develop purpurawith a normal platelet count. The purpuric rashis generally noted on the legs and buttocks andthe arthritis, and abdominal pain complete theclassic triad. (Rudolph CD,842)

155. (H) This scenario most likely represents bac-terial infection following the cat bite. Catbites are particularly prone to infection becausethe very long narrow teeth of the cat inocu-late the cat’s oral flora deep into the tissue. Of thechoices given, the most likely causative agentis P multocida. The drug of choice for treatmentis penicillin. However, because of the possibil-ity of a polymicrobial infection including anaer-obes and S aureus, the drug most often chosenin this scenario is oral amoxicillin-clavulanateor intravenous ampicillin-sulbactam sodium.(AAP:Red Book,487)

156. (C) This clinical picture is most consistent withhuman monocytic ehrlichiosis, most commonlyseen in the southeastern and south centralUnited States. The clinical findings closelyresemble Rocky Mountain spotted fever, anothertick-borne infection though the rash is seen lesscommonly. Leukopenia, hyponatremia, andliver dysfunction can be seen. Both are treatedwith doxycycline even in those less than 8 yearsof age. (AAP:Red Book, 281–283)

Page 313: Questions - uhs.edu.kh You suspect Lyme disease, most likely contracted by which of the following? (A) ingestion of unripe fruit (B) ingestion of spoiled fruit (C) drinking of contaminated

314 11: Practice Test

157. (A) Pharyngoconjunctival fever is an acute viralillness typically seen in summer-time pool out-breaks and manifests with fever, conjunctivitis,and pharyngitis. The most likely etiologic agentis adenovirus. (AAP:Red Book,202–203)

158. (G) Roseola (sixth disease) typically presents inchildren less than 24 months of age. Fever ischaracteristically high and persists 3–6 days.Frequently, the fever abates with the onset ofthe rash. Febrile seizures occur in 10–15% ofcases. Human herpes virus 6 is the mostcommon etiologic agent. (AAP:Red Book,375–377)

159. (I) Most cases of croup (laryngotracheobron-chitis) are due to infection with parainfluenza.Among other viruses, adenovirus, influenzavirus, and measles virus infections can alsocause croup. (AAP:Red Book,478–479)

160. (K) Rat bite fever is caused by Streptobacillusmoniliformis or S minus. The causative agentcolonizes the upper respiratory tract of mostrodents. Typically the fever does not begin atthe time of the bite but develops a few dayslater as the wound appear to be healing. Feveroccurs along with rash and migraotory pol-yarthritis in 50% of patients. In as many as 1/3of cases, close contact with rats without a bite

is noted, so history remains key to making thisdiagnosis. (AAP:Red Book,559–560)

161. (C) Sturge-Weber disease, also called encephalo-facial angiomatosis, is the most unique of theneurocutaneous conditions listed. It does nothave a clear inheritance pattern, lacks cutaneouspigmentation, and does not carry an increasedrisk of tumors. It is a progressive disorder andmay be associated with mental retardation,seizures, hemiparesis, and visual problems. Forchildren less severely affected, deteriorationafter age of 5 years is unusual, but learning dif-ficulties and seizures may persist. The nevusflammeus (port wine stain) generally is in thedistribution of the trigeminal nerve. (McMillan,2025; Lissauer, 310)

162. (B) Neurofibromatosis type 1 (von Recklinghausendisease) is an autosomal dominant neurocuta-neous condition with cafe au lait spots as thehallmark. Typically, the cafe au lait spots developduring the first year of life and increase in sizeand number for the first few years of life. Axillaryfreckling is closely associated with neurofibro-matosis. Cutaneous neurofibromas often are notapparent until puberty. Though inheritance isautosomal dominant, one-half of all cases are theresult of new mutation. (McMillan, 2019–2021)